You are on page 1of 137

CHAPTER 19

Permutations
& Combinations
It is one of the most logical chapters that you are considering for your test
preparation, so it’s a really good thing for those who are pretty uncomfortable with
the traditional Maths-essentially, the colossal equations, intricate graphs and
confusing formulae. Well, if you feel that all these things are intimidating and
discouraging, then here is a nirvana (or relief) from all such nasty things. Now,
what you need to do is just stop relying upon the formulae and try getting the Chapter Checklist
answers using logic (or call it common sense). And, the biggest advantage of using
Counting
the logic is that you would be able to develop a knack for replicating and
expanding the scope of logic in innumerable situations and settings; while there Permutations
are huge limitations in memorizing, recalling and applying the formulae in the Permutations of n Things
twisted and unfamiliar situations, especially under pressure. Usually, it is easy for Not All Different
our minds to apply the same logic in different situations, but it is truly painstaking Circular Permutations
in relating the same formula for different situations. For example, dividing the 10 Combinations
identical chocolates among 3 kids is same as the number of non-negative integral Distribution/Division of Distinct
solutions of a + b + c =10. things among Individuals/Groups
My only advice to you is that please stay open minded, logical and creative Distribution/Division of Identical
enough. If you struggle to get the answer, break the problem down into pieces or Objects Among Individuals/Groups
take a similar but small example in order to understand the underlying idea and its Algebraic Properties
logic. In P&C (Permutation & Combination) the understanding of problem is Re-Arrangement or Derangement
more important than knowing its solution, because the challenge is to decode the
Number Properties
thought process, the calculation part is too simple to be bothered about. Even if
you don’t know the formula/property you can get the required answer just by Geometrical Properties
CAT Test
becoming a bit imaginative, inventive and ingenious. As a business manager or a
leader of a team your job is to arrange the events, things and select the best people
and resources, so why don’t you start it right here and right now.
Therefore, being a very logical chapter it is one of the favorite chapters for the
paper setters of competitive exams. In CAT, you can expect approx. 1-3 problems
from this chapter in almost every session/slot. Other exams like XAT and IIFT too
ask approx. 2-4 sums from this chapter.
Permutations & Combinations 1017

Hint (1, H), (1, T ), (2, H ), (2, T ), (3, H ), (3, T ), (4, H ), (4, T ), (5, H ),
19.1 Counting (5, T ), (6, H ), (6, T ) i.e., the total number of all possible outcomes is
12. i.e., the total number of all possible outcomes is 12.
Fundamental principle of counting
(i) Multiplication : If one operation can be performed in ‘m’ Exp. 4) There are 6 trains running between
ways and corresponding to each way of performing the first Indore and Bhopal. In how many ways can a man
go from Indore to Bhopal and return by a different
operation a second operation can be performed in ‘n’ ways, train?
then the two operations together can be performed in m × n
Solution A man can go from Indore to Bhopal in 6 ways
ways. If after two operations are performed in any one of the by any one of the 6 trains available. Then he can return
m × n ways a third operation can be performed in p ways, then from Bhopal to Indore in 5 ways by the remaining
the three operations together can be performed in m × n × p 5 trains, since he cannot return by the same train by
ways and so on. which he goes to Bhopal from Indore.
Thus the required number of ways = 6 × 5 = 30
In general if there are n jobs (works/operations)
j1 , j2 , j3 , ..., jn such that job ji can be performed Exp. 5) In the previous question, if the person can
independently in mi ways. (i =1, 2, 3, ..., n). Then the total return by any one of the 6 trains, then find the
number of ways in which all the jobs can be performed is number of ways in which he can go from Indore to
m1 × m2 × m3 × ......× mn . Bhopal and return from Bhopal to Indore.
Here the different jobs/operations are mutually inclusive. It Solution He can go from Indore to Bhopal in 6 ways and
he can return from Bhopal to Indore in 6 ways also.
implies that all the jobs are being done in succession. In this
Hence the number of ways in which he can go from
case we use the word ‘and’ to complete the all stages of
Indore to Bhopal and return from Bhopal to Indore
operation and the meaning of ‘and’ is multiplication. = 6 × 6 = 36 ways.
Exp. 1) A student has to select a letter from vowels and Exp. 6) Four persons entered the lift cabin on the
another letter from consonants, then in how many ways can he ground floor of a six floor office. Suppose each of
make this selection? them can leave the cabin independently at any
Solution Out of 5 vowels he can select one vowel (letter) in 5 ways and floor beginning with the first. Find the total
out of 21 consonants he can select one consonant (letter) in 21 ways. number of ways in which each of the four persons
Thus a letter from the vowels and a letter from the consonants can be
can leave the cabin at different floors.
selected together in 5 × 21 = 105 ways.
Solution Suppose M1 , M2 , M3 and M4 are four persons.
(ii) Addition : If there are two jobs such that they can be M1 can leave at any of the 5 floors. So M1 can leave the
performed independently in m and n ways respectively, then cabin in 5 ways. Now, M2 can leave the cabin at any of
either of the two jobs can be performed in ( m + n) ways. remaining 4 floors. So M2 can leave the cabin in 4 ways.
Similarly M3 and M4 can leave the cabin in 3 ways and
In general if there are various jobs which are mutually
2 ways respectively. Thus, the total number of ways in
exclusive, then they can be performed in which each of the 4 persons can leave the cabin at
m1 + m2 + m3 + ... + mn ways. different floors is 5 × 4 × 3 × 2 = 120.
In this case we use the word ‘or’ between various jobs and the
Exp. 7) In the above question if the 4 persons can
meaning of ‘or’ is addition.
leave the cabin at any one of the 5 floors, then find
Exp. 2) A student has to select a letter either from vowels ‘or’ the number of ways in which this can be done.
from consonants, then in how many ways can he make this Solution Since each of the four person can leave the
selection? cabin at any one of the 5 floors. Hence each one of
M1 , M2 , M3 and M4 can leave the cabin in 5 ways.
Solution Out of 5 vowels he can select one letter in 5 ways.
Similarly he can select one letter from 26 consonants in Hence total number of ways in which each of the four
persons can leave the cabin at any of the 5 floors is
21 ways. Thus he can select one letter in 5 + 21 = 26 ways.
5 × 5 × 5 × 5 = 625
Exp. 3) If a die is cast and then a coin is tossed, find the
Exp. 8) In a MOCK CAT there are 3 sections viz,
number of all possible outcomes.
QA, English and D I containing 4, 6 and 11
Solution A die can fall in 6 different ways showing six different points
questions respectively. In how many ways can a
1, 2 , 3, 4, 5, 6, ... and a coin can fall in 2 different ways showing head
candidate select one question from each of the three
(H) or tail (T).
sections?
∴ The number of all possible outcomes from a die and a coin
Solution The candidate can select one question from Q A
= 6 × 2 = 12
in 4 ways, one question from English in 6 ways and one
1018 QUANTUM CAT
question from D I in 11 ways. Hence he can select one question Second section can be answered in 3 × 3 × 3 × 3 = 3 4 ways.
from each of three sections in 4 × 6 × 11 = 264 ways. Hence the whole test paper can be answered in 45 × 3 4 ways.
Exp. 9) A tricolour flag is to be formed having three Exp. 15) How many three letter words can be formed
adjacent strips of three different colours choosen from six using only consonants but each only once?
different colours. How many different coloured flags can
Solution There are only 21 consonants in English alphabet. So,
be formed with different design in which all the three
there are 21 ways of filling up the first place. Now, there are
strips are always in horizontal position ? only 20 ways of filling up the second place and the third place
Solution First strip can be coloured in 6 ways and second strip can be filled up in 19 ways.
can be coloured in 5 ways and third strip can be coloured in 4 Hence, the required number of words = 21 × 20 × 19 = 7980
ways. Hence all the three strips can be coloured in
Exp. 16) How many four digit numbers can be formed
6 × 5 × 4 = 120 ways.
with the digits 3, 5, 7 and 9 only?
Hence there can be 120 differently coloured flags.
4 4 4 4
Exp. 10) In how many ways can a chairman and Th H T U
vice-chairman be elected from a committee of 8 members
who are equally eligible for the post? A four digit number has 4 places viz, thousands, hundreds,
tens and unit place. All the four places can be filled up in
Solution The chairman can be elected in 8 ways, and the
4 × 4 × 4 × 4 = 256 ways since repetition of digits is not
vice chairman can be elected in 7 ways from the remaining
restricted.
7 members.
Hence the required number of ways = 8 × 7 = 56 ways. Exp. 17) How many 3 digit number can be formed
whose unit digit is always zero and repetition of digits is
Exp. 11) In how many ways can 5 prizes be given away
not allowed?
to 7 boys when each boy is eligible for all the prizes?
Solution First prize can be given away to any one of 7 boys in Solution 9 8 1 ⇒ 9 × 8 × 1 = 72
7 ways. Again second prize can be given away to any one of Since zero is fixed for the unit digit place. So only 9 digits are
the 7 boys since each boy is eligible for all the prizes. available for the hundreds place. Now only 8 digits are
Similarly third, fourth and fifth prize can also be given away available for the tens place since out of 10 digits (i.e., 0, 1, 2, 3,
in 7 ways. ..., 9) two digits at two places have been used. Therefore
Hence the required number of ways = 7 × 7 × 7 × 7 × 7 = 75 required possible numbers = 9 × 8 × 1 = 72.

Exp. 12) A test paper consists of 10 questions and each Exp. 18) How many numbers can be formed with digits
question has 4 choices. If each question is necessarily 2, 4, 6, 8 without repetition?
attempted, then find the number of ways of answering the Solution Case 1. There are 4 numbers of 1 digit.
test paper.
Solution Since each question can be answered in 4 ways. Case 2. 4 3 ⇒ 4 × 3 = 12
So, the total number of ways of answering 10 questions There are 12 numbers of 2 digits.
= 4 × 4 × 4 × 4 × 4...10 times = 410.
Case 3. 4 3 2 ⇒ 4 × 3 × 2 = 24
Exp. 13) A set of 6 questions contains true/false type
questions. Maximum how many students can take the test There are 24 numbers of 3 digits.
if all the students answer differently from others and must
attempt all the questions? Case 4. 4 3 2 1 ⇒ 4 × 3 × 2 × 1 = 24
Solution A question can be answered in two ways i.e. either There are 24 numbers of 4 digits.
true or false. Hence all the 6 questions can be answered in Hence total required numbers = 4 + 12 + 24 + 24 = 64
2 × 2 × 2 × 2 × 2 × 2 = 26 = 64 ways.
NOTE No any number of 5, 6, 7, .... digits can be formed since we
Thus there can be maximum 64 students. have only four digits and repetition of digits is not allowed.

Exp. 14) In a test paper first section contains 5 question Exp. 19) How many four digit numbers can be formed
each with 4 choices and second section contains using 5 only once but 5 must be used in all such numbers?
4 questions each with 3 choices. In how many different
ways can the paper be answered if all the questions are Case 1. ⇒ 8 × 9 × 9 × 1 = 648
Solution 8 9 9 1
attempted?
Solution First section can be answered in When 5 is fixed at unit place, then there are only 8 digits
4 × 4 × 4 × 4 × 4 = 45 ways available for thousands place (i.e., except to 0 and 5).
Permutations & Combinations 1019

Again hundreds and tens places can be filled by any one of at a time are
digits from 0 to 9 except 5. ab, ba, bc, cb, ac, ca
Case 2. 8 9 1 9 ⇒ 8 × 9 × 1 × 9 = 648 Therefore the number of permutations of three different
When 5 is fixed at tens, place then there are only 8 digits things taken two at a time is
available for thousands place and 9 digits are available for 3!
each of the hundreds and unit places.
3
P2 = =6
(3 − 2)!
Case 3. 8 1 9 9 ⇒ 8 × 1 × 9 × 9 = 648
Similarly, the permutations of the set of four letters
Here 5 is fixed at hundreds place. S , W , A, G number of taken two at a time is
Case 4. 1 9 9 9 ⇒ 9 × 9 × 9 = 729 SW , WS , SA, AS , SG, GS , WA, AW , WG, GW , AG, GA
Here 5 is fixed at thousands place and each of the hundreds, Therefore, the number of permutations of three different
tens and unit place can be filled up in 9 ways each. things taken two at a time is
Therefore, the total required numbers of ways 4!
= 648 + 648 + 648 + 729 = 2673
4
P2 = = 12
( 4 − 2)!
Exp. 20) How many four digit numbers can be formed NOTE n! = 12
. .3. 4 . . .( n − 1). n
with the digits 0, 2, 3, 5, 8, 9 if ( n − r )! = 12
. .3. 4 . . . ( n − r − 2)( n − r − 1)( n − r )
(i) repetition of digits is allowed? 0 ! = 1 = 1!
(ii) repetition of digits is not allowed?
Permutations of n Different Things
Solution (i) 5 6 6 6 ⇒ 5 × 6 × 6 × 6 = 1080
1. Number of permutations of n different things taken all
Since at thousands place zero can not be placed. at a time = n Pn = n!
(ii) 5 5 4 3 ⇒ 5 × 5 × 4 × 3 = 300 2. Number of permutations of n different things taken r at
n!
Since at thousands place zero cannot be placed so only a time = n Pr =
5 digits are available and there are only 5 remaining digits ( n − r )!
available for hundreds place and 4 digits for tens place and
3. Number of permutations of n different things taken at
3 digits for unit place.
most r at a time = n P1 + n P2 + n P3 + . . . + n Pr
19.2 Permutations 4. Number of permutations of n different things taken at
Out of the given set of finite number of things, you take all least r at a time = n Pr + n Pr + 1 + n Pr + 2 + . . .+ n Pn
or some of them and arrange in distinct possible ways, then 5. Number of permutations of n different things taken r at
each such arrangement is called the permutation. In a time, when one particular thing always occurs
Permutation, order of the things is very important. = r ⋅ ( n − 1 Pr − 1 )
Permutation means selection and arrangement both, while
6. Number of permutations of n different things taken r at a
Combination means only selection.
time, when one particular thing never occurs = n − 1 Pr
˜ A permutation is said to be a Linear Permutation if the
objects are arranged in a line. A linear permutation is 7. Number of permutations of n different things taken r at
simply called as a permutation. a time, when k particular things, always occur
= ( r Pk )( n − k Pr − k )
˜ A permutation is said to be a Circular Permutation if the
objects are arranged in the form of a circle. 8. Number of permutations of n different things taken r at
˜ The number of (linear) permutations that can be formed a time, when k particular things never occur = n − k Pr
by taking r things at a time from a set of n distinct things 9. Number of permutations of n different things, taken all
is denoted by n Pr or P ( n, r ), for every 1 ≤ r ≤ n. at a time, when m specified things always come
n! together = m!( n − m + 1)!
n
Pr = n( n − 1)( n − 2)( n − 3) . . . ( n − r + 1) =
( n − r )! 10. Number of permutations of n different things, taken all
Thus the permutations of three letters (say) a, b, c taken two at a time, when m specified things never come together
= n! − [ m!( n − m + 1)!]
1020 QUANTUM CAT
Introductory Exercise 19.1

1. Find the value of 9 P3 . Directions (for Q. Nos. 13 to 22) : Answer these questions
(a) 504 (b) 309 (c) 405 (d) 600 based on the following information.
2. Find the value of n, if nP5 = 20 nP3 . After completing his MBA, Zafar shifted to a new flat where he
bought a new bookshelf and a set of 7 novels. The names of these
(a) 5 (b) 8 (c) 6 (d) 4
novels, in the chronological order, are A, B, C, D, E, F and G.
3. If 56
Pr + 6 : Pr + 3 = 30800 : 1, find r.
54
13. Find the number of ways in which he can arrange all
(a) 14 (b) 20 the 7 novels in his bookshelf.
(c) 41 (d) 21 (a) 5040 (b) 4050 (c) 1 (d) 7
4. How many different numbers of 3 digits can be 14. If he wants to keep exactly 4 novels in his bookshelf,
formed with the digits 1, 2, 4, 5, 7, 8 ; none of the find the number of ways in which he can arrange
digits being repeated in any of the numbers so 4 novels.
formed? (a) 420 (b) 24 (c) 28 (d) 840
(a) 120 (b) 1200 15. If he wants to keep at most 4 novels in his bookshelf,
(c) 180 (d) 270
find the number of ways in which he can arrange the
5. How many even numbers less than 10,000 can be novels.
formed with the digits 3, 5, 7,8, 9 without any (a) 1100 (b) 1099 (c) 999 (d) 10
repetition? 16. If he wants to keep at least 4 novels in his bookshelf,
(a) 32 (b) 16 (c) 44 (d) 41
find the number of ways in which he can arrange the
6. How many numbers with different digits each novels.
greater than 4000 can be formed from the digits (a) 13440 (b) 1440 (c) 12440 (d) 840
0, 2, 5, 7, 8 ? 17. If he always wants to keep the first novel of the series
(a) 160 (b) 168 (c) 320 (d) 270
in his bookshelf, find the number of ways in which he
7. How many numbers greater than 5000 can be can arrange total 4 novels in his bookshelf.
formed with the digits, 3, 5, 7, 8, 9 no digit being (a) 120 (b) 480 (c) 720 (d) 24
repeated? 18. If he never wants to keep the last novel of the series
(a) 216 (b) 126 (c) 512 (d) 252
in his bookshelf, find the number of ways in which he
8. How many 6-digit telephone numbers can be can arrange total 4 novels in his bookshelf.
constructed with the digits 0, 1, 2, 3, 4, 5, 6, 7, 8, 9 (a) 360 (b) 256 (c) 240 (d) 666
if each number starts with 35 and no digit appears 19. If he always wants to keep the first two novels of the
more than once? series (A, B) in his bookshelf, find the number of ways
(a) 1600 (b) 1680 (c) 900000 (d) 9000
in which he can arrange total 4 novels.
9. How many 6-digit numbers can be formed with the (a) 345 (b) 210 (c) 240 (d) 360
digits 1, 2, 4, 5, 6, 7 (no digit being repeated) which 20. If he never wants to keep the last two novels of the
are divisible by 5? series (F , G) in his bookshelf, find the number of
(a) 555 (b) 156
ways in which he can arrange 4 novels.
(c) 120 (d) none of these (a) 360 (b) 240 (c) 256 (d) 120
10. Find the sum of all the four digit numbers which are 21. He wants to keep the novels in his bookshelf such
formed by the digits 1, 2, 5, 6. that the first two novels of the series are always kept
(a) 933510 (b) 93324 (c) 65120 (d) 8400
together, find the number of ways in which he can
11. How many even numbers greater than 300 can be arrange all the novels.
formed with the digits 1, 2, 3, 4, 5, such that no digit (a) 120 (b) 2440
being repeated in any such number? (c) 720 (d) 1440
(a) 111 (b) 600
22. He wants to keep the novels in his bookshelf such
(c) 900 (d) none of these
that the last two novels of the series are never kept
12. How many numbers, each lying between 100 and together, find the number of ways in which he can
1000, can be formed with the digits 0, 2, 3, 4, 5, arrange all the novels.
such that no digit being repeated? (a) 3600 (b) 2500
(a) 24 (b) 48 (c) 72 (d) 96 (c) 1440 (d) 2880
Permutations & Combinations 1021

Directions (for Q. Nos. 23 to 40) : Answer these questions 36. In the different arrangements of the word RAINBOW,
independently of each other. how many words are there in which exactly two
23. Find the number of ways in which the letters of the vowels always remain together?
word RAINBOW can be arranged. (a) 2880 (b) 1440 (c) 3200 (d) 1600
(a) 5040 (b) 4050 (c) 3040 (d) 8040 37. If all the words formed by the letters of the word
24. In the different arrangements of the word RAINBOW, RAINBOW are arranged in a dictionary form, what is
how many words begin with R? the position of the word RAINBOW in that dictionary?
(a) 720 (b) 360 (c) 1440 (d) 480 (a) 3136 (b) 3631 (c) 3361 (d) 1363
25. In the different arrangements of the word RAINBOW, 38. In how many ways can the letters of the word
how many words begin with R and end with W ? STRANGE be arranged so that the vowels appear at
(a) 120 (b) 240 (c) 180 (d) 360 the odd places?
26. In the different arrangements of the word RAINBOW, (a) 720 (b) 1092 (c) 1440 (d) 1772
how many words are there in which R and W are at 39. In how many ways can the letters of the word
the end positions? MOBILE be arranged so that the consonants always
(a) 120 (b) 180 (c) 210 (d) 240 occupy the odd places?
27. In the different arrangements of the word RAINBOW, (a) 18 (b) 36
how many words are there in which first and last (c) 54 (d) 72
letters are vowels? 40. In how many ways can the letters of the word
(a) 360 (b) 720 (c) 1440 (d) 2880
MOBILE be arranged so that at least two consonants
28. In the different arrangements of the word RAINBOW, remain together?
how many words are there in which R and W are (a) 2880 (b) 3200
together? (c) 576 (d) 1600
(a) 720 (b) 360 (c) 540 (d) 1440
Directions (for Q. Nos. 41 to 47) : Answer these questions
29. In the different arrangements of the word RAINBOW, independently of each other.
how many words are there in which R and W are
41. 5 men and 4 women are to be seated for a dinner, in
never together?
(a) 3600 (b) 2400 (c) 1774 (d) 1440 a row, such that no two persons of the same sex sit
together. Find the number of ways in which this
30. In the different arrangements of the word RAINBOW,
arrangement can be done.
how many words are there in which vowels are never
(a) 2880 (b) 20
together?
(c) 86400 (d) 9 ! − 2 (5 ! × 4 !)
(a) 720 (b) 1440 (c) 360 (d) 3660
42. 5 men and 4 women are to be seated for a dinner, in
31. In the different arrangements of the word RAINBOW,
a row, such that no two men sit together. Find the
how many words are there in which A is always
number of ways in which this arrangement can be
before I and I is always before O ?
done.
(a) 840 (b) 420 (c) 720 (d) 630
(a) 1440 (b) 2880 (c) 1720 (d) 720
32. In the different arrangements of the word RAINBOW,
43. 5 men and 4 women are to be seated for a dinner, in
how many words are there in which vowels are
a row, such that no two women sit together. Find the
always before the consonants?
number of ways in which this arrangement can be
(a) 72 (b) 144 (c) 96 (d) 124
done.
33. In the different arrangements of the word RAINBOW, (a) 43200 (b) 2880
how many words are there in which no two (c) 7200 (d) 21600
consonants are together?
44. 5 men and 4 women are to be seated for a dinner, in
(a) 441 (b) 420 (c) 360 (d) 144
a row, such that all the women do not sit together.
34. In the different arrangements of the word RAINBOW, Find the number of ways in which this arrangement
how many words are there in which vowels may can be done.
occupy only even positions? (a) 345600 (b) 348800 (c) 347200 (d) 216006
(a) 567 (b) 144 (c) 576 (d) 625
45. 5 men and 4 women are to be seated for a dinner, in
35. In the different arrangements of the word RAINBOW, a row, such that women occupy the even places. Find
how many words are there in which vowels occupy
the number of ways in which this arrangement can
odd positions?
be done.
(a) 676 (b) 625 (c) 343 (d) 576
(a) 2880 (b) 1440 (c) 720 (d) 2020
1022 QUANTUM CAT
46. 6 men and 3 women are to be seated for a dinner, in 54. Find the number of ways in which 12 different books
a row, such that no two women sit together. Find the can be arranged on a shelf so that all the 3 particular
number of ways in which this arrangement can be books shall not be together.
done. (a) 126 × 10! (b) 1320 × 9!
(a) 121500 (b) 151200 (c) 112500 (d) 6 ! × 120 (c) 387892 (d) 39916800
47. 4 men and 4 women are to be seated for a dinner, in 55. Find the number of ways in which 12 different books
a row, such that men and women sit alternately. Find can be arranged on a shelf so that none of the
the number of ways in which this arrangement can 3 particular books shall be together.
be done. (a) 126 × 9! (b) 1320 × 9!
(a) 1152 (b) 1252 (c) 720 × 10! (d) 261273600
(c) 576 (d) 40320
56. There are 3 distinct books on Physics, 4 distinct
Directions (for Q. Nos. 48 to 52) : Answer these questions books on Chemistry and 5 distinct books on Biology.
independently of each other. In how many ways can these books be placed on a
48. In how many ways can 12 soldiers stand in a queue? shelf if the books on the same subject are to be
(a) 11! (b) 12! together?
(c) 12! - 1 (d) 1 (a) 103680 (b) 17280
12 !
49. In how many ways can 12 soldiers stand in two (c) (d) 60
3!
queues such that each queue has 6 soldiers?
(a) 12
P6 × 12
P6 × 2 ! (b) P6 × 6P6
12 Directions (for Q. Nos. 57 to 60) : Answer these questions
independently of each other.
(c) 6 ! × 6 ! × 2 ! (d) P6 × 6 ! × 2 !
12

57. How many different signals can be given using any


50. In how many ways can 12 soldiers stand in three
number of flags from 6 flags of different colours?
queues such that each queue has same number of
(a) 1240 (b) 1956
soldiers? (c) 3976 (d) 1976
(a) 12
P4 × 8P4 × 4P4 (b) P4 × 3 !
12
58. How many different signals can be given using
(c) P4 ×
12 12
P4 × P4 × 3 ! (d)
12
P4 × 8P4 × 4P4 × 3 !
12
3 coloured flag from 6 flags of different colours?
51. In how many ways can 12 soldiers stand in three (a) 120 (b) 60 (c) 18 (d) 20
queues such that one queue has 3 soldiers, another 59. 10 students took a test and each one got the distinct
queue has 4 soldiers and so the remaining row has score. In how many ways can three distinct prizes be
5 soldiers? given?
(a) 12P5 × 7P4 × 3P3 (b) 12P3 × 9P4 × 5P5 (a) 270 (b) 360
9
(c) P3 × P4 × 5P5 × 3 !
12
(d) P3 ×
12
P4 × 12P5 × 3 !
12
(c) 720 (d) 540

52. In how many ways can 12 soldiers stand in a queue 60. There are 10 stations on Konkan Railway Line that
such that no other soldier stands between three connects Mumbai and Mangalore at the two ends of
it. How many different kinds of tickets of second
particular soldiers?
class must be printed in order to make a passenger
(a) 21772800 (b) 22778800
buy a ticket so that he can travel from one station to
(c) 21278800 (d) 362880
any other station?
Directions (for Q. Nos. 53 to 55) : Answer these questions (a) 90 (b) 45
independently of each other. (c) 135 (d) 100
53. Find the number of ways in which 12 different books 61. In how many ways three different rings can be worn
can be arranged on a shelf so that 2 particular books in four fingers such that at most one ring in any
shall not be together. finger can be worn?
(a) 399168000 (b) 299168000 (a) 24 (b) 12
(c) 199168000 (d) 369088000 (c) 36 (d) 120
Permutations & Combinations 1023

19.3 Permutations of n Things


Not All Different
1. Number of permutations of n things taken all at a time n!
permutations of n things =
when p of them are all alike and the rest are all p! q ! r !
n!
different = 3. Number of permutations of n things taken all at a time
p! when p1 things are alike of one kind, p2 things are
2. Number of permutations of n things taken all at a time alike of second kind, p3 things are alike of third kind,
when p things are alike of one kind, q things are alike … pr things are alike of r th kind, and rest all, if any,
of other kind, r things are alike of another kind and rest are different from p1 , p2 , p3 , . . . , pr , then the number
all, if any, are different from p, q, r, then the number of n!
of permutations of n things =
p1 ! p2 ! p3 !… pr !

Introductory Exercise 19.2

1. In how many ways can the letters of the word 10. How many words can be made from the word
REPEAT be arranged ? MATHEMATICS in which vowels are together?
(a) 2240 (b) 232230 (c) 360 (d) 235760 (a) 12960 (b) 120960
2. In how many ways can the letters of the word (c) 15400 (d) none of (a), (b), (c)
TAMANNA be arranged? 11. How many words can be made from the word
(a) 120 (b) 420 IMPORTANT in which both T do not come together?
(c) 840 (d) none of these (a) 141120 (b) 112244
3. In how many ways can the letters of the word (c) 113113 (d) 888222
RECUPERATE be arranged? 12. How many words can be made from the word TING
(a) 234000 (b) 123000 (c) 232300 (d) 302400 TING TRING in which vowels occupy even positions?
(a) 336000 (b) 85360
4. In how many ways can the letters of the word
(c) 113600 (d) none of these
ASSASSINATION be arranged?
(a) 181800 (b) 818100 (c) 108108 (d) 10810800 13. If the different permutations of the word
PRODIGIOUS are listed as in a dictionary, then what
5. In the above question (number 4) find the number of
is the rank of the word PRODIGIOUS?
arrangements of four different letters of the word in a
(a) 200966 (b) 609902
row?
(c) 12500 (d) 24800
(a) 360 (b) 180
(c) 540 (d) none of these 14. If all the letters of the word SEQUESTERED be
arranged as in a dictionary, what is 50 th word?
6. How many arrangements can be made out of the
(a) DEEEEQURSTS (b) RUQDESTESEE
letters of the word COMMITTEE, taken all at a time,
(c) ESSTREEUQDE (d) DQUESTREEES
such that the four vowels do not come together?
(a) 216 (b) 45360 (c) 1260 (d) 43200 15. How many 7 digit numbers can be formed using the
digits 1, 2, 0, 2, 4, 2, 4?
7. How many arrangements can be made out of the
(a) 120 (b) 360 (c) 240 (d) 424
letters of the word COMMITTEE, taken all at a time,
such that none of the four vowels come together? 16. How many 6 digit numbers can be formed out of the
(a) 5400 (b) 360 (c) 2160 (d) 14400 digits of the number 113113?
(a) 15 (b) 180 (c) 888 (d) 222
8. In how many ways can the letters of the word
DUMDUMDIGADIGA be arranged? 17. How many 6 digit numbers can be formed out of the
number 567724, which are even?
(a) 180 (b) 90
(a) 360 (b) 480
(c) 270 (d) none of these
(c) 180 (d) 220
9. How many different words can be formed with the
letters of the word NAINITAL such that each of the 18. How many numbers greater than a million can be
word begin with L and end with T ? formed with the digits 5, 5, 2, 2, 1, 7, 0?
(a) 90 (b) 80 (a) 980 (b) 1080
(c) 88 (d) 82 (c) 920 (d) 1200
1024 QUANTUM CAT
19. How many different words can be formed with the 23. There are three copies of each of 4 different books.
letters of the word REGURGITATE so that the two T's In how many ways can they be arranged on a shelf?
are always together? (a) 369600 (b) 366900 (c) 396600 (d) 306609
(a) 453600 (b) 543600 (c) 183600 (d) 5040
24. There are 3 red, 4 green and 5 pink marbles in a
20. The number of ways in which the letters of the word
bag. They are drawn one by one and arranged in a
SUMPTUOS can be arranged so that the two U's do
row. Assuming that all the 12 marbles are drawn,
not come together is :
(a) 5040 (b) 7560 determine the number of different arrangements.
(c) 38920 (d) none of (a), (b), (c) (a) 22770 (b) 27720 (c) 22077 (d) 27270
21. In how many ways the letters of the word AFLATOON 25. There are 12 umbrellas placed in a row. Out of these,
be arranged if the consonants and vowels must 3 identical umbrellas are of red colour, 4 identical
occupy alternate places? umbrellas are of pink colour and 5 identical
(a) 144 (b) 143 (c) 288 (d) 248 umbrellas are of white colour. In how many ways can
22. In how many ways can the letters of the word these 12 umbrellas be arranged so that at least one
SOOTHSAYER be arranged so that the two S's and umbrella is separated from the umbrellas of the
two O's come together? same colour?
(a) 23400 (b) 302050 (c) 12340 (d) 40320 12 !
(a) 27714 (b) 27720 (c) × 3 ! (d) 37216
3 ! 4 !5 !

Permutations Where Repetitions are Allowed


1. Number of permutations of n different things taken 3. Number of permutations of n different things taken at
exactly r at a time, when each may be repeated any least r at a time, when each may be repeated any
number of times in each arrangement = n r number of times in each arrangement
2. Number of permutations of n different things taken at = nr + nr + 1 + nr + 2 + K + nn
most r at a time, when each may be repeated any
n r ( n ( n − r + 1) − 1)
number of times in each arrangement =
n ( n r − 1) n −1
= n + n 2 + n 3 + ... + n r =
n −1

Introductory Exercise 19.3

1. How many numbers of 5 digits can be formed with 6. How many natural numbers not exceeding 4321 can
the digits 0, 2, 3, 4 and 5 if the digits may repeat? be formed with the digits 1, 2, 3, 4 if repetition is
(a) 2500 (b) 250 (c) 120 (d) 2400 allowed?
(a) 123 (b) 113 (c) 222 (d) 313
2. How many numbers each lying between 9 and 1000
can be formed with the digits 0, 1, 2, 3, 7, 8 7. In how many ways 4 rings of different types can be
(numbers can be repeated) ? worn in 3 fingers?
(a) 30 (b) 120 (c) 210 (d) 90 (a) 49 (b) 12 (c) 24 (d) 81
8. In how many ways can 5 prizes be given away to
3. How many five-figures numbers can be formed with
4 boys, when each boy is eligible for all the prizes?
the digits 1, 2, 3, 4, 5 given that an even digit may
(a) 1024 (b) 20 (c) 625 (d) 540
not occupy an even place if the digits may be
9. In how many ways can n balls be randomly
repeated?
distributed in n cells?
(a) 1125 (b) 5210 (c) 1152 (d) 2120
(a) n ! (b) nn (c) n (n − 1) (d) 2 n
4. How many of the numbers from 1000 to 9999
10. In the above question, what will be the number of
(both inclusive) do not have four different digits? ways if it is specified that each cell is to be
(a) 4446 (b) 4664 (c) 4464 (d) 6444 occupied?
5. Find the number of three digit numbers (repetitions (a) n ! (b) nn (c) n3 (d) 2 n
allowed) such that atleast one of the digits is 9. 11. In how many ways can 5 letters be posted in 4 letter
(a) 252 (b) 648 boxes?
(c) 468 (d) 864 (a) 512 (b) 1024 (c) 625 (d) 20
Permutations & Combinations 1025

3. Number of circular permutations of n different things


19.4 Circular Permutations n
Pr
1. Number of circular permutations of n different things taken r at a time =
r
taken all at a time = ( n − 1)!
4. Number of circular permutations of n different things
2. Number of circular permutations of n different things
taken r at a time in one direction (either clock-wise or
taken all at a time in one direction (either clock-wise or n
( n − 1)! pr
anti-clock-wise) = anti-clock-wise) =
2 2r

Introductory Exercise 19.4

1. In how many ways can 6 boys form a ring? two arrangements. In a year how many days they
(a) 120 (b) 720 dine together?
(c) 119 (d) none of (a), (b), (c) (a) 180 (b) 504 (c) 720 (d) 360
2. In how many ways can 6 boys be arranged at a round 10. Find the number of ways in which 10 different
table so that 2 particular boys may be together? flowers can be strung to form a garland so that
(a) 24 (b) 48 (c) 360 (d) 60 3 particular flowers are always together.
3. In how many ways can 6 beads be strung into a (a) 30240 (b) 30420
necklace? (c) 23400 (d) None of (a), (b), (c)
(a) 60 (b) 360 (c) 720 (d) 120 11. 3 male and 3 female singers sat around a circular
4. In how many ways can 5 men and 2 ladies be arranged table. Alka is a female singer did not sat adjacent to
at a round table if two ladies are never together? a male singer and Sonu is a male singer did not sat
(a) 5040 (b) 480 (c) 240 (d) 720 adjacent to a female singer. In how many ways this
could be done?
5. 9 persons were invited for a business meeting by
(a) 4 (b) 6 (c) 9 (d) 10
Ambani, where the host be seated at a circular table.
How many different arrangements are possible if two 12. Three couples book a table for six people at a
invitees viz., Sahara and Mahindra be seated on restaurant that has only circular tables. If no
husband sits opposite his wife, in how many ways
either side of the host Ambani?
(a) 10080 (b) 10800 (c) 9200 (d) 4600 can three couples be arranged to sit around the
table?
6. In a G-20 meeting there were total 20 people (a) 64 (b) 117 (c) 96 (d) 56
representing their own country. All the representative
13. Twelve students are called for the group discussion
sat around a circular table. Find the number of ways
of which no two students have the same rank. They
in which we can arrange them around a circular table
sit in a circle for the group discussion in such a way
so that there is exactly one person between two
that the students sitting just next to either side of
representatives namely Manmohan and Musharraf.
any student should have either higher rank or lower
(a) 2 × (17 !) (b) 2 × (18 !)
rank. Find the number of ways in which they can be
(c) (3 !) × (18 !) (d) (17!)
arranged.
7. 6 men and 6 women are to be seated at a circular (a) 14400 (b) 28560 (c) 86400 (d) 72000
table such that no two women are adjacent, then the 14. In a single-occupancy co-ed hostel, consisting of
number of arrangements is : 20 rooms, exactly 10 guys and 10 girls are allowed to
1
(a) 86400 (b) 46800 (c) 12! (d) (12 !) stay in. Each one of them stays in a separate room.
2
The hostel is designed in a circular form such that the
8. There are 3 boys and 4 girls, seated around a door of a room opens opposite the door of another
circular table so that no two boys are together. Find room. In how many ways can the rooms be allocated to
the number of ways in which this can be done. them so that whenever a guy looks either side of his
(a) 36 (b) 120 (c) 144 (d) 132 door there is always a girl next door?
9. 7 sisters dine at a round table. They dine together (a) 2 ! × (10 !) 2 (b) 20 ! − 19 !
till each of them dine with different neighbours i.e., 19 !
(c) 9 ! × 10 ! (d) 2 ! ×
they do not like to dine with same neighbours in any 9 ! × 10 !
1026 QUANTUM CAT
7. If n is odd, the greatest value of n C r = n C m where
19.5 Combinations
( n − 1) ( n + 1)
A selection that can be formed by taking some or all of a m= or m =
2 2
finite set of things (or objects) is called a Combination. r +1 r+2
8. C r +
r
Cr + C r + ... + n C r = n + 1C r + 1 ; r ≤ n
Let A, B, C be three letters, then we can combine any two of
them in the following ways: 9. n
C 0 + n C1 + n C 2 + n C 3 + ... + n C n = 2 n
AB, BC, CA 10. n
C1 + n C 2 + n C 3 + ... + n C n = 2 n − 1
Similarly, if A, B, C, D are four letters, then we can combine 11. n
C 0 + n C 2 + n C 4 + ... = 2 n −1
any two of them in the following ways: 12. n
C1 + n C 3 + n C 5 + ... = 2 n − 1
AB , AC , AD, BC , BD, CD
Similarly, we can combine any three of A, B, C and D as Combination of n Different Things
following: ABC, ABD, ACD, BCD 1. Number of combination of n different things taken r at a
The number of combinations of n dissimilar things taken r time, when x particular things always occur
at a time is denoted by n C r or C ( n, r ) or ( nr ) = n − xC r − x
n! 2. Number of combination of n different things taken r at a
n
Cr = ; Whereas C denotes the combination and
r ! ( n − r )! time, when x particular things never occur = n − x C r
r≤n 3. Number of combination of n different things taken r at a
NOTE In the combination order of the letters (or things/people) does time, when x particular things always occur and
not matter. That means AB and BA are same. Similarly, ABC, ACB, BAC, y particular things never occur = n − x − y C r − x
BCA, CAB, CBA are same, as order does not matter.
4. Number of combination of n different things taken r at a
Essential Properties time, when x particular things are not together in any
selection = n − x C r − x
n
Pr
1. n C r =
r!
5. Number of ways of selections of zero or more things
2. n C o = n C n =1 from a group of n distinct things
3. n
C r = n C n − r (0 ≤ r ≤ n) = n C 0 + n C1 + n C 2 + n C 3 + ...+ n C n = 2 n
4. n
C x = n C y ⇔ x + y = n or x = y; ( x, y ∈W ) 6. Number of ways of selections of one or more things
n +1 from a group of n distinct things
5. n
C r −1 + C r =
n
Cr
6. If n is even, the greatest value of n
C r = n C m where = n C1 + n C 2 + n
C 3 + ... + n C n = 2 n − 1 (Q n C 0 = 1)
m = n/2

Introductory Exercise 19.5

1. Find the vlaue of 8 C3 . 6. If C (2 n, 3 ) : C (n, 2 ) = 12 : 1, find n.


8
(a) 56 (b) 8! (c) 65 (d) 3 (a) 3 (b) 5 (c) 6 (d) 4
10
2. Find the value of C5. 7. What is the least possible value of n if :
n −1 n −1
(a) 525 (b) 126 (c) 252 (d) 50 C3 + C4 > nC3
3. If 13
Cx = 13
Cy and x ≠ y, what is the value of x + y ? (a) 6 (b) 8 (c) 7 (d) 12
(a) 6 (b) 7 (c) 13 (d) 26 8. If Cx = 56 and Px = 336, find n and x.
n n

4. 17
Cr = 17
Cr + 3 . Find the value of r. (a) 7, 3 (b) 8, 4 (c) 8, 3 (d) 9, 6
(a) 17 (b) 6 (c) 7 (d) 13 9. What is the value of x when 11Cx is maximum ?
5. 7
P3 = n.7C3 , find the value of n. (a) 7 (b) 6
(c) 5 (d) both (b) and (c)
(a) 4 (b) 8 (c) 6 (d) 9
Permutations & Combinations 1027

12
10. What is the maximum value of Cx for any natural 22. In the previous question (no. 21) how many
number x ? committees can be formed if 3 particular members
(a) 6 (b) 7 (c) 8 (d) 4 must not be included?
14 (a) 6 (b) 8 (c) 10 (d) 61
11. For what value of x, Cx is maximum?
23. A committee of 3 experts is to be selected out of a
(a) 5 (b) 6 (c) 7 (d) 8
panel of 7 persons, three of them are engineers,
12. How many different committees of 5 members may three of them are managers and one is both engineer
be formed from 6 gentlemen and 4 ladies? and manager. In how many ways can the committee
(a) 181 (b) 357 (c) 603 (d) 252 be selected if it must have at least an engineer and a
13. In a test paper there are total 10 questions. In how manager?
many different ways can you choose 6 questions to (a) 33 (b) 22 (c) 11 (d) 66
answer? 24. A committee of 5 persons is to be formed out of
(a) 210 (b) 540 6 gents and 4 ladies. In how many ways this can be
(c) 336 (d) none of these done, when at most two ladies are included?
14. In the above question (no. 13) if the question (a) 186 (b) 168 (c) 136 (d) 169
number 1 is compulsory in how many ways can you 25. In an election, a voter may vote for any number of
choose to answer 6 questions in all? candidates not greater than the number to be
(a) 53 (b) 63 (c) 126 (d) 210 chosen. There are 7 candidates and 4 members are
15. Droupdi has 5 friends. In how many ways can she to be chosen. In how many ways can a person vote?
invite one or more of them to a dinner? (a) 89 (b) 98 (c) 79 (d) 101
(a) 31 (b) 55 (c) 13 (d) 25 26. At an election there are 5 candidates and 3 members
16. In how many ways can a committee of 6 members be are to be elected and a voter is entitled to vote for
formed from 7 men and 6 ladies consisting of 4 men any number to be elected. In how many ways a voter
and 2 ladies? can a vote?
(a) 252 (b) 525 (c) 625 (d) 256 (a) 25 (b) 53 (c) 35 (d) 15

17. A committee of 5 persons is to be formed from a 27. A question has two parts, part A and part B, each
group of 6 gentlemen and 4 ladies. In how many containing 8 questions. If the students has to choose
ways can this be done if the committee is to be 6 from part A and 5 questions from part B, in how
included atleast one lady? many ways can he choose the questions?
(a) 123 (b) 113 (a) 1268 (b) 1788 (c) 1024 (d) 1568
(c) 246 (d) 945 28. An examinee is required to answer six questions out
18. A committee of 7 persons is to be chosen from of twelve questions which are divided into two groups
13 persons of whom 6 are Americans and 7 are each containing six questions and he is not permitted
Indians. In how many ways can the selection be to answer more than four questions from any group.
made so as to retain a majority of Indians? In how many ways can he answer six questions?
(a) 945 (b) 1057 (a) 750 (b) 850 (c) 580 (d) 570
(c) 923 (d) 1056 29. An examination paper consists of 12 questions
19. In how many ways can a committee of 4 men and divided into two parts, part A and part B. Part A
3 women be appointed from 6 men and 8 women? contains 7 questions and part B contains 5 questions.
(a) 480 (b) 408 A candidate is required to attempt 8 questions,
(c) 420 (d) 840 selecting atleast 3 from each part. In how many ways
can he select the questions?
20. In the previous question (no. 19) what will be the
(a) 240 (b) 60 (c) 420 (d) 480
number of committees in which Miss A refuses to
serve if Mr. B is a member? 30. In MOCK CAT Quantitative Aptitude Section was
(a) 210 (b) 420 divided into 3 groups of 5, 5 and 4 questions
(c) 630 (d) none of these respectively. A candidate is required to answer
6 questions taking at least two questions from each
21. In how many ways 7 members forming a committee
of the first two groups and atleast one question from
out of 11 be selected so that 3 particular members
the third group. In how many ways can he make up
must be included? his choice?
(a) 60 (b) 130 (c) 80 (d) 70 (a) 1200 (b) 1400 (c) 1284 (d) 1560
1028 QUANTUM CAT
31. Mr. Daruwala has 18 acquaintances of whom 13 are 40. A cricket team of 11 players is to be formed from
relatives. In how many ways he may invite 10 guests 20 players including 6 bowlers and 3 wicket keepers.
so that 8 of whom are relatives? In how many different ways can a team be formed so
(a) 12870 (b) 22022 that the team contain exactly 2 wicket keepers and
(c) 20222 (d) 12780 atleast 4 bowlers?
32. If there are 11 players in a cricket team, all of whom (a) 22725 (b) 27225
shake hands with each other, how many shake hands (c) 22275 (d) none of (a), (b), (c)
take place in the team? 41. A team of 11 cricketers containing 1 wicket keeper,
(a) 36 (b) 55 2 bowlers, 3 all rounders and 5 batsmen, is to be
(c) 66 (d) none of these formed from a group of 25 cricketers containing
33. Amitabh has a list of 24 friends. He wishes to invite 2 wicket keepers, 8 bowlers, 5 all rounders and
some of them in such a manner that he can enjoy 10 batsmen?
maximum number of parties, but in each party the (a) 141120 (b) 111240
number of friends (i.e., invitees) be same and each (c) 101240 (d) none of (a), (b), (c)
party must have different set of persons. Then how 42. Out of 3 books on Economics, 4 books on Corporate
many parties can Amitabh enjoy? Strategy and 5 books on Philosophy, how many
(a) 2704156 (b) 357600 collection consists of exactly one book on each
(c) 235763 (d) 270156 subject?
34. An ice-cream parlour offers only family packs of (a) 40 (b) 36 (c) 60 (d) 120
ice-creams with 11 different flavours. If each 43. In the previous question (no. 42) what is the number
member of a family loves different flavours, then of different collections of books if atleast one book
maximum how many such families can purchase the on each subject is taken?
ice-cream if each family contains equal number of (a) 2365 (b) 3255 (c) 4224 (d) 1236
persons? 44. A box contains 7 red, 6 white and 4 blue balls. How
(a) 246 (b) 462 many selection of three balls can be made so that all
(c) 123 (d) 11C2 three are red balls?
35. In the previous question (no. 34) what is the (a) 35 (b) 70 (c) 42 (d) 17
maximum possible number of member in a family? 45. In the previous question (no. 44) if the red coloured
(a) 4 (b) 5 ball is not taken, then the number of selections is :
(c) 6 (d) 8 (a) 30 (b) 120
36. In a meeting everyone had shaken hands with (c) 60 (d) none of (a), (b), (c)
everyone else, it was found that 66 handshakes were 46. In the question (no. 44), if there is one ball of each
exchanged. How many members were present in the colour, then the number of selections is :
meeting? (a) 148 (b) 124 (c) 168 (d) 186
(a) 10 (b) 14 (c) 12 (d) 8
47. An urn contains 5 different red and 6 different green
37. In how many ways can a cricket team of eleven balls. In how many ways can 6 balls be selected so
players be chosen out of a batch of 16 players if a that there are atleast two balls of each colour?
particular player is always chosen? (a) 425 (b) 245 (c) 125 (d) 625
(a) 2002 (b) 3003 (c) 1003 (d) 7603
48. Find the number of different words that can be
38. In the previous question (no. 37) if a particular formed from 15 consonants and 5 vowels by taking
player is never chosen, then the number of ways in 2 consonants and 4 vowels in each word.
which a cricket team of eleven players can be chosen (a) 625 (b) 630 (c) 525 (d) 615
is :
(a) 2365 (b) 2359 (c) 1365 (d) 1056 49. Find the number of ways of selecting 4 letters from
the word EXAMINATION.
39. A cricket team of 11 players is to be formed from (a) 136 (b) 126 (c) 252 (d) 525
16 players including 4 bowlers and 2 wicket keepers.
In how many different ways can a team be formed so 50. How many words can be formed by using 4 letters at
that the team has atleast 3 bowlers and atleast one a time out of the letters of the word MATHEMATICS?
(a) 2445 (b) 2454
wicket keeper?
(a) 2472 (b) 2274 (c) 2427 (d) 1236 (c) 1243 (d) 1454
Permutations & Combinations 1029

51. In how many ways can 3 ladies and 3 gentlemen be parathas in its menu. The menu is designed in such
seated at a round table, so that any two and only two a way that nth pack contains n parathas. Also, no two
of ladies sit together? packs contain common parathas. However, you can
(a) 72 (b) 36 (c) 24 (d) 144 choose any r parathas from any pack (r ≤ n), but not
52. Eighteen guests have to be seated half on each side from different packs, for any particular order.
of a long table. Four particular guests desire to sit on The parathas can be ordered online only through
one particular side and three others on other side. home delivery service, and only one order in a day is
Determine the number of ways in which the sitting, accepted from a particular customer. Shivesh Raina
arrangements can be made. — my foodie friend— wants to order 3 different
(a) 18! (b) 462 × (9 !)2 parathas. Then, on a particular day, how many
(c) (9 !)2
(d) 7 ! × (9 !)2 different sets of parathas can he choose from?
(a) 5985 (b) 1140 (c) 5040 (d) 20 C1 × 206C3
53. Find the value of 10
C1 + C2 +
10 10
C3 + ... + 10
C10 .
(a) 100 (b) 10 2
(c) 2 10
(d) 210 − 1 Directions (for Q. Nos. 61 and 62) : Answer these questions
based on the following information.
54. Find the value of 10
C2 + C3 + ... +
10 10
C10 .
To win the Mathematics Olympiad you can choose any one
(a) 100 (b) 10 − 10 (c) 2
2 10
− 11 (d) 210 − 2 module from 40 different modules. The number of problems
55. Find the value of 10
C1 + C2 +
10 10
C3 + K + 10
C9 . in each module is same as the number of module, i.e., n th
(a) 100 (b) 10 − 10 (c) 10 − 11 (d) 2 (2 9 − 1)
2 2 module has n problems. The module 1 has the toughest
problem and so the module 40 has the easiest problems.
56. Find the value of 10
C0 + 10
C2 + 10
C4 + K .
61. Nilekani, an aspirant, attempts only 2 problems from
(a) ∞ (b) 2 9
(c) 2 10
−1 (d) 4 9 − 1
a particular module. The number of ways in which he
57. Find the value of [(10 C1 + 10
C3 + 10
C5 + ... ). can attempt 2 distinct problems is
(a) 10850 (b) 10660 (c) 50425 (d) 20 C1 × 206C3
× (20 C1 + 20C3 + 20C5 + ... ) × (30 C1 + 30
C3 + 30
C5 + ... )
× ....× (100 C1 + 100C3 + 100C5 + .... )] 62. Narainmurthi, another aspirant, attempts exactly
(a) 540 (b) 552 (c) 2540 (d) 2 55
−1 38 problems from a particular module. The number of
ways in which he can attempt 38 distinct problems is
58. Larry has a long weekend off for 4 days that starts from (a) 2460 (b) 1640 (c) 820 (d) 4038
Thursday and lasts till Sunday. She wants to attend
some spiritual enrichment sessions that are held every
Directions (for Q. Nos. 63 to 66) : Answer these questions
day for four hours. In how many ways can she attend based on the following information.
one or more sessions during this long weekend? This week Cinemax, a multi screen cinema, is showing 4 new
(a) 15 (b) 16 (c) 4 (d) 4 × 4C1 movies. Shahid Chopra and Priyanka Kapoor, the movie freak
couple, who never watch movies individually or separately, can
59. Madhulika is planning to join International School for
watch as many movies as possible if they get good reviews, else they
languages, which imparts the professional courses
may not go for any movie if the movies are not entertaining enough.
on numerous languages under various programs.
Program 1 offers 1 language, program 2 offers 63. Find the total number of ways in which they can
2 languages, program 3 offers 3 languages, and so watch any number of new movies this week,
on. No two programs offer any common language. irrespective of the order of the movie?
One can choose any number of languages from a (i) 4
C0 + 4C1 + 4C2 + 4C3 + 4C4
certain program, but all the languages must be (ii) 2 4 (iii) 42 (iv) 4
chosen from any one program only. Currently, there (a) Only (i) (b) Only (ii) and (iii)
are total 10 programs on offer with the program (c) Either (i) or (iv) (d) (i), (ii) and (iii)
10 offering exactly 10 languages. Madhulika is
willing to enroll for exactly 4 languages, find the 64. If his girlfriend Priyanka insists on watching at least
number of ways in which she can study any one new movie this week, irrespective of the order of
4 languages. the movie, then the number of ways in which they
(a) 462 (b) 210 (c) 40 (d) 10 C1 × 49C4 can watch the movies is
(i) 4
C1 + 4C2 + 4C3 + 4C4 (ii) 2 4 − 1
60. Paratha Inc. is a chain of food joints, which
specializes in various kinds of parathas − the Indian (iii) 3 (iv) 2 4 − 4C0
dish, usually stuffed with various kinds of vegetables (a) Only (iii) (b) Only (iv)
cooked and smashed. It has total 210 different (c) All, except (iii) (d) Only (i) and (ii)
1030 QUANTUM CAT
65. If his ex-girlfriend Kareena Gandhi also joins the 70. If he decides to read at least 50 scripts this year,
couple and now all of them want to watch at least then the number of ways in which he can read these
2 movies, irrespective of the order of the movie, then scripts, irrespective of their order, is
the number of ways in which they can watch the 1
(a) (2100 − 100C50 ) (b) 2 49
movies is 2
(i) 4 C2 + 4C3 + 4C4 (ii) 2 4 − 5 (c) 2100 − 251
1
(d) 2 99 + (100 C50 )
(iii) 2 (iv) 2 4 − 4C0 − 4C1 2
(a) Only (iii) 71. A coaching institute imparts training in 10 different
(b) Only (iv) subjects. It wants to develop various modules in such
(c) All, except (iii) a way that each module has equal number of
(d) Only (i) and (ii) subjects and no two modules have all the same
66. If Shahid and Priyanka realize, before they start subjects. Find the number of subjects in each
watching any movie, that they have to do a long due module when maximum number of modules can be
shopping and so they decide not to watch more than developed.
3 movies this week, then the number of ways, (a) 5 (b) 4 (c) 8 (d) 6
irrespective of the order of the movie, in which they 72. A sweet shop sells 9 varieties of sweets. The
can watch the movies is shopkeeper wants to pick not more than one piece of
(i) 4
C0 + 4C1 + 4C2 + 4C3 any variety of sweets to make the assortment of
(ii) 2 4 − 1 sweets. He then packs equal number of pieces in
(iii) 2 4 − 4C4 each box from variety of sweets in order to have
maximum number of distinct assortments. How
(iv) 6
(a) Only (iii) (b) Only (iv) many varieties of sweets should he pack in a box so
(c) Except (i) only (d) Only (i), (ii) and (iii) that each box contains distinct assortment of
sweets?
Directions (for Q. Nos. 67 to 70) : Answer these questions (a) 3 (b) 4
based on the following information. (c) 5 (d) both (b) and (c)
Amirbhai Moviewala, regarded for his perfection, is a very
fastidious actor as far selection of the scripts is concerned. Directions (for Q. Nos. 73 and 74) : Answer these questions
based on the following information.
This year he has been offered as many as 100 scripts to read
A team of 8 software engineers works on a project, for an
before he confirms to act in any movie which would be based
American client, in Bengaluru, India. Every team member
on these scripts only. Facing the problem of plenty and
wants to go to the client’s site in USA, as many times as it’s
hectic shooting schedule this year, he may not go through
possible for them. However, as per the company’s policy every
some or all of the scripts. Also, he is not looking for any other member will visit the site same number of times in a year. Each
assignment in whatsoever media or role-small screen, team member has to travel in groups of the same size each time
commercials, reality shows, cameo role etc. the team goes to client’s site. Not all members can be same in
67. Find the number of ways in which he can read these any two groups visiting the client’s site in a year.
scripts, irrespective of their order. 73. Maximum how many different groups can be formed?
(a) 100 (b) 101 (a) 4 (b) 16 (c) 70 (d) 32
(c) 2100 (d) 2100 − 1
74. Maximum how many times a team member can go
68. If he decides not to read more than 50 scripts, then to the client’s site in a year?
the number of ways in which he can read these (a) 32 (b) 35 (c) 49 (d) 7
scripts, irrespective of their order, is
Directions (for Q. Nos. 75 to 79) : Answer these questions
(a) 250 (b) 250 − 1
1 based on the following information.
(c) 2100 − 250 − 1 (d) (2100 + 100C50 ) Shashi Huzoor – a prolific writer – is very vocal and
2
straightforward in his views. He loves to opine through
69. If he decides to read at least 2 but not more than
twitter, a website that allows anybody to tweet (or publish
98 scripts, then the number of ways in which he can
digitally) a message containing not more than 140
read these scripts, irrespective of their order, is
characters including the number of blank spaces.
(a) 97 (b) 2 97
(c) 2 100
− 202 (d) 2100 − 2 98 − 2
Permutations & Combinations 1031

But, it’s not possible to tweet a blank message. Recently he got Combination of n Identical Things
mired in a nasty tweet. Since then he has a superstition that if 1. Number of ways of selections of r things from a group
he tweets a post that uses odd number of alphanumeric of n identical things = 1.
characters, from the set of 26 alphabets and 10 numerals, it 2. Number of ways of selections of at most r things from a
brings misfortune and a lot of trouble for him. So he prefers to group of n identical things = r + 1
use only even number of alphanumeric characters in all his 3. Number of ways of selections of at least r things from a
tweets. That is to write a post on twitter he may select either 2 group of n identical things = ( n − r ) + 1
characters or 4 characters or 6 characters, and so on, from the 4. Number of ways of selections of at least one thing from
set {a, b, c, ...., z, 0, 1, 2, ......, 9} to write a tweet. Repetition of a group of n identical things = n
any character does not matter to him; that is total number of 5. Number of ways of selections of zero or more things
characters in a tweet can be either odd or even. from a group of n identical things = n + 1
75. Find the number of ways in which he can select the Combination of n Things Not All Different
characters to tweet for his ardent followers.
Number of ways of selection of some or all the things from a
(a) 36 (b) 236 (c) 436 − 1 (d) 235 − 1
group of distinct subgroups:
76. Sunanda Khushker, his beautiful wife, loves his 1. Number of ways of selection of zero or more things out
everything but his superstition with the usage of odd of ( p + q + r + ... ) things, of which p are alike of one
number of characters. This invokes her to insist him kind, q are alike of second kind, r are alike of third
to write a tweet using nothing but an odd number of kind, and so on = [( p + 1) ( q + 1) ( r + 1)... ]
characters. Succumbed to his wife’s love and 2. Number of ways of selection of at least one thing out of
insistence he decides to tweet a post using odd ( p + q + r + ... ) things, of which p are alike of one kind,
number of characters only. Find the number of ways q are alike of second kind, r are alike of third kind, and
in which he can select the characters for tweeting. so on = [( p + 1) ( q + 1) ( r + 1) K ] − 1
(a) 235 + 1 (b) 236 (c) 418 − 1 (d) 235 3. Number of ways of selection of zero or more things
77. If he selects even number of characters from
from p identical things of one kind, q identical things
of second kind, r identical things of third kind, and
alphabets and even number of characters from
from remaining n things which are all different
numerals, separately, then the number of ways in
= [( p + 1) ( q + 1) ( r + 1)2 n ]
which he can select the characters for tweeting.
(a) 235 + 1 (b) 234 − 1 4. Number of ways of selection of at least one thing from
(c) 436 − 1 (d) 235 − 1 p identical things of one kind, q identical things of
second kind, r identical things of third kind, and from
78. Seeing his dilemma, one of his fans tweeted back remaining n things which are all different
that Mr. Shashi should not be bogged down and = [{( p + 1) ( q + 1) ( r + 1)2 n } − 1]
further advises him that if he just selects an odd
5. The number of ways of choosing k objects, from
number of characters from alphabets and odd
p objects of one kind, q objects of second kind, and so
number of characters from numerals, separately, no
on, is the coefficient of x k in the expansion
trouble will occur to him. Then the number of ways
in which he can select the characters for tweeting.
(1 + x + x 2 + K + x p ) (1 + x + x 2 + K + x q ) ...
(a) 235 + 1 (b) 234 6. The number of ways of choosing k objects from
(c) 235 − 2 (d) 235 − 1 p objects of one kind, q objects of second kind, and so
on, such that one object of each kind must be included
79. Mr. Shashi is also a globetrotter. Once on board he
is the coefficient of x k in the expansion
meets a numerologist, who tells him not to worry as
( x + x + .... + x ( x + x + K + x q ) K
2 p) 2
even if he selects an odd number of characters from
alphabets and an odd number of characters from
7. The number of ways in which r things can be selected
numerals, separately, but if he selects overall an
from a group of n things of which p are identical is
r
even number of characters from the set of 36
alphanumerical characters, all sorts of problems will
∑ n − p C r ; if r ≤ p
0
be vanished. Then the number of ways in which he 8. The number of ways in which r things can be selected
can select the characters for tweeting is from a group of n things of which p are identical is
(a) 235 + 1 (b) 235 − 1 r
(c) 2 − 2
35
(d) 234 ∑ n − p C r ; if r > p
r=p
1032 QUANTUM CAT
Introductory Exercise 19.6

Directions (for Q. Nos. 1 and 2) : Answer these questions 13. Find the number of ways of selecting one candle of
based on the following information. each colour.
A set contains 9 letters such that (a) 10 (b) 11 (c) 2 (d) 1
S = {A, A, B, B, B, C, C, C, C}. 14. Find the number of ways of selecting at least one
1. Find the number of ways of selection of 4 letters. candle of each colour.
(a) 7 (b) 11 (c) 15 (d) 18 (a) 0 (b) 72 (c) 2 (d) 10

2. Find the number of ways of selection of 4 letters 15. Find the number of ways of selecting 2 candles of
such that each of A, B, C must be there. each colour.
(a) 3 (b) 4 (c) 5 (d) 6 (a) 0 (b) 1 (c) 2 (d) 10

Directions (for Q. Nos. 3 to 10) : Answer these questions 16. Find the number of ways of selecting at least
based on the following information. 2 candles of each colour.
(a) 12 (b) 2 (c) 10 (d) 11
At DLF mall Gurugram, there is a store that sells premium
T-shirts. At this store there is a shelf that has exactly 10 red 17. Find the number of candles of selecting at most
tees (or T-shirts) of same colour, size and style; and I am 2 candles of each colour.
interested in buying tees from this shelf only. (a) 0 (b) 16 (c) 18 (d) 81

3. Find the number of ways of selecting 1 tee. 18. Find the number of ways of selecting 2 candles.
(a) 0 (b) 1 (c) 2 (d) 10 (a) 0 (b) 1 (c) 2 (d) 10

4. Find the number of ways of selecting at least 1 tee. 19. Find the number of ways of selecting 1 white, 2 red,
(a) 0 (b) 1 (c) 2 (d) 10 3 green and 2 yellow candles.
(a) 0 (b) 1 (c) 2 (d) 10
5. Find the number of ways of selecting at most 1 tee.
(a) 0 (b) 1 (c) 2 (d) 10 20. Find the number of ways of selecting all the candles
of each colour.
6. Find the number of ways of selecting 4 tees. (a) 0 (b) 1 (c) 2 (d) 10
(a) 0 (b) 1 (c) 2 (d) 10
21. Find the number of ways of selecting 4 candles.
7. Find the number of ways of selecting at least 4 tees. (a) 0 (b) 29 (c) 2 (d) 10
(a) 4 (b) 1
(c) 2 (d) none of these 22. Find the number of ways of selecting 6 candles.
(a) 0 (b) 36 (c) 40 (d) 72
8. Find the number of ways of selecting at most 4 tees.
(a) 5 (b) 1 (c) 4 (d) 10 23. Find the number of ways of selecting at least
6 candles.
9. Find the number of ways of selecting any number of (a) 0 (b) 140 (c) 72 (d) 120
tees.
(a) 0 (b) 1 (c) 2 (d) 11 24. Find the number of ways of selecting at most
6 candles.
10. Find the number of ways of selecting 10 tees. (a) 7 (b) 72 (c) 120 (d) 140
(a) 0 (b) 1 (c) 2 (d) 10
25. Find the number of ways of selecting 6 candles, such
Directions (for Q. Nos. 11 to 25) : Answer these questions that there must be at least one candle of each colour.
based on the following information. (a) 9 (b) 10 (c) 11 (d) 100
There are 2 white candles, 3 red candles, 4 green candles and
Directions (for Q. Nos. 26 to 34) : Answer these questions
3 yellow candles in a brand new packet. All the candles are
based on the following information.
uniform in shape and size, except their colour, as told above.
An electronic goods store has 3 boxes of pendrives. The
11. Find the number of ways of selecting any number of capacity of each pendrive is n GB, where n ∈ N .
candles. The first box has 3 identical pendrives, each of 4 GB storage
(a) 212 (b) 240 capacity; second box has 4 identical pen-drives, each of
(c) 72 (d) 13
2 GB storage capacity; and third box has 5 pendrives, each
12. Find the number of ways of selecting at least one of varying or distinct storage capacities other than 2 GB and
candle. 4 GB. All the pendrives are of the same make, colour and
(a) 222 − 1 (b) 239 (c) 72 (d) 12
design.
Permutations & Combinations 1033

26. Find the number of ways of selecting any number of 34. Find the number of ways of selecting any 2 identical
pen drives. pendrives.
(a) 120 (b) 600 (c) 520 (d) 640 (a) 0 (b) 1 (c) 2 (d) 4
27. Find the number of ways of selecting at least one 35. Find the number of ways of selecting any 2 distinct
pendrive. pendrives.
(a) 639 (b) 119 (c) 519 (d) 384 (a) 19 (b) 20 (c) 21 (d) 66
28. Find the number of ways of selecting at most Directions (for Q. Nos. 36 and 37) : Answer these questions
1 pendrive. based on the following information.
(a) 9 (b) 8 (c) 0 (d) 1 A shop has 10 mobile recharge coupons of Vodafone. Out of
29. Find the number of ways of selecting 1 pendrive from which 4 are identical of same value and remaining each
each box. coupon is of distinct value.
(a) 5 (b) 3 (c) 60 (d) 50 36. Find the number of ways of selecting 7 coupons.
30. Find the number of ways of selecting 3 pendrives. (a) 3 (b) 21 (c) 42 (d) 22
(a) 49 (b) 36 (c) 64 (d) 220 37. Find the number of ways of selecting 3 coupons.
31. Find the number of ways of selecting 3 distinct (a) 6 (b) 21 (c) 22 (d) 42
pendrives.
(a) 10 (b) 36 (c) 35 (d) 3 Combination of Contiguous Things
32. Find the number of ways of selecting 2 pen-drives of (A) Linear Combination
4 GB, 3 pendrives of 2 GB and 4 pen-drives of
I. Number of selections of k consecutive things out of n
different storage capacities other than 2 GB and
things in a row = ( n − k + 1)
4 GB.
(a) 5 (b) 10 II. Number of selection of k things out of n things in a row,
(c) 24 (d) 25 such that no two of the selected objects are next to each
33. Find the number of ways of selecting at most
other = n − k + 1C k
2 pendrives from the first box, at most 3 pendrives (B) Circular Combination
from the second box and at most 4 pendrives from
the third box.
Number of selections of k consecutive things out of n things
n, when k < n
(a) 372 (b) 72 in a circle = 
(c) 36 (d) 488 1, when k = n

Introductory Exercise 19.7


Directions (for Q. Nos. 1 to 3) : Answer these questions Directions (for Q. Nos. 4 to 13) : Answer these questions
based on the following information. independently of each other.
Last Diwali Chhota Bheem got 10 distinct coloured candles 4. In an amphitheater there are 10 distinguishable
placed in a row. Then he asked his friend to lit 3 candles. seats placed in a row. A family of five wants to sit
1. Find the number of ways of selection of candles so together, while rest of the attendees can sit anywhere
that all the 3 candles are consecutive. without any restriction. In how many ways can 5
(a) 19 (b) 20 (c) 8 (d) 66 seats be selected so that this family can sit together?
(a) 4 (b) 5 (c) 6 (d) 10 C5
2. Find the number of ways of selection of candles so
that all the 3 candles are not consecutive. 5. A rectangular field of 100 × 10 is plotted into 10
(a) 112 (b) 120 square plots of 10 × 10. Happy Housing is a first
(c) 132 (d) 66 customer who wants to buy 4 contiguous square plots
3. Find the number of ways of selection of candles so to develop a society for women who are victims of
that at least 2 candles are consecutive. domestic violence. In how many ways can he select 4
(a) 66 (b) 33 such plots that are all adjacent to each other?
(c) 17 (d) 64 (a) 7 (b) 6
(c) 5 (d) 40
1034 QUANTUM CAT
6. Anna Hazare – a social activist who became world 11. There are twelve security guards made to stand
famous after his crusade against corruption – wants around a circular football stadium. Any six guards,
to sit on a fast for 4 days back-to-back in the next who are adjacent to each other, need a replacement.
week. Sitting on fast is usually indicative of protest in In how many ways can these guards be selected for
a democratic system to let the government/authority replacement?
know the differing, rising and unheard opinion of its (a) 36 (b) 18 (c) 2 (d) 12
stakeholders. 12. Five friends were dining at a plush restaurant sitting
Find the number of selection of days in which he can at a round dining table. During their dinner a wine
sit on fast for 4 consecutive days. glass fell off the table and wine was, of course,
(a) 4 (b) 5 (c) 6 (d) 2 spilled on the two guys sitting next to each other.
7. Divya, who just got married, is allowed to work from Find the number of ways in which any two chairs can
home. The only restriction is that in every calendar be selected on which these two persons are sitting
year she has to work in the office for exactly 4 months next to each other, provided each chair is
but she cannot work in the office for any two distinguishable.
consecutive months. In a particular calendar year, in (a) 2 (b) 10 (c) 4 (d) 5
how many different ways can she choose to work from 13. A group of ten friends went to Lonavala in the late
her office? night. After reaching there all of them were almost
(a) 369 (b) 156 nonplussed to realize that there is a nip in the air.
(c) 126 (d) 123 Luckily there was a hut that had a bone-fire for sale.
8. At a very small railway station in Rajasthan, out of All of them paid money for tea and bone-fire and
eight terrorists four terrorists enter into four distinct then sat around the fire. But it wasn’t too late when
bogies of a passenger train that has total ten bogies, four adjacent chairs caught the fire, which went
so that they could place the bombs in the train. unnoticed until one of the guys felt the burning
However, no two terrorists enter in the contiguous sensation. Find the number of ways in which these
bogies that are directly connected to each other. In four chairs can be selected, provided each chair is
how many ways can these terrorists board the train? distinguishable.
(a) 58800 (b) 54200 (a) 2 (b) 10 (c) 4 (d) 5
(c) 14700 (d) 2450 Directions (for Q. Nos. 14 and 15) : Answer these questions
9. Four teams – A, B, C and D, are attending a meet. based on the following information.
They are made to sit in the 21 chairs placed beside All the triangles are formed from the 10-sided polygon by
one another in a row. The number of members in connecting all the vertices mutually.
these teams are 2, 3, 5 and 6, respectively. However,
14. Find the number of triangles so that at least one side
no two members of different teams sit in the
of the triangle coincides with the side of the polygon.
adjacent chairs and all the members of each team (a) 120 (b) 70 (c) 80 (d) 66
must sit together in the adjacent seats. In how many
ways can they sit in these chairs? 15. Find the number of triangles so that none of the
(a) 3036800 (b) 373248000 sides of the triangle coincides with the side of the
(c) 61036800 (d) 125646000 polygon.
(a) 20 (b) 80 (c) 64 (d) 50
10. On the very auspicious day of Karva Chauth, six
couples of my college decide to book the whole 16. Sunburn is one of the biggest New Year carnivals of
mini-theater, however they prefer to sit on the top Goa. The party happens inside the restricted circular
row only, which has twenty seats. They sit in such a premises, which has exactly 10 gates. The event
manner that the couple sits in any two adjacent managers want 3 entrance gates and 7 exit gates for
chairs but there must be at least one seat vacant the heck of safety and security concerns. However,
between any two couples. All the seats of top row are they want that none of the 3 entrances should be
fixed in such a way that there is no gap between any adjacent to each other. Each of the gates have a
two seats. Find the number of ways of sitting in the designated number, as in gate number 1, 2, ...., 10.
seats of top row. In how many ways can 3 gates be selected from the
(a) 4456200 (b) 519300 10 gates in order to have the entrance facility?
(c) 697480 (d) 3870720 (a) 150 (b) 50 (c) 60 (d) 160
Permutations & Combinations 1035

17. Out of the following six statements how many 18. Suchirupa is a fitness conscious woman. She joined
statements are INCORRECT? a club in her neighbourhood that offers seven fitness
(i) Number of ways of arranging 6 different red activities to choose from. However, on a particular
chairs and 4 different blue chairs in a row such day only one activity is supposed to be done by any
that the chairs of the same color are always of the members of this club. If Suchirupa does not
together = (6 ! × 4 !) × 2 ! do the same activity on any two consecutive days, in
(ii) Number of ways of arranging 6 different red how many ways can she choose to work out the next
chairs and 4 different blue chairs in a row such week?
that all the red chairs are together = 5 ! × 6 ! (a) 42 × 55
(iii) Number of ways of arranging 6 different red chairs (b) 7 × 63 × 53
and 4 different blue chairs in a row such that no (c) 7 × (6 )6
two blue chairs are together = 7C4 × 4 ! × 6 !
(d) 56
(iv) Number of ways of arranging 5 different red
chairs and 5 different blue chairs in a row such 19. Deepika has two best friends, Yuvraj and Siddhartha.
that no two adjacent chairs are of the same Everyday, she practices badminton with one of them,
color = 2 × (5 !)2 but she practices at least once a week with
(v) Number of ways of arranging 6 identical red Siddhartha. If she practices badminton with
chairs and 4 identical blue chairs in a row such Siddhartha more than once in a week she must
that no two blue chairs are together = 7C4 practice with him on consecutive days only. She
(vi) Number of ways of arranging 6 identical red practices it only once in a day. In how many ways
chairs and 4 different blue chairs in a row such can she practice the badminton with her friends?
that no two blue chairs are together = 4 ! × 7C4 (a) 206 (b) 28
(a) 0 (b) 1 (c) 2 (d) 4 (c) 49 (d) 48

19.6 Distribution/Division of Distinct Things Among


Individuals/Groups
1. Number of ways in which ( m + n) different things can 5. The number of ways in which mn different items can be
be divided into two unequal groups containing m and n divided equally into m groups, each containing
things, if the order of the group is not important n objects when the order of the group is not important
( m + n)!  ( mn)!  1
= m + nCm × nCn = =
m! n! m
 ( n!)  m!
2. Number of ways in which ( m + n) different things can 6. The number of ways in which mn different items can be
be divided into two unequal groups containing m and n divided equally into m groups, each containing
things, if the order of the group is important n objects when the order of the groups is important
( m + n)!  ( mn)! 
= × 2!
m! n! = m
 ( n!) 
3. The number of ways in which ( m + n + p) different
things can be divided into three different groups 7. The number of ways of dividing 2n different things into
containing m, n and p things respectively, if the order three groups of n each when the order of groups is not
(2n)! 1
of the group is not important important = ; the division by 2! indicates that
m+ n + p n+p ( m + n + p)! (n!) 2 2!
= Cm × Cn × C p =
p
m! n! p! since 2 groups have equal number of things, so the two
groups are indistinguishable.
4. The number of ways in which ( m + n + p) different
things can be divided into three different groups 8. The number of ways of dividing 3n different things into
three groups of n each when the order of groups is
containing m, n and p things respectively, if the order
( m + n + p)!  (2n)! 
of the group is important = × 3! important =  2
m ! n ! p!  ( n!) 
1036 QUANTUM CAT
9. The number of ways of dividing 3n different things into Exp. 2) Find the number of ways of distributing the
three groups of n each when the order of groups is not 4 distinct articles between 2 girls.
(3n)! 1 Solution Let a , b , c and d be the four distinct articles to be
important = ; the division by 3! indicates that
( n!) 3 3! distributed in two girls.
Then you have the following distribution.
since 3 groups have equal number of things, so the
three groups are indistinguishable. Girl 1 Girl 2
10. The number of ways of dividing 3n different things into i – a, b, c, d
three groups of n each when the order of groups is ii a b, c, d
 (3n)!  iii b a, c, d
important =  3 iv c a,b, d
 ( n!) 
v d a, b, c
11. The number of ways in which n distinct things can be vi a, b c, d
distributed to r different persons = r n vii a, c b, d
NOTE Usually the phrase ‘distribution’ indicates that the things viii a,d b, c
are given out (or doled out) to different people and so ix b, c a, d
the order becomes important. As, normally, people are
distinct so it becomes essential to distinguish that who receives what.
x b, d a, c
The phrase ‘division’ usually suggests that the different things are just xi c, d a, b
clubbed in different sets or groups. They may be kept in identical boxes xii a, b, c d
or their receivers are identical, and therefore they are naturally xiii a, b, d c
indistinguishable. That’s why no rearranging or ordering matters.
xiv a, c, d b
However, authors/paper-setters normally use both the
phrases interchangeably. So, it is your responsibility to figure it out xv b, c, d a
whether order is important or not; and that can be done on the basis of xvi a, b, c, d –
the situation and language of the problem.
Since each girl is distinct, so the number of ways in which the
Exp. 1) Find the number of ways of dividing 4 distinct 2 groups can be arranged is 2! ways.
articles between 2 groups. Therefore the number of ways of distribution of 4 distinct
Solution Let a, b, c and d be the four distinct articles to be articles between 2 distinct girls = 2! × 8 = 16
divided in two groups. Alternatively
Then you have the following groups. Case I (4, 0) Number of ways = 4C 4 × 0C 0 × 2 = 1 × 2 = 2

i – Case II (3, 1) Number of ways = 4C 3 × 1C1 = 4 × 1 × 2 = 8


a, b, c, d
Case III (2, 2) Number of ways
ii a b, c, d
1 1
iii b a, c, d = 4 C 2 × 2C 2 ×
× 2= 6×1× × 2= 6
2 2
iv c a, b, d Therefore, total number of ways in which 4 distinct articles
v d a, b, c can be divided in 2 groups =2 + 8 + 6 = 16.
vi a, b c, d Exp. 3) Find the number of ways of distributing
vii a, c b, d 4 distinct articles between 2 groups, such that one group
viii a, d b, c has 1 article and another one has 3 articles.
Solution Let a , b , c and d be the four distinct articles to be
Therefore, total number of ways in which 4 distinct articles divided in two groups, such that one group has 1 article and
can be divided in 2 groups is 8. another one has 3 articles. Then you have the following
groups.
Alternatively
Case I (4, 0) Number of ways = 4C 4 × 0C 0 = 1 × 1 = 1 i a b, c, d

Case II (3, 1) Number of ways = C 3 × C1 = 4 × 1 = 4


4 1 ii b a, c, d

Case III (2, 2) Number of ways iii c a, b, d


1 1 iv d a, b, c
= 4 C 2 × 2C 2 × = 6 × 1 × = 3
2 2 Therefore, total number of ways in which 4 distinct articles
Therefore, total number of ways in which 4 distinct articles can be divided in 2 groups such that one group has 1 article
can be divided in 2 groups =1 + 4 + 3 = 8. and another one has 3 articles is 4.
Permutations & Combinations 1037

Alternatively Number of ways in which 4 distinct articles the two groups become identical. Therefore, the actual
can be divided in 2 groups such that one group has 1 article number of ways of division of articles
and another one has 3 articles 1  4!  1
= ( 4 C 2 × 2C 2 ) × = × =3
= 4C1 × 3C 3 =
4!
=4 2!  2! 2!  2!
1! 3 !
Exp. 6) Find the number of ways of dividing 4 distinct
Exp. 4) Find the number of ways of dividing 4 distinct articles between 2 girls, equally.
articles between 2 girls, such that one girl gets 1 article and Solution Let a, b, c and d be the four distinct articles to be
another girl gets 3 articles. distributed in two girls, equally.
Solution Let a, b, c and d be the four distinct articles to be Then you have the following distribution.
distributed in two girls, such that one girl gets 1 article and
another girl gets 3 articles. Girl 1 Girl 2
Then you have the following distribution. i a, b c, d

Girl 1 Girl 2 ii a, c b, d
iii a, d b, c
i a b, c, d
ii b a, c, d iv b, c a, d
iii c a, b, d v b, d a, c
iv d a, b, c vi c, d a, b
v a, b, c d Since each girl is distinct, so the number of ways in which the
vi a, b, d c 2 groups can be arranged in 2! ways.
vii a, c, d b Therefore the number of ways of distribution of 4 distinct
articles equally among 2 distinct girls = 2! × 3 = 6
viii b, c, d a
Alternatively Number of ways in which 4 distinct articles
Since each girl is distinct, so the number of ways in which the can be divided equally in 2 groups
2 groups can be arranged in 2! ways. 4!
= 4 C 2 × 2C 2 = =6
Therefore the number of ways of distribution of 4 distinct 2! 2!
articles equally among 2 distinct girls = 2! × 4 = 8 But since both the 2 groups have equal number of articles, so
Alternatively: Number of ways in which 4 distinct articles the two groups become identical. Therefore, the actual
can be divided in 2 groups such that one group has 1 article number of ways of division of articles
and another one has 3 articles 1  4!  1
= ( 4 C 2 × 2C 2 ) × =   × =3
4! 2!  2! 2! 2!
= 4 C1 × 3C 3 = =4
1! 3 ! However, the 2 girls are distinct so the required number of
Now these 2 groups of articles can be arranged among distribution of articles
themselves in 2! ways. 1  4!  1
= ( 4 C 2 × 2C 2 ) × × 2! =   × × 2! = 6
Therefore the number of ways of distribution of 4 distinct 2!  2! 2! 2!
articles among 2 distinct girls = 2! × 4 = 8

Exp. 5) Find the number of ways of distributing 4 19.7 Distribution/Division of


distinct articles between 2 groups, equally.
Solution Let a, b, c and d be the four distinct articles to be Identical Objects Among
divided in two groups, equally. Individuals/Groups
Then you have the following groups.
1. The number of ways of dividing n identical items
i a, b, c, d among r persons (where, 0 ≤ r ≤ n), each one of whom
ii a, c b, d can receive any number (0, 1, 2,..., n) of items
iii a, d b, c = n + r − 1C r − 1
Therefore, total number of ways in which 4 distinct articles Example : Divide 16 identical balls among 3 kids, in
can be divided equally in 2 groups is 3. such a way that any kid can get any number of balls is
Alternatively Number of ways in which 4 distinct articles 16 + 3 − 1
C 3 − 1 = 18C 2
can be divided equally in 2 groups
4! 2. The number of ways of dividing n identical items
= 4 C 2 × 2C 2 = =6
2! 2! among r persons (where, 0 ≤ r ≤ n), each one of whom
But since both the groups have equal number of articles, so
must receive at least one item = n − 1C r −1
1038 QUANTUM CAT
Example : Divide 16 identical balls among 3 kids, in B. For a natural number n, if the given inequation is
such a way that each kid must get at least 1 ball is x 1 + x 2 + x 3 + . . . + x r ≤ n.
16−1
C 3−1 = 15
C2 Then, add a dummy variable x r + 1 in the given
equation, so that inequality can be converted into
3. The number of ways in which n identical items can be equality before finding the number of solutions.
divided into r groups so that no group contains less
5. If x i ≥ 0, the number of integral solutions to
than m items and more than k items (where m < k) is
x1 + x 2 + x 3 + ..+ x r ≤ n is same as the number of
coefficient of x n in the expansion of
integral solutions of
( x m + x m + 1 + K+ x k ) r , x1 + x 2 + x 3 + ... + x r + x r + 1 = n. Thus the required
for every m ≤ x i ≤ k number of integral solutions is n + r C r .
Example : Divide 16 identical balls among 3 kids, so Example : If x i ≥ 0, the given inequation
that each kid gets at least 2 balls but not more than x1 + x 2 + x 3 + ... ≤ n can be expressed as
7 balls is coefficient of x 16 in the expansion of x1 + x 2 + x 3 + x 4 = n
(x 2 + x 3 + x 4 + x 5 + x 6 + x 7 ) 3 so the required number of integral solutions is n+3
C3.
4. The number of ways in which n identical items can be C. For a natural number n, number of solutions of
divided into r groups such that the group x i contains | x 1 | + | x 2 | + . . . + | x m | = n.
minimum a i and maximum bi items (where a i ≤ bi ) is
6. The total number of integral solutions of | a| + | b| = n
coefficient of x n in is 4n
( x a1 + x a1 + 1 + ...+x b1 )( x a2 + x a2 + 1 + K + x b2 ) 7. The total number of integral solutions of
... ( x ar + x ar + 1 + K + x br ) | a| + | b| + | c | = n is 4n 2 + 2
Example : Divide 16 identical balls among 3 kids, 8. The total number of integral solutions of
such that first kid cannot have less than 3 and more  8n 
| a | + | b |+ | c | + | d | = n is   ( n 2 + 2)
than 5 balls, second kid cannot have less than 2 and 3
more than 8 balls, and the third kid cannot have less
D. For a natural number n, number of terms of
than 4 and more than 7 balls is coefficient of x 16 in
binomial and multinomial expressions.
(x 3 + x 4 + x 5 ) (x 2 + x 3 + x 4 + x 5 + x 6 + x 7 + x 8 )
9. Total number of terms in ( x + y) n = n +1
(x + x + x + x )
4 5 6 7
10. Total number of terms in
m+ n −1
( a1 + a 2 + K a n ) m = Cn − 1
19.8 Algebraic Properties 11. Total number of terms in (1 + x + x 2 + K x n ) m is
A. For a natural number n, the given equation is ( m. n) +1
x1 + x2 + x3 + K + xr = n xn + 1 − 1
NOTE 1+ x + x2 + K + xn =
1. If x i ≥ 0, the number of integral solutions is x −1
n + r −1
Cr − 1 Exp. 1) Suppose you have 4 identical chocolates in your
2. If x i ≥1, the number of integral solutions is pocket. In how many ways can you distribute the four
n −1 identical chocolates between the two kids (or children)?
Cr − 1
Solution In the following you can see that there are total
3. If m ≤ x i ≤ k , the number integral solutions is 5 ways of distributing the 4 chocolates between 2 kids.
coefficient of x n in the expansion of
(x m + x m + 1 + K + x k ) r Number of Chocolates

4. If a1 ≤ x1 ≤ b1 , a 2 ≤ x 2 ≤ b2 , ..., a r ≤ x r ≤ br the First Kid 4 3 2 1 0


number of integral solutions is coefficient of x n in Second Kid 0 1 2 3 4
( x a1 + x a1 + 1 + K + x b1 ) ( x a2 + x a2 + 1 + ... + x b2 ) ...
NOTE Here chocolates are identical and kids are distinct. So, we focus
( x a r + x a r + 1 + K + x br ) on kids, as they are distinguishable, but not on the chocolates.
Permutations & Combinations 1039

Alternatively Look at the other way of understanding the HHHHU


same problem. In this illustration there are 5 distinct HHHUH
possibilities of distributing the 4 chocolates between 2 kids. HHUHH
The number of chocolates on the left side of the bar (vertical HUHHH
line) is given to the first kid and the number of chocolates UHHHH
on the right side of the bar is given to the second kid. Thus, Thus you can see that there are total 5 shortest ways of
you can see that the number of ways of distributing the reaching C from A.
4 chocolates is same as the number of ways of arranging Mathematically, the answer is similar to arranging 5 objects
4 circles and 1 bar in a row. (steps) in a row such that 4 of them are identical. That
is 5!/ 4! = 5
So, essentially, what you are doing is nothing but
distributing the total 5 steps in 2 different directions. Thus
you can use the same logic in various kinds of problems.
Exp. 2) Find the number of ways in which you can
distribute all the 12 identical chocolates among the 3 kids.
Solution This problem is like dividing 12 circles by 2 bars
(vertical lines). Now, the number of ways of arranging
(12 + 2)! 14!
12 circles and 2 bars = = = 91
And, you already know how to arrange the 5 objects in a 12! × 2! 12! × 2!
row, which can be simply done in 5! ways. But since all the Therefore, the number of ways in which 12 identical
4 circles are identical, so the actual number of ways in chocolates can be divided among 3 kids is 91.
which 5 objects can be arranged is 5!/4! = 5 ways. Hint The equivalent algebraic equation would be a + b + c = 12, such
that a , b, c ≥ 0 .
NOTE As we need only one wall or partition to divide any physical
NOTE You must be wondering that since the two kids are distinct,
object into two parts, so we need only one bar to divide any number of
then why I have considered them identical. The reason is that the order
circles into 2 parts.
of kids does not matter at all, as the kids are equivalent to the bars and
Alternatively Now look at the other way of the role of bars is just to separate the circles. So whenever the order is
understanding the same problem. not important, things (or persons) may act as if they are identical.

Find the number of non-negative integral solutions of Exp. 3) Find the number of ways in which you can
a +b=4 distribute all the 12 identical chocolates among the 5 kids.
Therefore, we have (a, b) Solution As you have to distribute the chocolates among 5 kids
that means you need to have 4 bars to make 5 divisions among
= (4, 0), (3, 1), (2, 2), (1, 3), (0, 4). 12 circles.
Thus you can see that there are total 5 solutions to the above Thus the answer to this problem is same as finding the
equation. number of ways in which 16 objects (12 circles + 4 bars) can
be arranged in a row, out of which 12 objects (circles) are
Alternatively Now look at the other problem, which
identical and 4 other objects (bars) are also identical.
works with the same logic as the original problem does. That is the number of ways of arrangements of 12 circles and
Find the number of distinct shortest paths from A to C in the 12 + 4)! 16!
4 bars in a row = = = 1820
following diagram, which is a rectangular grid of 12! × 4! 12! × 4!
4 horizontal steps and 1 upward step. Therefore, the number of ways in which 12 identical
chocolates can be divided among 5 kids is 1820.
D C
Hint The equivalent algebraic equation would be
a + b + c + d + e = 12, such that a , b, c , d , e ≥ 0
U
Exp. 4) Find the number of ways in which you can
A B distribute all the 12 identical chocolates among the 3 kids,
such that each of them must get at least one chocolate.
In order to reach C from A you have to take any 4 horizontal Solution To simplify the solution what you need to do is
steps and any 1 upward step in anyone of the 5 following distribute 1 chocolate to each of the 3 kids. Since chocolates are
orders. identical, so it does not matter who gets which chocolate.
Now you have just 9 chocolates left to distribute among
1040 QUANTUM CAT
3 kids, in the second round. But the best part is that now you Exp. 5) Find the number of ways in which you can
are totally free to give any number of chocolates, out of
distribute all the 12 identical chocolates among the 3 kids,
9 chocolates, to any of the 3 kids. That means even if you give
0 chocolate to a particular kid, it won’t violate the basic such that each of them must get at least two chocolates.
restriction, as each kid has already received 1 chocolate in Solution To simplify the solution what you need to do is give
the first round. 2 chocolates to each of the 3 kids. Since chocolates are
Then the modified problem is like distributing 9 identical identical, so it does not matter who gets which chocolates.
chocolates among 3 kids, without any specific restriction. Now you have just 6 chocolates left to be distributed among
And, this is equivalent to arranging the 9 circles and 2 bars, 3 kids, in the second round.
such that all the 9 circles are identical and the 2 bars are But the best part is that now you are totally free to give any
identical. number of chocolates, out of 6 chocolates, to any of the
That is the number of ways of arrangements of 9 circles and 3 kids. That means even if you give 0 chocolate to a particular
( 9 + 2)! 11! kid, it won’t violate the basic restriction, as each kid has
2 bars = = = 55 already received 2 chocolates, in the first round.
9! × 2! 9! × ! 2!
Then the modified problem is like distributing 6 identical
Therefore, the number of ways in which 12 identical
chocolates among 3 kids, without any specific restriction.
chocolates can be divided among 3 kids, such that each kid
must get at least 1 chocolate is 55. And, this is equivalent to arranging the 6 circles and 2 bars,
such that all the 6 circles are identical and the 2 bars are
Hint The equivalent algebraic equation would be a + b + c = 12 , such that identical.
a , b, c ≥ 1. or a + b + c = 9 , such that a , b, c ≥ 0 That is the number of ways of arrangements of 6 circles and
NOTE Whenever you see the lower limits imposed on the problem, ( 6 + 2)! 8!
2 bars = = = 28
you must give away the minimum required number of items to the 6 ! × 2 ! 6! × 2 !
deserving persons in the first round, then remaining number of items Therefore, the number of ways in which 12 identical
can be given away (or distributed) without any restriction. So, our chocolates can be divided among 3 kids, such that each kid
objective is to deal with the restrictions, first. Then it becomes a very must get at least 2 chocolates is 28.
flexible problem to deal with.
Hint The equivalent algebraic equation would be a + b + c = 12 such that
AlternativelyThe given problem can be stated as a , b , c ≥ 2 or a + b + c = 6, such that a , b , c ≥ 0
a + b + c =12 ; a, b, c ≥1
NOTE Whenever you see the lower limits imposed in the problem,
Before the distribution of chocolates : you must give away the minimum required number of items to the
deserving persons in the first round, then remaining number of items
can be given away (or distributed) without any restriction. So, our
objective is to do away with (or get rid of) the restrictions, first. Then it
After distributing minimum 1 chocolate to each of the becomes a very flexible problem to deal with.
3 kids :
Alternatively The given problem can be stated as
a + b + c =12 ; a, b, c ≥ 2
Now, you can see that only 9 chocolates are remaining to be Before the distribution of chocolates:
distributed among the 3 kids.
And, the best thing is that you are totally free to give any
number of chocolates to anyone. As in, you can give all the
After distributing the minimum 2 chocolates to each of the
9 chocolates to one kid and no chocolates to the other 2 kids
3 kids:
or you can give equal number of chocolates to each of them.
And, that you can do in 55 ways, as explained below.
After ensuring that each kid must get 1 chocolate, the
distribution problem transforms to
a + b + c = 9; a, b, c ≥ 0 Now, you can see that only 6 chocolates are remaining to
And, the number of solutions to this equation be distributed among the 3 kids. And, the best thing is that
(9 + 2)! 11! you are totally free to give any number of chocolates to
= = = 55 anyone. As in, you can give all the 6 chocolates to one kid
9! × 2! 9! × 2!
and no chocolates to the other 2 kids or you can give equal
NOTE Once you have ensured that each kid is getting the minimum number of chocolates to each of them. And, that you can
requisite number of items, you are completely free to distribute the
items using the very basic logic of distribution. do in 28 ways, as explained on next page.
Permutations & Combinations 1041

After ensuring that each kid must get 1 chocolate, the Exm. 8) Find the number of ways in which you can
distribution problem transforms to distribute all the 12 identical chocolates among the 3 kids,
a + b + c = 6; a, b, c ≥ 0 such that each of them must get at least five chocolates.
And, the number of solutions to this equation Solution Total number of identical chocolates = 12
(6 + 2)! 8!
= = = 28 In the first round total number of chocolates to be given to
6! × 2! 6! × 2! the 3 kids = 3 × 5 = 15
Since there are fewer chocolates available than the required
NOTE Once you have ensured that each kid is getting the minimum
requisite number of items, you are completely free to distribute the
number of chocolates, so there is no way to distribute the
items using the very basic logic of distribution. given chocolates.
Thus the required number of ways = 0.
Exp. 6) Find the number of ways in which you can Hint The equivalent algebraic equation would be a + b + c = 12, such that
distribute all the 12 identical chocolates among the 3 kids, a , b , c ≥ 5.
such that each of them must get at least three chocolates.
Solution Total number of identical chocolates = 12 Exp. 9) Find the number of ways in which you can
In the first round total number of chocolates given to the distribute 12 identical chocolates among the 3 kids, such
3 kids = 3 × 3 = 9 that one kid gets at least 1 chocolate, another kid gets at
Number of chocolates remaining to be distributed without least 2 chocolates and the third kid gets at least
any restriction = 12 − 9= 3 3 chocolates.
Now this will be equivalent to arranging 5 objects (3 circles + Solution Total number of identical chocolates = 12
2 bars) in a row such that all the 3 circles are identical and all
In the first round total number of chocolates given to the
the 2 bars are also identical.
3 kids = 1+2+3 = 6
Thus the number of ways of arrangements of 3 circles and
( 3 + 2)! 5! Number of chocolates remaining to be distributed without
2 bars = = = 10 any restriction = 12 − 6= 6
3 ! × 2! 3 ! × 2!
Now, this will be equivalent to arranging 8 objects
Therefore, the number of ways in which 12 identical
chocolates can be divided among 3 kids, such that each kid (6 circles + 2 bars) in a row such that all the 6 circles are
must get at least 3 chocolates is 10. identical and all the 2 bars are also identical.
Hint The equivalent algebraic equation would be a + b + c = 12, such that
That is the number of ways of arrangements of 6 circles and
8!
a , b, c ≥ 3 or a + b + c = 3, such that a , b, c ≥ 0 2 bars = = 28.
6! × 2!
NOTE Whenever you see the lower limits imposed in the problem,
Therefore, the number of ways in which 12 identical
you must give away the minimum required number of items to the
deserving persons in the first round, then remaining number of items chocolates can be divided among 3 kids, such that one kid
can be given away (or distributed) without any restriction. So our gets at least 1 chocolate, another kid gets at least 2 chocolates
objective is to do away with (or get rid of) the restrictions, first. Then, it and the third kid gets at least 3 chocolates.
becomes a very flexible problem to deal with.
Hint The equivalent algebraic equation would be a + b + c = 12, such that
Exp. 7) Find the number of ways in which you can a ≥ 1, b ≥ 2 , c ≥ 3 or a + b + c = 6, such that a , b, c ≥ 0
distribute all the 12 identical chocolates among the 3 kids, NOTE Whenever you see the lower limits imposed in the problem,
such that each of them must get at least four chocolates. you must give away the minimum required number of items to the
deserving persons in the first round, then remaining number of items can
Solution Total number of identical chocolates = 12
be given away (or distributed) without any restriction. So our objective is to
In the first round total number of chocolates given to the do away with (or get rid of) the restrictions, first. Then it becomes a very
3 kids = 3 × 4 = 12 flexible problem to deal with.
Number of chocolates remaining to be distributed without
any restriction = 12 − 12 = 0 Exp. 10) Find the number of ways in which you can
Now there are no any chocolate left to be distributed. distribute 12 identical chocolates among 3 kids, such that
Thus, there is only 1 way to distribute the 12 identical there is one particular kid who gets the chocolates in the
chocolates among 3 kids such that each of them must get at multiples of 3 only.
least 4 chocolates. And that can be expressed as following.
Solution The equivalent algebraic equation would be
(4, 4, 4) 3 x + y + z = 12
For x = 0, we have y + z = 12 and the number of solutions
Hint The equivalent algebraic equation would be a + b + c = 12, such that 13 !
= = 13
a , b, c ≥ 4 . 12! × 1!
1042 QUANTUM CAT
For x = 1, we have y + z = 9 and the number of solutions Therefore total number of non-negative integral solutions
10! = 6 × 3 = 18
= = 10
9! × 1! {0, 0, 12} , {1, 1, 10} , {2, 2, 8} , {3, 3, 6} , {5, 5, 2} , {6, 6, 0}
For x = 2, we have y + z = 6 and the number of solutions {0, 12, 0} , {1, 10, 1} , {2, 8, 2} , {3, 6, 3} , {5, 2, 5} , {6, 0, 6}
7! {12, 0, 0} , {10, 1, 1} , {8, 2, 2} , {6, 3, 3} , {2, 5, 5} , {0, 6, 6}
= =7
6! × 1! So the number of ways in which 12 identical chocolates can
For x = 3, we have y + z = 3 and the number of solutions be distributed among 3 kids such that exactly 2 kids get the
4! same number of chocolates = 18.
= =4
3 ! × 1!
For x = 4, we have y + z = 0 and the number of solutions Exp. 13) Find the number of ways in which you can
1! distribute 12 chocolates among 3 kids, such that no two
= =1
0! × 1! kids get the equal number of chocolates.
Therefore, total number of solutions Solution Total number of ways in which 12 chocolates can be
(1 + 13) distributed among 3 kids, without any restriction
= 1 + 4 + 7 + 10 + 13 = × 3 = 35 (12 + 2)!
2 = = 91
12! × 2!
Exp. 11) Find the number of ways in which you can But, this distribution involves those cases too, in which 2 or
distribute 12 chocolates among 3 kids, such that there is all the 3 kids get same number of chocolates.
one particular kid who gets the chocolates in the multiples Case I : When all 3 kids get the same number of chocolates.
of 3 only and no kid gets less than 1 chocolate. x = y =z
Solution Since, x , y , z ≥ 1 and x gets chocolates in multiple of 3, ⇒ 1 way only
we must give 3 chocolates to x and 1 chocolate to each of y and Case II : When any 2 kids get same number of chocolates.
z. Now, we are free to give the remaining 7 chocolates without x + y + z = 12
any restriction. ⇒ 2x + z = 12
The equivalent algebraic equation would be 3 x + y + z = 7 ⇒ 18 ways
For x = 0, we have y + z = 7 and the number of solutions Case III: When none of the kids get same number of
8! chocolates.
= =8
7 ! × 1! x < y < z, y < z < x, z < x < y,
x < z < y, y < x < z, z < y < x
For x = 1, we have y + z = 4 and the number of solutions
5! Let α denotes the number of ways when each of the kids get
= =5 the same number of chocolates, β denotes the number of
4! × 1!
ways when exactly two kids get the same number of
For x = 2, we have y + z = 1 and the number of solutions chocolates and γ denotes the number of ways when none of
2! the kids get same number of chocolates. Therefore,
= =2
1! × 1! α + β + γ = 91
Therefore, total number of solutions = 2 + 5 + 8 = 15 ⇒ 1 + 18 + 6( k) = 91
⇒ 6k = 72 = γ
Exp. 12) Find the number of ways in which you can Therefore number of ways in which 12 identical chocolates
distribute 12 chocolates among 3 kids, such that exactly
can be distributed among 3 kids in such a way that no two
two kids get the same number of chocolates.
kids get same number of chocolates = 72
Solution Let the three kids get x, y and z number of chocolates.
Alternatively There are 4 possible cases when each of the
Since two kids get same number of chocolates, so we can
assume x = y , Then we have, three kids gets distinct number of chocolates.
x + y + z = 12 Case I: {0, 1, 11} , {0, 2, 10} , {0, 3, 9} , {0, 4, 8} , {0, 5, 7} → 5
⇒ 2x + z = 12 Case II: {1, 2, 9} , {1, 3, 8} , {1, 4, 7} , {1, 5, 6} → 4
12 − z Case III: {2, 3, 7} , {2, 4, 6} → 2
⇒ x=
2 Case IV: {3, 4, 5} → 1
⇒ {y , z} ≡ {0, 12}, {1, 10} , {2, 8} , {3, 6} , {5, 2} , {6, 0} Total 12 (=5+4+2+1) triplets. Each such triplet can be
⇒ {x , y , z} ≡ {0, 0, 12} , {1, 1, 10} , {2, 2, 8}, {3, 3, 6} , arranged in 3! = 6 ways. Therefore there are total 72 ways to
{5, 5, 2} , {6, 6, 0} express a + b + c = 12, such that a, b and c are distinct.
Thus there are 6 distinct pairs of {x, z}, such that x = y ≠ z Thus, the number of ways in which 12 identical chocolates
Now, each such triplet {x , y , z} can be arranged in 3 ways can be distributed among 3 kids such that no two kids get the
 3 !
=  . same number of chocolates = 72.
 2! 
Permutations & Combinations 1043

Exp. 14) Find the number of ways in which you can Solution odd + odd + odd = odd
distribute 12 chocolates among 3 kids, such that all the Since 12 is not an odd number, so we cannot distribute odd
number of chocolates to each of the three kids.
three kids get distinct number of chocolates.
The required number of ways = 0.
Solution It is the same problem as previous one, which is
stated in different words. Exp. 17) Find the number of ways in which you can
Hence, the answer is 72. distribute 12 chocolates among 3 kids, such that each kid
Exp. 15) Find the number of ways in which you can gets the even number of chocolates.
distribute 12 chocolates among 3 kids. – Archimedes, Solution Even + Even + Even = Even. Therefore it is possible to
distribute even number of chocolates, as shown below.
Bernoulli and Carl Gauss, such that Archimedes gets
{0, 0, 12} , {0, 2, 10}, {0, 4, 8} , {0, 6, 6} → 3+6+6+3 = 18 ways
more than Bernoulli and Bernoulli gets more than Carl
{2, 2, 8} , {2, 4, 6} → 3+6 = 9 ways
Gauss.
{4, 4, 4} → 1 way
Solution Total number of ways in which 12 chocolates can be Total number of ways in which this can be done
distributed among 3 kids, without any restriction
= 18 + 9 + 1 = 28 ways
(12 + 2)!
= = 91
12 ! × 2 ! Exp. 18) Find the number of ways in which you can
Now, we know that there are 13 different relations between distribute all the 12 chocolates among 3 kids, such that at
the number of chocolates that 3 persons can receive. least one of them must get more than 6 chocolates.
Case I: When all 3 kids get the same number of chocolates. Solution In order to ensure that at least one of them must get
a=b=c more than 6 chocolates you have to find the number of ways in
⇒ 1 way only which one or more than one kid gets more than 6 chocolates
Case II: When any 2 kids get same number of chocolates. using the inclusion-exclusion principle, as explained below.
a + b + c = 12 (i) Finding the number of ways in which any one kid gets
⇒ 2a + c = 12 more than 6 chocolates.
⇒ 18 ways So, first of all keep aside 7 chocolates and distribute the
Case III: When none of the kids get same number of remaining 5 chocolates among 3 kids.
chocolates. Now, the number of ways in which you can distribute 5
a > b > c, b > c > a, c > a > b, identical chocolates among 3 kids without restriction
(5 + 2)! 7!
a > c > b, b > a > c, c > b > a = = = 21
(5 ! × 2!) 5 ! × 2!
Let α denotes the number of ways when each of the kids get
the same number of chocolates, β denotes the number of Now, you can give away the 7 chocolates, which you had
ways when exactly two kids get the same number of kept aside, to any one of the 3 kids in 3 C1 = 3 ways.
chocolates and γ denotes the number of ways when none of Therefore, the number of ways in which you can
the kids get same number of chocolates. Therefore, distribute 12 identical chocolates among 3 kids such that
α + β + γ = 91 any one = of the 3 kids must get more than 6 chocolates
⇒ 1 + 18 + 6 ( k) = 91 = 3 × 21 = 63
⇒ 6k = 72 ⇒ k = 12 (ii) Finding the number of ways in which any two kids get
So, out of 6 relations, as suggested in Case III, only 1 relation more than 6 chocolates.
is in our favour (a > b > c). So, for that you have to keep aside 14 (=7+7) chocolates.
Therefore, the required number of ways in which 12 identical But, you have only 12 chocolates, so you cannot distribute
1
chocolates can be distributed among A, B and C = 72 × = 12 12 chocolates among 3 kids in such a way that at least
6 2 kids get more than 6 chocolates.
Alternatively If a + b + c = n; a > b > c ≥ 0, the number of That is, the number of ways in which you can distribute
n2 + 6  12 identical chocolates among 3 kids such that any 2 kids
solutions =  
 12  must get more than 6 chocolates = 0.
We have, a + b + c = 12; a > b (iii) Finding the number of ways in which all three kids get
more than 6 chocolates.
Therefore, the required number of ways
So, for that you have to keep aside 21 (=7+7+7) chocolates.
122 + 6  150  But, you have only 12 chocolates, so you cannot distribute
= =  = 125
.  = 12
 12   12  12 chocolates among 3 kids in such a way that all the
3 kids get more than 6 chocolates.
Exp. 16) Find the number of ways in which you can That is, the number of ways in which you can distribute
distribute 12 chocolates among 3 kids, such that each kid 12 identical chocolates among 3 kids such that all the
gets the odd number of chocolates. 3 kids must get more than 6 chocolates = 0.
1044 QUANTUM CAT
Now using Inclusion-Exclusion principle, you can find the Now using Inclusion-Exclusion principle, you can find the
number of ways in which you can distribute 12 identical number of ways in which you can distribute 12 identical
chocolates among 3 kids such that at least one of them must chocolates among 3 kids such that at least one of them
get more than 4 chocolates = 63 − 0 + 0 = 63. must get more than 4 chocolates = 108 − 18 + 0 = 90.
 (5 + 2)!  3
Hint 3
C 1  − C 2 ( 0) + C 3( 0) = 63
3
 ( 7 + 2)!  3  (2 + 2)!  3
 − C2   + C 3 ( 0) = 90
3
C 1
 5! × 2! 
Hint
 7! × 2!   2! × 2! 

Exp. 19) Find the number of ways in which you can Alternatively Total number of ways in which you can
distribute all the 12 chocolates among 3 kids, such that at distribute 12 identical chocolates among 3 kids without any
least one of them must get more than 4 chocolates. (12 + 2)! 14!
restriction = = = 91
Solution In order to ensure that at least one of them must get 12! × 2! 12! × 2!
more than 4 chocolates you have to find the number of ways in
which one or more than one kid gets more than 4 chocolates And, the number of ways in which you can distribute not
using the inclusion-exclusion principle, as explained below. more than 4 chocolates to any of the 3 kids = 1
(i) Finding the number of ways in which any one kid gets
more than 4 chocolates.
Therefore, the number of ways in which you can distribute
So, first of all keep aside 5 chocolates and distribute the
12 identical chocolates among 3 kids such that at least one
remaining 7 chocolates among 3 kids. of them must get more than 4 chocolates = 91 − 1 = 90.
Now, the number of ways in which you can distribute Hint There is only one way in which none of the 3 kids gets more that
7 identical chocolates among 3 kids without restriction 4 chocolates, which is (4, 4, 4). In all other 90 combinations at least one kid
(7 + 2)! 9! must get more than 4 chocolates.
= = = 36
7 ! × 2! 7 ! × 2 !
Exp. 20) Find the number of ways in which you can
Now, you can give away the 5 chocolates, which you had distribute all the 12 chocolates among 3 kids, such that at
kept aside, to any one of the 3 kids in 3 C1 = 3 ways.
least one of them must get more than 2 chocolates.
Therefore, the number of ways in which you can Solution In order to ensure that at least one of them must get
distribute 12 identical chocolates among 3 kids such that more than 2 chocolates you have to find the number of ways in
any one of the 3 kids must get more than 4 chocolates which one or more than one kid gets more than 2 chocolates
= 3 × 36 = 108 using the inclusion-exclusion principle, as explained below.
(ii) Finding the number of ways in which any two kids get (i) Finding the number of ways in which any one kid gets
more than 4 chocolates. more than 2 chocolates.
So, first of all keep aside 10 (=5+5) chocolates and So, first of all keep aside 3 chocolates and distribute the
distribute the remaining 2 chocolates among 3 kids. remaining 9 chocolates among 3 kids.
Now, the number of ways in which you can distribute Now, the number of ways in which you can distribute
2 identical chocolates among 3 kids without restriction 9 identical chocolates among 3 kids without restriction
( 2 + 2)! 4! ( 9 + 2)!
= = =6 = =
11!
= 55
2! × 2! 2! × 2! 91 × 2! 9! × 2!
Now you have 2 identical packets of chocolates with you, Now, you can give away the 3 chocolates, which you had
such that each packet has 5 chocolates in it. kept aside, to any one of the 3 kids in 3 C1 = 3 ways.
Now, you can give away the 2 identical packets of
chocolates, which you had kept aside, to any 2 of the Therefore, the number of ways in which you can
3 kids in 3 C 2 = 3 ways. distribute 12 identical chocolates among 3 kids such that
any one of the 3 kids must get more than 2 chocolates
Therefore, the number of ways in which you can = 3 × 55 = 165.
distribute 12 identical chocolates among 3 kids such that
(ii) Finding the number of ways in which any two kids get
any two of the 3 kids must get more than 4 chocolates
more than 2 chocolates.
= 3 × 6 = 18.
So, first of all keep aside 6 (=3+3) chocolates and
(iii) Finding the number of ways in which all three kids get
distribute the remaining 6 chocolates among 3 kids.
more than 4 chocolates.
So, for that you have to keep aside 15 (=5+5+5) chocolates. Now, the number of ways in which you can distribute
But, you have only 12 chocolates, so you cannot distribute 6 identical chocolates among 3 kids without restriction
12 chocolates among 3 kids in such a way that all the ( 6 + 2)! 8!
= = = 28
3 kids get more than 4 chocolates. That is, the number of 6! × 2! 6! × 2!
ways in which you can distribute 12 identical chocolates Now you have 2 identical packets of chocolates with you,
among 3 kids such that all the 3 kids must get more than such that each packet has 3 chocolates in it.
4 chocolates = 0.
Permutations & Combinations 1045

Now, you can give away the 2 packets of chocolates, Now you can give away the remaining 7 chocolates, which
which you had kept aside, to any 2 of the 3 kids in you had kept aside, to any one of the 3 kids in 3 ways.
3
C 2 = 3 ways. Therefore, the number of ways in which you can distribute
Therefore, the number of ways in which you can 12 identical chocolates among 3 kids such that any one of the
distribute 12 identical chocolates among 3 kids such that 3 kids must get more than 6 chocolates = 12 × 3 = 63.
any two of the 3 kids must get more than 2 chocolates Number of ways in which you can distribute 12 identical
= 3 × 28 = 84. chocolates among 3 kids such that any 2 kids get more than
6 chocolates = 0. Similarly, the number of ways in which you
(iii) Finding the number of ways in which all three kids get
can distribute 12 identical chocolates among 3 kids such that
more than 2 chocolates.
all 3 kids get more than 6 chocolates = 0.
So, for that you have to keep aside 9 (= 3+3+3) chocolates
and distribute the remaining 3 chocolates among 3 kids. Now using Inclusion-Exclusion principle, you can find the
number of ways in which you can distribute 12 identical
Now, the number of ways in which you can distribute
chocolates among 3 kids such that at least one of them must
3 identical chocolates among 3 kids without restriction
( 3 + 2)! 5! get more than 4 chocolates = 63 − 0 + 0 = 63 .
= = = 10 Thus, the number of ways in which you can distribute
3 ! × 2! 3 ! × 2!
12 identical chocolates among 3 kids such that none of the
Now you have 3 identical packets of chocolates with you,
3 kids get more than 6 chocolates = 91 − 63 = 28
such that each packet has 3 chocolates in it.
Now, you can give away the 3 packets of chocolates, Alternatively There are following 3 cases: when the
which you had kept aside, to the 3 kids in 3 C 3 = 1 way. highest value is 6, when the highest value is 5 and when the
Therefore, the number of ways in which you can highest value is 4.
distribute 12 identical chocolates among 3 kids such that {6, 6, 0} , {6, 5, 1} , {6, 4, 2} , {6, 3, 3}
at least two of the 3 kids must get more than 2 chocolates
= 1 × 10 = 10.
→ 3 + 6 + 6 + 3 = 18 ways
Now using Inclusion-Exclusion principle, you can find the {5, 5, 2}, {5, 4, 3} → 3+6 = 9 ways {4, 4, 4}→ 1 way
number of ways in which you can distribute 12 identical Therefore, we can distribute 12 chocolates among 3 kids
chocolates among 3 kids such that at least one of them
such that no kid gets more than 6 chocolates in 28 ways
must get more than 2 chocolates = 165 − 84 + 10 = 91
(=18+9+1).
 ( 9 + 2)!  3  ( 6 + 2)!  3  (3 + 2) ! 
 − C2   + C 3  = 91
3
Hint C1 
 9! × 2!   6! × 2!   3! × 2!  Exp. 22) Find the number of ways in which you can
distribute all the 12 chocolates among the 3 kids, such that
Alternatively Total number of ways in which you can
none of them gets less than 1 chocolate and more than
distribute 12 identical chocolates among 3 kids without any 6 chocolates.
(12 + 2)! 14!
restriction = = = 91 Solution The problem suggests that each kid has to get at least
12! × 2! 12! × 2! 1, but at most 6 chocolates.
And, the number of ways in which you can distribute not In order to ensure that no any kid gets more than
more than 2 chocolates to any of the 3 kids = 0 6 chocolates subtract the number of ways in which at least
one kid must get more than 6 chocolates from the total
Therefore, the number of ways in which you can distribute number of ways.
12 identical chocolates among 3 kids such that at least one How to calculate the total number of ways in which each kid
of them must get more than 4 chocolates = 91 − 0 = 91 gets at least 1 chocolate: In order to ensure that each kid must
get at least 1 chocolate keep aside 3 chocolates and distribute
Exp. 21) Find the number of ways in which you can the remaining 9 chocolates without any other constraint. The
distribute all the 12 chocolates among 3 kids, such that number of ways in which it can be done is
none of them gets more than 6 chocolates. ( 9 + 2)! 11!
= = 55
Solution In order to ensure that at least one kid must get more 9! × 2! 9! × 2!
than 6 chocolates, keep aside 7 chocolates and later on give it to However, the remaining 3 chocolates can be given in such a
any of the 3 kids. Now, you have just 5 chocolates to be way that each kid gets exactly 1 chocolate. This distribution
distributed among 3 kids, which you can distribute in any can be done in 1 way. Therefore, the number of ways in
manner because when you give 7 chocolates to any one of the which 12 chocolates can be distributed among 3 kids such
3 kids he will certainly have more than 6 chocolates. that each kid must get at least 1 chocolate = 55 × 1 = 55.
Therefore, the number of ways in which you can distribute How to calculate the total number of ways in which at least
5 identical chocolates, without any restriction one kid must get more than 6 chocolates and none of them
(5 + 2)! 7! gets less than 1 chocolate:
= = = 21
5 ! × 2! 5 ! × 2!
1046 QUANTUM CAT
(i) As you know that if you want to ensure that each kid Thus, the number of solutions of a + b + c =12, such that
must get at least 1 chocolate then you actually have only a, b, c ≥1 and one of a, b, c ≥ 6 is 55 − 30 = 25.
9 chocolates to be distributed.
Now you have to ensure that any one kid must get more
NOTE You cannot give more than 6 chocolates to any 2 or 3 kids.
than 6 chocolates, so keep aside 6 chocolates. Then you Alternatively There are following 3 cases: when the
have only 3 chocolates out of the 9 chocolates to be highest value is 6, when the highest value is 5 and when the
distributed among 3 kids.
highest value is 4.
Therefore, the number of ways in which 3 chocolates can be
( 3 + 2)! 5! {6,5,1},{6,4,2},{6,3,3}→ 6 + 6 + 3 = 15 ways
distributed among 3 kids = = = 10
3 ! × 2! 3 ! × 2! {5,5,2} ,{5,4,3} → 3 + 6 = 9 ways
Now, you can give away 3 chocolates, which you had {4, 4, 4} → 1 way
kept aside, to any one of the 3 kids in 3 C1 = 3 ways.
Therefore, we can distribute 12 chocolates among 3 kids
Therefore, the number of ways in which you can such that no kid gets less than 1 chocolate or more than
distribute 12 identical chocolates among 3 kids such that 6 chocolates in 25 ways (= 15 + 9 +1)
any one of the 3 kids must get more than 6 chocolates and
none of them gets less than 1 chocolate = 3 × 10 = 30 Exp. 23) Find the number of ways in which you can
(ii) Number of ways in which you can distribute 12 identical distribute at most 12 chocolates among 3 kids.
chocolates among 3 kids such that any 2 kids get more Solution First of all you must understand that the maximum
than 6 chocolates and none of them gets less than 1 number of chocolates that you can distribute is 12. It implies
chocolate = 0. that out of the 12 chocolates it is not necessary that you
(iii) Similarly, the number of ways in which you can distribute distribute all the chocolates. If you distribute all the chocolates
12 identical chocolates among 3 kids such that all 3 kids then the kids will receive the maximum number of
get more than 6 chocolates and none of them gets less 12 chocolates, but if you retain some chocolates with yourself,
than 1 chocolate = 0. then the kids will receive less than 12 chocolates.
It implies that greater the number of chocolates you retain
Now using Inclusion-Exclusion principle, you can find the
with yourself, lesser the number of chocolates you will
number of ways in which you can distribute 12 identical
distribute among the 3 kids. That means if you distribute
chocolates among 3 kids such that at least one of them
X chocolates among 3 kids and retain Y chocolates with
must get more than 1 chocolate = 30 − 0 + 0 = 30. yourself, then
Thus, the number of ways in which you can distribute X + Y = 12
12 identical chocolates among 3 kids such that none of Now, let the number of chocolates received by first kid,
them gets less than 1 chocolate and more than 6 second kid and third kid be a , b and c, the appropriate
chocolates = 55 − 30 = 25 relation would be
AlternativelyLet A, B and C be three kids. Then, a + b + c ≤ 12 or a + b + c + Y = 12
a + b + c =12, such that a, b, c ≥1 Now, the above equation is equivalent to the problem of
distributing 12 identical chocolates among 4 kids. This, in
⇒ a + b + c = 9, such that a, b, c ≥ 0 turn, is equivalent to the problem of arranging the 15 objects
The number of solutions to the above equation (12 circles + 3 bars) in a row, out of which 12 circles are
identical and 3 bars are also identical. That is the number of
(9 + 2)! 11!
= = = 55 ways of arrangements of 12 circles and 3 bars =
15 !
= 455
9!× 2! 9!× 2! 12!× 3 !
Now, a + b + c =12, such that a, b, c ≥1 Therefore, the number of ways in which at most 12 identical
⇒ a + b + c = 9, such that a, b, c ≥ 0 chocolates can be divided among 3 kids is 455.
Alternatively This solution is indeed not the smartest
⇒ a + b + c = 3, such that a, b, c ≥ 0 and any one of one, but it gives you a glimpse of what you can do to
a, b, c ≥ 6 understand it how it works actually when you are not able to
The number of solution to the above equation solve the problem smartly and quickly.
Case I: a + b + c = 12 The number of solutions
 (3 + 2)! 
= 3C 1   =
14!
= 14C 2
 3!× 2!  12!× 2!
 5!  Case II: a + b + c = 11 The number of solutions
=3×   = 30 13 !
 3!× 2 !  = = 13C 2
11! × 2!
Permutations & Combinations 1047

Case III: a + b + c = 10 The number of solutions This, in turn, is equivalent to the problem of arranging the 8
12! objects (5 identical circles + 3 identical bars) in a row.
= = 12C 2
10!× 2! That is the number of ways of arrangements of 5 circles and
Case XII: a + b + c = 10 The number of solutions 8!
3 bars = = 56
3! 5 !× 3 !
= = 3C 2
1!× 2! Therefore, the number of ways in which at most 5 identical
Case XIII: a + b + c = 0 The number of solutions chocolates can be distributed among 3 kids is 56.
2! Therefore, the number of ways in which you can distribute
= = 2C 2
0!× 2! not less than 6 but not more than 12 identical chocolates
Therefore, total number of solutions among 3 kids = number of ways in which you can distribute
= 2C 2 + 3C 2 + ... + 12C 2 + 13C 2 + 14C 2 at most 12 identical chocolates among 3 kids− number of
ways in which you can distribute at most 5 identical
15 !
= 14 + 1C 2 + 1 = 15C 3 = = 455 chocolates among 3 kids = 455 − 56 = 399
12!× 3 !
Hint Number of non negative integral solutions of 6 ≤ a + b + c ≤ 12 =
r +1 r +2 n +1
Hint r
Cr + Cr + C r + K + nC r = Cr + 1; r ≤ n. Otherwise, Number of non negative integral solutions of a + b + c ≤ 12 − Number of
2! 3! 4! 14 ! non negative integral solutions of a + b + c ≤ 5
+ + + ... + = 1 + 3 + 6 + K + 91
0 ! × 2 ! 1! × 2 ! 2 ! × 2 ! 12 ! × 2 ! = ( 2C 2 + 3C 2 + .... + 14
C 2) − ( 2C 2 + 3C 2 + K + 7C 2)
13 × 14 × 15
= = 455 = ( 15C 3) − ( 8 C 3) = 455 − 56 = 399
6
n( n + 1) n( n + 1)( n + 2) Alternatively This solution is indeed not the smartest
Hint Since 1 + 3 + 6 + ... + =
2 6 one, but it gives you a glimpse of what you can do in exam
when you are not able to solve the problem smartly and
Exp. 24) Find the number of ways in which you can quickly.
distribute at least 6 and at most 12 chocolates among
3 kids. Case I : a + b + c =12. The number of solutions
Solution First of all you must understand that the number of 14!
= = 14C 2
chocolates that you can distribute is maximum 12 and 12! × 2!
minimum 6. That means if you distribute X chocolates among
the 3 kids and retain Y chocolates with yourself, then
Case II : a + b + c =11. The number of solutions
X + Y = 12 13!
= = 13C 2
Now, let the number of chocolates received by first kid, 11! × 2!
second kid and third kid be a , b and c, then the appropriate Case III : a + b + c =10. The number of solutions
relation is
12!
a + b + c + Y = 12 = = 12C 2
Now, the above equation is equivalent to the problem of 10! × 2!
distributing 12 identical chocolates among 4 kids. This, in Case IV : a + b + c = 9. The number of solutions
turn, is equivalent to the problem of arranging 15 objects
11!
(12 circles + 3 bars) in a row, out of which 12 circles are = = 11C 2
identical and 3 bars are also identical. 9! × 2!
That is the number of ways of arrangements of 12 circles and Case V : a + b + c = 8. The number of solutions
15 !
3 bars = = 455 10!
12!× 3 ! = = 10C 2
Therefore, the number of ways in which at most 12 identical 8!× 2!
chocolates can be divided among 3 kids is 455. Case VI : a + b + c = 7. The number of solutions
Now, assume that instead of 12 chocolates you have just 9!
= = 9C 2
5 chocolates. And, you can distribute at most 5 chocolates to 7!× 2!
the 3 kids. That means if you distribute P chocolates among
the 3 kids and retain Q chocolates with yourself, then
Case VII : a + b + c = 6. The number of solutions
P + Q =5 8!
= = 8C 2
Now, let the number of chocolates received by first kid, 6!× 2!
second kid and third kid be a , b and c, then the appropriate Therefore, the required number of solutions
relation is, a + b + c + Q = 5
Now, the above equation is equivalent to the problem of = ( 8 C 2 + 9C 2 + K + 14
C2 )
distributing 5 identical chocolates among 4 kids. = 28 + 36 + 45 + 55 + 66 + 78 + 91 = 399
1048 QUANTUM CAT
NOTE Now, you must have observed that the previous solution is Solution Even though the three dice are identical, still for the
more logical and this one is more mechanical (or manual). But, this heck of our convenience we call them A , B and C
solution is helping you to visualize that what the actual logic behind the
Then, 5 ≤ a + b + c ≤ 15, such that ( a , b , c) =1, 2, 3 , 4, 5 , 6
previous solution is. For smaller values alternative solution may appear to
be easy one, but for greater values the previous solution is faster. That is, 2 ≤ a + b + c ≤ 12, such that ( a , b , c) = 0, 1, 2, 3 , 4, 5 , 6
Now, we can find the required number of ways as shown
Exp. 25) You have 2 dice and you throw them below.
simultaneously. Find the number of ways in which the 4!
Case I : a + b + c = 2, Number of ways = =6
total sum you get in any throw is not less than 5 and not 2!× 2!
more than 10. 5!
Case II : a + b + c = 3, Number of ways = = 10
Solution Even though the two dice are identical, still for the 3 !× 2!
6!
heck of our convenience we call them A and B Case III : a + b + c = 4, Number of ways = = 15
That is, 5 ≤ a + b ≤ 10, such that, ( a , b) =1, 2, 3 , 4, 5 , 6 4!× 2
Then, 3 ≤ a + b ≤ 8, such that, ( a , b) = 0, 1, 2, 3 , 4, 5 , 6 7!
Case IV : a + b + c = 5, Number of ways = = 21
Now, we can find the required number of ways as shown 5 !× 2!
8!
below. Case V : a + b + c = 6, Number of ways = = 28
4! 6!× 2!
a + b = 3, Number of ways = =4 9!
3 !× 1! Case VI : a + b + c = 7, Number of ways = − 3 = 33
5! 7 !× 2!
a + b = 4, Number of ways = =5 10!
4!× 1! Case VII : a + b + c = 8, Number of ways = − 9 = 36
6! 8!× 2!
a + b = 5, Number of ways = =6 11!
5 !× 1! Case VIII : a + b + c = 9, Number of ways = − 18 = 37
7! 9!× 2!
a + b = 6, Number of ways = =7 12!
6!× 1! Case IX : a + b + c = 10, Number of ways= − 30 = 36
8! 10!× 2!
a + b = 7, Number of ways = −2 = 6 13 !
7 !× 1! Case X : a + b + c = 11, Number of ways = − 45 = 33
9! 11!× 2!
a + b = 8, Number of ways = −4 = 5 14!
8!× 1! Case XI : a + b + c = 12, number of ways = − 63 = 28
To get the above values you can use the following approach. 12!× 2!
Draw a 6 × 6 grid, as a die has only 6 numbers (1, 2, …, 6) on it. Thus the total number of ways
= 6 + 10 + 15 + 21 + 2( 28 + 33 + 36) + 37 = 283
a+b=6
6 7 8 9 10 11 12 Hint From Case VI to Case XI, you see that I have subtracted some values.
a+b=5 For example, in case VI, there are 3 ways/ arrangements (7, 0, 0), (0, 7, 0),
(0, 0, 7) which do not satisfy the given criteria, as you cannot use any value
5 11
a+b=4 greater than 6. That’s why they have been discarded from the maximum
possible number of ways/ solutions.
a+b=3 4 10
Similarly, in case VII, there are 9 solutions (8, 0, 0), (0, 8, 0), (0, 0, 8), (7, 1, 0),
(7, 0, 1), (1, 0, 7), (1, 7, 0), (0, 1, 7), (0, 7, 1) which do not satisfy, as you cannot
3 9
use any value greater than 6.
2 8 The same process is being followed for other cases too.
a+b=8
1 7
Exp. 27) Find the number of integral solutions of
a+b=7 |x| +|y| = 12.
0 1 2 3 4 5 6 Solution It can be solved simply, by putting the minimum and
Now, you can find the number of ways by looking at the maximum possible values.
number of intersections on corresponding diagonals. As, x y Number of Solutions
a + b = 3 has 4 intersections, so it has 4 solutions. Similarly,
a + b = 6 has 7 intersections, so it has 7 solutions. But, since 0 ± 12 2
6 is the highest number that a die has, so after it reaches 6 it ±1 ± 11 4
starts to decrease in the same manner as it increases before 6. ±2 ± 10 4
Thus the total number of ways = 4 + 5 + 6 + 7 + 6 + 5 = 33 …… …… ……
Exp. 26) You have 3 dice and you throw them ± 10 ±2 4
simultaneously. Find the number of ways in which the ± 11 ±1 4
total sum that you get in any throw is not less than 5 and ± 12 0 2
not more than 15.
Total number of solutions = 2 + 11( 4) + 2 = 48.
Permutations & Combinations 1049

Alternatively Since |± k | = k, it means each k accepts Case I : When none of the x, y and z is zero, then the
2 non-zero values. But since 0 is a neutral number in the number of solutions of
sense that it has no effect of positive or negative sign, so we x + y + z = 12 = 11C 2 = 55
have to consider it separately.
But, since each of | x|, | y| and | z| takes 2 non zero values −
Case I : When none of x and y is 0.
one positive and one negative, so we will have 8 different
x + y =12; x, y ≥1 ⇒ x + y =10; x, y ≥ 0 ways (2 × 2 × 2) to represent the 55 solutions.
Therefore, the number of solutions of the equation Therefore, the number of solutions of | x| + | y| + | z| =12 is
11! 8 × 55 = 440
x + y =10; x, y ≥ 0 is = 11
10!× 1! Case II : When exactly one of the x, y and z is zero, the
And, the number of solutions of the equation | x | + | y | =10; number of solutions of
x, y ≥ 0 is 11 × (2 × 2) = 44 x + y + z = 12 = 11C1 = 11
Case II : When one of x and y is 0 But, when x = 0, | y| + | z| will have 4 different ways (2 × 2) to
That is ( x, y) ≡≡ (0, 12), (0, − 12), (12, 0), ( −12, 0). So it has represent the 11 solutions
4 solutions. Thus, the total number of required solutions of Similarly, when y = 0, | x | + | y | will have 4 different ways to
| x | + | y | =10; x, y ≥ 0 is 44 + 4 = 48 represent the 11 solutions
NOTE There are 44 solutions which have no zeros and 4 solutions Similarly, when z = 0, | x| + | y | will have 4 different ways to
have exactly one zero. represent the 11 solutions
Alternatively Look at the following coordinate graph. Therefore, the number of solutions of
The graph is drawn such that − 12 ≤ x ≤ 12 and −12 ≤ y ≤ 12. z +|y| +||
|| z = 12 is 12 × 11 = 132
The number of solutions is equal to the number of Case III : When exactly 2 of x, y and z are zero.
intersections of the diagonals. As, there are 48 intersection Then the number of solutions of x + y + z =12 is 3.
points. So the number of solutions of | x | + | y | =12 is 48.
But, since there are two ways to express a non-zero value
(0, 12) ( + / −), so the total number of solutions of | x| + | y| + | z| =12
is 2 × 3 = 6
Thus, by adding the number of solutions of case I, II and III,
we get the total number of solutions, which is
(–12, 0) (12, 0) 440 + 132 + 6 = 578.
Exp. 30) How many 3 digit numbers are there in which
sum of all the three digits is 12?
Solution a + b + c = 12; a ≥ 1, b ≥ 0, c ≥ 0
(0, –12)
⇒ a + b + c = 11; a ≥ 1, b ≥ 0, c ≥ 0
Exp. 28) Find the integral solutions of|x| +|y|≤ 12. Therefore the required number of solutions
= 11 + 3 − 1C 3 − 1 = 13C 2 = 78
Solution The number of integral solutions of|x| + |y| ≤12 will
be the sum of the number of integral solutions of Alternatively: Total number of solutions of
(|x| + |y| = 0) + (|x| + |y| = 1) + (|x| + |y| = 2) + K + (|x| + |y| = 12) a + b + c = 12;
= 1 + 4 + 8 + K + 48 ( a ≥ 0, b ≥ 0, c ≥ 0) is 14 C 2 = 91
= 1 + 4 (1 + 2 + 3 + K + 12) = 313 Number of solutions of a + b + c = 12, when
a = 0, b ≥ 0, c ≥ 0 is 13 C1 = 13
Exp. 29) Find the integral solutions of|x| +|y +|z| = 12.
Therefore the required number of solutions
Solution The total number of integral solutions of a + b + c = 12, when
|x| + |y|+||
z = n is 4n2 + 2. So the number of integral solutions a ≥ 1, b ≥ 0, c ≥ 0 is 78 ( = 91 − 13).
of the given equation is 4(122 ) + 2 = 578
Alternatively a + b + c =12, such that a ≥ 1, b ≥ 0, c ≥ 0
Alternatively The given problem can be solved in the For a = 1, b + c = 11 ⇒ The number of solutions = 12
C1 =12
following way.
For a = 2, b + c = 10 ⇒ The number of solutions = 11
C1 =11
1050 QUANTUM CAT
For a = 3, b + c = 9 ⇒ The number of solutions= 10
C1 =10 Exp. 33) Find the total number of terms in the
……………… expansion of ( a + b + c) 2 .
Solution Look at the following expansions. These
For a = 12, b + c = 0 ⇒ The number of solutions = 1C1 =1
expansions show how the different terms are formed.
Therefore, he required answer is However, the coefficients of these terms may not be correct
78( =12 + 11 + 10 + K + 3 + 2 +1). as our emphasis is on finding the number of terms only.
+1 r +2 n +1 ( a + b) 2 = a 0b 2 + a1 b1 + a 2 b 0
NOTE Since, r C r + r Cr + C r + K + nC r = Cr + 1; r ≤ n,
Therefore 1C 1 + 2C 1 + 3C 1 + K + 12
C1 = 13
C 2 = 78 ( a + b) 3 = a 0b 3 + a1 b 2 + a 2 b1 + a 0b 3
( a + b) 4 = a 0b 4 + a1 b 3 + a 2 b 2 + a 3 b1 + a 4 b 0
Exp. 31) How many 3 digit number are there in which ( a + b + c) 2 = a 0b 0c2 + a 0b1 c1 + a 0b 2 c0 + a1 b 0c1 + a1 b1 c0 + a 2 b 0c2
sum of all the three digits is not more than 12?
Solution a + b + c ≤ 12, such that a ≥ 1, b ≥ 0, c ≥ 0. So looking at the various terms of the expansion of above
For a + b + c = 1, the number of solutions is C 2 = 1 2 polynomials we can conclude that the number of terms in any
polynomial expansion is like distributing n identical items
For a + b + c = 2, the number of solutions is 3 C 2 = 3
among m distinct persons. And thus we get the following
For a + b + c = 3, the number of solutions is 4 C 2 = 6
formula.
… … … m + n −1
( a1 + a2 + K + an) m = Cn − 1
For a + b + c = 12, the number of solutions is
13
C 2 = 78 Exp. 34) Find the total number of terms in (1 + x + x 2 ) 3 .
Therefore the required number
Solution (1 + x) 2 = 1 + x 2 + 2 x
= 1 + 3 + 6 + ... + 78
= 364 (1 + x) 3 = 1 + 3 x + 3 x 2 + x 3
Hint 1 + (1 + 2) + (1 + 2 + 3) + K + (1 + 2 + 3 + .... n) (1 + x) 4 = 1 + 4x + 6x 2 + 4x 3 + x 4
n( n + 1)( n + 2) 
=   (1 + x + x 2 ) 2 = 1 + 2x + 3 x 2 + 2x 3 + x 4
 6 
(1 + x + x 2 ) 3 = 1 + 3 x + 6x 2 + 7 x 3 + 6x 4 + 3 x5 + x 6
Exp. 32) Find the number of terms in ( x + y) n This shows that total number of terms in
n −1 n−2 2 (1 + x + x 2 + .... x n) m = (m.n) + 1
Solution ( x + y) = C 0( x ) + C1 ( x
n n n n
y) + C 2 ( x
n
y )
+ K + C n − 1 ( xy n − 1 ) + nC n( y n)
Exp. 35) What is the coefficient of a 2b 3 c 5 in ( x + y + z) 10 ?
(a) There are n + 1 terms in the expansion of ( x + y) n
10!
(b) In each term sum of the indices of x and y is equal ton. Solution By Multinomial Theorem, it is
2! 3 !5 !

Introductory Exercise 19.8


Directions (for Q. Nos. 1 to 12) : Answer these questions 3. Find the number of ways in which these CDs can be
based on the following information. divided into three groups of 6 CDs, 4CDs and 2 CDs.
A film library at FTII (Film and Television Institute of India) (a) 12680 (b) 18360
Pune has 12 distinct CDs on French cinema. 12 !
(c) 13860 (d)
1. Find the number of ways in which these CDs can be 6! + 4! + 2!
divided into two groups such that one group has 4. Find the number of ways in which these CDs can be
8 CDs and another one has 4 CDs. gifted to three French students who had recently
(a) 594 (b) 495 (c) 945 (d) 32
visited FTII from France such that one student has
2. Find the number of ways in which these CDs can be 6 CDs, second student has 4 CDs and the third one
gifted to two French students who had recently has 2 CDs.
visited FTII from France such that one student has (a) 31860 (b) 82160
8 CDs and another one has 4 CDs. (c) 13860 (d) 83160
(a) 990 (b) 900 (c) 945 (d) 1188
Permutations & Combinations 1051

5. Find the number of ways in which these CDs can be 15. Find the number of ways of distributing
divided equally into two groups. 12 chocolates among 4 kids such that each of them
(a) 1024 (b) 492 (c) 462 (d) 924 must receive at least two chocolates.
6. Find the number of ways in which these CDs can be (a) 35 (b) 1 (c) 8 (d) 65
distributed to two French students equally. 16. Find the number of ways of distributing
(a) 594 (b) 990 (c) 984 (d) 924 12 chocolates among 4 kids such that each of them
7. Find the number of ways in which these CDs can be must receive at least three chocolates.
divided equally into three groups. (a) 36 (b) 1 (c) 3 (d) 16
(a) 34560 (b) 34650 (c) 7575 (d) 5775 17. Find the number of ways of distributing
8. Find the number of ways in which these CDs can be 12 chocolates among 4 kids such that no kid
distributed among three French students equally. receives less than 2 chocolates and more than 4
(a) 34650 (b) 272100 chocolates.
(c) 43650 (d) 207900 (a) 38 (b) 19 (c) 16 (d) 32

9. Find the number of ways in which these CDs can be 18. Find the number of ways of distributing
distributed to 6 French students equally. 12 chocolates among 4 kids such that these kids get
12 ! 12 ! 1, 2, 4, and 5 chocolates.
(a) (b) × 6!
(2 !)6 (6 !)2 (a) 25 (b) 24 (c) 52 (d) 64
12 ! 19. Find the number of ways of distributing
(c) × 6! (d) 32000084
6!×2! 12 chocolates among 4 kids such that taller the kid
fewer the chocolates he gets. However, these kids get
10. Find the number of ways in which these CDs can be
1, 2, 4, and 5 chocolates as per their heights,
distributed to 6 French students such that anyone
assuming no two kids are of the same height.
may have any number of CDs.
(a) 25 (b) 24
12 !
(a) 612 (b) 12 6 (c) (d) (6 !)12 (c) 52 (d) 1
6!
20. Find the number of ways of distributing
11. Find the number of ways in which these CDs can be
12 chocolates among 4 kids equally.
distributed to 4 French students such that each one (a) 4 (b) 4 4
must have at least two CDs. 12 !
(c) (d) 1
(a) 3450650 (b) 7050120 4!
(c) 504090 (d) 6900520
Directions (for Q. No. 21 to 30) : Answer the following
12. Find the number of ways in which these CDs can be questions independently of each other.
distributed to 4 French students such that no one
21. Find the total number of non-negative integral
has more than three CDs.
12 ! 12 ! solutions of a + b + c = 30.
(a) (b) × 4! (a) 306 (b) 406 (c) 396 (d) 496
64 (3 !)4
12 ! 22. Find the total number of positive integral solutions of
(c) (d) 360960
1296 a + b + c = 30.
(a) 405 (b) 406 (c) 303 (d) 496
Directions (for Q. Nos. 13 to 20) : Answer the questions
based on the following information. Once Ruskin Bond meets 23. Find the total number of even positive integral
4 kids on his way to Dehradun. While chatting with them he solutions of a + b + c = 30.
decides to distribute all the 12 identical chocolates among (a) 128 (b) 72 (c) 91 (d) 104
these kids. 24. Find the total number of odd positive integral
solutions of a + b + c = 30.
13. Find the number of ways of distributing
(a) 30 (b) 42 (c) 1 (d) 0
12 chocolates among 4 kids such that each of them
can receive any number of chocolates. 25. Find the total number of positive integral solutions of
(a) 455 (b) 272 (c) 575 (d) 34650 a + b + c = 30 such that a , b and c are distinct.
(a) 366 (b) 256 (c) 178 (d) 398
14. Find the number of ways of distributing
12 chocolates among 4 kids such that each of them 26. Find the total number of non-negative integral
must receive at least one chocolate. solutions of a + b + c = 30 such that a < b < c.
(a) 365 (b) 1 (c) 8! (d) 165 (a) 63 (b) 85 (c) 105 (d) 75
1052 QUANTUM CAT
27. Find the total number of non-negative integral 40. Find the number of integral solutions of the
solutions of a + b + c = 30 such that a ≤ b. equation, such that (a , b, c, d ) ≤ 0.
(a) 231 (b) 225 (c) 256 (d) 132 (a) 256 (b) −256 (c) 425 (d) −425
28. Find the total number of positive integral solutions of Directions (for Q. Nos. 41 to 44) : Answer the questions
3 a + 2 b + c = 30. based on the following information. There are three integers
(a) 33 (b) 21 (c) 61 (d) 67 a, b, c such that a + b + c = p.
29. Find the total number of ways of selecting the 30 41. If p = 8 and 0 ≤ (a , b, c) ≤ 4, find the number of
roses from the truckload of red, pink and yellow
solutions of the given equation.
roses. (a) 15 (b) 24 (c) 12 (d) 30
(a) 496 (b) 30! (c) 303 (d) 330
42. If p = 8 and 0 ≤ (a , b, c) ≤ 3, find the number of
30. Find the total number of ways of selecting the
solutions of the given equation.
30 roses from truckload of red, pink and yellow
(a) 6 (b) 12
roses, such that in each selection there are roses of
(c) 3 (d) 21
all the three colours.
(a) 404 (b) 24 (c) 406 (d) 303 43. If p = 8 and 0 ≤ (a , b, c) ≤ 2, find the number of
Directions (for Q. Nos. 31 to 38) : Answer the questions solutions of the given equation.
based on the following information. There are four numbers (a) 7 (b) 6
a, b, c, d such that a + b + c + d =12. (c) 5 (d) none of these

31. Find the number of non-negative integral solutions of 44. If p = 8 and 0 ≤ (a , b, c) ≤ 1, find the number of
the equation. solutions of the given equation.
(a) 25 (b) 244 (c) 255 (d) 425 (a) 2 (b) 4
32. Find the number of positive integral solutions of the (c) 6 (d) 0
equation. Directions (for Q. Nos. 45 to 47) : Answer the questions
(a) 125 (b) 165 (c) 143 (d) 156 based on the following information.
33. Find the number of integral solutions of the There are four integers a , b , c, d such that a + b + c + d = p
equation, such that 2 ≤ a , b, c, d ≤ 4. 45. If p = 30 and 0 ≤ (a , b, c) ≤ 15, find the number of
(a) 19 (b) 24 (c) 52 (d) 34 solutions of the given equation.
34. Find the number of integral solutions of the (a) 2736 (b) 2456
equation, such that a , b, c, d ≥ 2. (c) 5226 (d) 3436
(a) 35 (b) 24 (c) 52 (d) 34
46. If p = 30 and 0 ≤ (a , b, c) ≤ 10, find the number of
35. Find the number of integral solutions of the solutions of the given equation.
equation, such that a > 0 , b > 1, c > 2 , d > 3. (a) 285 (b) 286
(a) 10 (b) 12 (c) 34 (d) 24 (c) 528 (d) 304
36. Find the number of integral solutions of the 47. If p = 30 and 0 ≤ (a , b, c) ≤ 8, find the number of
equation, such that a > − 4 , b > −3 , c > −2 , d > −1. solutions of the given equation.
(a) 1330 (b) 1331 (c) 1690 (d) 340 (a) 25 (b) 10 (c) 0 (d) 30
37. Find the number of integral solutions of the Directions (for Q. Nos. 48 to 59) : Answer the questions
equation, such that 1 ≤ (a , b) ≤ 4 , c = 6 , d = 0. based on the following information. Hirawala is a uber rich
(a) 18 (b) 9 (c) 5 (d) 4 guy who lives in a swanky place, called Hiranandini, in
38. Find the number of integral solutions of the Mumbai. He has 15 diamond rings and 5 daughters.
equation, such that −2 ≤ (a , b, c, d ) ≤ 19.
48. If the rings are distinct, find the number of ways of
(a) 1991 (b) 1771 (c) 2552 (d) 434
distributing these rings among his 5 daughters such that
Directions (for Q. No. 39 to 40) : Answer the questions no any two daughters get equal number of rings but
based on the following information. There are four numbers each daughter gets at least one ring and at most 5 rings.
a, b, c, d such that a + b + c + d = −12. 15 !
(a) 15C5 (b) ×5
39. Find the number of integral solutions of the 1! 2 ! 3 ! 4 ! 5 !
equation, such that (a , b, c, d ) < 0. 15 ! 15 !
(c) (d)
(a) 195 (b) −165 (c) 92 (d) 165 1! 2 ! 3 ! 4 ! 5 ! 1! 2 ! 3 ! 4 !
Permutations & Combinations 1053

49. If the rings are distinct, find the number of ways of 56. If the rings are identical, find the number of ways of
distributing these rings among his 5 daughters such distributing these rings among his 5 daughters such
that elder the daughter fewer the rings she gets. that elder the daughter fewer the rings she gets. The
Thus the youngest daughter gets 5 rings and the youngest daughter gets 5 rings and the eldest
eldest daughter gets just 1 ring. No any two daughter gets 1 ring only. There are no twins among
daughters are of the same age. his daughters.
15 ! 15 ! (a) 3 (b) 2 (c) 1 (d) 6
(a) × 5! (b)
1! 2 ! 3 ! 4 ! 5 ! 1! 2 ! 3 ! 4 ! 5 !
57. If the rings are identical, find the number of ways of
15 ! 15 !
(c) 15 (d) ×5 distributing these rings among his 5 daughters such
( C5 ) 5 ! 1 ! 2 ! 3! 4! 5! that none of the daughters gets less than 2 rings and
50. If the rings are distinct, find the number of ways of more than 4 rings.
(a) 51 (b) 15 (c) 52 (d) 25
distributing these rings among his 5 daughters such
that elder the daughter fewer the rings she gets. The 58. If the rings are identical, find the number of ways of
youngest daughter gets 5 rings and the eldest distributing these rings among his 5 daughters
daughter gets just 1 ring. There are no twins among equally.
his daughters. (a) 5 (b) 1 (c) 10 (d) 25
15 ! 7 15 ! 59. If the rings are identical, find the number of ways of
(a) × (b) × 3!
3 ! 5 ! 144 1! 2 ! 3 ! 4 ! 5 ! distributing these rings among his 5 daughters such
15 ! 8 that a daughter may get zero or all the rings.
(c) 155 (d) ×
3 !5 ! 121 (a) 38 (b) 76 (c) 3876 (d) 34
51. If the rings are distinct, find the number of ways of Directions (for Q. Nos. 60 to 63) : Solve the following
distributing these rings among his 5 daughters such problems independently of each other.
that none of the daughters gets less than 2 rings and Rollmall is a fast food outlet, which delivers the veg and non-veg
more than 4 rings. rolls in its vicinity. Bawarchi is an employee, who is assigned to
15 ! 15 ! pack the rolls in the boxes for delivery. He has 5 rolls in hand to
(a) (b)
2! 2! 3! 4!4! 2! 3! 3! 3! 4! pack in the 3 boxes so that none of the boxes remains empty.
(c) (15C5 ) × 5 ! (d) none of these 60. Find the number of ways of packing the rolls, if each
52. If the rings are distinct, find the number of ways of role and each box both are distinct.
(a) 150 (b) 50 (c) 60 (d) 90
distributing these rings among his 5 daughters equally.
15 ! 15 ! 61. Find the number of ways of packing the rolls, if each
(a) 1 (b) × 5 ! (c) (d) 15C5
(3 !)5 (3 !)5 roll is identical but each box is distinct.
(a) 3 (b) 5 (c) 6 (d) 4
53. If the rings are distinct, find the number of ways of
62. Find the number of ways of packing the rolls, if each
distributing these rings among his 5 daughters such roll is distinct but all the boxes are identical.
that a daughter may get zero or all the rings. (a) 120 (b) 24 (c) 25 (d) 40
(a) 75 (b) 1
(c) 155 (d) 515 63. Find the number of ways of packing the rolls, if all
the rolls and all the boxes are identical.
54. If the rings are identical, find the number of ways of (a) 3 (b) 0 (c) 1 (d) 2
distributing these rings among his 5 daughters such
Directions (for Q. Nos. 64 to 70) : Answer the following
that no daughters get equal number of rings but
questions independently of each other.
each daughter gets at least one ring and at most
5 rings. 64. In how many ways can 12 identical bouquets be kept
(a) 120 (b) 75 (c) 12 (d) 34 in 3 identical crates?
(a) 20 (b) 16 (c) 19 (d) 25
55. If the rings are identical, find the number of ways of
distributing these rings among his 5 daughters such 65. In how many ways can 12 identical bouquets be kept
that elder the daughter fewer the rings she gets. in 3 distinct crates?
(a) 19 (b) 91 (c) 81 (d) 61
Thus the youngest daughter gets 5 rings and the
eldest daughter gets only 1 ring. No any two 66. In how many ways can 12 identical bouquets be kept
daughters are of the same age. in 3 distinct crates so that no box remains empty?
(a) 1 (b) 5 (c) 15 (d) 3 (a) 36 (b) 55 (c) 66 (d) 91
1054 QUANTUM CAT
67. In how many ways can 12 identical bouquets be kept Directions (for Q. No. 71-73) : Answer the following
in 3 distinct crates so that at least 2 bouquets must questions independently of each other.
be there in each bouquet?
71. In how many ways can 4 balls be selected from the
(a) 28 (b) 24 (c) 42 (d) 36
box that contains 10 balls of distinct colours?
68. In how many ways can 12 identical bouquets be kept (a) 360 (b) 320 (c) 90 (d) 210
in 3 distinct crates so that the smallest box has at
72. In how many ways can 4 balls be selected from the
least 2 bouquets, the largest box has at least
box that contains balls of 10 distinct colours, in
4 bouquets and the third one has at least 3 bouquets?
(a) 10 (b) 20 (c) 21 (d) 32 which there are thousands of balls of each colour?
(a) 124 (b) 210
69. In how many ways can 12 identical bouquets be kept (c) 715 (d) 640
in 3 distinct crates so that no two boxes have the
73. In how many ways can 10 cookies be selected from a
same number of bouquets?
bakery that sells the cookies of 4 distinct types, such
(a) 36 (b) 32 (c) 72 (d) 64
that there must be at least one cookie of each type?
70. In how many ways can 12 identical bouquets be kept (a) 12
in 3 distinct crates so that each box has the same (b) 21
number of bouquets? (c) 84
(a) 36 (b) 1 (c) 2 (d) 12 (d) none of the above

19.9 Re-arrangement or Derangement


Rearrangement of objects such that none of the objects (ii) The number of ways in which n different letters can be
occupies its original place. placed in their n addressed envelopes so that all the
(i) The number of ways in which exactly r letters can be letters are in the wrong envelopes
placed in wrongly addressed envelopes when n letters  1 1 1 1
are placed in n addressed envelopes = n! 1 − + − + ... + ( −1) n 
 1! 2 ! 3! n !
 1 1 1 1
= n Pr 1 − + − + .... + ( −1) r 
 1! 2! 3! r!

Introductory Exercise 19.9


Directions (for Q. Nos. 1 to 4) : Answer the following 3. Find the number of ways of placing 6 letters in
questions independently of each other. 6 envelopes, which have different addresses in such
a way that at least two letters are correctly placed in
1. Find the number of ways in which all the 6 different
the right envelops.
letters are placed in 6 addressed envelops so that all
(a) 191 (b) 171 (c) 362 (d) 121
the letters are in the wrong envelops.
(a) 265 (b) 365 (c) 252 (d) 356 4. For every distinct natural number a , b, c, d , e , f < 7 , a
2. Find the number of ways of placing 6 letters in number N = (a − 1)(b − 2 ) (c − 3 )(d − 4 )(e − 5 )(f − 6 ) is
6 envelopes which have different addresses in such a an integer. Find the number of distinct sets
way that only two letters are correctly placed and {a , b, c, d , e , f } for which N is a non-zero integer.
4 letters are not in the right envelops. (a) 365 (b) 256
(a) 235 (b) 135 (c) 152 (d) 325 (c) 265 (d) none of these
Permutations & Combinations 1055

19.10 Number Properties


If a natural number N can be expressed in the canonical (x) Total number of ordered pairs (x, y) such that LCM of x
form as N = a p b q c r ...., whereas a, b, c, Kare distinct prime and y is a composite number = (2 p + 1)(2q + 1)(2r + 1)K
numbers. Then Exp. 1) Find the number of factors 72.
(i) Total number of factors of a composite number
Solution Let us consider N = 72, which can be expressed as
= ( p + 1)( q +1)( r + 1)K
72 = 2 3 × 3 2 . Now, there are total 12 factors of 72; namely 1, 2,
(ii) Sum of factors of a composite number
3, 4, 6, 8, 9, 12, 18, 24, 36, 72. Let’s see how it happens.
( a p + 1 − 1) ( b q + 1 − 1) ( c r + 1 − 1) 1 = 20 3 0 , 2 = 21 3 0 , 3 = 20 31 ,
= × × ×...
(a ) − 1 ( b − 1) ( c − 1) 4 = 22 3 0 , 6 = 21 31 , 8 = 23 3 0 ,
(iii) Product of factors of a composite number 9 = 20 3 2 , 12 = 22 31 , 18 = 21 3 2 ,
= N 1/ 2 (total number of factors of N ) 24 = 23 31 , 36 = 22 3 2 , 72 = 23 3 2
(iv)Total number of odd factors of a composite number Thus you can see that all the factors of 72 are due to only two
= ( p + 1)( q +1)( r + 1); where each of p, q, rK must be prime numbers namely 2 and 3. What’s happening here is
odd prime number only. That means if that we are taking all the possible combinations of 2j and 3 k.
Where j = 0, 1, 2, 3 and k = 0, 1, 2. In other words we can
N =2 k a p b q c r ...., then eliminate 2 k from the factors
select prime number 2 in four ways (as 20 , 21 , 22 , 23 ) and
of N to obtain number of odd factors of N . prime number 3 in three ways (as 3 0 , 31 , 3 2 ) to get the
(v) Total number of even factors of a composite number factors of 72.
= Total number of factors of N − total number of odd It implies that 72 = 23 × 3 2 has ( 3 + 1)( 2 + 1) factors = 12 factors.
factors. Thus it can be concluded that N = a p × b q × cr × .... has
(vi) Total number of co-primes to N , which are less than ( p + 1)( q + 1)(r + 1)K factors.
 1   1   1
N = N 1 −  1 −  1 −  K NOTE When N is a perfect square total number of distinct factors
 a   b  c would be odd and when N is a non perfect square total number of
distinct factors would be even.
(vii) Sum of all the co-primes of N which are less than
N Exp. 2) Find the sum of the factors of 72.
N = × Number of co-primes to N which are less
2 Solution Let us consider N = 72, which can be expressed as
than N . 72 = 23 × 3 2 . Now, there are total 12 factors of 72.
(viii) Total number of ways in which N can be written as a
( 20 3 0),( 20 31 ),( 20 3 2 ), ( 21 3 0),( 21 31 ),( 21 3 2 ),
product of two co-primes = 2 n − 1 ; where n is the
( 22 3 0),( 22 31 ),( 22 3 2 ), ( 23 3 0),( 23 31 ),( 23 3 2 )
number of distinct prime factors of N .
Therefore sum of these factors
(ix) Total number of ways in which N can be expressed as a
= ( 20 3 0) + ( 20 31 ) + ( 20 3 2 ), ( 21 3 0) + ( 21 31 ) + ( 21 3 2 )
product of two factors
+ ( 22 3 0) + ( 22 31 ) + ( 22 3 2 ) + ( 23 3 0) + ( 23 31 ) + ( 23 3 2 )
Total number of factors of N
= = ( 20 + 21 + 22 + 23 )( 3 0 + 31 + 3 2 )
2
( 23 + 1 − 1) ( 3 2 − 1)
(a) If N has odd number of factors (i.e. when N is a = ×
perfect square), the number of ways in which N can 2−1 3 −1
be expressed as a product of two ‘distinct’ factors Thus we can conclude that sum of all the factors of
(Total number of factors of N ) −1 ( a p + 1 − 1) ( b q + 1 − 1) cr + 1 − 1
= N= × × ×....
2 ( a − 1) ( b − 1) ( c − 1)
(b) If N has odd number of factors (i.e. when N is a Exp. 3) Find the product of all the factors of 72.
perfect square), the number of ways in which N can
Solution Let us consider N =72, which can be expressed as
be expressed as a product of two factors, including
the ‘similar’ factors 72 = 23 × 3 2
(Total number of factors of N ) + 1 Now, there are total 12 factors of 72; namely 1, 2, 3, 4, 6, 8, 9,
= 12, 18, 24, 36, 72.
2
1056 QUANTUM CAT
We can write these factors in pairs like we will simply ignore those numbers which are multiple of
72 = 1 × 72, 72 = 2 × 36, 2 or 3 but less than or equal to 72.
72
72 = 3 × 24, 72 = 4 × 18, Number of multiples of 2 = = 36, so the number of
2
72 = 6 × 12, 72 = 8 × 9
numbers which are not the multiple of
Therefore, product of all the factors 72  1
= 72 × 72 × 72 × 72 × 72 × 72 = 726 2 = 72 − = 721 − 
2  2
It implies that product of all the factors of 72
Number of multiples of 3 = = 24, so the number of
N = ( N ) half of the total number of factor 3
numbers which are not the multiple of
NOTE When N is a perfect square, the number of factors would be 72  1
3 = 72 − = 721 − 
odd. In that case express N as ( N ) 2 so the product of all the factors of 3  3
N = ( N ) total number of factors
Therefore, number of numbers which are neither the
multiple of 2 nor of 3
Exp. 4) Find the total number of odd factors of 72.
 1  1
= 721 −  1 − 
Solution Let us consider N = 72, which can be expressed as  2  3
72 = 23 × 3 2 It implies that the number of co-primes of 72 which are less
 1  1
Now, there are total 12 factors of 72. than 72 = 72 1 −  1 −  = 24
 2  3
( 20 3 0),( 20 31 ),( 20 3 2 ), ( 21 3 0),( 21 31 ),( 21 3 2 ),
Therefore number of co-primes to N, which is less than
( 22 3 0),( 22 31 ),( 22 3 2 ), ( 23 3 0),( 23 31 ),( 23 3 2 )
 1  1  1
N = N 1 −  1 −  1 −  ....
But if we remove those factors which involve any positive  a  b  c
power of 2, that is 21 , 22 , 23 , we will have those factors only
which are odd. Thus there are 3 odd factors in 72 namely 1, 3 Exp. 7) Find the sum of all the co-primes of 72, which are
and 9. less than 72.
So we can conclude that if N = 2k a p b qcr K; where a , b , cK are Solution Let us consider N = 24, which can be expressed as
odd prime numbers, then the number of odd factors will be 24 = 22 × 3
obtained by disregarding (or ignoring) the even prime
number. Therefore in this case total number of odd factors of Therefore number of co-primes to 24, which are less than
N = ( p + 1)( q + 1)(r + 1).....  1  1
24 = 24 1 −  1 −  = 8
 2  3
Exp. 5) Find the total number of even factors of 72. Thus there are total 8 co-prime factors of 24; namely 1, 5, 7,
Solution Let us consider N = 72, which can be expressed as 11, 13, 17, 19, 23. Now you can see the conjugate pairs of
co-prime factors which are (1, 23), (5, 19), (7,17), (11, 13).
72 = 23 × 3 2
It shows that sum of both the terms in each parenthesis is 24.
Now, there are total 12 factors of 72; namely 1, 2, 3, 4, 6, 8, 9, Thus the sum of all the co-prime factors of 24, not exceeding
12, 18, 24, 36, 72. Simply if we subtract the odd factors from 24 = 24 × 4
total factors, we will have only even factors. It implies that the sum of all the co-primes of N which are
Thus number of even factors = total number of factors − total N
odd factors less than N = × Number of co-primes to N which are less
2
Thus you can see 72 has 3 odd factors and 9 even factors. than N.

Exp. 6) Find the total number of co-primes to 72, which Exp. 8) Find the total number of ways in which 6, 30,
are less than 72. 210 and 60 can be written as a product of two co-primes.
Solution Let us consider N = 72, which can be expressed as Solution Let us consider N = 6, which can be expressed as
72 = 23 × 3 2 6 = 2 × 3.
Now, there are total 12 factors of 72; namely 1, 2, 3, 4, 6, 8, 9, Now, there are total 4 factors of 6; namely 1, 2, 3, 6.
12, 18, 24, 36, 72. You know that since 2 is a prime factor of 72, Then 6 can be expressed as product of two co-prime factors
so all the multiples of 2 (i.e., 2, 4, 6, 8, …, 72) will be factors of in two ways: 1 × 6 and 2 × 3,
72. Simply there are 72/2 = 36 such numbers. Again since 3 is Again consider N = 30, which can be expressed as
a factor of 72, so all the multiples of 3 (i.e., 3, 6, 6, 9, ..., 72) will 30 = 2 × 3 × 5
be factors of 72. Simply there are 72/3 = 24 such numbers. Now, there are total 8 factors of 30; namely 1, 2, 3, 5, 6, 10, 15,
It is also very clear that if any number less than or equal to 30.
72 is multiple of either 2 or 3 cannot be co-prime to 72. So in
Then 30 can be expressed as product of two co-prime factors
order to get co-prime numbers of 72 which are less than 72,
in four ways: 1 × 30, 2 × 15, 3 × 10 and 5 × 6.
Permutations & Combinations 1057

Similarly, if we consider N = 210, which can be expressed as And 36 can be expressed as a product of two factors
210 = 2 × 3 × 5 × 7 ( 9 + 1)
including the product of similar factors = =5
Now, there are total 16 factors of 210; namely 1, 2, 3, 5, 6, 7, 2
10, 14, 15, 21, 30, 35, 42, 70, 105, 210. Therefore it implies that, when N is a perfect square number
Then 210 can be expressed as product of two co-prime (a) The total number of ways in which N can be
factors in eight ways: expressed as a product of two ‘distinct’ factors
1 × 210, 2 × 105 , 3 × 70, 5 × 42, (Total number of factors of N) − 1
6 × 35 , 7 × 30, 10 × 21 and 14 × 15. =
2
Once again, if we consider N = 60, which can be expressed as
(b) The total number of ways in which N can be
60 = 22 × 3 × 5. Now, there are total 12 factors of 60; namely 1,
expressed as a product of two factors, including the
2, 3, 4, 5, 6, 7, 10, 12, 15, 20, 30, 60. Then there will be 6 pairs:
‘similar’ factors
1 × 60, 2 × 30, 3 × 20, 4 × 15 , 5 × 12 and 6 × 10. But 2 × 30 and
(Total number of factors of N) + 1
6 × 10 are not valid. factor =
2
Then 60 can be expressed as product of two co-prime factors
in four ways: 1 × 60, 3 × 20, 4 × 15 and 5 × 12. Exp. 10) Find the total number of ordered pairs ( x, y)
Let N = a p × b q × cr ×K and N = L × R, where L and R are such that LCM of x and y is 72.
co-prime (or relatively prime) numbers. Solution Let us consider 72 is the LCM of two numbers x and y
Then a p can be placed in two ways either at the place of L or
72 = 22 × 3 2
R. Similarly b q can be placed in two ways either at the place
of L or R. Similarly cr can be placed in two ways either at the Then the possible pairs can be determined as follows.
place of L or R. And so on... The various ways to distribute 23 to ( x , y) are
Therefore if there are n factors of N, the number of ways in ( 23 , 1), ( 23 , 2), ( 23 , 22 ),
which N can be expressed as product of two co-prime factors ( 23 , 23 ), ( 22 , 23 ), ( 2, 23 ), (1, 23 ). Thus it can be distributed in
= 21×4 24×2 2×44
K3 × 2 = 2n
n times
2 × 3 + 1 = 7 ways.
Similarly, 3 2 can be distributed to ( x , y) in 2 × 2 + 1 = 5 ways.
But since L and R are interchangeable, so the number of ways
in which N can be expressed as product of two factors Therefore the total number of ways = 7 × 5 = 35
2n These pairs are as follows:
= = 2n − 1
2 (72, 1), (72, 3), (72, 9), (24, 9), (8, 9),
(72, 2), (72, 6), (72, 18), (24, 18), (8, 18)
Exp. 9) Find the total number of ways in which 72 and (72, 4), (72, 12), (72, 36), (24, 36), (8, 36)
36 can be expressed as a product of two factors. (72, 8), (72, 24), (72, 72), (24, 72), (8, 72)
Solution Let us consider N = 72, which can be expressed as (36, 8), (36, 24), (36, 72), (12, 72), (4, 72)
72 = 23 × 3 2 . (18, 8), (18, 24), (18, 72), (6, 72), (2, 72)
Now, there are total 12 factors of 72; namely 1, 2, 3, 4, 6, 8, 9, (9, 8), (9, 24), (9, 72), (3, 72), (1, 72)
12, 18, 24, 36, 72. Now you see that Thus the total number of ordered pairs ( x , y) such that LCM
72 = (1 × 72), 72 = ( 2 × 36), of x and y is 72 = ( 2 × 3 + 1)( 2 × 2 + 1) = 35
72 = ( 3 × 24), 72 = ( 4 × 18), Thus the total number of ordered pairs ( x , y) such that LCM
72 = ( 6 × 12) of x and y is a composite number = ( 2p + 1)( 2q + 1)( 2r + 1).
It implies that total number of ways in which N can be
expressed as a product of two factor
Total number of factors of N
19.11. Geometrical Properties
=
2 (i) In a plane if there are n points of which no three are
Let us consider N = 36, which can be expressed as collinear, then
36 = 22 × 3 2
(a) The number of straight lines that can be formed by
Now, there are total 9 factors of 36; namely 1, 2, 3, 4, 6, 9, 12,
joining them = n C 2
18, 36.
Now you see that (b) The number of triangles that can be formed by
36 = (1 × 36); 36 = ( 2 × 18) joining them = n C 3
36 = ( 3 × 12); 36 = ( 4 × 9) (c) The number of quadrilaterals that can be formed by
36 = ( 6 × 6) joining them = n C 4
Thus 36 can be expressed as a product of two distinct factors
( 9 − 1) (d) The number of polygons with k sides that can be
= =4
2 formed by joining them = n C k
1058 QUANTUM CAT
(e) The number of diagonals in a polygon of n sides n2 + n + 2
= nC2 − n =
2
(ii) In a plane if there are n points out of which m points
are collinear, then 3
2 4
(a) The number of straight lines that can be formed by
joining them = n C 2 − mC 2 +1 1 10
9 11 5
(b) The number of triangles that can be formed by
joining them = n C 3 − mC 3 8 7
6
(c) The number of polygons with k sides that can be
formed by joining them = n C k − mC k (b) If n straight lines are intersecting a circle such that
(iii) If n points are given on the circumference of the no two lines are parallel and no three lines are
circle, then concurrent, the maximum number of parts/regions
(a) The number of straight lines that can be formed by into which these lines divide the circle is
joining them = n C 2 n( n +1) n2 + n + 2
n
C 0 + n C1 + n C 2 = +1 =
(b) The number of triangles that can be formed by 2 2
joining them = n C 3
(c) The number of quadrilaterals that can be formed by
joining them = n C 4
(d) The number of polygons with k sides that can be
formed by joining them = n C k
(e) The number of diagonals in a polygon of n-sides
= nC2 − n (c) If n points lie on the circumference of a circle and
are connected by straight lines intersecting the
(iv) The number of regions in which a line/plane/space/
circle such that no three lines are concurrent (that is
sphere can be divided by n points/lines/circles/
no three lines ever pass through the same point),
ellipses
then the maximum number of parts/regions into
Division by Division of Maximum Number of Parts / which these lines divide the circle is
Regions n
C 0 + n C 2 + n C 4 . The first few values are: 1, 2, 4,
1
n Points Line 8, 16, 31, 57, 99, 163, 256,
∑ Ck = nC0 + nC1
n
(1-Dimensional) k= 0

2
n Lines Plane
∑ Ck = nC0 + nC1 + nC2
n
(2-Dimensional) k= 0

3 1 2 4 8
n Planes Solid
∑ Ck = nC0 + nC1 + nC2 + nC3
n
(3-Dimensional) k= 0
(d) If n points are given on the circumference of a circle
4
and the chords determined by them are drawn. If no
n Hyper Planes Space
∑ Ck = nC0 + nC1 + nC2
n
(4-Dimensional) three chords have a common point, number of
k= 0
triangles all of whose vertices lie inside the circle
+ nC 3 + nC 4 n!
= nC6 =
(a) If n straight lines are drawn in the plane such that no 6!( n − 6)!
two lines are parallel and no three lines are
concurrent, then the maximum number of
parts/regions into which these lines divide the plane
is
n( n +1)
n
C 0 + n C1 + n C 2 = +1
2
Permutations & Combinations 1059

Every set of six points on the circumference of (k) If n circles are intersecting each other, maximum
the circle can be paired in one and only one way number of points of intersection
such that the three lines joining pairs will form = 2 × ( n C 2 ) = n P2 = n ( n − 1)
an admissible triangle.
(l) If n points in the plane are connected by straight lines in
Conversely, every admissible triangle has sides all the possible ways, and of these no two are coincident
leading to six points on the circumference. or parallel and no three of them are concurrent except at
Hence the number of admissible triangles is the given points, the number of points of intersection
n!
= nC6 = other than the given points of the lines so formed is
6!( n − 6)! 1 n!
×
(e) If n circles are drawn in the plane such that no 8 ( n − 4)!
two of them are tangent, none of them lies 2 4
entirely within or outside of another one and no
three of them are concurrent, the maximum
1 1 2 3
number of parts/regions into which these
circles divide the plane is n( n − 1) + 2
n=1 n=2
(f) If n ellipses are drawn in the plane such that no
two of them are tangent, none of them lies 8 14
entirely within or outside of another one and no 1 6
2
1 4 2
three of them are concurrent, the maximum 9 10
7 5 13 7
number of parts/regions into which these 6 5 12 11
ellipses divide the plane is 2n( n − 1) + 2 3 4 8 3
(g) If there are n overlapping triangles, the
maximum number of regions into which these n=3 n=4
triangles divide the plane is 3n 2 − 3n + 2
2 6 14 26
(h) If n planes are drawn in the space such that no
four planes intersect at a single point and the
intersection of any three planes are non-parallel
lines and no two planes are parallel, the
maximum number of spacial regions into which n=1 n=2 n=3 n=4
these planes divide the space is 2 8 20
n 3 + 5n + 6 11
n
C 0 + n C1 + n C 2 + n C 3 = 10 1 2 12
6 9
3
13
18 19
(i) The maximum number of regions into which a 8
4

3-dimensional cube/sphere/cylinder can be 17 7 5 14


6
16 15
partitioned by exactly n planes is
n 3 + 5n + 6 n=1 n=2 n=3
n
C 0 + n C1 + n C 2 + n C 3 =
6 (v) Bounded and unbounded Regions
(j) A great circle is a circle drawn on a sphere is Since every new line added on the plane increases the
known as an ‘equator’. That is, its center is also number of unbounded regions by 2, so by using this logic
the centre of the sphere. There are n great we can determine the number of unbounded and bounded
circles on a sphere such that no three of which regions. As n th line creates n new regions and that of the n
meet at any point, the number of regions into new regions introduced, n − 2 of them must be new bounded
which these great circles divide the sphere is regions. And the maximum number of bounded regions is
n2 − n + 2 the difference of total number of regions and unbounded
regions.
1060 QUANTUM CAT
Refer to the following table.
New unbounded Now bounded
Number of Total Total new regions Total unbounded Total bounded
regions added by regions added by
lines n regions added by nth line regions regions
nth line nth line
1 2 1 2 (1) 0 (0)
2 4 2 4 2 0 0
3 7 3 6 2 1 1
4 11 4 8 2 3 2
5 16 5 10 2 6 3
6 22 6 12 2 10 4
…… …… …… …… …… …… ……
…… …… …… …… …… …… ……
n n2 + n + 2 n 2n 2 n2 − 3n + 2 n−2
n 2

NOTE The figures, as shown in the table above, written in (e) Total number of quadrilaterals if m parallel lines
parentheses () indicate the exception to the pattern.
intersect other n parallel lines = mC 2 × n C 2
l
Total number of regions may vary depending on the way each line
is drawn. In order to have maximum number of regions no two lines (vii) Number of Triangles in any Triangle
should be parallel and no three lines should pass through any
intersecting point.
If you draw m lines from a vertex,
l
In the above table, the second column by default talks about these new lines will divide the side
MAXIMUM number of total regions. of the original triangle in m +1units
l
Total number of unbound regions varies as per NUMBER of lines
(or parts).
drawn only, not on the WAY the lines are drawn on the plane.
l
Total number of BOUNDED regions may vary depending on the way (a) When lines are drawn from only
each line is drawn. In order to have maximum number of regions no one vertex
two lines should be parallel and no three lines should pass through
any intersecting point. If a side of a triangle is divided
l
In the above table, the sixth column by default talks about n ( n + 1)
MAXIMUM number of total bounded regions. into n units, the total number of triangles =
l
Each new line drawn on the plane increases minimum 1 new
2
bounded region and maximum n − 2 bounded regions. NOTE In the above diagram, there are 6 triangles.
l
Minimum number of bounded regions given by each nth line is
n − 2. (b) When lines are drawn from any two vertices
If each side of a triangle is divided into n units, the
(vi) Number of Squares/Rectangles/Quadrilaterals
total number of triangles = n 3
(a) Total number of squares in a square having n
columns and n rows
n( n + 1)(2n + 1)
= 12 + 2 2 + 3 2 + .... + n 2 = Σn 2 =
6
(b) Total number of rectangles in a square having
n columns and n rows
2 NOTE In the above diagram, there are 27 triangles.
 n( n + 1) 
= 13 + 2 3 + 3 3 + .... + n 3 = Σn 2 =   (c) When the original triangle is an equilateral triangle
 2 
and lines are drawn parallel to the sides of original
(c) Total number of squares in a rectangle having triangle, as shown in the following figures.
m columns and n rows
= m ⋅ n + ( m − 1)( n − 1) + ( m − 2)( n − 2) +K + 0
(d) Total number of rectangles in a rectangle having
m columns and n rows
= (1 + 2 + 3 + K + m)(1 + 2 + 3 +K + n)
Permutations & Combinations 1061

If each side of a triangle is divided into n units – C. Total number of triangles pointing downward
 n( n + 2)(2n − 1) 
A. Total number of triangles = 
 24 
 n( n + 2)(2n + 1) 
= 
Or, total no. of triangles pointing downward
 8  1
24 ( n) ( n + 2)(2n − 1), when n is even
Or, total number of triangles =
( −1) n + 4n 3 +10n 2 + 4n − 1  1 ( n 2 − 1) (2n + 3), when n is odd
= 24
16
NOTE
Or, total number of triangles l
In the above diagram, figure (i) has 13 triangles and figure (ii) has
 n( n + 2)(2n + 1) 27 triangles.
 , when n is even l
In the above diagram, figure (i) has 10 upward pointing triangles
8
 and figure (ii) has 20 upward pointing triangles.
[ n( n + 2)(2n + 1) −1] , when n is odd l
In the above diagram, figure (i) has 3 downward pointing triangles
 8 and figure (ii) has 7 downward pointing triangles.

Let n be the number of units in each side of the original


B. Total number of triangles pointing upward
n ( n + 1)( n + 2) triangle and k be the number of units in each side of the
= triangle that can be formed out of the given triangle.
6
The following table gives the number of triangles for different values of n and k for Upward pointing triangles.
n k Total
1 2 3 4 5 6 7 8 9 10 Triangles

1 1 1
2 3 1 4
3 6 3 1 10
4 10 6 3 1 20
5 15 10 6 3 1 35
6 21 15 10 6 3 1 56
7 28 21 15 10 6 3 1 84
8 36 28 21 15 10 6 3 1 120
9 45 36 28 21 15 10 6 3 1 165
10 55 45 36 28 21 15 10 6 3 1 220
…… …… …… …… …… …… …… …… …… …… …… ……

The following table gives the number of triangles for different values of n and k for Downward pointing triangles.
n k Total
1 2 3 4 5 6 7 8 9 10 Triangles
1 0
2 1 1
3 3 3
4 6 1 7
5 10 3 13
6 15 6 1 22
7 21 10 3 34
8 28 15 6 1 50
9 36 21 10 3 70
10 45 28 15 6 1 95
…… …… …… …… …… …… …… …… …… …… …… ……
1062 QUANTUM CAT
(viii) Number of Triangles, Diagonals and Intersections
in a Regular Polygon
A. Number of triangles formed by the vertices of a
regular polygon of n sides .
(a) Number of triangles formed by joining the
3 vertices of n - sided polygon 5 Diagonal Endpoints 6 Diagonal Endpoints 6 Diagonal Endpoints
n( n − 1)( n − 2) with False Triangle
N = nC3 = ; ∀n ≥3
6 Number of triangles with 3 Diagonal Endpoints = n C 3
(b) Number of triangles having one side common
with that of the polygon N 1 = n( n − 4); ∀ n ≥ 3 Number of triangles with 4 Diagonal Endpoints
(c) Number of triangles having two sides = 4( n C 4 )
common with that of the polygon Number of triangles with 5 Diagonal Endpoints
N 2 = n; ∀ n ≥ 3 = 5( n C 5 )
(d) Number of triangles so that at least one side of
the triangle coincides with the side of the Number of triangles with 6 Diagonal Endpoints
polygon = n C 6 − [ a 3 ( n)( 3 C 3 ) + a 4 ( n) ( 4 C 3 ) + a 5 ( n)( 5 C 3 )
= ( N 1 + N 2 ) = n( n − 3) + a 6 ( n)( 6 C 3 ) + a 7 ( n)( 7 C 3 ) + δ 2 ( n)( n 2 C 3 )]
(e) Number of triangles having no side common
Therefore, the number of triangles generated by
with that of the polygon
intersecting diagonals of an n -sided regular polygon is
n( n − 4)( n − 5)
N 0 = N − (N1 + N 2 ) = ; ∀ n≥6 n
C 3 + 4( n C 4 ) + 5( n C 5 ) + n C 6
6
B. Number of triangles formed by intersecting − [ a 3 ( n) ( 3 C 3 ) + a 4 ( n) ( 4 C 3 ) + a 5 ( n) ( 5 C 3 )
diagonals of a regular polygon of n - sides.
+ a 6 ( n) ( 6 C 3 ) + a 7 ( n) ( 7 C 3 ) + δ 2 ( n) ( n / 2 C 3 )]
Let us assume N (d k ) indicates the number of
triangles with k number of diagonal endpoints For example, if we consider an 8-sided regular polygon, the
total number of triangles
Number of sides
Number of Triangles
in a polygon = 8C 3 + 4( 8 C 4 ) + 5( 8 C 5 ) + 8C 6 − [8(1) + 0( 4 C 3 )
n N ( d3 ) N ( d4 ) N ( d5 ) N ( d6 ) Total + 0( 5 C 3 ) + 0( 6 C 3 ) + 0( 7 C 3 ) + 1( 4 C 3 )]
3 1 1
4 4 4 8 = 56 + 280 + 280 + 28 − (8 + 4)
5 10 20 5 35 = 56 + 280 + 280 + 16 = 632
6 20 60 30 110 Hint The expression a 3( n)( 3C 3) + a 4 ( n)( 4 C 3) + a 5( n)( 5C 3)
7 35 140 105 7 287 + a 6 ( n) ( 6 C 3) + a 7( n) ( 7C 3) + δ 2( n) ( n / 2C 3) indicates the number of false
8 56 280 280 16 632 triangles. It happens when more than two diagonals intersect at any point
9 84 504 630 84 1302 inside the regular polygon.
10 120 840 1260 180 2400
I. Here a m ( n) denotes the number of intersection
Let us consider an example of a regular octagon (interior) points other than the centre when m diagonals
(8-sided polygon), as shown below. By connecting intersect each other.
all the vertices it produces 8 C 2 = 28 lines and II. Here k C 3 denotes the number of false triangles
20 ( = 28 − 8) diagonals. corresponding to a common point created by the
intersection of k line segments for every k > 2.
III. Here δ 2 ( n) denotes the contribution of the centre-point
for even value of n. When n is odd, there is no centre
point inside the regular polygon.
0, if n is odd
Therefore, δ 2 ( n) = 
A Regular Octagon 3 Diagonal Endpoints 4 Diagonal Endpoints 1, otherwise
Permutations & Combinations 1063

NOTE By observation, the number of diagonals m is 2, 3, 4, 5, 6 or 7. In the following figure, a grid of 3 × 5 dimensions is shown,
In particular, it is impossible to have 8 or more diagonals of a regular
n-gon meeting at a point other than the centre.
in which there are 4 horizontal lines and 6 vertical lines
intersect each other. Using these lines as roads you can go
The maximum number of diagonals of a regular n-gon ( n > 4) that meet
from one corner to another one.
at a point other than the center is m = 2, if n is odd or n is 6
D C
m = 3, if n is even but not divisible by 6,
m = 4, if n is 12
m = 5, if n is divisible by 6 but not 30, and,
m = 7, if n is divisible by 30.
A B
The number of intersection points formed inside a regular n-gon by its
diagonals. In mathematics, we are usually interested in determining the
Total shortest paths and maximum number of paths available
am(n ) [Intersection Points Other Intersection
Than Centre]
from one point to another one.
Centre Points
n m Suppose you have to go from A to C using the shortest
Point With-
With
a2(n ) a3(n ) a4 (n ) a5(n ) a6 (n ) a7 (n ) out paths, without backtracking, then how many shortest paths
Centre Centre
are there between A and C?
5 2 5 5 5
If you look closely, then you will realize that while going
6 2 12 1 12 13
from A to C you can move in only 2 directions – either
7 2 35 35 35
rightward or upward. In other words, to reach from A to C,
8 3 40 8 1 48 49
you have to use the combination of rightward and upward
9 2 126 126 126 moves.
10 3 140 20 1 160 161
As you see in the diagram there are 5 RIGHT steps
11 2 330 330 330
(between A and B) and 3 UP steps (between B and C). So
12 4 228 60 12 1 300 301
it’s obvious that to reach from A to C, you have to take total
... ... ... ... ...
8( = 5 + 3) steps. Now, it’s your choice how you choose these
15 2 1365 1365 1365 steps. Suppose if you want to go from A to C, then you can
... ... ... ... ... choose your paths as given below.
18 5 1512 216 54 54 1 1836 1837
RRRRRUUU or RRRRURUU or RRRRUURU or … or
... ... ... ... ...
UUURRRRR. It implies that you have to find out that in how
30 7 13800 2250 420 180 120 30 1 16800 16801
many ways you can select 5 RIGHT steps and 3 UP steps
For a generic polygon, maximum number of intersection points formed out of total 8 step. And this can be done in
inside a regular n-gon by its diagonals is n C 4 , because every four
8!
vertices would act as the end points of a unique pair of intersecting
8
C 5 × 8− 5C 3 = 8C 5 × 3C 3 = ways
diagonals. But it can be less, because in a regular n-gon it may happen 5! × 3!
that three or more diagonals meet at an interior point, and then some
In a sense, it shows that in how many ways you can arrange
of the n C 4 intersection points will coincide.
your 8 steps. Since out of 8 steps 5 steps are identical
Also, as we know that no three diagonals meet when n is odd, it implies
(RIGHTWARD) and 3 steps are identical (UPWARD).
that for odd n the number of intersection points
a m ( n) = nC 4 .
8!
Here k C 3 denotes the number of false triangles corresponding to a
5! × 3!
common point created by the intersection of k line segments for every So all the 8 steps can be arranged in 8!(5! × 3!) ways.
k > 2.
Since, k > 2, it implies that this term a 2( n)( n C 3) makes no sense, as you
In general, if there is a grid of m horizontal lines and n
cannot select 3 things out of 2 things, and that’s why it has been
vertical lines, there will be ( m −1) steps in the horizontal
discarded from the formula. direction and ( n −1) steps in the vertical direction, thus the
Here δ 2( n) denotes the contribution of the centre-point for even value total number of steps will be ( m − 1) + ( n − 1) = ( m + n) − 2.
of n. Therefore, δ 2( n) = 
0 , if n is odd
. Therefore, the total number of shortest paths that can be
1, otherwise
taken to reach from one corner to the opposite corner
Paths and Grids When m horizontal lines intersect the n ( m + n − 2)!
= ( m + n − 2 ) C m − 1 × n − 1C n − 1 = .
vertical lines then a grid or network is formed. ( m − 1)!( n − 1!)
1064 QUANTUM CAT
Introductory Exercise 19.10

Directions (for Q. Nos. 1 and 2) : Answer these questions Directions (for Q. Nos. 11 to 14) : Answer these questions
based on the following information. based on the following information.
There are four non-collinear points on a plane. There are twelve points on a plane, out of which four are
collinear points.
1. Find the maximum number of straight lines formed
by joining the four points. 11. Find the maximum number of straight lines formed
(a) 3 (b) 2 (c) 1 (d) 6 by joining the twelve points.
(a) 33 (b) 60 (c) 62 (d) 61
2. Find the maximum number of triangles formed by
joining the four points. 12. Find the maximum number of triangles formed by
(a) 4 (b) 2 (c) 1 (d) 6 joining the twelve points.
(a) 215 (b) 216 (c) 220 (d) 196
Directions (for Q. Nos. 3 to 6) : Answer these questions
based on the following information. 13. Find the maximum number of quadrilaterals formed
by joining the twelve points.
There are ten points on a plane, such that no three of them are
(a) 495 (b) 491 (c) 494 (d) 400
collinear.
14. Find the maximum number of hexagons formed by
3. Find the maximum number of straight lines formed
joining the twelve points.
by joining the ten points.
(a) 900 (b) 924 (c) 906 (d) 609
(a) 90 (b) 30 (c) 45 (d) 20
Directions (for Q. Nos. 15 to 18) : Answer these questions
4. Find the maximum number of triangles formed by
joining the ten points. based on the following information.
(a) 100 (b) 120 (c) 90 (d) 60 There are n points on a plane, out of which m are collinear
points, such that m < n
5. Find the maximum number of hexagons formed by
joining the ten points. 15. Find the maximum number of straight lines formed
(a) 210 (b) 120 (c) 240 (d) 66 by joining them.
(a) n − m C2 + 1 (b) nC2 − mC2 − 1
6. Find the maximum number of diagonals in a (c) n−m
C2 − 1 (d) nC2 − mC2 + 1
decagon formed by joining the ten points.
16. Find the maximum number of triangles formed by
(a) 30 (b) 45 (c) 35 (d) 20
joining them.
Directions (for Q. Nos. 7 to 10) : Answer these questions (a) n ! − m ! (b) m C3 − nC3
n−m
based on the following information. (c) C3 − C3
m
(d) nC3 − mC3
There are fifteen points on the circumference of a circle. 17. Find the maximum number of quadrilaterals formed
7. Find the maximum number of straight lines formed by joining them.
by joining the fifteen points. (a) m C4 − nC4 (b) nC4 + mC4
(a) 105 (b) 210 (c) 15 (d) 60 (c) C4 − C4
n m
(d) nC4 − mC4 + 1

8. Find the maximum number of triangles formed by 18. Find the maximum number of hexagons formed by
joining the fifteen points. joining them.
(a) 555 (b) 455 (c) 660 (d) 560 (a) nC6 − mC6 (b) nC6 − mC6 + 6
(e) 525 (c) C6 + C6
n m
(d) nC6 + mC6 − 6

9. Find the maximum number of quadrilaterals formed Directions (for Q. Nos. 19 to 22) : Solve the following
by joining the fifteen points. problems independently of each other.
(a) 365 (b) 1035 19. Find the number of diagonals in a heptagon.
(c) 1155 (d) 1365 (a) 14 (b) 21 (c) 12 (d) 16
10. Find the maximum number of octagons formed by 20. Find the number of diagonals in the n-sided polygon.
joining the fifteen points. (a) nC2 − n (b) nC2
(a) 6543 (b) 2345 n(n − 3 )
(c) nC2 + n (d)
(c) 7275 (d) 6435 2
Permutations & Combinations 1065

21. A polygon has 54 diagonals. Find the number of its 29. If a plane is divided into 22 distinct regions, find the
sides. minimum number of coplanar lines which divide this
(a) 18 (b) 12 plane in such a way that neither two lines are parallel
(c) 14 (d) 9 nor any three lines are concurrent.
(a) 3 (b) 4 (c) 8 (d) 6
22. Find the value of m, if there are maximum 28 triangles
30. The maximum number of disjoint regions that can be
formed by joining total 9 points on a plane, out of
formed in the plane by n simple closed curves which
which m points are collinear.
pair-wise meet in at most two points is
(a) 3 (b) 4
(a) n3 − n + 2 (b) n2 − n + 1
(c) 1 (d) 8
(c) n − n + 2
2
(d) n(n − 1)
Directions (for Q. Nos. 23 and 24) : Answer these questions 31. If there are 6 points on the circumference of the
based on the following information. circular sheet, find the maximum possible number of
Two parallel lines each have a number of distinct points marked triangular regions that can be cut out of the circular
on them. On one line there are two points P and Q. On the other sheet such that none of the vertices of these
line there are eight points.
triangles are the points of the circumference.
23. Find the maximum number of different triangles (a) 3 (b) 2 (c) 1 (d) 15
which could be formed having three of the ten points
as their vertices. Directions (for Q. Nos. 32 to 35) : Answer these questions
(a) 3 (b) 2 (c) 64 (d) 6 based on the following information.
Once, Amey and his friends decided to celebrate their success after
24. Find the maximum number of triangles which must
have P as their vertex. they secured their admissions into various IIMs (Indian Institutes
(a) 3 (b) 2 (c) 36 (d) 6 of Management). They ordered a thin crust cheese pizza from Pizza
Hut and they told Pizza Hut to deliver it without cutting into pieces.
Directions (for Q. Nos. 25 to 27) : Solve the following
The pizza was large enough to satiate their cravings. Once they
problems independently of each other.
receive the pizza, they were so excited that they didn’t mind cutting
25. If 20 straight lines be drawn in a plane such that no the pizza into any shape and size. However, while cutting the pizza
two of them being parallel and no three of them they cut it from edge to edge, that is they started from any point on
being concurrent, maximum how many points of the circumference and didn’t lift the knife until it reaches another
intersection will there be?
point on the circumference. It’s like they cut it along various chords
(a) 95 (b) 380
of a circle.
(c) 400 (d) 190
32. At most, how many pieces would they get if they
26. If m parallel lines in a plane are intersected by a
made 8 cuts in this pizza?
family of n parallel lines, find the maximum number
(a) 37 (b) 32 (c) 58 (d) 36
of parallelograms thus formed.
(a) m n (b) (m + 1) (n + 1) 33. If they wanted to have 20 pieces of this pizza,
minimum how many cuts would be required?
(m − n) mn(m − 1) (n − 1)
(c) (d) (a) 4 (b) 5 (c) 10 (d) 6
n! 4
34. Amey had just made 7 cuts making maximum possible
27. In a plane there are 37 straight lines, of which number of pieces such that everyone present there
13 pass through the point A and 11 pass through the could have had one piece each. But before anybody
point B. Besides, no three lines pass through one could have picked up a piece of it few more friends
point, no line passes through both the points A and arrived at the party. Thus to serve the new guests at
B, and no two lines are parallel. Find the maximum his party, he made one more cut keeping in mind that
number of points of intersection of the straight lines. each new guest would get exactly one piece of pizza.
(a) 535 (b) 525 (c) 235 (d) 355 What’s the maximum number of new guests (friends)
Directions (for Q. Nos. 28 to 31) : Answer these questions that showed up at the party?
(a) 8 (b) 7 (c) 9 (d) 6
independently of each other.
35. If Amey marked 6 points on the circumference of the
28. If 6 straight lines be drawn in a plane such that no
pizza and then the pizza was cut into pieces such that
two of them being parallel and no three of them
all the cuts were straight and must passed through
being concurrent, the maximum number of regions
these points on the circumference, at most how many
into which this plane will be divided pieces they would have got out of this pizza?
(a) 30 (b) 22 (c) 18 (d) 15 (a) 99 (b) 69 (c) 66 (d) 25
1066 QUANTUM CAT
Directions (for Q. Nos. 36 to 38) : Answer these questions (iii) Any cut must last from one marked point to another
based on the following information. marked point.
Actually few of his friends love cheese so much so that they (iv) One has to cut the paper as many times as the number
don’t want to eat the pieces which have less cheese on it as of chords are possible with n points on the
they find it tasteless. For them, size of the piece does not circumference of a circle.
matter as long as it has cheese uniformly spread on it. The (v) One cannot displace the pieces until all the possible
problem is that there is no cheese spread on the edge (or cuts are made.
circumference) of the pizza in order to avoid any untoward 41. Find the maximum possible number of triangles
incident of the cheese falling on the clothes while eating. A which do not have any of its vertices out of the n
piece of pizza that contains some part of circumference (or marked points.
edge) has less or no cheese spread on it is called as Edger (a) 0 (b) 3 (c) nC3 (d) nC6
and another piece of pizza that is from the middle part of
42. If n = 6, find the maximum possible number of
pizza which has proper cheese spread on it is called as triangles that can be cut out of the circular sheet
Cheeser. such that none of the vertices of these triangles are
36. If there are 15 such guys and girls who don’t eat the the points of the circumference.
Edger and each such person must get a Cheeser one (a) 3 (b) 6
to eat, minimum how many cuts are required to get (c) 1 (d) 2
the desired pieces?
(a) 6 (b) 7 (c) 8 (d) 9 Directions (for Q. Nos. 43 and 44) : Answer these two
questions based on the following information.
37. If there are 25 such guys and girls who don’t want to
eat Edger one and if only 8 cuttings are made to cut Khayamati is not just a foodie and extravagant but also a
this pizza, minimum how many such persons have to party animal. On her birthday her friends got an elephant
compromise with an Edger piece of pizza? size cake that was supposed to be distributed among all her
(a) 0 (b) 1 (c) 2 (d) 4 friends. The cake was cylindrical in shape. To cut this cake
38. Minimum how many cuts are required if they need they got a very large knife which was able to cross through
more number of Cheeser than that of Edger pieces? the whole cake from any direction or angle. That is a single
(a) 3 (b) 6 (c) 9 (d) 7 cut would be enough to make two pieces of the whole cake.
Directions (for Q. Nos. 39 and 40) : Answer these questions By the way, they made n cuts to the cake from any arbitrary
based on the following information. direction they could make. It was notable that until all the
A Venn diagram has two circles, which cuts the plane into n cuts were done no one removed even a single piece.
4 regions and another Venn diagram has 3 circles, which 43. If n = 6, find the maximum possible number of
cuts the plane into 8 regions. pieces that can be sliced out of that cake.
39. Find the maximum possible number of regions on a (a) 32 (b) 62 (c) 42 (d) 22
plane that can be created by a Venn diagram with 6 44. After cutting the cake if they got total 300 pieces,
circles. find the minimum possible number of cuts required.
(a) 28 (b) 32 (c) 16 (d) 36 (a) 13 (b) 17
40. If you have ovals instead of circles, find the (c) 11 (d) 12
maximum possible number of regions on a plane Directions (for Q. Nos. 45 to 60) : Solve these problems
that can be created by a Venn diagram with 5 ovals.
independently of each other.
(a) 34 (b) 42 (c) 18 (d) 26
45. Find the total number of squares and total number of
Directions (for Q. Nos. 41 and 42) : Answer these questions
rectangles, respectively, in the given figure. The main
based on the following information.
figure itself is a square.
In a class of origami, in Japan, my teacher Yamamotoyama
gave me a circular paper, a knife and a scissors. She asked
me to cut the paper in order to make triangles out of the
circular sheet. She laid down some rules which were
mandatory to follow for every origami student.
(i) A sheet of paper has n points marked on its circumference.
(a) 2 and 4 (b) 2 and 2
(ii) One must cut through the marked points only. (c) 3 and 4 (d) 3 and 9
Permutations & Combinations 1067

46. Find the total number of squares and total number of 56. Find the total number of squares when 8 vertical
rectangles in the following figure. The main figure lines intersect 15 horizontal lines such that all the
itself is a square. vertical lines are parallel and equidistant to each
other, and all the horizontal lines are also parallel
and equidistant to each other.
(a) 336 (b) 456 (c) 1096 (d) 376
57. How many total squares are there in a standard
chess board (or check board)?
(a) 65 (b) 204 (c) 256 (d) 206
(a) 10 and 6 (b) 10 and 16
(c) 14 and 36 (d) 14 and 22 58. How many total rectangles are there in a standard
chess board (or check board)?
47. Find the total number of squares and total number of
(a) 1296 (b) 512 (c) 0 (d) 2025
rectangles in the following figure.
59. Find the total number of squares in the following
The smallest unit (or cell) in this figure is a square.
diagram, where the main figure (i.e., the largest one)
itself is a square.

(a) 10 and 20 (b) 10 and 16


(c) 14 and 45 (d) 14 and 35
48. Find the total number of squares in a square having
(a) 42 (b) 62 (c) 27 (d) 40
10 rows and 10 columns.
(a) 385 (b) 360 (c) 100 (d) 275 60. Find the total number of triangles in the given
49. Find the total number of rectangles in a square diagram.
having 10 rows and 10 columns.
(a) 3225 (b) 1625 (c) 1044 (d) 3025
50. Find the total number of squares in a rectangle
having 8 rows and 15 columns.
(a) 345 (b) 456 (c) 567 (d) 606
(a) 3 (b) 27 (c) 9 (d) 8
51. Find the total number of rectangles in a square
having 8 rows and 15 columns.
(a) 4321 (b) 6320 (c) 4320 (d) 3456
Directions (for Q. Nos. 61 to 66) : Solve the following
problems independently of each other.
52. Find the total number of quadrilaterals in a square
having 10 rows and 10 columns. 61. Find the total number of the shortest routes from one
(a) 3025 (b) 7744 (c) 2401 (d) 2601 corner to its opposite corner in the given diagram.

53. Find the total number of quadrilaterals in a rectangle


having 8 rows and 15 columns.
(a) 4321 (b) 4320 (c) 1234 (d) 5440
54. Find the total number of quadrilaterals when (a) 10 (b) 15 (c) 6 (d) 9
8 parallel lines intersect another set of 15 parallel 62. Find the total number of the shortest routes from
lines. A to C in the given diagram.
(a) 3636 (b) 2940 (c) 1990 (d) 3654 D C
55. Find the total number of rectangles when 8 vertical
lines intersect 15 horizontal lines such that all the
vertical lines are parallel and equidistant to each
other, and all the horizontal lines are also parallel
and equidistant to each other. A B
(a) 3996 (b) 6040 (c) 2940 (d) 6734 (a) 4200 (b) 2402 (c) 2400 (d) 4800
1068 QUANTUM CAT
63. Find the total number of the shortest routes from Directions (for Q. Nos. 67 to 70) : Solve the following
one corner to its opposite corner in the following problems based on the following information.
diagram. An industrial corridor is laid out along the Special
Economic Zone (SEZ), which is in the shape of a regular
polygon. Every vertex of the SEZ has exactly one
manufacturing plant.
A particular road connects any two plants. No single road
connects directly more than two plants. Engineers have laid
down all the possible roads connecting all the plants with
(a) 42 (b) 84 (c) 92 (d) 40 each other. Each road is designed to offer the shortest
64. Find the total number of the shortest routes from distance between any two plants.
one corner to its opposite corner in the following Also, engineers have laid down all the possible rail-loops.
diagram. Each loop connects any three plants. Every loop is designed
along the shortest possible distance between any two plants
connected through that loop.
Wherever any two or more than two roads intersect inside
the SEZ (polygon), exactly one warehouse is to be built
there.
Wherever any three or more than three roads intersect inside
(a) 800 (b) 400 (c) 388 (d) 380 the SEZ (polygon), exactly one government office is to be
built there.
65. Find the total number of the shortest paths from A to
Wherever any five or more than five roads intersect inside
C in the following diagram.
the SEZ (polygon), exactly one airport is to be built there.
D C
Q R 67. Maximum how many rail-loops are there if the SEZ
has 16 plants?
(a) 480 (b) 256
P S (c) 560 (d) none of these
A B 68. Maximum how many warehouses are there if the SEZ
has 25 plants?
In this diagram, PQRS is a military area so one
(a) 7220 (b) 12650
cannot enter inside the premises.
(c) 14400 (d) none of these
However, one may go along PQ, QR, PS and SR.
69. Maximum how many government offices are there if
(a) 100 (b) 110
the SEZ has 49 plants?
(c) 168 (d) 204
(a) 0
66. Find the total number of triangles in a pentagon (b) 7
when all its vertices are joined with each other. (c) 50
(d) none of the above
70. Maximum how many airports are there if the SEZ has
32 plants?
(a) 56 (b) 108
(a) 10 (b) 35 (c) 25 (d) 20 (c) 8648 (d) none of these
CAT-Test
Questions Helping you bell the CAT

LEVEL 01 > BASIC LEVEL EXERCISE


1 A letter lock consists of 4 rings, each ring contains 9 If all vowels occupy odd places, how many words can be
9 non-zero digits. This lock can be opened by setting a formed from the letters of the word HALLUCINATION?
4 digit code with the proper combination of each of the (a) 129650 (b) 1587600
4 rings. Maximum how many codes can be formed to (c) 78500 (d) none of these
open the lock? 10 In how many ways can the letters of the word
(a) 49 (b) 9 P4 SUCCESSFUL be arranged?
(c) 94 (d) none of these (a) 1215700 (b) 1251600
(c) 151200 (d) none of these
2 In the previous question, if the lock opens in only one
arrangement of 4 digits, how many unsuccessful 11 If all S′s come together, then in how many ways the
attempts are possible in which lock cannot be opened? letters of the word SUCCESSFUL be arranged?
(a) 49 − 1 (b) 94 − 1 (a) 10080 (b) 40080
9 (c) 2378 (d) none of these
(c) C 4 (d) none of these
12 If all C’s occur together and all U′s also occur together,
Directions (for Q. Nos. 3 to 7) : Answer these questions based on then how many arrangements are possible of the word
the following information. SUCCESSFUL?
A group consists of four straight couples. That means each couple (a) 5745 (b) 2760 (c) 6720 (d) 5432
is having a male and a female. 13 What is the sum of all the 4 digit numbers which can be
formed with the digits 1, 2, 3, 4 without repetition?
3 In how many ways could they be arranged in a straight (a) 15560 (b) 87660 (c) 45600 (d) 66660
line such that the men and the women occupy alternate
14 What is the sum of all 5 digit numbers which can be
positions?
formed with the digits 0, 1, 2, 3, 4 without repetition?
(a) 1152 (b) 1278
(a) 2599980 (b) 235500 (c) 923580 (d) 765432
(c) 1296 (d) none of these
15 Which one of the following polygon has as many
4 In how many ways can they be arranged in a straight diagonals as the number of sides in it?
line such that no two men were sitting together? (a) 4 (b) 6 (c) 5 (d) 7
(a) 1242 (b) 1440 (c) 3880 (d) 2880 16 In how many ways can the letters of the word
5 In how many ways can they be seated such that each PROPORTION be arranged by taking 4 letters at a time?
husband sits with his own wife? (a) 123 (b) 758 (c) 658 (d) 578
(a) 384 (b) 275 17 In the previous question (no. 16) how many words can
(c) 184 (d) none of these be formed without changing the relative positions of the
vowels and consonants?
6 In how many ways can they be seated around a circular
(a) 217 (b) 720 (c) 920 (d) 1040
table such that the men and women occupy the alternate
positions? 18 There are 16 executives including two brothers, Lehman
and Mckinsey. In how many ways can they be arranged
(a) 288 (b) 144 (c) 72 (d) 720
around the circular table if the two brothers cannot be
7 In how many ways can they be seated on the single seated together?
bench such that all the men sit together and all the (a) (14 !)⋅ 13 (b) 14 P3
women sit together? 14 !
(c) (d) none of these
(a) 1512 (b) 1296 3!
(c) 1152 (d) none of these 19 In the previous question how many arrangements are
8 How many different words can be made using the letters of possible if there is exactly one executive between these
the word ‘HALLUCINATION’ if all consonants are together? two brothers?
(a) 129780 (b) 1587600 (a) (14 !)2 (b) (14 !)
(c) 35600 (d) none of these (c) 2⋅(14 !) (d) none of these
1070 QUANTUM CAT
20 In how many different ways can 6 different balls be 33 In how many ways can 15 billiard balls be arranged in a
distributed to 4 different boxes, when each box can hold row if 3 are red, 4 are white and 8 are black?
any number of ball? (a) 12 (b) 18
(a) 2048 (b) 1296 (c) 96 (d) none of these
(c) (24)2 (d) 4096 34 Seven delegates are to address a meeting. If a particular
21 In how many different ways can 6 identical balls be speaker is to speak before another particular speaker,
distributed to 4 different boxes, when each box can have find the number of ways in which this can be arranged.
any number of balls? (a) 1220 (b) 2520 (c) 3250 (d) 7826
(a) 12 (b) 18 (c) 84 (d) 88 35 How many 7 digit telephone numbers can be formed
22 If a + b + c = 21, what is the total number of from the digits 0, 1, 2, …, 9, where each telephone
non-negative integral solutions? number begins with digit 2?
(a) 123 (b) 253 (c) 321 (d) 231 (a) 106 (b) 610 (c) 10! (d) 10 P6
23 If a + b + c = 21 what is the total number of positive 36 A man has 4 sons. There are 6 proposals of marriage for
integral solutions? his sons. In how many ways can they select a proposal
(a) 109 (b) 190 (c) 901 (d) 910 for their marriages, such that none of them marry with
24 What is the total number of ways of selecting atleast one more than one girl?
object from 2 sets of 12 different objects, each set (a) 180 (b) 270
contains 6 objects? (c) 360 (d) none of these
(a) 4096 (b) 4095 37 In how many ways can 4 books be selected out of
(c) 2048 (d) none of these 16 books on different subjects?
25 What is the total number of ways of selecting atleast (a) 1208 (b) 1820
one object from 2 different sets, each set containing (c) 1296 (d) 1860
6 identical objects? 38 In how many ways can 4 books be arranged out of
(a) 24 (b) 48 (c) 76 (d) 96 16 books on different subjects?
26 What is the total number of ways of selecting atleast one (a) 34650 (b) 43680
item from each of the two sets containing 6 different (c) 43890 (d) none of these
items each? 39 In how many ways can 16 books on different subjects be
(a) 2856 (b) 3969 divided equally into 4 groups?
(c) 480 (d) none of these 16 !
(a) (b) 4 ! × (16 !)
27 What is the total number of ways of selecting atleast one (4 !)
item from each of the two sets containing 6 identical 16 !
(c) (d) none of these
items each? (4 !)5
(a) 16 (b) 36 (c) 32 (d) 18
40 In a badminton tournament each player played one
28 A girl has to climb 12 steps. She climbs in either a single game with all the other players. How many players
step or 2 steps simultaneously. In how many ways can participated in the tournament if they played 105 games
she do it? in all?
(a) 233 (b) 223 (c) 322 (d) 232
(a) 35 (b) 12
29 In how many ways can 8 identical apples be divided (c) 15 (d) none of these
among 3 sisters?
41 There are 9 subjects in eighth standard but there are
(a) 25 (b) 65 (c) 45 (d) 24
only 6 periods in a day. In how many ways can the time
30 In how many ways can 100 soldiers be divided into 4 table be formed?
squads of 10, 20, 30, 40 respectively? (a) 60480 (b) 23460
(a) 1700 (b) 18! (c) 56780 (d) 61280
(c) 190 (d) none of these
42 How many 6 digit numbers can be formed using the
31 What is the total number of 4 digit numbers that can be digit 2 two times and the digit 4 four times?
formed using the digits 0, 1, 2, 3, 4, 5 without (a) 16 (b) 15
repetition, such that the number is divisible by 9? (c) 18 (d) 24
(a) 36 (b) 28
(c) 15 (d) 18 43 There are 4 different monitors and 6 different mother
boards. How many different arrangements can be made
32 How many 5 digit numbers contain exactly two 7’s in to purchase a monitor and a motherboard?
them? (a) 12 (b) 24
(a) 4268 (b) 6804 (c) 2340 (d) 1269 (c) 18 (d) 64
Permutations & Combinations 1071

44 The number of five digit numbers having atleast one of Directions (for Q. Nos. 56 to 63) : There are 5 different caps
their digits repeated is : c1 , c2 , c3 , c4 and c5 and 5 different boxes B1 , B2 , B3 , B4 and B5 .
(a) 900 (b) 1000 The capacity of each box is sufficient to accomodate all the 5 caps.
(c) 62784 (d) none of these 56 If any box can have any number of caps, in how many
45 Four dice are rolled. The number of possible outcomes in ways can all the caps be distributed?
which atleast one die shows 4 is : (a) 3125 (b) 1235 (c) 2358 (d) 1248
(a) 671 (b) 168 57 In how many arrangements does B1 have cap C1?
(c) 176 (d) none of these
(a) 5! (b) 54
46 The number of signals that can be generated by using 5 (c) 5 P4 (d) none of these
differently coloured flags, when any number of them
may be hoisted at a time is : 58 If all the caps are of different colours and each box can
(a) 235 (b) 253 have only one cap, in how many ways can you arrange
(c) 325 (d) none of these the caps among the 5 boxes?
(a) 120 (b) 180
47 If the letters of the word VERMA are arranged in all (c) 360 (d) none of these
possible ways and these words are written out as in a
dictionary, then the rank of the word VERMA is : 59 If atleast one cap has to be distributed and the caps have
(a) 108 (b) 117 to be arranged such that any box can have a maximum
(c) 810 (d) 180 of one cap only, in how many ways can you arrange the
caps among 5 boxes?
48 How many 5 digit numbers divisible by 3 can be formed (a) 1 (b) 16
using the digits 0, 1, 2, 3, 4 and 5, without repetition? (c) 1545 (d) none of these
(a) 108 (b) 216 (c) 810 (d) 180
60 If all the caps are identical, in how many ways can the
49 In how many ways can a mixed double game can be caps be arranged in the different boxes such that no box
arranged from amongst 8 married couples if no husband
is empty?
and wife play in the same game? (a) 1 (b) 2 (c) 6 (d) 8
(a) 840 (b) 240
(c) 480 (d) none of these 61 If C1 and C 5 are similar in all aspects, in how many ways
can you arrange the caps in such a way that all the boxes
50 If n objects are arranged in a row, then the number of
have one cap?
ways of selecting three of these objects so that no two of
(a) 70 (b) 60 (c) 75 (d) 80
them are next to each other is :
(a) nC 3 (b) n − 2C 3 62 If B 3 can have only C1 or C 5, in how many ways can you
n arrange the caps such that all boxes have one cap?
(c) C 2 (d) none of these
(a) 480 (b) 420 (c) 48 (d) 88
51 The number of times the digit 8 will be written when
listing the integers from 1 to 1000 is :
63 If B1 and B 5 have the caps C1 and C 5 among themselves,
(a) 100 (b) 200 (c) 300 (d) 400 in how many ways can you arrange the caps among the
5 boxes ?
52 Five balls of different colours are to be placed in three (a) 400 (b) 485 (c) 500 (d) 365
boxes of different sizes. Each box can hold all five balls.
64 Romeo and Juliet write love-letters to none but to each
The number of ways in which we can place the balls in
other. In a given period of time, Romeo writes 4 letters
the boxes so that no box remains empty is :
and Juliet writes 2 letters. During this period, at any
(a) 30 (b) 150 (c) 600 (d) 900
given point of time Romeo writes greater than or equal
53 The number of squares on a chessboard is : to the number of letters written by Juliet. Find the
(a) 102 (b) 108 (c) 216 (d) 204 number of ways of writing love-letters to each other.
54 A train going from Lucknow to Mumbai stops at 7 (a) 10 (b) 8 (c) 6 (d) 9
intermediate stations. Five persons enter the train during 65 Malvika wants to colour a cubical box from outside. In
the journey with five different tickets of the same class. how many ways can she colour it if she wants to colour
How many different set of tickets they could have had ? each face of the box with either blue or pink colour?
(a) 98280 (b) 2898 (a) 6 (b) 12 (c) 10 (d) 18
(c) 7325 (d) none of these 66 A gym opens twice a day – morning and evening. But, I
55 In how many ways can a committee of 4 women and don’t work out twice a day. I attend it only 4 times a
5 men be chosen from 9 women and 7 men, if Mr. A week. In how many ways can I attend the gym in a
refuses to serve on the committee if Ms. B is a member? week?
(a) 1608 (b) 1860 (a) 560 (b) 350
(c) 1680 (d) 1806 (c) 280 (d) none of these
1072 QUANTUM CAT
67 An n-digit number is a positive number with exactly 74 Find the total number of three digit numbers, which
n-digits. Nine hundred distinct n-digit numbers are to be have their digits in increasing order, as in, for a three
formed using only the three digits 2, 5 and 7. The digit number abc, there must be a < b < c.
smallest value of n for which this is possible is (a) 84 (b) 67 (c) 97 (d) 79
(a) 6 (b) 7 (c) 8 (d) 9 75 There are three straight lines dividing the circle into
68 Which of the following is not a possible number of maximum possible number of regions. If three additional
regions into which three straight lines, of infinite extent, lines are drawn, what is the maximum possible number
divide a plane? of additional regions that can be formed in the circle?
(a) 6 (b) 9
(a) 4 (b) 5 (c) 6 (d) 7
(c) 14 (d) none of these
69 If you travel between Mumbai and Pune, you have to go
76 Spiritual Guru H. E. Dalai Lama is on a world tour to
through a 10 km long tunnel. Lately there has been a
surge in the incidents of loot and plunder and many speak on peace and compassion. His itinerary mentions
passengers have been harassed and injured inside the only four continents – Asia, Africa, Europe and America
tunnel. Considering the enormity of the law and order – for this visit. What is the minimum number of
problem, highway police decided to establish 4 security countries he should visit to ensure that at least 21 Asian
posts in order to ensure the safety of life and luggage of or at least 16 African or at least 11 European or at least
commuters. The minimum distance between any two 6 American countries must be visited?
posts is 1 km. Each security post must be n km (a) 24 (b) 51 (c) 54 (d) 57
(n = 0, 1, 2, 3 . . . ) apart from entry/exit of the tunnel and 77 Arya, Bran, Cersei, Daenerys, Petyr and Tyrion are
none of them will be outside the tunnel. Find the seated in six chairs placed in a row. Maximum how
number of ways of selecting the spots for upcoming many seating arrangements are possible if Arya must sit
security posts. next to Bran, but Petyr cannot sit next to Tyrion?
(a) 330 (b) 110 (c) 66 (d) 120 (a) 98 (b) 196 (c) 144 (d) 256
70 A rectangle with sides (2m − 1) and (2n − 1) is divided 78 How many 6 digit telephone numbers can be formed if
into squares of unit length by drawing parallel lines as each number starts with 91 and sum of the digits is
shown in the diagram, then the number of rectangles equal to 16?
possible with odd side lengths is (a) 210 (b) 336 (c) 168 (d) 84
(a) (m + n + 1)2 (b) 4( m + n − 1)
79 Jwala Gutta and P. V. Sindhu play a one-day series.
2 2
(c) m n (d) mn(m + 1)(n + )
They stop playing matches as soon as one of them wins
71 A black ant has to go from the origin to a point (6, 4) on 3 matches and in this series no match is drawn. In how
the Cartesian plane, using the coordinate axes. many ways can this series be won?
However, it avoids crossing the point (4, 1), as a red ant (a) 20 (b) 24 (c) 12 (d) 6
is sitting there. Find the total number of shortest paths
80 Let there be a random number n such that the product of
that it can go along.
its digits is 6. How many values of n are there, if n lies
(a) 150 (b) 160 (c) 190 (d) 240 between 109 and 1010?
72 The number of seven digit integers, with sum of digits (a) 10 (b) 60 (c) 90 (d) 100
equal to 10 and formed by using the digits 1, 2, and 3 81 Sarvesh wants to prepare a bouquet with roses, lilies,
only, is
orchids and carnations, so he walked into a large garden
(a) 55 (b) 66 (c) 77 (d) 88
of flowers. In how many ways can he prepare a bouquet
73 Find the maximum possible number of regions created of 15 flowers such that he selects at least 1 rose, 2 lilies,
by 6 overlapping triangles. 3 orchids and 4 carnations for his bouquet?
(a) 36 (b) 63 (c) 92 (d) 96 (a) 24 (b) 120 (c) 56 (d) 112
LEVEL 02 > HIGHER LEVEL EXERCISE
1 In how many ways can the following prizes be given 8 In the previous question, how many seating arrangements
away to a class of 30 students, first and second in are possible if 3 girls sit together in a back row on adjacent
Mathematics, first and second in Physics, first in seats?
Chemistry and first in English? (a) (3!)2 (b) 6!
30 ! (c) 5! (d) none of these
(a)
4!
9 In how many ways 5 plus (+ ) signs and 5 minus (–) signs
(b) (30)4 × (29)2
be arranged in a row so that no two minus signs are
(c) (30)3 − 1 together?
(d) (30)4 × (29)4 (a) 6 (b) 7 (c) 8 (d) 10
2 If Pr stands for r Pr , then the value of 10 In how many ways can 11 identical books on English
1 + 1. P1 + 2 . P2 + 3 . P3 + K + n . Pn is : and 9 identical books on Maths be placed in a row on a
(n − 1)! (n + 1)! shelf so that two books on Maths may not be together?
(a) (b) (a) 110 (b) 220 (c) 330 (d) 440
2 2
(c) 2 (n − 1)! (d) (n + 1)! 11 Find the total number of factors of 1680.
(a) 40 (b) 50 (c) 60 (d) 30
3 In an examination hall there are four rows of chairs.
n
Each row has 6 chairs one behind the other. There are 12 Σ k
C r equals :
two classes sitting for the examination with 12 students k=m
n+ 1 n+ 1
in each class. It is desired that in each row, all students (a) Cr −1 (b) C r + 1 − mC r
n+ 1 n+ 1
belong to the same class and that no two adjacent rows (c) C r + 1 − mC r + 1 (d) C r + 1 + mC r + 1
are alloted to the same class. In how many ways can
these 24 students be seated? 13 In a certain test there are n questions. In this test 2n − k
(a) 2 (12!)2 (b) 3! × (12!)2 students gave wrong answers to atleast k questions,
where k = 1, 2, 3, … , n. If the total number of wrong
(c) (6 !)2 (d) none of these
answers given is 2047, then n is equal to :
4 How many 3 digit even numbers are there such that if 3 (a) 10 (b) 11 (c) 12 (d) 13
is one of the digits then 9 is the next digit? 14 The number of triangles whose vertices are at the
(a) 365 (b) 536 vertices of an octagon but none of the sides of such
(c) 210 (d) 156 triangles are taken from the sides of the octagon.
5 A number plate of a vehicle has always a fixed code (a) 8 (b) 15
UP–32 for Lucknow city followed by the number of (c) 16 (d) none of these
particular vehicle which is in two parts. First part is 15 The number of ways in which we can select 5 numbers
occupied by 2 English alphabets and second part is
from the set of numbers {1, 2, 3, …, 25} such that none
occupied by 4 digit numbers (0001, 0002, … 9999). If
of the selections includes four consecutive numbers is :
UP − 32 
the latest registration number of vehicle is  , (a) 53109 (b) 13350
SK − 0123 (c) 10035 (d) none of these
find the number of vehicles registered before this vehicle
number in Lucknow.
16 The number of integral solutions for the equation
(a) 2449744 (b) 4779644 a + b + c + d = 12, where (a, b, c, d ) ≥ − 1 is :
(c) 4669235 (d) 9235888222 (a) 19C 3 (b) 18C 4
20
6 How many 4 digit numbers divisible by 4 can be formed (c) C4 (d) none of these
without using the digits 0, 6, 7, 8, 9 if the repetition of 17 If n = kC 2, the value of nC 2 is :
digits is not allowed? (a) 2 (k + 2C 4 ) (b) (n − 2)! (c) (k − 2)! (d) 3 (k + 3C 4 )
(a) 67 (b) 68
(c) 24 (d) 48 18 The number of positive integral solutions of abc = 42 is :
(a) 17 (b) 27 (c) 21 (d) 3! × 42
7 In how many ways three girls and nine boys can be
seated in two vans, each having numbered seats, 3 in the 19 There are three piles of identical red, green and blue
front and 4 at the back? balls and each pile contains atleast 10 balls. The number
(a) 14C12 (b) 94 of ways of selecting 10 balls if twice as many red balls as
(c) 14 P12 (d) none of these green balls are to be selected is :
(a) 1 (b) 2 (c) 4 (d) 6
1074 QUANTUM CAT
20 Between Shirdee and Khandala, there are 10 intermediate 32 There are 10 lamps in a hall. Each one of them can be
stations. The number of ways in which a train can be made switched on independently. The number of ways in
to stop at 4 of these stations so that no two of these halting which the hall can be illuminated is :
stations are consecutive is : (a) 1022 (b) 1023
(a) 35 (b) 70 (c) 105 (d) 10 (c) 1024 (d) none of these
21 The number of ways in which an examiner can assign 33 How many 10 digits numbers can be formed by using
30 marks to 8 questions, giving not less than 2 marks to the digits 2 and 3?
any question is : (a) 210 (b) 102
(a) 11 (b) 21C7 (c) 18 (d) 235 (c) 10! (d) none of these
22 The exponent of 3 in 33! is : 34 How many subsets containing at most n elements from
(a) 15 (b) 2 the set of (2n + 1) elements can be selected?
(c) 11 (d) none of these (a) 2n (b) 2n − 1
n+ 1
(c) 2 (d) 22n
23 The total number of natural numbers of 6 digits that can
be formed with digits 1, 2, 3, 4, if all the digits are to 35 The sum of the divisors of 23 ⋅ 34 ⋅ 52 is :
appear in the same number atleast once, is : (a) 15625 (b) 1234
(a) 740 (b) 1830 (c) 56265 (d) 56789
(c) 1560 (d) none of these 36 The number of ways of selecting 10 balls out of an
24 The total number of seven digit numbers, the sum of unlimited number of white, red, blue and green balls is :
whose digits is even are : (a) 268 (b) 286 (c) 246 (d) 468
(a) 250000 (b) 4500000 37 If 10 objects are arranged in a row, then the number of
(c) 35 × 105 (d) none of these ways of selecting three of these objects so that no two of
them are next to each other is :
25 The number of ways in which 4 pictures can be hung (a) 56 (b) 65 (c) 28 (d) 13
from 6 picture nails on the wall is :
38 The number of non-negative integral solutions of
(a) 46 (b) 4 P6 (c) 6 P4 (d) 64
x1 + x 2 + x 3 + x 4 ≤ n (where n is a positive integer) is :
26 The total number of ways of selecting 6 coins out of n+ 3 n+ 2 n+ 4 n+ 4
(a) C3 (b) C3 (c) C4 (d) C3
10 one rupee coins, 6 fifty paise coins and 8 twenty
paise coins is : 39 The total number of permutations of (n > 1) different
(a) 28 (b) 14 (c) 13 (d) 19 things taken not more than r at a time, when each thing
27 Nargis has 8 children and she takes 3 at a time to may be repeated any number of times is :
nr − 1
children’s park as often as she can without taking the (a) (b) rn − 1
same 3 children together more than once. The number (n − 1)
of times she will go to the park is : n (nr − 1)
(c) (d) none of these
(a) 56 (b) 14 (c) 28 (d) 76 (n − 1)
28 In the previous problem (no. 27), the number of times 40 Number of rectangles on a chessboard is :
each child will go to the park is: (a) 1008 (b) 1296 (c) 1124 (d) 1600
(a) 14 (b) 8
41 Ten persons, amongst whom A, B and C are to speak at
(c) 21 (d) none of these
a function. The number of ways in which it can be done
29 The sides AB, BC, CA of a triangle ABC have 3, 4 and 5 if A wants to speak before B and B wants to speak before
interior points respectively on them. The total number of C is :
triangles that can be constructed by using these points as 10 ! 9!
vertices is : (a) (b)
6 6
(a) 105 (b) 307 (c) 420 (d) 205 (c) 9C 6 (d) none of these
30 The number of all the possible selections which a
42 The number of natural numbers which are smaller than
student can make for answering one or more questions
2⋅ 108 and which can be written by means of the digits
out of 10 given questions in a paper, when each question
1 and 2 is :
has an alternative is :
(a) 678 (b) 786 (c) 766 (d) 677
(a) 1345 (b) 23560 (c) 541340 (d) 59048
31 The number of ways in which a team of eleven players 43 Tweleve persons are arranged in a row. The number of
can be selected from 22 players including 2 of them and ways of selecting four persons so that no two persons
excluding 4 of them is : sitting next to each other are selected is :
(a) 15C10 (b) 16C10 (a) 124 (b) 136
(c) 126 (d) none of these
(c) 16C 9 (d) none of these
Permutations & Combinations 1075

44 The number of n-bit (digits 0 and 1) strings having 53 In how many ways we can select four cards of different
exactly k zeros with no two zeros consecutive is, where suits and different values from a deck of 52 cards?
2k < n. 13!
(a) 10! (b)
(a) n + 1 − k C k (b) n + k C k 10 !
n− k n−1 13!
(c) Ck (d) Ck (c) (d) none of these
9!
45 A question paper has two parts–Part A and Part B. Part A
contains 5 questions and part B has 4. Each question in 54 How many 5 digit numbers can be formed having the
part A has an alternative. A student has to attempt digits 0 three times and 3 two times?
atleast one question from each part. Find the number of (a) 4 (b) 6 (c) 8 (d) 10
ways in which the student can attempt the question 55 How many different four digit numbers can be formed
paper. with the digits 1, 2, 3, 4, 5, 6, 7, 8 and 9 such that the
(a) 3360 (b) 1258 (c) 3850 (d) 3630 digit 5 appears exactly once?
46 How many different vehicle licence plates can be made if (a) 1024 (b) 2048
the licences contain 2 letters of the English alphabet (c) 4096 (d) none of these
followed by a three digit number if repetitions are 56 How many different four digit numbers can be formed
allowed? using digits 1, 2, 3, 4, 5, 6, 7, 8, 9 without repetition
(a) 54320 (b) 67534 (c) 23456 (d) 675324 such that the digit 5 appears exactly once?
47 In the previous question (no. 46), how many different (a) 1243 (b) 1234 (c) 1344 (d) 1355
plates can be formed if the repetitions are not allowed? 57 How many different eight digit numbers can be formed
(a) 468000 (b) 13000 using only four digits 1, 2, 3, 4 such that the digit
(c) 15680 (d) none of these 2 occurs twice?
48 2m people are arranged along two sides of a long table (a) 20412 (b) 12042
with m chairs each side. r men wish to sit on one (c) 25065 (d) none of these
particular side and s on the other. In how many ways 58 In how many ways can 8 distinct things be divided
can they be seated? (r, s ≤ m) among three people such that any one can receive any
(a) 48C r (b) 68 m number of things?
(2m + r)! (a) 1556 (b) 6561 (c) 8C 2 (d) 7 C 2
(c) (d) none of these
r !⋅ s !
59 There are 6 numbered chairs placed around a circular
49 Management city has m parallel roads running East-West table. 3 boys and 3 girls want to sit on them such that
and n parallel roads running North-South. How many neither of two boys nor two girls sit adjacent to each
shortest possible routes are possible to go from one other. How many such arrangements are possible?
corner of the city to its diagonally opposite corner? (a) 36 (b) 58
(a) m + nC m − 1 (b) m + nC n (c) 72 (d) none of these
(c) ( m + n− 2)C( m − 1) (d) none of these 60 In how many ways one black and one white rook can be
50 Priyanka has 11 different toys and Supriya has 8 placed on a chessboard so that they are never in an
different toys. Find the number of ways in which they attacking position?
can exchange their toys so that each keeps her initial (a) 1234 (b) 3136 (c) 9516 (d) 1024
number of toys. 61 There are 9 pairs of white shoes and 6 pairs of black
(a) 19C11 (b) 18C10 (c) 20C11 (d) 19C11 − 1 shoes contained in a box. We are allowed to draw only
th
one shoe at a time. Minimum how many shoes are
51 The sum of the numbers of the n term of the series
required to be drawn out to get one pair of white shoes?
(1) + (1 + 2) + (1 + 2 + 3) + (1 + 2 + 3 + 4) + …
(a) 11 (b) 22 (c) 33 (d) 15
(1 + 2 + 3 + … n) :
n+ 1 n+ 1 n+ 2 62 In the previous question, minimum how many shoes
(a) C3 (b) C2 (c) nC 2 (d) C3
must be drawn out to get at least 1 pair of either black
52 Two packs of 52 playing cards are shuffled together. or white shoes?
Find the number of ways in which a man can be dealt (a) 15 (b) 18 (c) 16 (d) 22
26 cards so that he does not get two cards of the same 63 There are four different coloured balls and four boxes of
suit and same denomination. the same colour as that of each ball is. Find the number
52! of ways in which exactly one ball can be put in a box so
(a) 52C 26 × 226 (b)
(2!)26 that the colour of the box and that of ball is distinct.
(c) 226 (d) none of these (a) 16 (b) 10
(c) 4C 2 (d) none of these
1076 QUANTUM CAT
64 In the previous question (no. 63) find the number of Directions (for Q. Nos. 75 to 82) : There are 3 pots and 4 coins.
ways in which only two balls can be put in the correct All these coins are to be distributed into these pots where any pot
boxes i.e., the colour of box and the colour of contained can contain any number of coins.
ball be same. 75 In how many ways these coins can be distributed if all
(a) 3 (b) 6 the coins and all the pots are different?
(c) 9 (d) none of these (a) 34 (b) 43 (c) 3 P4 (d) 4 P3
65 Find the number of digits required to write down the 76 In how many ways these coins can be distributed if all
number of pages in a 300 pages book. the coins and all the pots are identical?
(a) 792 (b) 279 (a) 4 (b) 6 (c) 8 (d) 1
(c) 729 (d) none of these
77 In how many ways all these coins can be distributed if
66 Find the number of numbers that can be formed using all coins are identical but all pots are different?
all the digits 1, 2, 3, 4, 3, 2, 1 only once so that the odd (a) 15 (b) 16 (c) 17 (d) 81
digits occupy odd places only. 78 In how many ways all these coins can be distributed if
(a) 9 (b) 16 (c) 18 (d) 27 all coins are different but all pots are identical?
Directions (for Q. Nos. 67 to 74) : There are 5 different boxes (a) 14 (b) 21
and 7 different balls. All the 7 balls are to be distributed in the 5 (c) 27 (d) none of these
boxes placed in a row so that any box can receive any number of 79 In how many ways all these coins can be distributed
balls. such that no pot is empty if all coins are different but all
pots are identical?
67 In how many ways can these balls be distributed into
(a) 16 (b) 6 (c) 42 (d) 21
these boxes?
(a) 57 (b) 7 5 (c) 7 C 5 (d) 7 P5 80 In how many ways all these coins can be distributed
such that no pot is empty if all coins are identical but all
68 In how many ways can these balls be distributed so that pots are different?
no box is empty? (a) 6 (b) 3 (c) 9 (d) 27
(a) 7! (b) 16800
81 In how many ways all these coins can be distributed if
(c) 1775 (d) none of these all coins are identical and two pots are also identical?
69 Suppose all the balls are identical, then in how many (a) 1 (b) 10 (c) 9 (d) 11
ways can all these balls be distributed into these boxes? 82 In how many ways all these coins can be distributed if out
(a) 110 (b) 220 (c) 330 (d) 440 of 4 coins 2 coins are identical and all pots are different?
70 Suppose all the balls and all the boxes are also identical. (a) 45 (b) 27
Then in how many ways can all these balls be (c) 54 (d) none of these
distributed into these boxes? 83 Once Munnabhai gets admitted to a psychiatric hospital.
(a) 12 (b) 13 (c) 16 (d) 255 In his room, there are two bulbs connected to two
71 In the previous question (no. 70) in how many ways can different switches, independently. One night, while
all these balls be distributed into these boxes so that no accompanying him, his best buddy Circuit notices that
box remains empty and no two consecutive boxes have there is, initially, no light in the room, but whenever a
the same number of balls? mosquito bites him he switches on the light and then
(a) 1 (b) 4 immediately switches it off. Throughout the night,
(c) 12 (d) none of these Munnabhai presses the switch (on/off) six times. Finally
when Munnabhai stops playing around with the
72 In how many ways can these balls be distributed so that
switches, Circuit notices that there is no light in the
box 2 and box 4 contain only 1 and 2 balls respectively?
room. In how many ways Munnabhai ends up having no
(a) 5522 (b) 8505
light in his room by pressing the given switches on or off
(c) 2305 (d) none of these
exactly six times?
73 In how many ways can these balls be distributed into (a) 3 (b) 4 (c) 6 (d) 8
these boxes if ball 2 can be put into either box 2 or box
84 Mr. Been, who is getting older and senile, is standing on
4?
a crossroad totally confused about his directions. He can
(a) 12360 (b) 31250 (c) 13490 (d) 31526
move in any of the four directions – North, South, East
74 In how many ways can these balls be distributed such or West. He takes some steps and then comes back to
that no box is empty and ball 2 and ball 4 cannot be put the same point at the crossroad. In how many ways can
in the same box? he end up being at the same point where he is initially
(a) 1200 (b) 15000 standing by taking total 8 steps?
(c) 3800 (d) none of these (a) 4900 (b) 6400 (c) 360 (d) 238
Permutations & Combinations 1077

85 Indian Democratic Alliance (IDA) is a coalition of three 91 If the rings are distinct, find the number of ways of
distinct regional parties. IDA decides to contest on distributing these rings among his 5 daughters such that
100 seats for the upcoming parliamentary elections. IDA elder the daughter fewer the rings she gets. However
wants to share these seats in such a way that no any ally none of them gets less than 1 and more than 5.
gets the same number of seats to contest and no two 15! 15!
(a) (b)
allies contest for the same seat. Also each of the allies 3! × (5!)2 (2!)5 × (5!)2
must get at least 1 seat to contest. Find the number of 15!
ways of allocating the number of seats for the three (c) (d) none of these
(2!) × (3!)2 × 5!
2
constituents of the IDA?
(a) 4704 (b) 4851 (c) 4884 (d) 3667 92 If the rings are identical, find the number of ways of
distributing these rings among his 5 daughters such that
86 Bill Gates personally announced that he would be elder the daughter fewer the rings she gets. However
building 49 toilets in 3 select Indian villages A, B and C, none of them gets less than 1 and more than 5.
as part of the pilot project. In how many ways these
(a) 180 (b) 150 (c) 75 (d) 6
toilets can be built in 3 villages such that village A gets
93 Find the number of positive integral solutions of the
more toilets than that of village B and village B gets
equation x1 x 2 x 3 x 4 = 462.
more toilets than that of village C?
(a) 128 (b) 1024 (c) 256 (d) 64
(a) 98 (b) 196
(c) 188 (d) none of these 94 Find the number of integral solutions of the equation
x1 x 2 x 3 x 4 = 462.
87 H2O is a chain of salons, employing one hair stylist at
(a) 2048 (b) 256 (c) 24 (d) 1024
each salon. It has got centralized system in place to
provide appointments through phone calls for all its 6 95 Rumour has it that Runnbeer, an alpha male, who easily
salons located in different areas in Pune. A family of four gets to date the hometown hotties at the drop of his hat.
seeking the appointment for all its members for the hair Of late, he has been very promiscuous and ditched his
cut. On a particular day, in how many ways the salons two girls, who stay in the same city at two distinct
can give the appointments to them? locations Powai (P) and Qababchowk (Q).
(a) 3024 (b) 2524 (c) 3384 (d) 3084 D (0, 6) C(10, 6)

88 A natural number N is such that 10 10


< N < 10
12
and the
sum of its digits is 3. Find the total number of values of N.
(a) 121 (b) 111 (c) 222 (d) 144 Q (8, 4)

89 A spiritual organization, working for the world peace, P (4, 3)

has inducted 32 young volunteers none of whom knew


each other before joining this organization. During the
induction program they interacted with each other. Next A (0, 0) B(10, 0)
day it was observed that most of the volunteers became Today, Runnbeer has got a new date and so he wants to take his
friends on the Facebook, but one-fifth of the girls denied date from her apartment at Arthur road ( A) to a restaurant at
becoming Facebook friend with one-third of the boys. Churchgate (C). However, he does not want to go through the
Considering the natural Facebook friendship among places P and Q as his deserted girls, Priyankhan and Qatrinah,
some of the volunteers, the head of the organization stay at these two places only. Find the total number of the
asked them to visit the various regions of the world and shortest paths that he can follow to reach C from A, considering
spread the message of the world peace. She told them the roadmap given in the diagram.
that a region would have to be visited by exactly (a) 3298 (b) 6058 (c) 3880 (d) 3148
2 volunteers who are the Facebook friends and no any
96 Sixty delegates from three different countries
such pair would have to visit more than one region.
Which is the possible number of regions that these participated in a recent Disaster Management Conclave.
Each such country is divided into four states and each
volunteers could have visited?
such state is further divided into five cities. Each
(a) 525 (b) 496 (c) 421 (d) 487
delegate was representing a different city. In that
90 Find the total number of non-negative integers less than conclave any two delegates shook hands with each other
1000 for which the sum of the digits is 10. exactly two times. However, if any two delegates are
(a) 63 (b) 66 (c) 67 (d) 56 from the same country they shook hands five times and
if any two delegates are from the same state they shook
Directions (for Q. Nos. 91 and 92) : Hirabhai after marrying their hands nine times. What was the total number of
Hiraben settled in Hiranandini, Mumbai. He has 15 diamond handshakes amongst the delegates?
rings and 5 daughters. Among his 5 daughters, he has only one (a) 1200 (b) 5730 (c) 7370 (d) 3540
daughter who is not twins.
1078 QUANTUM CAT
97 DHL is a leading logistics company in India. In order to incentives. One day, he performs seven online
optimize its resources it has virtually divided the whole transactions, and in every transaction he empties his one
market region into four zones and each zone is again wallet completely into another wallet. After seven
divided into five blocks. There are two dedicated trucks transactions he has ` 100 in PayPal wallet. During this
of DHL carrying the goods between any two blocks procedure neither he pays any amount to nor receives
within each zone. Also there are three dedicated trucks any amount from any third party by any means. In how
carrying the goods between any two blocks of different many different ways can he make these transactions?
zones. Exactly, how many trucks of DHL are engaged in (a) 14 (b) 21 (c) 42 (d) 49
the stated market? 102 Kamlesh, the sulochan boy, has 100 identical
(a) 72 (b) 660 (c) 530 (d) 484 matchsticks. How many distinct rectangles can he form
98 There are 6 red and 4 green points on the circumference using at most 100 matchsticks?
of a circle. How many convex polygons can be formed (a) 1225 (b) 625
such that all the vertices of the polygon must be from (c) 25 (d) none of these
these ten points only and having at least one green 103 Two teams – Anadi and Khiladi – are playing a cricket
vertex? match. Anadi requires 12 runs in the last 3 balls to win
(a) 1226 (b) 530 (c) 730 (d) 926 the match. Any team can score 0 to 6 runs, except 5
99 If m and n are relatively prime and mn = 10 !, find the runs, from a particular ball. In how many different ways
Team Anadi can score 12 runs without any no-ball and
m m
number of all rational numbers such that 0 < < 1. wide-ball in the last 3 balls?
n n
(a) 11 (b) 12
(a) 8 (b) 15
(c) 13 (d) none of these
(c) 30 (d) 21
104 Each employee in our office at Lamamia must wear a
100 How many distinct triangles are there in this figure? shirt, a tie and a pair of pants to dress up oneself. Thus,
any employee in our office can wear exactly 105 distinct
combinations with the help of different colours. No two
employees have equal number of quantities of all the
three things individually. That is no two employees have
x shirts, y ties and z pairs of pants. No two employees
have the same colours of their clothes. Maximum how
many employees are there in our office?
(a) 32 (b) 66
(c) 48 (d) 64 (a) 24 (b) 27 (c) 35 (d) 105
105 How many 8 digit numbers can be formed with the
101 Sarvesh Verma has 3 mobile wallets – PayPal, PayTM
digits 1, 2, 3, 4, 5, 6, 7, 8 such that no digit repeats and
and PayU – installed in his smartphone. Initially, he has none of the patterns 12, 34, 56, or 78 appears in any
only ` 100 in PayPal wallet and other two wallets have arrangement?
nothing. He can transfer his money from one wallet to (a) 24096 (b) 24024
any other wallet without any transaction charges or any (c) 16296 (d) none of these

LEVEL 03 > Final Round


1 Maximum number of points of intersection of 6 circles, is 5 Two straight lines intersect at a point O. Points
(a) 30 (b) 28 A1, A2, A3, A4, A5, …, Am are taken on one line and
(c) 15 (d) none of these points B1, B 2, B 3, …, B n on the other. If the point O is not
2 Maximum number of points of intersection of 6 straight included, the number of triangles that can be drawn
lines is :
using these points as vertices, is :
(a) 30 (b) 15
(c) 28 (d) none of these (a) nC 2 + mC 2 (b) 2nC 2
m+ n
3 Maximum number of points into which 3 circles and 3 (c) C2 (d) none of these
lines intersect is :
6 How many different nine digit numbers can be formed
(a) 21 (b) 9 (c) 27 (d) 3!
from the number 22 33 55 888 by rearranging its digits
4 Eight identical coins are arranged in a row. The total
so that the odd digits occupy even positions?
number of ways in which the number of heads is equal to
the number of tails, is : (a) 60 (b) 75
(a) 35 (b) 15 (c) 140 (d) 70 (c) 88 (d) 77
Permutations & Combinations 1079

7 The straight lines l1, l2, l3 are parallel and lie in the 17 In how many ways a cricketer can make a century with
same plane. A total number of m points on l1, n points on fours and sixes only?
(a) 6 (b) 9
l2, k points on l3 are used to produce the triangles, the
(c) 8 (d) 10
maximum number of triangles formed with vertices at
these points are : 18 The number of permutations of the letters of the word
(a) mC 3 × nC 3 × kC 3 LUMINARY such that neither the pattern LURY nor
(m + n + k ) MINA occurs is :
(b) C3 − ( C3 + C3 + C3)
m n k
(a) 46800 (b) 24600
(m + n + k )
(c) C3 (c) 40086 (d) none of these
(d) none of the above 19 10 students are to be seated in two rows equally for the
8 Given that n is the odd, the number of ways in which MOCK CAT in a room. There are two sets of papers,
three numbers in AP can be selected from 1, 2, 3, 4, …, code A and code B. Each of the two rows can have only
n is : one set of paper but different that from the other row. In
(n − 1)2 how many ways these students can be arranged?
(a) (b) n2 (c) n3 (d) (n − 2)2
4 (a) 2775600 (b) 1200560
9 Eight straight lines are drawn in the plane such that no (c) 125600 (d) 7257600
two lines are parallel and no three lines are concurrent. 20 Aman Verma and Mini Mathur jointly host a TV
The number of parts into which these lines divide the programme in which one particular day n guests attend
plane, is : their show. In that show, each guest shakes hands with
(a) 73 (b) 37 (c) 17 (d) 72 every other guest and each guest shakes hands with each
10 Number of divisors of the form 4n + 2 (n ≥ 0) of the of the hosts. If there happens to be total 65 handshakes,
integer 240 is : find the number of guests who attend the show.
(a) 6 (b) 4 (c) 3 (d) 12 (a) 13 (b) 14
(c) 10 (d) 9
11 The number of rectangles excluding squares from a
21 The number of ways in which an examiner can assign
rectangle of size of 12 × 8 is :
50 marks to 10 questions giving not less than 3 marks to
(a) 1234 (b) 625
any question is :
(c) 2460 (d) 256 29 47
(a) C9 (b) C3
12 Two lines intersect at O. Points A1, A2, … , An are taken 52 40
(c) C2 (d) C10
on one of them and B1, B 2, … , B n on the other the
number of triangles that can be drawn with the help of 22 For x ∈ R , let [x] denote the largest integer less than or
these (2n + 1) points is : equal to x, then the value of
(a) n (b) n2 (c) n3 (d) n4
1  1 1  1 2  1 99 
E= + + + + +…+ + is :
13 The number of ways in which 9 identical balls can be  3  3 100   3 100   3 100 
placed in three identical boxes is :
9! (a) 22 (b) 66
(a) 12 (b) 6 (c) 9 (d) (c) 33 (d) none of these
3!
23 The number of positive integers from 1 to 106 (both
14 The number of ways in which 30 coins of one rupee each
inclusive) which are neither perfect squares, nor cubes,
be given to six persons so that none of them receives less
nor perfect fourth powers is :
than 4 rupees is :
(a) 918990 (b) 998911
(a) 246 (b) 462
30 ! (c) 998910 (d) none of these
(c) (d) none of these
6! 24 Kumar’s family consists of a grandfather, ‘m’ sons and
daughters and ‘2n’ grand children. They are to be seated
15 Find the number of circles that can be drawn out of
in a row for dinner. The grandchildren wish to occupy
10 points, of which 7 points are collinear is :
the n seats at each end and the grandfather refuses to
(a) 35 (b) 85
have a grandchild on either side of him. In how many
(c) 70 (d) 124
ways can the Kumar’s family be made to sit?
16 The number of natural numbers of two or more than two (a) (2n !)(m − 1)!
digits in which digits from left to right are in increasing
(b) (2n !)(m !)(m − 1)
order is :
(c) (2m !)(n !)
(a) 205 (b) 520
(d) none of the above
(c) 502 (d) none of these
1080 QUANTUM CAT
25 Six Ps have to be placed in the squares of 34 Find the number of ways of putting five distinct rings on
the diagram given below such that each four fingers of the left hand. (Ignore the difference of
column contains at least one P. In how size of rings and the fingers)
many ways this can be done? (a) 1250 (b) 6720
(a) 26 (b) 13 (c) 5260 (d) none of these
(c) 8 (d) 24 35 If n distinct things are arranged in a circle, find the
26 A box contains two red, three green and four blue balls. number of ways of selecting three of these things so that
In how many ways can three balls be drawn from the no two of them are next to each other.
box if atleast one green ball is to be included in the 1
(a) n (n − 2)(n − 3) (b) n (n − 4)(n − 5)
draw? 6
(a) 23 (b) 64 1 2
(c) (n + 3n − 5) (d) none of these
(c) 46 (d) none of these 3
27 Ajay has 7 relatives, 3 men and 4 women. His wife Kajol 36 The number of different selections of 5 letters from 1A,
also has 7 relatives, 3 women and 4 men. In how many 2 B’s, 3C’s, 4D’s and 5 E’s is :
ways can they invite 3 men and 3 women so that 3 of (a) 71 (b) 41
them are the Ajay’s relatives and 3 his wife Kajol’s? (c) 117 (d) none of these
(a) 485 (b) 458 37 In the entrance test of Wharton Business School, the
(c) 365 (d) none of these maximum marks for each of three papers is n and that
28 Serena and Venus were only two women participating in for the fourth paper is 2n. Find the number of ways in
a chess tournament. Every participant played two games which a candidate can get 3n marks.
with every other participant. The number of games that 1 1
(a) (n + 1)(n + 2) (b) (n + 1)(5n2 + 10n + 6)
men played between themselves proved to exceed by 66, 6 6
compared to the number of games the men played with 1 2
(c) (n + 3n − 6) (d) none of these
women. How many participants were there? 2
(a) 156 (b) 610 (c) 13 (d) 108
38 An examination consists of 4 papers. Each paper has a
29 In the previous questions (no. 28) how many games maximum of ‘n’ marks. Find the number of ways in
were played? which a students can get 2n marks in the examination.
(a) 208 (b) 156 1 1
(a) (n2 − 5n + 4) (b) (n + 1)(2n2 + 4n + 3)
(c) 316 (d) none of these 3 3
1
30 Six Burfis and six rasgullas are to be distributed among (c) (n + 1)(n + 4) (d) none of these
ten girls so that each girl receives at least one piece of 6
sweets out of burfis and rasgullas. Find the number of 39 Mark Shekhar Garg goes to a school to demonstrate how
ways in which the sweets can be distributed. the kids can use the Facebook harmlessly and take the
(a) 26250 (b) 22560 optimum advantage of socialization with fellow kids
(c) 22330 (d) none of these online at the early age of their social being. He tells the
31 Mr. John has x children by his first wife and Ms. Bashu has kids, after demonstrating the usage and safety features,
x + 1 children by her first husband. They marry and have that to make someone your facebook friend you have to
children of their own. The whole family has 10 children. send her/him a request and the other person has to
Assuming that two children of the same parents do not accept it; as in out of two classmates A and B if first A
fight, find the maximum number of fights that can take sends request to B then B will accept it or if first B sends
place among children.
request to A then A will accept it and thus both A and B
(a) 33 (b) 22
will become facebook friends.
(c) 111 (d) none of these
As every kid responds to his instructions, within a
32 Find the number of natural numbers which are smaller
minute Mark finds that every student in the class is now
than 2⋅ 10 and are divisible by 3 which can be written
8
facebook friend of the other. In that class the number of
by means of the digits 0, 1 and 2.
female students is twice that of male students. He also
(a) 7272 (b) 4373
notices that the number of friend-requests sent by the
(c) 3437 (d) none of these
female kids is equal to the number of friend-requests
33 Find the number of whole numbers formed on the sent by the male kids. Which of the following can be the
screen of a calculator which can be recognised as number of friend-requests sent by female students to
numbers with (unique) correct digits when they are read male students of that class?
inverted. The greatest number that can be formed on the (a) 42 (b) 99
screen of the calculator is 999999. (c) 105 (d) 120
(a) 98970 (b) 89912 (c) 110050 (d) 100843
Permutations & Combinations 1081

40 A king calls 11 masseurs for his queen, as she wants the two times a day. However, any two adjoining
body massage from Monday to Friday, while on restaurants do not give change to each other at all. On a
weekends she wants to go on the long trips with the certain day at most how many times the change can be
king. She wants everyday a new masseur, so basically given to each other among all the restaurants on the roof
5 masseurs are required. But a confidant of the king tells top?
him that out of 11 masseurs 5 are familiar with the (a) 3970 (b) 5080 (c) 4370 (d) 6750
queen before her marriage and she used to get very close
45 Find the total number of quadratic polynomial
to them. But the king has no idea of these 5 people who
ax 2 + bx + c, if a, b, c are some three positive distinct
are familiar with the queen. In spite of knowing this very
odd fact he takes the risk of telling the masseurs that integers less than 2000, such that ( x + 1) is the factor of
they can start giving massage to his lovely queen, as he the quadratic polynomial ax 2 + bx + c.
doesn’t want to make any fuss over it. However, he (a) 19996002 (b) 1860004
wants to identify the 5 masseurs who are close to the (c) 19876004 (d) none of these
queen. Based on the queen’s personal behaviour with
46 Find the number of isosceles triangles with integer sides,
him the wise king gets an idea that if in any week out of
if no side exceeds 2014 unit.
the 5 familiar masseurs any 3 masseurs give the massage
(a) 2426980 (b) 402800
to the beautiful queen he will certainly identify who all
these 5 people familiar with the queen are. Minimum (c) 2028098 (d) 3120600
how many weeks are required to identify these 47 A five-digit number is called a mountain number if the
5 masseurs familiar with the queen? first three digits increase and the last three digits
(a) 3 (b) 4 decrease. For example, 35763 is a mountain number,
(c) 5 (d) 7 but 35663 is not. How many five-digit numbers greater
than 70,000 are mountain numbers?
41 There is a set of 10 integers {0, 1, 2, 3, ., 9}. In how
(a) 69 (b) 36 (c) 63 (d) 96
many ways can you permute this set of integers such
that either 3 and 6 or 5 and 4 or 6 and 5 appear 48 If a triangle is inscribed in a regular decagon such that
consecutively? all the three vertices of the triangle are the vertices of
the decagon, but none of the triangle’s sides is the side
(For example, we do not count (4, 5, 0, 6, 9, 2, 3, 7, 8,
of the decagon. How many such triangles can be
1), as we want 5 and 4 to appear consecutively in that
formed?
order. But we count (7, 2, 0, 1, 4, 3, 6, 5, 8, 9), both 3
(a) 40 (b) 50
and 6, and 6 and 5 appear consecutively in it.)
(a) 962640 (b) 960049 (c) 60 (d) 70
(c) 368460 (d) 313200 49 A wise old woman calls upon all her 7 sons at her
eleventh hour and hands them over the keys of a box in
42 How many integer solutions are there to the equation
which she keeps all her precious jewelry. She tells them
x1 + x 2 + x 3 + x 4 = 30, such that 3 ≤ x1, x 2, x 3, x 4 ≤ 10.
that she has locked the box with multiple locks so that
(a) 246 (b) 184 the box can be opened only when majority of you are
(c) 1084 (d) none of these present with the keys I am giving to you. After giving
n−1
43 If C r = (k 2 − 3) nC r + 1, then k ∈ R each of her sons the keys of certain locks she takes her
(a) (− α − 2] (b) [ 2, α ) last breath. How many locks she uses to secure her
(c) [ − 3, 3] (d) ( 3, 2] jewelry and how many keys does she hand over to each
of the seven sons?
44 A shopping mall in Kashipur, a meditative and idyllic
(a) 35, 20 (b) 42, 21
town near Nainital, has three food courts – Chinese,
(c) 28, 21 (d) 36, 18
Thai and Italian, situated on its rooftop. Each food court
is designed in such a way that all the restaurants are 50 An engineer in the urban planning department of IIM
situated in a circle. Chinese, Thai and Italian food courts Kashipur, situated near Jim Corbett National Park in
have 20, 15 and 10 restaurants, respectively. Having the Uttarakhand, wants to connect all the 25 buildings
change (coins and paper currency in smaller (blocks) mutually. Each road connects only one pair of
denomination) is usually a terrible job due to digital blocks. No single road connects more than 2 blocks. The
payments and rush in the food courts, so restaurants try roads are designed in such a way that one can reach
to avoid giving the change to each other. Still it is found from one block to any other block through a sequence of
that each restaurant gives the change to any other roads laid out at the campus. The number of roads (R)
restaurant at most three times a day and if the in the campus may satisfy the condition
restaurants belong to the same food court then each (a) 25 ≤ R ≤ 625 (b) 24 ≤ R ≤ 288
restaurant gives the change to other restaurant at most (c) 24 ≤ R ≤ 300 (d) 25 ≤ R ≤ 225
Quantitative Aptitude UANTUM CAT

Answers
Introductory Exercise 19.1
1 (a) 2. (b) 3. (c) 4. (a) 5. (d) 6. (b) 7. (a) 8. (b) 9. (c) 10. (b)
11. (a) 12. (b) 13. (a) 14. (d) 15. (b) 16. (a) 17. (b) 18. (a) 19. (c) 20. (d)
21. (d) 22. (a) 23. (a) 24. (a) 25. (a) 26. (d) 27. (b) 28. (d) 29. (a) 30. (b)
31. (a) 32. (b) 33. (d) 34. (b) 35. (d) 36. (a) 37. (b) 38. (c) 39. (b) 40. (c)
41. (a) 42. (b) 43. (a) 44. (a) 45. (a) 46. (b) 47. (a) 48. (b) 49. (d) 50. (d)
51. (c) 52. (a) 53. (a) 54. (a) 55. (d) 56. (a) 57. (b) 58. (a) 59. (c) 60. (a)
61. (a)

Introductory Exercise 19.2


1 (c) 2. (b) 3. (d) 4. (d) 5. (a) 6. (d) 7. (a) 8. (d) 9. (a) 10. (b)
11. (a) 12. (a) 13. (b) 14. (a) 15. (b) 16. (a) 17. (c) 18. (b) 19. (a) 20. (b)
21. (c) 22. (d) 23. (a) 24. (b) 25. (a)

Introductory Exercise 19.3


1 (a) 2. (c) 3. (a) 4. (c) 5. (a) 6. (d) 7. (d) 8. (a) 9. (b) 10. (a)
11. (b)

Introductory Exercise 19.4


1 (a) 2. (b) 3. (a) 4. (b) 5. (a) 6. (b) 7. (a) 8. (c) 9. (d) 10. (a)
11. (a) 12. (a) 13. (c) 14. (a)

Introductory Exercise 19.5


1 (a) 2. (c) 3. (c) 4. (c) 5. (c) 6. (b) 7. (b) 8. (c) 9. (d) 10. (a)
11. (c) 12. (d) 13. (a) 14. (c) 15. (a) 16. (b) 17. (c) 18. (b) 19. (d) 20. (c)
21. (d) 22. (b) 23. (a) 24. (a) 25. (b) 26. (a) 27. (d) 28. (b) 29. (c) 30. (b)
31. (a) 32. (b) 33. (a) 34. (b) 35. (c) 36. (c) 37. (b) 38. (c) 39. (a) 40. (b)
41. (a) 42. (c) 43. (b) 44. (a) 45. (b) 46. (c) 47. (a) 48. (c) 49. (a) 50. (b)
51. (a) 52. (b) 53. (d) 54. (c) 55. (d) 56. (b) 57. (c) 58. (a) 59. (a) 60. (a)
61. (b) 62. (c) 63. (d) 64. (a) 65. (c) 66. (d) 67. (c) 68. (d) 69. (c) 70. (d)
71. (a) 72. (d) 73. (a) 74. (b) 75. (d) 76. (d) 77. (b) 78. (b) 79. (d)

Introductory Exercise 19.6


1 (b) 2. (a) 3. (b) 4. (d) 5. (c) 6. (b) 7. (d) 8. (a) 9. (d) 10. (b)
11 (b) 12. (b) 13. (d) 14. (b) 15. (b) 16. (a) 17. (d) 18. (d) 19. (b) 20. (b)
21 (b) 22. (c) 23. (b) 24. (d) 25. (a) 26. (d) 27. (a) 28. (b) 29. (a) 30. (a)
31 (c) 32. (a) 33. (a) 34. (c) 35. (c) 36. (c) 37. (d)

Introductory Exercise 19.7


1 (c) 2. (a) 3. (d) 4. (c) 5. (a) 6. (a) 7. (c) 8. (a) 9. (b) 10. (d)
11. (d) 12. (d) 13. (b) 14. (b) 15. (d) 16. (b) 17. (a) 18. (c) 19. (b)

Introductory Exercise 19.8


1 (b) 2. (a) 3. (c) 4. (d) 5. (c) 6. (d) 7. (d) 8. (a) 9. (a) 10. (a)
11. (b) 12. (c) 13. (a) 14. (d) 15. (a) 16. (b) 17. (b) 18. (b) 19. (d) 20. (d)
Permutations & Combinations 1083

21. (d) 22. (b) 23. (c) 24. (d) 25. (a) 26. (d) 27. (c) 28. (c) 29. (a) 30. (c)
31. (d) 32. (b) 33. (a) 34. (a) 35. (a) 36. (a) 37. (c) 38. (b) 39. (b) 40. (c)
41. (a) 42. (c) 43. (d) 44. (d) 45. (a) 46. (b) 47. (b) 48. (d) 49. (b) 50. (a)
51. (d) 52. (c) 53. (d) 54. (a) 55. (a) 56. (b) 57. (a) 58. (b) 59. (c) 60. (a)
61. (c) 62. (c) 63. (d) 64. (c) 65. (b) 66. (b) 67. (a) 68. (a) 69. (c) 70. (d)
71. (d) 72. (c) 73. (c)
Introductory Exercise 19.9
1 (a) 2. (b) 3. (a) 4. (c)
Introductory Exercise 19.10
1 (d) 2. (a) 3. (c) 4. (b) 5. (a) 6. (c) 7. (a) 8. (b) 9. (d) 10. (d)
11. (d) 12. (b) 13. (c) 14. (b) 15. (d) 16. (d) 17. (c) 18. (a) 19. (a) 20. (a)
21. (b) 22. (d) 23. (c) 24. (c) 25. (d) 26. (d) 27. (a) 28. (b) 29. (d) 30. (c)
31. (c) 32. (a) 33. (b) 34. (a) 35. (a) 36. (b) 37. (d) 38. (d) 39. (b) 40. (b)
41. (d) 42. (c) 43. (c) 44. (a) 45. (d) 46. (c) 47. (c) 48. (a) 49. (d) 50. (b)
51. (c) 52. (a) 53. (b) 54. (b) 55. (c) 56. (a) 57. (b) 58. (a) 59. (d) 60. (c)
61. (a) 62. (b) 63. (c) 64. (d) 65. (b) 66. (b) 67. (c) 68. (b) 69. (a) 70. (d)

Level 01 Basic Level Exercise


1 (c) 2. (b) 3. (a) 4. (d) 5. (a) 6. (b) 7. (c) 8. (b) 9. (b) 10. (c)
11. (a) 12. (c) 13. (d) 14. (a) 15. (c) 16. (b) 17. (b) 18. (a) 19. (c) 20. (d)
21. (c) 22. (b) 23. (b) 24. (b) 25. (b) 26. (b) 27. (b) 28. (a) 29. (c) 30. (d)
31. (a) 32. (b) 33. (d) 34. (b) 35. (a) 36. (c) 37. (b) 38. (b) 39. (c) 40. (c)
41. (a) 42. (b) 43. (b) 44. (c) 45. (a) 46. (c) 47. (c) 48. (b) 49. (a) 50. (b)
51. (c) 52. (b) 53. (d) 54. (a) 55. (d) 56. (a) 57. (b) 58. (a) 59. (c) 60. (a)
61. (b) 62. (c) 63. (c) 64. (d) 65. (c) 66. (a) 67. (b) 68. (b) 69. (a) 70. (c)
71. (b) 72. (c) 73. (c) 74. (a) 75. (d) 76. (b) 77. (c) 78. (d) 79. (a) 80 (d)
81. (c)

Level 02 Higher Level Exercise


1 (b) 2. (d) 3. (a) 4. (a) 5. (b) 6. (c) 7. (c) 8. (d) 9. (a) 10. (b)
11. (a) 12. (c) 13. (b) 14. (c) 15. (a) 16. (a) 17. (d) 18. (b) 19. (c) 20. (a)
21. (b) 22. (a) 23. (c) 24. (b) 25. (c) 26. (a) 27. (a) 28. (c) 29. (d) 30. (d)
31. (c) 32. (b) 33. (a) 34. (d) 35. (c) 36. (b) 37. (a) 38. (c) 39. (c) 40. (b)
41. (a) 42. (c) 43. (c) 44. (a) 45. (d) 46. (d) 47. (a) 48. (d) 49. (c) 50. (d)
51. (b) 52. (a) 53. (c) 54. (a) 55. (b) 56. (c) 57. (a) 58. (b) 59. (c) 60. (b)
61. (b) 62. (c) 63. (d) 64. (b) 65. (a) 66. (c) 67. (a) 68. (b) 69. (c) 70. (b)
71. (a) 72. (b) 73. (b) 74. (b) 75. (a) 76. (a) 77. (a) 78. (a) 79. (b) 80. (b)
81. (c) 82. (c) 83. (d) 84. (a) 85. (a) 86. (d) 87. (a) 88. (d) 89. (d) 90. (a)
91. (d) 92. (a) 93. (c) 94. (a) 95. (d) 96. (b) 97. (c) 98. (d) 99. (a) 100. (d)
101. (c) 102. (b) 103. (c) 104. (b) 105. (b)

Level 03 Final Round


1. (a) 2. (b) 3. (c) 4. (d) 5. (d) 6. (a) 7. (b) 8. (a) 9. (b) 10. (b)
11. (c) 12. (c) 13. (a) 14. (b) 15. (b) 16. (c) 17. (b) 18. (c) 19. (d) 20. (c)
21. (a) 22. (c) 23. (c) 24. (b) 25. (a) 26. (b) 27. (a) 28. (c) 29. (b) 30. (a)
31. (a) 32. (b) 33. (d) 34. (b) 35. (b) 36. (a) 37. (b) 38. (b) 39. (c) 40. (c)
41. (a) 42. (a) 43. (d) 44. (b) 45. (d) 46. (c) 47. (b) 48. (b) 49. (a) 50. (c)
QUANTUM CAT
Hints & Solutions
Introductory Exercise 19.1
9!  n n! 
1 9
P3 = Q Pr = 
6!  (n − r)! Case 4. 4 3 2 1 ⇒ 4 × 3 × 2 × 1 = 24
9 × 8 × 7 × 6! There are 24 even numbers of 4 digits
= = 9 × 8 × 7 = 504
6! Thus there are total 1 + 4 + 12 + 24 = 41 even numbers.
2 n
P5 = 20 . nP3 NOTE In each case 8 is fixed at unit place, since if the unit
n! n! (n − 3)! digit is an even digit then the whole number is an even
⇒ = 20 . ⇒ = 20 number.
(n − 5)! (n − 3)! (n − 5)!
(n − 3)(n − 4)(n − 5)! 6 3 4 3 2 ⇒ 3 × 4 × 3 × 2 = 72
⇒ = 20
(n − 5)!
Thousands place can assume only 3 values viz., 5, 7, 8.
⇒ (n − 3)(n − 4) = 20 Since required numbers are greater than 4000.
⇒ n = 8 (n = − 1 is inadmissible) 4 4 3 2 1 ⇒ 4 × 4 × 3 × 2 × 1 = 96
Alternatively Go through options.
Ten thousands place can assume all the remaining non-zero
56
Pr + 6 30800 digits and thousands place can assume zero also except the
3 =
54
Pr + 3 1 digit which has been filled up at thousands place.
Therefore total required numbers
56 ! = number of 4 digit numbers + number of 5 digit numbers
(50 − r)! 30800 = 72 + 96 = 168
=
54 ! 1
(51 − r)! 7 4 4 3 2 ⇒ 4 × 4 × 3 × 2 = 96

56 × 55 × (51 − r)! 30800 Thousands place cannot assume 3 since required numbers
⇒ = are greater than 5000.
(50 − r)! 1
(51 − r)! 10 5 4 3 2 1 ⇒ 5 × 4 × 3 × 2 × 1 = 120
⇒ =
(50 − r)! 1 Total required numbers = 96 + 120 = 216
(51 − r) × (50 − r)! 10
⇒ = = 51 − r = 10 ⇒ r = 41 8 8 7 6 5 ⇒ 8 × 7 × 6 × 5 = 1680
(50 − r)! 1
3 and 5 have been already used, so we have 8 digits for
Alternatively Go through options thousands place, then 7 digits for hundreds place, 6 digits
4 Since there are 6 digits available and out of 6 digits we have for tens digit and remaining 5 digits for unit place.
to take only 3 digits for the arrangement. Therefore, Alternatively 8 P = 8 ! = 8 × 7 × 6 × 5 × 4 !
6! 4
4! 4!
required number of 3 digits numbers = 6P3 = = 120
3!
= 8 × 7 × 6 × 5 = 1680
Alternatively 6 5 4 ⇒ 6 × 5 × 4 = 120 9 A number is divisible by 5 only if its unit digit is either 5 or 0.
We have 6 digits to fill up hundreds place and only 5 digits So we fix 5 as unit digit and then we fill up the remaining
for tens place and only 4 digits for unit place since places.
repetition is not allowed.
5 4 3 2 1 ⇒ 5 × 4 × 3 × 2 × 1 × 1 = 120
5 Case 1. There is only one even number of one digit.
Hence required number = 120
Case 2. 4 1 = 4 ×1 = 4
10 4 3 2 1 ⇒ 4 × 3 × 2 × 1 = 24
There are only 4 even numbers of two digit
There are total 24 numbers of 4 digit.
(Numbers are 38, 58, 78 and 98)
Since we have only 4 digits it means we use each of the
⇒ 4 × 3 × 1 = 12  24
Case 3. 4 3 1 digit 6  =  times in each of the unit, tens hundreds and
 4
There are only 12 even numbers of 3 digit. thousands place.
Permutations & Combinations 1085

Therefore the sum of the digits in the units place 17 Number of permutations of 7 different novels taken 4 at a
= 6 (1 + 2 + 5 + 6) = 84 time, when the first novel is always there = 4 ⋅ (6 P3 ) = 480
Hence choice (b) is the correct one.
Similarly, the sum of the digits in each of tens, hundreds.
and thousands places = 84 Hint Since one novel is always there in the shelf, so he has to
Hence the required sum of all the numbers take any 3 novels from the remaining 6 novels to arrange them.
But the first novel, which is already there in the bookshelf, can
= 84 × 1 + 84 × 10 + 84 × 100 + 84 × 1000 be arranged in 4 ways, as there are four places available to
= 84 (1 + 10 + 100 + 1000) arrange the first novel. X , Y and Z denote the other 3 novels.
= 84 × 1111 = 93324 _ X _ Y_ Z _

11 Case 1. 3 3 1 ⇒ 3× 3×1 = 9 18 Number of permutations of 7 different novels taken 4 at a


time, when the last novel is never kept in the shelf = 6P4 = 360
When 2 is fixed at unit place we have 3, 4 and 5 i.e., 3 digits Hence choice (a) is the correct one.
for hundreds place and remaining 3 digits (out of 5) for the
tens place. Hint Since the last novel is not to be considered for the
arrangements, so there are only 6 novels left from which any 4
2 3 1 ⇒ 2× 3×1 = 6 novels will be considered for the arrangement.

When 4 is fixed at unit place, we have 3 and 5 i.e., only two 19 Number of permutations of 7 different things taken 4 at a
digits for hundreds place and remaining 3 digits for tens time, when 2 particular novels always occur
place. = (4 P2 )(5 P2 ) = 240
Hence choice (c) is the correct one.
Case 2. 4 3 2 2 = 48
Hint Since 2 novels are already selected, so only 2 novels has to
4 digit numbers = 48 be selected from remaining 5 novels. Now arrange the 2 novels
that you have just selected. The remaining3rd novel can be
Case 3. 4 3 2 1 2 = 48
arranged in 3 ways and the 4th novel can be arranged in 4
5 digit numbers = 48 ways. Therefore total number of arrangements = 4 × 3 × (5 P2 ).
If you have any confusion, then solve this problem after
Thus there are total 15 + 48 + 48 = 111 even numbers
studying the Combinations.
greater than 300.
20 Number of permutations of n different novels taken 4 at a
12 4 4 3 = 48 time, when 2 particular novels never occur = 5P4 = 120
Hundred’s place cannot assume 0. Hence choice (d) is the correct one.
13 The number of permutations in which he can arrange all 21 Number of permutations of 7 different novels, taken all at a
his 7 novels together = 7 ! = 5040 time, when 2 specified novels always come together is
Hence choice (a) is the correct one. 2! × (6 !) = 1440
14 The number of ways in which he can arrange any 4 novels Hence choice (d) is the correct one.
7! 7! 7 × 6 × 5× 4 × 3× 2×1 Hint Since 2 novels come together, then consider them as a
= 7 P4 = = = = 840 single novel. Now there will be total 6 novels only, which can
(7 − 4)! 3! 3× 2×1
be arranged in 6! ways. But as the 2 novels can be mutually
Hence choice (d) is the correct one. arranged in 2! ways, so the total number of arrangements
= 2! × 6!.
15 The number of ways in which he can arrange at most
4 novels 22 Number of permutations of 7 different novels, taken all at a
time, when 2 specified novels never come together is
= P1 + P2 + P3 + P4
7 7 7 7
7 ! − [ 2!(6 !)] = 7 × (6 !) − 2 × (6 !) = 6 ! × 5 = 3600
= 7 + 42 + 210 + 840 = 1099 Hence choice (a) is the correct one.
Hence choice (b) is the correct one.
Hint Number of permutations when 2 novels never come
Hint Either he can arrange 1 novel or 2 novels or 3 novels or together = Total number of permutations − Number of
4 novels. But he cannot arrange more than 4 novels. permutations when the 2 specified novels are always together.

16 The number of ways in which he can arrange at least 23 There are 7 letters in the word RAINBOW and each letter is
4 novels used only once. So all the 7 letters can be arranged in
7 ! ways.
= P4 + P5 + P6 + P7
7 7 7 7
7 ! = 7 × 6 × 5 × 4 × 3 × 2 × 1 = 5040.
= 840 + 2520 + 5040 + 5040 = 13440
24 If we fix R as the initial letter, then we have to arrange only
Hence choice (a) is the correct one. 6 remaining letters.
Hint Either he can arrange 4 novels or 5 novels or 6 novels or 7 Hence required number of permutations = 6 !
novels. But he would not have to arrange less than 4 novels. = 6 × 5 × 4 × 3 × 2 × 1 = 720
1086 QUANTUM CAT
25 If we fix R and W as the first and last letters then we have to RAINBOW can be arranged in which A is before I and I is
arrange only 5 remaining letters which can be arranged in before O = 4 ! × 35 = 24 × 35 = 840.
5! = 120 ways. Alternatively All the 7 letters of the word RAINBOW
26 When R and W are the first and last letters of all the words can be arranged in 7! ways and 3 particular letters (A,I,O)
then we can arrange them in 5! ways. Similarly when W can be arranged in 3! ways.
and R are the first and last letters of the words then the But the given condition is satisfied by only 1 out of 6 ways.
remaining letters can be arrange in 5! ways. Hence required number of arrangements
Thus the total number of permutations = 2 × 5! 7!
= = 7 × 6 × 5 × 4 = 840
= 2 × 120 = 240 3!
Alternatively Except R and W all the remaining
32 Let all the vowels be in a single packet and all the
5 letters can be arranged in 5! ways and R and W can be consonants be in the other packet.
arranged in 2! ways at the end positions.
Now all the vowels can be arranged in 3! ways and all the
Thus total number of permutations consonants can be arranged in 4! ways.
= 2! × 5! = 2 × 120 = 240
Hence total number of arrangements
27 The first and last letters can be arranged in 3 P2 ways and
= 3! × 4 ! = 6 × 24 = 144
the remaining letters can be arranged in 5! ways.
33 First arrange the 3 vowels which can be done in 3! ways.
Hence required number of permutations
Now there are 4 places created by the 3 vowels which can
= P2 × 5! = 6 × 120 = 720
3
be filled up by 4 consonants in 4 P4 ways.
28 Assume R and W as a single letter, then we have only Therefore total number of required permutations
6 letters for the arrangement. = 3! × 4P4 = 6 × 24 = 144
That is A, B, I, N, O, RW, which can be arranged in 6! ways.
34 1 2 3 4 5 6 7
Now, R and W can also be mutually arranged in 2! ways. In order that vowels may occupy even places arrange all the
Therefore, total number of arrangements = 2! × 6 ! = 1440. 3 vowels, then arrange all the four consonants in four
29 Total number of words = 7 ! = 5040 places.
Thus the required number of arrangements = 3! × 4 != 144
Number of words in which R and W are always together
35 1 2 3 4 5 6 7
= 1440
First of all arrange any 3 consonants at even places in
Number of words in which R and W are never together 4
P3 ways. Now the newly created four odd places can be
= 5040 − 1440 = 3600.
filled by the remaining letters which includes 3 vowels and
Hence choice (a) is the correct one. 1 consonants, which can be done in 4 P4 ways. Hence the
30 There are three vowels viz., A, I and O required number of permutation
First of all arrange all the four consonants which can be
= 4P3 × 4P4 = 24 × 24 = 576
done in 4! ways. Now we have 5 places available for the
three vowels to be filled up, which can be done in 5 P3 ways. 36 First of all arrange all the four consonants R, N, B, W in
5! 4! ways.
Thus the total number of arrangements = 4 ! × 5P3 = 24 × Then there are 5 places to be filled up by the vowels. But
2!
any two vowels are always together then we assume that
= 24 × 60 = 1440
there are only two vowels which can be filled in 5 places in
31 First of all we arrange all the 4 consonants in 4! ways. 5
P2 ways. But we have to take any two vowels together out
Now we have 5 places for the 3 vowels as of 3 vowels then this can be done in 3 P2 ways.
1 C1 2 C2 3 C3 4 C4 5 Hence the total number of permutation = 5P2 × 3P2 × 4 !
1. When all the three vowels AIO are together then = 20 × 6 × 24
5 places can be filled in 5 ways. = 2880
2. When A and I are together and O is separated from ‘‘AI’’
then the 5 places can be filled up in 10 ways. Alternatively Since there can be only 3 cases
3. When I and O are together and A is separated from ‘‘IO’’ (i) When A, I and O are separate from each other
then the 5 places can be filled up in 10 ways. (ii) When A, I and O can always together
4. When A, I and O are separated from each other then (iii) When any two vowels out of three vowels A, I and O
the 5 places can be filled up in 10 ways. are together.
Thus there are total 5 + 10 + 10 + 10 = 35 ways in which Now we need to calculate the value of Case (iii).
5 places can be filled up such that A is before I and I is ∴ Required number of permutations
before O. = Total number of permutations – [Case (i) + Case (ii)]
Hence total number of ways in which all the 7 letters of
∴ 7 ! − (1440 + 720) = 5040 − (2160) = 2880
Permutations & Combinations 1087

37 The correct order of the letters is as follows : 42 Since no two men can sit together, it means there must be
A, B, I, N, O, R, W 4 women separating all the 5 men. So, first of all let the
Number of words begin with A = 6! 4 women sit in a row, then allow the 5 men to sit in such a
way so that no two men sit together.
Number of words begin with B = 6!
Essentially, this is the same problem as the previous one.
Number of words begin with I = 6!
Number of words begin with N = 6! Therefore the required number of permutations

Number of words begin with O = 6! = 4 ! × 5! = 2880.

Number of words begin with RAB = 4! 43 Since no two women can sit together, it means there must
be at least 3 men separating all the 4 women. But since men
Number of words begin with RAIB = 3! are more than 3, so let them sit before the women take
Now the next word is RAINBOW (it is the first word which their places. When men sit at 5 places (in 5! ways), there
begins with RAIN) will be 6 places available for 4 women.
So the ranking of the word RAINBOW Therefore number of required permutations
= 5 × 6 ! + 4 ! + 3! + 1 = 3631 = 5! × 6P4 = 43200.
38 There are 5 consonants and 2 vowels in the word
STRANGE out of 7 places for the 7 letters, 4 places are odd 44 Total number of persons = 9
and 3 places are even. So they can be arranged, without any restriction, in
2 vowels can be arranged in 4 odd places in 4 P2 ways and 9! ways.
then 5 consonants can be arranged in the remaining But when all the four women sit together, consider them as
5 places in 5 P5 ways. one unit, then the number of ways in which they can be
Hence, the required number of ways = 4P2 × 5P5 = 1440 arranged = (5 + 1)! × 4 ! = 6 ! × 4 !
2 4 6 Therefore the required number of ways in which all the
39 four women do not sit together
1 3 5
First of all arrange 3 vowels at 3 even places, which can be = 9 ! − (6 ! × 4 !) = 9 × 8 × 7 × (6 !) − (6 ! × 4 !)
done 3! ways. Again arrange the 3 consonants at 3 odd = 6 !(7 × 8 × 9 − 4 × 3 × 2 × 1)
places which can be done in 3! ways. = 345600.
Hence required number of permutations = 3! × 3! = 36
45 There are total 9 places out of which 4 are even and rest
ways 5 places are odd.
Alternatively 3 3 2 2 1 1 = 36 ways 4 women can be arranged at 4 even places in 4! ways.
Assume vowels are even numbers say 2, 4, 6 and and 5 men can be placed in remaining 5 places in 5 P5 ways.
consonants are odd numbers say 1, 3, 5. Hence, the required number of permutations
Now, if we have to form the numbers in which odd integers = 4 ! × 5P5 = 24 × 120 = 2880
always occupy odd places then this can be done as above.
(O, E, O, E, O, E ) 46 First of all the 6 men can be arranged in 6! ways then the
3 women can be arranged in 7 places in 7 P3 ways.
40 At least two consonants remain together means
(i) Any two consonants are together Therefore the required number of arrangements
(ii) All the three consonants are together = 6 ! × 7 P3 = 720 × 210 = 151200.
which implies that all of the consonants are not separated 47 There are two cases : In the first case a man is sitting at the
from each other. first chair and in the second case a woman is sitting at the
Hence required number of permutations first chair.
= (Total number of permutations
– number of permutations when all of the vowels M W M W M W M W
are separated from each other) 1 2 3 4 5 6 7 8
= 6 ! − (3! × P3 ) = 720 − 144 = 576
4
W M W M W M W M
41 Since neither two men nor two women sit together, it
First case first chair can be occupied by any 1 of the
means they sit on the alternate positions.
4 men, third chair can be occupied by 1 of the remaining
M W M W M W M W M 3 men, fifth chair can be occupied by the 1 of the remaining
2 men and the seventh chair can be occupied by the last
Therefore first of all we arrange 4 women in 4! ways man. Thus chair numbers 1, 3, 5 are 7 can be occupied by
then we arrange 5 men in newly created 5 places in the 4 men in 4! ways. Similarly the remaining chairs can be
5
P5 = 5! ways. Thus the total number of arrangements occupied by the 4 women in 4! ways. Thus all the 8 people
= 4 ! × 5! = 2880.
1088 QUANTUM CAT
can be seated in 4 ! × 4 ! ways. 56 All the 3 books on Physics can be mutually arranged in
Second case in this case also, all the 8 people can be 3! ways.
arranged in 4 ! × 4 ! ways. Therefore total number of Similarly, 4 books on Chemistry can be mutually arranged
required ways = 4 ! × 4 ! + 4 ! × 4 ! = 2(4 ! × 4 !) = 1152 in 4! ways.
48 12 soldiers can stand in a queue in 12! ways. And 5 books on Biology can be arranged in 5! ways.
49 Any 6 soldiers can stand in one queue and the remaining Besides, the three sets of Physics, Chemistry and Biology
6 soldiers can stand in another queue. And then the two can be mutually arranged in 3! ways.
queues can be mutually arranged. Therefore, required number of permutations
Therefore the required number of arrangements = 3! × (3! × 4 ! × 5!) = 103680.
= 12P4 × 6P6 × 2! = 12P6 × 6 ! × 2! 57 The signals can be made by using at a time one or two or
50 The required number of arrangements three or four or five or six flags out of 6 different coloured
flags.
= 12
P4 × P4 × P4 × 3!
8 4
∴ Required number of flags
51 The required number of ways = 12P3 × 9P4 × 5P5 × 3!.
= 6P1 + 6P2 + 6P3 + 6P4 + 6P5 + 6P6
52 Assume that the 3 soldiers constitute only one soldier. So
= 6 + 30 + 120 + 360 + 720 + 720 = 1956
there are now 12 − 3 + 1 = 10 soldiers. Thus 10 soldiers can
be arranged in 10! ways. And the 3 soldiers can be mutually 58 Required number of signals = 6P3 = 120.
arranged in 3! ways. Therefore the required number of 59 Required number of permutations = 10P3 = 720
ways
Alternatively First prize can be given to any of the
= 10 ! × 3! = 21772800.
10 students and second prize can be given to any one of the
53 Required number of ways = 12! − (11 ! × 2!) remaining 9 students and the third prize can be given to
= 11 ! × (12 − 2) any one of the remaining 8 students.
= 11 ! × 10 = 399168000 Therefore it can be done in
54 Total number of permutations = 12! 10
P1 × 9P1 × 8P1 = 10 × 9 × 8 = 720 ways.
Number of permutations when three particulars books are 60 A passanger from any station may purchase ticket for
together = 10 ! × 3! anyone of the other 9 stations. Therefore, there must be
∴Number of permutations when three particular books are 9 tickets in each station.
not together = 12! − 10 ! × 3! Therefore total number of different tickets = 9 × 10 = 90
= 10 !(12 × 11 − 6) = 10 ! × 126 = 457228800 61 Since we cannot wear more than one ring in any finger
NOTE In this question out of the 3 particular books any means repetition is not allowed.
2 particular books can be together but all of the 3 particular Hence, the first ring can be worn in any of the available
books cannot be together. 4 fingers in 4 ways and second ring can be worn in any of
the remaining 3 fingers in 3 ways and third ring can be
55 First of all arrange the remaining 9 books in 9! ways. Then worn in any of the remaining 2 fingers in 2 ways.
there will be 10 new places on the either side of these books So the required number of ways in which all the 3 rings can
where the 3 books can be arranged in 10 P3 = 720 ways. be worn in 4 fingers = 4 × 3 × 2 = 24.
Therefore the total number of ways in which none of the Alternatively We have 4 places to be filled up by
3 particular books is together = 9 ! × 720 = 261273600. 3 different things (i.e., rings), which can be done in 4 P3 ways.
4
P3 = 24

Introductory Exercise 19.2


1 In the given word there are 6 letters of which E occurs 3 There are total 10 letters of which R occurs 2 times,
2 times. E occurs 3 times.
6! 10 !
Hence the required number of ways = Hence the required number of ways = = 302400
2! 2!. 3!
6 × 5× 4 × 3 × 2 × 1
= = 360 13!
2 4 Required number of permutations =
3!. 4 !. 2!. 2!
2 There are total 7 letters of which A occurs 3 times and
N occurs 2 times. = 10810800
7! 5 There are 6 different letters in the word which are A, S, I, N, T,
Hence, the required number of ways = = 420
3!. 2! O.
Permutations & Combinations 1089

The number of arrangements of 6 different letters taken = 181440 − 40320 = 141120


4 at a time = 6P4 = 360 12 T,T,T, N,N,N, G,G,G, R, I,I,I
6 There are total 9 letters in the word COMMITTEE in which There are total 13 leters of which 3 are T ’s, 3 are N’s, 3 are
there are 2M’s, 2T’s, 2E’s. G’s, 3 are I’s.
∴ The number of ways in which 9 letters can be arranged Also there are 6 even places and 7 odd places. 3Is
9! (i.e., vowels) can be arranged in the 6 even places in
= = 45360 6
P3
2! × 2! × 2! = 20 ways.
3!
There are 4 vowels O, I, E, E in the given word. If the four
Now, we are left with 10 places in which 10 letters can be
vowels always come together, taking them as one letter we 10 !
have to arrange 5 +1 = 6 letters which include 2Ms and 2Ts filled up in = 16800
6! 3! × 3! × 3!
and this be done in = 180 ways. ∴ The total number of permutations in which vowels
2! × 2!
occupy even places = 20 × 16800 = 336000
In which of 180 ways, the 4 vowels O, I, E, E remaining
4! 13 Correct order of the letters in D, G, I, I, O, O, P, R, S, U
together can be arranged in = 12 ways.
2! Word begin with Number of words
∴ The number of ways in which the four vowels always D
9!
come together = 180 × 12 = 2160. 2! 2!
Hence, the required number of ways in which the four 9!
vowels do not come together = 45360 − 2160 = 43200 G
2! 2!
7 Consonants : C, M, M, T, T Vowels : O, I, E, E 9!
I
Total number of consonants = 5 2!
5! Word begin with Number of words
Number of ways of arranging 5 consonants = = 30 9!
2! 2! O
Total number of places created for vowels = 6 2!
8!
Number of ways of arranging 4 consonants in 6 places PD
2! 2!
= 6P4 = 360
8!
PG
But since two vowels are same, so the 4 vowels can be 2! 2!
arranged in 360 / 2 = 180 ways 8!
PI
Therefore the required number of ways = 30 × 180 = 5400 2!
14 ! 8!
8 = 113513400 PO
4 ! × 2! × 2! × 2! × 2! × 2! 2!
7!
9 When L and T are fixed as first and last letters of the word, PRD
2! 2!
then we have only 6 letters to be arranged. 7!
6! PRG
Hence required number of permutations = = 90 2! 2!
2! × 2! × 2!
7!
PRI
10 M, M, T, T, H, C, S A, A, E, I 2!
144
42444
3 1424
3
7 4 5!
PRODG
When all the vowels are together then n = 7 + 1 = 8 2!
8! × 4! Now the next two words are
∴ Required number of permutations = = 120960
2! × 2! × 2! PRODIGIOSU and PRODIGIOUS
Hence the rank of the word PRODIGIOUS
Hint (M, T and A occur 2 times and A A E I can be arranged
4!  9!   9 !  8!   8 !
mutually in ways) = 2  + 2   + 2  + 2 
2!  2! 2!  2!  2! 2!  2!
11 I,M,P,O,R,T,A,N,T  7 !  7 ! 5!
9! + 2  + + +2
Total number of permutations = = 181440  2! 2! 2! 2!
2!
 9 !  8 ! 5!
Number of permutations when 2T ’s are together = 3   + 3  + 7 ! + +2
 2!  2! 2!
8 ! × 2!
= = 8 ! = 40320 = 544320 + 60480 + 5040 + 60 + 2
2!
∴Number of permutations when 2T ’s are not together = 609902
1090 QUANTUM CAT
14 The correct order of letters is D, E, E, E, E, Q, R, S, S, T, U. Therefore number of numbers when 2T ’s are together
4!
Number of words begin with DEEEEQR = = 12 10 ! 2!
2! = × = 453600
2! × 2! × 2! 2!
Number of words begin with DEEEEQS = 4 ! = 24 ↑ ↑ ↑ ↑
4! R E G T
Number of words begin with DEEEEQT = = 12
2! Here R is 2 times, E is 2 times, G is 2 times and T is 2 times.
Now, the next two words are 20 S U M P T O
DEEEEQURSST and DEEEEQURSTS. SU
Hence the 50 th word is DEEEEQURSTS. Number of ways in which 2U’s are always together is
15 There are 7 digits 1, 2, 0, 2, 4, 2, 4 in which 2 occurs 7 ! 2!
× = 2520
3 times, 4 occurs 2 times. 2! 2!
7! 8!
∴ Number of 7 digit numbers = = 420 Total number of permutations = = 10080
3! × 2! 2!. 2!
∴Number of ways in which 2U’s are not together
But out of these 420 numbers there are some numbers
= 10080 − 2520 = 7560
which begin with ‘0’ and they are not 7-digit numbers. The
number of such numbers beginning with ‘0’. 21 A F L T O N
6! A O
= = 60 There are 4 consonants F, L, T and N and 4 vowels A, A, O, O
3! × 2!
4!
Hence the required number of 7 digits numbers 4 vowels can be arranged in = 6 ways.
2!. 2!
= 420 − 60 = 360
Alternatively Now the 4 consonants can be arranged in 4 places in 4!
= 24 ways.
6 6 5 4 3 2 1 ⇒ 6 . (6 !)
Therefore required number of permutations
Since the left most place can assume only non-zero digit = (6 × 24) × 2! = 288
hence it can consider only 6 digits. Now the remaining
6 places can be filled up in normal ways. Hint There are two cases :
(i) OCOCOCOC (ii) COCOCOCO
But since 2 occurs 3 times, 4 occurs 2 times, therefore
where O → vowel, C → consonant)
6 . (6 !)
required number of 7 digit numbers = = 360 22 S O T H A Y E R
3!. 2!
SO
6!
16 Required number of 6 digit numbers = = 15 Since two S’s and two O’s together therefore we can
4 !. 2! consider there are only 8 letters, which can be permuted in
8 ! × 2! × 2!
17 5 4 3 2 1 3 ⇒ 3 . (5!) = 8 ! = 40320 ways
2! × 2!
Unit place can assume any of the three even digits 23 Total number of books = 3 × 4 = 12 in which each of
viz.,2 ,4 ,6 and rest of the places can be filled up in 5! ways.
4 different books is repeated 3 times.
But since digit 7 occurs 2 times.
Hence the required number of permutations
3. (5!) 12! 12!
∴ Required number of numbers = = 180 = = = 369600
2! 3! × 3! × 3! × 3! (3!)4
18 6 6 5 4 3 2 1 24 There are total 3 + 4 + 5 = 12 marbles of which 3 are red
Since left most place can assume only non-zero digit and (alike), 4 are green (alike) and 3 are pink (alike).
12!
rest can assume all the given digits. ∴ The required number of arrangements =
Therefore the number of numbers = 6 . (6 !) 3! × 4 ! × 5!
But since 5 occurs 2 times and 2 occurs 2 times = 27720
6 (6 !) 25 Total number of ways of arranging the 12 umbrellas
Hence the required number of numbers = 12!
2!. 2! =
3! 4 ! 5!
= 1080 Number of ways of arranging 12 umbrellas when the
19 R E G U I T A umbrellas of the same colour are together = 3!
REG T Therefore the required number of ways
There are total 11 letters, but since 2T ’s are together. 12!
= − 3! = 27714
3! 4 ! 5!
Permutations & Combinations 1091

Introductory Exercise 19.3


1 Ten thousands place can assume only non-zero digits Number of 4 digit numbers beginning with
hence ten thousands place can be filled up in 4 ways and 1 = 4 × 4 × 4 = 64
thousands place can be filled up in 5 ways since repetition Number of 4 digit numbers beginning with
is allowed (and 0 can be filled up in this place). Similarly 2 = 4 × 4 × 4 = 64
hundreds, tens and unit places can be filled up in 5 ways Number of 4 digit numbers beginning with
each. 3 = 4 × 4 × 4 = 64
Number of 4 digit numbers beginning with 41 = 4 × 4 = 16
∴The required number of numbers = 4 × 5 × 5 × 5 × 5
Number of 4 digit numbers beginning with 42 = 4 × 4 = 16
= 2500 Number of 4 digit numbers beginning with 431 = 4
2 The numbers lying between 9 and 1000 consist of 2 or 3 Number of 4 digit numbers beginning with 432 = 1
digits in which repetitions are allowed. Total number of 4 digit numbers
Case 1. (For two digit numbers) = 64 + 64 + 64 + 16 + 16 + 4 + 1 = 229
As the tens place can be filled by only non-zero digits and Hence the total number of natural numbers not exceeding
unit digit can be filled up in 6 ways. 4321 is 4 + 16 + 64 + 229 = 313
∴The number of 2 digit numbers = 5 × 6 = 30 7 First ring can be worn in any of the 3 fingers similarly
Case 2. (For three digit numbers) second, third and fourth ring can be worn in any of the
three fingers.
Hundreds place can be filled up by only non-zero digits
hence it can be done in 5 ways and the rest of the places i.e., ∴ 4 rings can be worn in 3 × 3 × 3 × 3 = 81 ways.
tens and unit places can be filled up in 6 ways each. 8 First prize can be given away to 4 boys in 4 ways.
∴ The number of 3 digit numbers = 5 × 6 × 6 = 180 Similarly second, third, fourth and fifth prizes can also be
given away to four boys in 4 ways.
∴ Total number of numbers = 30 + 180 = 210
Hence the required number of ways in which all the 5
3 Since even digit may not occupy an even place therefore prizes can be given away to 4 boys
even places can be filled up in only 3 ways (with odd digits) = 4 × 4 × 4 × 4 × 4 = 1024
and rest of the 3 odd places can be filled up in 5 ways with
9 The first ball can be placed in any one of the n cells in
all the 5 digits. n ways. The second ball can also be placed in any one of the
Therefore the required number of numbers n cells in n ways.
∴The first and second balls can be placed in n cells in n × n
= 5 3 5 3 5
i.e., n2 ways.
= 5 × 3 × 5 × 3 × 5 = 1125 Similarly each of the rest balls can be placed in n ways.
Hence the required number of ways = n × n × . . . × n times
4 Number of numbers of 4 digit in which repetition allowed
= 9 × 10 × 10 × 10 = 9000 = nn
Number of numbers of 4 digit in which repetitions is not
10 Since each cell is to be occupied, 1 ball is to be placed in
allowed = 9 × 9 × 8 × 7 = 4536
one cell. First ball can be placed in any one of the n cells in
Hence the required number of numbers of 4 digit
n ways, 2 nd ball can be placed in any one of the remaining
= 9000 − 4536 = 4464
(n − 1) cells in (n − 1) ways, similarly 3rd , 4th . . . ball can be
5 Total number of 3 digit numbers = 9 × 10 × 10 = 900
arranged in (n − 2), (n − 3). . . . ways.
Number of 3 digit numbers in which none of the digits is 9 Hence the required number of ways
= 8 × 9 × 9 = 648
= n (n − 1)(n − 2). . . . 2 . 1 = n !
∴Number of 3 digit numbers in which there is atleast one
digit is 9 = 900 − 648 = 252 11 First letter can be posted in 4 letter boxes in 4 ways.
Similarly second letter can be posted in 4 letter boxes in
6 Total one digit numbers = 4
4 ways and so on.
Total 2 digit numbers = 4 × 4 = 16 Hence all the 5 letters can be posted in
Total 3 digit numbers = 4 × 4 × 4 = 64
= 4 × 4 × 4 × 4 × 4 = 1024
1092 QUANTUM CAT
Introductory Exercise 19.4
1 Required number of permutations = 5! = 120 These 8 flowers can be arranged 7! ways but the 3 flowers
which are together can be arranged mutually in 3! ways.
2 Assume there are only 5 boys since two boys always remain
Hence, the required number of ways = 3! × 7 ! = 6 × 5040
together and these two boys can be arranged mutually in
= 30240
2 ways.
∴ Required number of permutations = 4 ! × 2 = 48 11 Sonu can have only male singer as his neighbours which
can be mutually arranged in 2! ways.
1
3 Required number of permutations = × 5! = 60 Similarly Alka can have only female singers as her
2
neighbours which can be arranged in 2! ways.
(Since there is no difference in clockwise and anticlockwise
arrangements) Hence the required number of arrangements = 2! × 2! = 4
12 Let H1, H 2, H 3 be the respective husbands of W1, W2, W3.
4 Total number of arrangements = 6 ! = 720
As they sit in a circular arrangement, so the total number of
Number of arrangements in which two ladies are together
ways in which they can sit around the table without any
= 2 × 5! = 240 restriction = 5! = 120
∴Number of arrangements in which two ladies are never
Now, we have to subtract all such cases in which the couple
together = 720 − 240 = 480
sits on the opposite chairs.
Alternatively First of all place the five men in 4! ways,
then place the two ladies in any 5 spaces in 5 P2 ways. Case I When exactly one couple sits on opposite chairs.
Let us consider H1 sits opposite W1. So, H 2 can sit on any of
Hence, the required number of ways = 4 ! × 5P2 = 480
the 4 chairs in 4 ways and then W2 can sit on any of the
5 Assuming Sahara, Ambani and Mahindra as a single 2 chairs in 2 ways. After that H 3 can sit on any of the 2 chairs
personality, then there are total (10 − 3) + 1 = 8 person, in 2 ways and W3 can sit on the remaining chair in 1 way.
which can be arranged in 7! ways. Therefore when a couple H1 − W1 sits opposite all others can
Also Sahara and Mahindra can be mutually permuted in be arranged in 4 × 2 × 2 = 16 ways. But, since there are three
2 ways. Therefore required number of arrangements couples, so any one couple can be selected in 3 ways.
= 2 × 7 ! = 10080 Therefore, the total number of ways in which a couple sits
on opposite chairs = 3 × 16 = 48 ways.
6 A person can be chosen out of 18 people in 18 ways to be
seated between Musharraf and Manmohan. Now consider Case II When 2 couples sit on opposite chairs.
Musharraf, Manmohan and the third person, sitting When two couples sit on opposite chairs, the third couple
between them, as a single personality, we can arrange will naturally be sitting on opposite chairs. So there are
them in 17! ways but Musharraf and Manmohan can also 0 ways in which 2 couples sit on opposite chairs.
be arranged in 2 ways. Case III When all three couples sit on opposite chairs.
∴ Required number of permutations = 18 × (17 !) × 2 Arrange H1 and W1 opposite each other, which can be done
in 1 way. Now H 2 can be arranged on any 4 chairs and W2
= 2 × 18 !
can be arranged in 1 way as she has to sit opposite her
7 6 Men can be arranged in 5! ways then 6 women can be husband. And then H 3can sit in 2 ways and her wife W3 can
arranged in 6 places in 6! ways. sit in 1 way. Therefore the number of ways in which all the
Hence the required number of ways = 5! × 6 ! three couples sit opposite = 1 × 4 × 1 × 2 × 1 = 8 ways
= 120 × 720 = 86400 Therefore the required number of ways in which no couple
8 First of all 4 girls can be arranged in 3! ways then 3 boys sits opposite each other = 120 − (48 + 0 + 8) = 64
can be seated in 4 places in 4 P3 ways. 13 First of all arrange the six students with the higher ranks
(1, 2, 3, 4, 5 and 6). Now, arrange the remaining students
∴ Required number of arrangements = 3! × P3 4
(of lower ranks 7, 8, 9, 10, 11 and 12) between them.
= 6 × 24 = 144
The number of ways in which first 6 candidates can be
9 Clearly 7 sisters can sit around a table in 6! ways. But in arranged in 5!ways and the remaining 6 candidates can be
clockwise and anticlockwise arrangements each of the arranged in 6 ! ways.
ladies have the same neighbours. Therefore the required number of ways = 5! × 6 ! = 86400
1
So, the required number of ways = × (6 !) = 360 14 First of all 10 girls can be given 10 alternate rooms in
2 10 !ways, as rooms are distinguishable. Now 10 guys can be
Hence they dine together only 360 days. given 10 alternate rooms in 10 ! ways. However, girls and
guys can exchange their rooms in 2 ways.
10 Since 3 particulars flowers are together
Therefore the required number of ways = 2! × (10 !)2
Hence there can be total (10 − 3) + 1 = 8 flowers.
Permutations & Combinations 1093

Introductory Exercise 19.5


8! 8 ×7 × 6 ⇒ n=8
1 8
C3 = = = 56
3! 5! 6 Alternatively Go though options.
10 × 9 × 8 × 7 × 6 × 5! 9 If n is odd, the greatest value of nC r = nC m
2 10
C5 = = 252
5 × 4 × 3 × 2 × 1 × (5!)
(n − 1) (n + 1)
where m = or m =
3 Q 13
C x = 13C y 2 2
⇒ either x = y or x + y = 13 (11 − 1) (11 + 1)
∴ x= = 5 or x = =6
But since x ≠ y, therefore x + y = 13 2 2
4 Since 17 C r = 17C r + 3 Hence (d) is the correct choice.

⇒ either r = r + 3 or r + r + 3 = 17 10 If n is even, the greatest value of nC r = nC m


r≠r+ 3 n 12
But since where m=
⇒ x= =6
2 2
∴ r + r + 3 = 17 ⇒ r =7
14
5 7
P3 = n ⋅ C 3
7 11 14C x is maximum when x = =7
2
⇒ 5× 6 ×7 = n × 5×7 ⇒n = 6 Alternatively Go through options.
Alternatively n
Pr = r !(nC r ) 12 Since there are total 10 persons and out of these 10 persons
⇒ r ! = 3! = 6 = n we have to select any 5 persons which can be done in
10
6 2n
C 3 : nC 2 = 12 : 1 C 5 ways.
10 !
2n (2n − 1)(2n − 2) n (n − 1) ∴ 10
C5 = = 252
⇒ : = 12 : 1 5! × 5!
3× 2×1 2×1
⇒ (2n − 1) = 9 13 Out of 10 questions, 6 questions can be selected in
10
⇒ n=5 C 6 ways
Alternatively Go through options.
∴ 10
C 6 = 210

7 Go through options : The least possible value of n is 8. To 14 Since question number 1 is compulsory so we have to
verify the result put the vlaue of n as given in the choices. choose only 5 questions from rest of the 9 questions, which
Let us consider choice (a) i.e., n = 6, then can be done in 9C 5 ways
5
C 3 + 5C 4 > 6C 3 ∴ 9
C 5 = 126

10 + 5 > 20, which is wrong. 15 She may invite one or more friends by selecting either 1 or
Again consider choice (c) i.e., n = 7, then 2 or 3 or 4 or 5 friends out of 5 friends.
6
C3 + C4 > C3
6 7 ∴1 friend can be selected out of 5 in 5C1 ways
2 friends can be selected out of 5 in 5C 2 ways
20 + 15 > 35, which is also wrong.
Now consider choice (b) i.e., n = 8, then 3 friends can be selected out of 5 in 5C 3 ways
7
C 3 + 7C 4 > 8C 3 4 friends can be selected out of 5 in 5C 4 ways
5 friends can be selected out of 5 in 5C 5 ways
35 + 35 > 56, which is correct.
Hence the least possible value of n = 8. Hence the required number of ways
Now you need not to check the option (d). = 5C1 + 5C 2 + 5C 3 + 5C 4 + 5C 5
(n !) = 5 + 10 + 10 + 5 + 1 = 31
n
Px (n − x )! 336 Alternatively 5
C1 + 5C 2 + 5C 3 + 5C 4 + 5C 5
8 n = =
Cx (n !) 56 = 25 − 1 = 31
( x !)(n − x )!
n Since, C1 + nC 2 + nC 3 + … + nC n = 2n − 1
n
Px
⇒ = x!= 6
n
Cx 16 4 men can be selected out of 7 men in 7 C 4 ways
⇒ x ! = 3! and 2 ladies can be selected out of 6 ladies in 6C 2 ways
⇒ x=3 Hence, the required number of ways = 7C 4 × 6C 2
Now since nP3 = 336 = 35 × 15 = 525
⇒ n (n − 1)(n − 2) = 8 × 7 × 6
1094 QUANTUM CAT
17 A committee of 5 persons is to be formed 23 3 experts including atleast an engineer and a manager can
from 6 gentlemen and 4 ladies by taking. be selected by taking.
(i) 1 lady out of 4 and 4 gentlemen out of 6. (i) 2 managers out of 3 and 1 engineer out of 3.
(ii) 2 ladies out of 4 and 3 gentlemen out of 6. (ii) 1 manager out of 3 and 2 engineer out of 3.
(iii) 3 ladies out of 4 and 2 gentlemen out of 6. (iii) 2 persons out of 6 (3 managers and 3 engineers) and
(iv) 4 ladies out of 4 and 1 gentleman out of 6. 1 person out of one who is both engineer and manager.
In case I the number of ways In case I, the number of ways = 3C 2 × 3C1 = 9
= 4C1 × 6C 4 = 4 × 15 = 60 In case II, the number of ways = 3C1 × 3C 2 = 9

In case II the number of ways In case III, the number of ways = 6C 2 × 1C1 = 15
= 4C 2 × 6C 3 = 6 × 20 = 120 Hence, the required number of ways = 9 + 9 + 15 = 33.

In case III the number of ways 24 A committee of 5 persons, consisting of at most two ladies,
can be formed in the following ways.
= C 3 × C 2 = 4 × 15 = 60
4 6
(i) selecting 5 gents only out of 6.
In case IV the number of ways (ii) selecting 4 gents only out of 6 and one lady out of 4.
= 4C 4 × 6C1 = 1 × 6 = 6 (iii) selecting 3 gents only out of 6 and two ladies out of 4.
Hence, the required number of ways In case I, the number of ways = 6C 5
= 60 + 120 + 60 + 6 = 246 In case II, the number of ways = 6C 4 × 4C1
18 A committee of 7 persons retaining a majority of Indians In case III, the number of ways = 6C 3 × 4C 2
can be made from 6 Americans and 7 Indians by taking ∴Required number of ways
(i) 1 American out of 6 and 6 Indians out of 7. = 6C 5 + 6C 4 × 4C1 + 6C 3 × 4C 2
(ii) 2 Americans out of 6 and 5 Indians out of 7.
(iii) 3 Americans out of 6 and 4 Indians out of 7. = 6 + 60 + 120 = 186
In case I the number of ways = 6C1 × 7C 6 = 6 × 7 = 42 25 A voter may cast vote for either 1 candidate
or 2 candidates or 3 candidates or 4 candidates out of 7.
In case II the number of ways = C 2 × C 5 = 15 × 21 = 315
6 7
The voter may cast vote for 1 candidate in 7 C1 ways
In case III the number of ways
The voter may cast vote for 2 candidates in 7 C 2 ways
= 6C 3 × 7C 4 = 20 × 35 = 700
The voter may cast vote for 3 candidates in 7 C 3 ways
Hence, the required number of ways The voter may cast vote for 4 candidate in 7 C 4 ways
= 700 + 315 + 42 = 1057 Hence, the required number of ways
19 Required number of ways = 6C 4 × 8C 3 = 15 × 56 = 840 = 7 C1 + 7 C 2 + 7 C 3 + 7 C 4
20 The number of committees in which both Miss. A and Mr. B = 7 + 21 + 35 + 35 = 98
are always members. We can select rest 3 men from the
26 Required number of ways = 5C1 + 5C 2 + 5C 3
remaining 5 in 5C 3 ways and rest 2 women from the
= 5 + 10 + 10 = 25
remaining 7 in 7 C 2 ways.
27 Required number of ways = 8C 6 × 8C 5 = 28 × 56 = 1568
∴The required number of ways in which both Miss. A
and Mr. B are always included 28 Required number of ways
= C 3 × C 2 = 10 × 21 = 210
5 7 = (6C 4 × 6C 2 ) + (6C 3 × 6C 3 ) + (6C 2 × 6C 4 )

Hence the required number of ways in which Miss. A and = (15 × 15) + (20 × 20) + (15 × 15)
Mr. B do not serve together = 840 − 210 = 630 = 225 + 400 + 225 = 850
21 Since 3 particular members are already selected, then we 29 Required number of ways
are required to select only 4 members out of the remaining = 7C 3 × 5C 5 + 7C 4 × 5C 4 + 7C 5 × 5C 3
8 members. It can be done in 8C 4 ways.
= 35 × 1 + 35 × 5 + 21 × 10
∴ 8
C 4 = 70 = 35 + 175 + 210 = 420
22 Since 3 particular members are not included hence we have 30 Required number of ways
to select 7 members out of remaining 8 members. It can be = (5C 2 × 5C 3 × 4C1 ) + ( 5C 3 × 5C 2 × 4C1 ) + (5C 2 × 5C 2 × 4C 2 )
done in 8C7 ways
= (10 × 10 × 4) + (10 × 10 × 4) + (10 × 10 × 6) = 1400
∴ 8
C7 = 8
31 Required number of ways
= 13C 8 × 5C 2 = 1287 × 10 = 12870
Permutations & Combinations 1095

32 One hand shake requires two different persons. ∴Required number of ways
∴ Number of hand shakes = (4C 3 × 2C1 × 10C7 ) + (4C 3 × 2C 2 × 10C 6 )
= number of ways of selecting two persons out of 11 persons + (4C 4 × 2C1 × 10C 6 ) + (4C 4 × 2C 2 × 10C 5 )
= C 2 = 55
11
= (3 × 1 × 7 ) + (3 × 2 × 6) + (4 × 1 × 6) + (4 × 2 × 5)
n = 960 + 840 + 420 + 252
33 If n is even, C r is maximum when r =
n
2
= 2472
24
∴ Number of invitees in a party = = 12 40 Total number of bowles = 6
2
and maximum possible number of parties = 24
C12 Total number of wicket keepers = 3
Total number of normal players = 11 [ 20 − (6 + 3)]
= 2704156 Possible combinations :
34 If n is odd, nC r is maximum when
Wicket Normal
n −1 n+1 Bowlers
r= or r = keepers players
2 2
4 2 5
11 − 1
∴ r= =5 5 2 4
2
6 2 3
11 + 1
or r= =6
2 ∴Required number of ways
∴ 11
C 5 = 11C 6 = 462 = (6C 4 × 3C 2 × 11C 5 ) + (6C 5 × 3C 2 × 11C 4 )
35 Possible number of members in a family is either 5 or 6. + (6C 6 × 3C 2 × 11C 3 )
So, the maximum possible number of family members = 6 = 20790 + 5940 + 495 = 27225
36 Let there were n persons in the meeting, then number of 41 Total number of wicket keepers = 2
handshakes = nC 2
Total number of bowlers = 8
∴ n
C 2 = 66 Total number of batsman = 10
n! Total number of all rounders = 5
⇒ = 66
2!(n − 2)! 1 Wicket keeper can be selected out of 2 in 2C1 ways
n (n − 1) 2 Bowlers can be selected out of 8 in 8C 2 ways
⇒ = 66
2
5 Batsmen can be selected out of 10 in 10C 5 ways
⇒ n (n − 1) = 132
⇒ n = 12 3 All rounders can be selected out of 5 in 5C 3 ways
Alternatively Go through options. ∴Required number of ways = 2C1 × 8C 2 × 10C 5 × 5C 3
37 Since a particular player is always chosen therefore we = 2 × 28 × 252 × 10 = 141120
have to choose only 10 players out of remaining 15 players. 42 1 book on Economics can be collected out of 3 in 3C1 ways
∴ Required number of ways = 15C10 = 3003
1 book on Corporate strategy can be collected out of 4 in
38 Since a player is never chosen, hence we have to choose 4
C1 ways 1 book on Philosophy can be collected out of
11 players out of remaining 15 players. 5 in 5C1 ways
∴ Required number of ways = C11 = 1365
15
Hence the required number of collections each of which
39 Total number of bowlers = 4 consists of exactly one book on each subject
Total number of wicket keepers = 2 = 3C1 × 4C1 × 5C1
Rest (normal) players = 10
= 3 × 4 × 5 = 60
Possible Combinations :
43 At least one book on Economics can be collected out of
Bowlers
Wicket Normal 3 = 3C1 + 3C 2 + 3C 3 = 23 − 1 = 7 ways.
keepers players
Similarly one book on corporate strategy can be collected
3 1 7 out of 4 = 4C1 + 4C 2 + 4C 3 + 4C 4 = 24 − 1 = 15 ways.
3 2 6
Similarly one book on Philosophy can be collected out of
4 1 6 5 = 5C1 + 5C 2 + 5C 3 + 5C 4 + 5C 5 = 25 − 1 = 31 ways.
4 2 5
Hence the required number of ways = 7 × 15 × 31 = 3255
1096 QUANTUM CAT
44 Since all the three balls are red which can be selected from 51 Let L1, L 2, L 3 be 3 ladies and G1, G 2, G 3 be 3 gents.
7 red balls only.
Any two of the ladies out of 3 can be selected in 3C 2 ways
∴ Required number of selections = 7C 3 = 35
and can be arranged mutually in 2! ways.
45 Since red balls cannot be taken thus we can select 3 balls Now we are left with 3 gents and 1 lady these 4 persons can
out of 6 white and 4 blue balls. be arranged at a round table in 3! ways.
∴ Required number of selections = 10C 3 = 120 ways
Now we have only two places among 4 persons to place the
46 The required number of selections = 7C1 × 6C1 × 4C1 = 168 two selected ladies which can be done in 2 P1 ways.
47 Total number of red balls = 5 Hence the required number of arrangements
Total number of green balls = 6 = 3C 2 × 2! × 3! × 2P1
Possible combinations are : = 3 × 2 × 6 × 2 = 72
Red Green 52 Since 4 guests desire to sit on one side and the other
2 4 3 guests desire to sit on the other side.
3 3 A1 A2 A3 A4
4 2 A

∴Required number of selections B


= ( 5C 2 × 6C 4 ) + ( 5C 3 × 6C 3 ) + ( 5C 4 × 6C 2 ) = 425 B1 B2 B3
48 Required number of words = 15C 2 × 5C 4
Hence we are left with 11 guests only out of which we
= 105 × 5 = 525 choose 5 guests for the side A in 11C 5 ways and remaining
49 There are 11 letters in the given word of which 2 are A’s, 6 guests can be selected for the side B in 6C 6 ways.
2 are I’s, 2 are N’s and the remaining 5 letters are different.
Thus we have 11 letters of 8 different kinds viz., Further in each side 9 guests can be arranged in 9! ways.
( A, A ), (I, I ), (N , N ), E, X, M, T, O. Hence the required number of arrangements
A group of 4 letters can be classified as follows : = 11C 5 × 6C 6 × 9 ! × 9 !
(i) Two alike of one kind and two alike of another kind. = 462 × (9 !)2
(ii) Two alike and the other two different.
(iii) All four different. 53 10
C1 + 10
C 2 + 10C 3 + K + C10 = 210 − 1
10

In case I, the number of ways = 3C 2 = 3 54 Q 10C1 + 10


C2 + 10
C3 + K + C10 = 210 − 1
10

In case II, the number of ways = 3C1 × 7C 2 = 63 ∴ 10


C2 + 10
C3 + K + 10
C10 = (210 − 1) − 10 = 210 − 11
In case III, the number of ways = C 4 = 70
8
55 Q 10C1 + 10
C2 + 10
C3 + K + C9 +
10 10
C10 = 210 − 1
Hence, the required number of ways = 3 + 63 + 70 = 136
∴ 10
C1 + 10
C2 + 10
C3 + K + 10
C9
50 There are 11 letters in the given word where M occurs
2 times, A occurs 2 times and T occurs 2 times and rest = (2 10
− 1) − 1 = 2 10
−2
letters occur one times each. = 2(29 − 1)
i.e., M, A, T, H, E, I, C, S
56 10
C0 + 10
C 2 + 10C 4 + . . . + C10 = 210 − 1 = 29
10
M, A, T
A group of 4 letters can be selected in the following 57 [(10C1 + 10
C3 + 10
C 5 + . . . ) × (20C1 + 20
C3
manner. + 20
C 5 + . . . ) × (30C1 + 30
C3 + 40
C5 + . . . )
(i) Two alike of one kind and two alike of other kind.
× ... × ( 100
C1 + C3 +
100 100
C 5 + . . . )]
(ii) Two alike and the other two different.
(iii) All four different. = 29 × 219 × 229 × . . . × 299
4!
Case I. 3C 2 × = 18 = 29 + 19 + 29 + K + 99 = 2540
2! 2!
4! 58 Larry can attend or miss the first session in 2 ways.
Case II. 3C1 × 7C 2 × = 756
2! She can attend or miss the second session in 2 ways.
Case III. C 4 × 4 ! = 1680
8
She can attend or miss the third session in 2 ways.
∴ Total required words of 4 letters = 18 + 756 + 1680 She can attend or miss the fourth session in 2 ways.
= 2454 Thus she can attend the sessions in 2 × 2 × 2 × 2 = 24 ways.
Permutations & Combinations 1097

But this includes the case when she does not attend even a 66 They can watch at most 3 movies.
single session. Therefore she can attend one or more
Therefore the required number of ways in which they can
sessions in 24 − 1 = 15 ways. watch the movies
Alternatively = 4C 0 + 4C1 + 4C 2 + 4C 3
She can attend the one session in 4C1ways = 24 − 4C 4 = 24 − 1
4
She can attend the two sessions in C 2 ways 67 He may choose to read none, one or any number of scripts
She can attend the three sessions in 4C 3 ways offered.
4
She can attend the four sessions in C 4 ways Therefore the number of ways in which he can choose to
read the scripts
Therefore she can attend 1 or 2 or 3 or 4 sessions in
= 100C 0 + 100C1 + 100C 2 + K + 100C100 = 2100
4
C1 + 4C 2 + 4C 3 + 4C 4 = 4 + 6 + 4 + 1 = 15 ways.
68 Since he reads at most 50 scripts.
59 The number of ways in which Madhulika can select exactly
four languages = C4 + C4 + C4 + . . . +
4 5 6 10
C4 Therefore the number of ways in which he can choose to
read the scripts
= 11
C 5 = 462
= 100C 0 + 100
C1 + 100
C 50 = 2100 −
r +1 r +2 n+1
NOTE (Q C r +
r
Cr + Cr + K + Cr =
n
C r + 1; r ≤ n) (100C 51 + 100
C 52 + . . . + 100
C100 )
Madhulika cannot select 4 languages from program 1, 2 or 3 as
⇒ ( 100
C0 + 100
C1 + K + 100
C 49 )
none of them has 4 languages.
+ 100
C 50 = 2 100
−( 100
C 51 + 100
C 52 + K + 100
C100 )
60 210 = 1 + 2 + 3 + K + 20.
⇒ P+ 100
C 50 = 2 100
−P
It means total 20 packs of parathas are there and each pack
⇒ P+ 100
C 50 = 2 100
− P − 100C 50 + 100
C 50
has distinct number of parathas. The number of ways in
which Shivesh can select any 3 parathas ⇒ P+ 100
C 50 = 2 100
− (P + 100
C 50 ) + 100
C 50
= 3C 3 + 4C 3 + 5C 3 + K + 20
C3 ⇒ 2(P + 100
C 50 ) = 2 100
+ 100
C 50
= 21
C 4 = 5985 1
⇒ P+ 100
C 50 = (2100 + 100
C 50 )
2
61 Since Nilekani attempts 2 problems so he can’t attempt the
∴ (100C 0 + 100
C1 + 100
C2 + K + 100
C 49 )
module 1, as it has only 1 problem.
1 100
Therefore the number of ways in which he can attempt + 100
C 50 = (2 + 100
C 50 )
2
2 distinct problems
= 2C 2 + 3C 2 + 4C 2 + K + C2 =
40
C 3 = 10660
41 Hint (100 C 0 + 100
C1 + K + 100
C 49 )

62 Since Narainmurthi attempts 38 problems, so he has to =(100


C51 + 100
C52 + K + 100C100 )
attempt them from module 38 or module 39 or module 40 And 100
C0 + 100C1 + K + 100C100 = 2100
only, as other modules don’t have sufficient number of
69 The number of ways in which he can choose to read the
problems to attempt from. scripts
= 38
C 38 + C 38 +
39
C 38 =
40
C 39 = 820
41
= 100C 2 + 100
C3 + K + 100
C 98 = 2100 − (100C 0
63 They can choose not to watch any movie or can go for 1 or 2 + 100
C1 + 100
C 99 + 100
C100 )
or 3 or all the four movies. Therefore the required number
⇒ 100C 2 + 100
C3 + K + 100
C 98 = 2100 − (1 + 100 + 100 + 1)
of ways in which they can watch the movies
4
C 0 + 4C1 + 4C 2 + 4C 3 + 4C 4 = 24 = 16 = 42 ⇒ 100
C2 + 100
C3 + K + 100
C 98 = 2100 − 202

64 They can watch either 1 or more movies. 70 Since he reads at least 50 scripts.
Therefore the required number of ways in which they can Therefore the number of ways in which he can choose to
read the scripts
watch the movies
= 100C 50 + 100
C 51 + 100
C100 = 2100 − (100C 0
= 4C1 + 4C 2 + 4C 3 + 4C 4 = 24 − 4C 0 = 24 − 1
+ 100
C1 + K + 100
C 49 )
65 They can watch 2 or more movies.
⇒ 100
C 50 + ( 100
C 51 + 100
C 52 + K + 100
C100 )
Therefore the required number of ways in which they can
watch the movies 4 C2 + 4 C3 + 4 C4 = 24 − 4 C0 − 4 C1 = 24 − 5 =2 100
−( 100
C0 + 100
C1 + K + 100
C 49 )
1098 QUANTUM CAT
⇒ 100
C 50 + P = 2100 − P 75 Mr. Shashi uses only even number of characters to write a
⇒ 100
C 50 + P = 2100 − P − 100C 50 + 100C 50 tweet from the set of 36 characters.
Q n
C 0 + nC 2 + nC 4 + K + nC n = 2n − 1
⇒ 100
C 50 + P = 2 100
− (P + 100
C 50 ) + 100
C 50
∴ n
C 2 + nC 4 + . . . nC n = 2n − 1 − nC 0
⇒ 2(100
C 50 + P ) = 2 100
+ 100
C 50
⇒ n
C 2 + nC 4 + K nC n = 2n − 1 − 1
1 100 100
⇒ 100
C 50 + P = (2 + C 50 )
2 ∴ 36
C2 + C4 + K
36 36
C 36 = 235 − 1
1
⇒ 100
C 50 + P = 299 + (100C 50 ) 76 Mr. Shashi uses only odd number of characters to write a
2
tweet from the set of 36 characters.
1 100
∴ (100C 50 + C 51 +
100 100
C100 ) = 299 + ( C 50 ) Q C1 + nC 3 + nC 5 + K = 2n − 1
n
2
∴ 36
C1 + 36
C3 + C5 + K +
36 36
C 35 = 235
Hint (100 C51 + 100
C52 + K + 100
C100 )
77 Mr. Shashi uses only even number of characters from
= (100 C 0 + 100
C1 + K + 100
C 49 )
26 alphabets and even number of characters from
n 10 numerals to write a tweet.
71 If n is even, the greatest value of C r = C m; where m =n n
2 ∴ (26C 0 + C2 +
26 26
C4 K 26
C 26 )
It implies that the maximum number of modules can be
( C0 +
10 10
C2 + 10
C4 . . . C10 ) = 225 × 29 = 234
10
 10
developed when each module has 5  =  subjects.
 2 Thus we get the answer 234, which includes a case in which
And the maximum number of modules would be 0 characters are used. But, we know that he cannot
10
C 5 = 252. tweet without writing anything in it, so 1 case has to be
subtracted.
72 If n is odd, the greatest value of nC r = nC m where
(n − 1) (n + 1) Therefore the number of ways in which he can select the
m= or m = required characters = 234 − 1
2 2
It implies that the maximum number of distinct 78 Mr. Shashi uses only odd number of characters from
assortments can be obtained when each assortment has 26 alphabets and odd number of characters from 10
(9 − 1) (9 + 1)
= 4 or = 5 varieties of sweets. numerals to write a tweet.
2 2
And the maximum number of distinct assortments ∴ (26C1 + 26
C3 + 26
C5 K 26
C 25 )
= 9C 4 = 9C 5 = 126 ( C1 +
10
C3 +
10 10
C5 . . . 10
C9)= 2 25
× 29 = 234
n
73 If n is even, the greatest value of nC r = nC m; where m = 79 When Mr. Shashi selects odd number of characters from
2
26 alphabets and odd number of characters from 10
It implies that the maximum number of groups can be numerals, he naturally he selects total even number of
 8
formed when in each group there are 4  =  members. characters.
 2
Thus the number of ways in which he can select the
And the number of groups = 8C 4 = 70 characters
74 For maximum number of trips/groups 4 members group = (26C1 + 26
C3 + 26
C5 K 26
C 25 )
will be travelling each time. So a particular member can go
( C1 +
10 10
C3 + 10
C5 . . . 10
C9)
with any 3 members out of the remaining 7 members.
Therefore the maximum number of trips by a particular =2 25
×2 =2 9 34

member = 7C 3 = 35

Introductory Exercise 19.6


1 The number of ways of choosing 4 letters from 2As, 3Bs Therefore the required number of ways of choosing
and 4Cs = coefficient of x 4 in (1 + x + x 2 ) 4 letters from {A, A, B, B, B, C, C, C, C} is 11.
(1 + x + x 2 + x 3 )(1 + x + x 2 + x 3 + x 4 ) Hence choice (b) is the answer.
By multiplication we get (1 + K + 11 x 4 + K + x 9 ). Alternatively
4 Case I : 0, 0 4 {CCCC}
Thus we can see that the coefficient of x is 11.
Permutations & Combinations 1099

Case II : 0, 1, 3 {ABBB}, {ACCC}, {BCCC}, {CBBB} 9 Number of ways of selecting any number of tees
Case III : 0, 2, 2 {AABB}, {AACC}, {BBCC} = selecting no tee or 1 tee or 2 tees or 3 tees or … or 10 tees
Case IV : 1, 1, 2 {AABC}, {BBCA}, {CCAB} = 1 + 1 + 1 + 1 + K + 1 = 11
2 The number of ways of choosing 4 letters, such that at least Hence choice (d) is the answer.
1 letter must be taken from each of 2As, 3Bs and 10 Number of ways of selecting 10 tees = 1
4Cs = coefficient of x 4 in
Hence choice (b) is the answer.
( x + x 2 )( x + x 2 + x 3 )( x + x 2 + x 3 + x 4 )
11 Number of ways of selection of zero or more things out of
By multiplication we get ( p + q + r + K ) things, of which p are alike of one kind, q
( x + . . . + 3x + K + x ).
3 4 9 are alike of second kind, r are alike of third kind, and so on
= [( p + 1)(q + 1)(r + 1). . . ]
Thus we can see that the coefficient of x 4 is 3.
Therefore the required number of ways of choosing Therefore the number of ways of selection of any number
(≥ 0) of candles out of (2 + 3 + 4 + 3) candles, of which 2
4 letters from {A, A, B, B, B, C, C, C, C} is 3.
are alike of white colour, 3 are alike of red colour, 4 are
Hence choice (a) is the answer. alike of green colour and 3 are alike of yellow colour
Alternatively = (2 + 1)(3 + 1)(4 + 1)(3 + 1) = 240.
{AABC}, {ABBC}, {ABCC} Hence choice (b) is the answer.
Alternatively
NOTE The above solution shows that out of 2 white candles
First take out 1A, 1B, 1C. you may select any number of candles (0 or 1 or 2); or out of
3 red candles you may select any number of candles (0 or 1 or 2
Now you can select either 1A in 1 way or 1B in 1 way or 1C or 3); or out of 4 green candles you may select any number of
in 1 way. candles (0 or 1 or 2 or 3 or 4); or out of 3 yellow candles you
So the total number of selections = 1 + 1 + 1 = 3 . may select any number of candles (0 or 1 or 2 or 3).
Alternatively Essentially, it manifests that you may select any number of
{1A, 1B and 2C} or {1A, 2B, 1C} or {2A, 1B, 1C} candles (0 or 1 or 2 or … or 12) from the whole lot of 12 candles.

(1 × 1 × 1) + (1 × 1 × 1) + (1 × 1 × 1) = 3 12 Number of ways of selection of one or more things out of


( p + q + r + . . . ) things, of which p are alike of one kind, q
3 Number of ways of selecting 1 tee = 1 are alike of second kind, r are alike of third kind, and so on
Hence choice (b) is the answer. = [( p + 1)(q + 1)(r + 1) K ] − 1
4 Number of ways of selecting at least 1 tee Therefore number of ways of selection of at least one (≥ 1)
= selecting 1 tee or 2 tees or 3 tees or….or 10 tees candle out of (2 + 3 + 4 + 3) candles, of which 2 are alike of
= 1 + 1 + 1 + K + 1 = 10 white colour, 3 are alike of red colour, 4 are alike of green
Hence choice (d) is the answer. colour and 3 are alike of yellow colour
= [(2 + 1)(3 + 1)(4 + 1)(3 + 1) − 1] = 239.
5 Number of ways of selecting at most 1 tee = selecting no tee
or 1 tee = 1 + 1 = 2 Hence choice (b) is the answer.
Hence choice (c) is the answer. Hint As it’s clear from the previous question that there are
6 Number of ways of selecting 4 tees = 1 240 ways in which you may select 0 or 1 or 2 or … or
12 candles. It obviously includes 1 way in which none of the
Hence choice (b) is the answer. candles can be selected. So if we subtract 1 way from 240 ways,
7 Number of ways of selecting at least 4 tees we get the answer in which at least one candle must be
= selecting 4 tees or 5 tees or 6 tees or ... or 10 tees selected.

=1 + 1 + 1K+ 1 =7 NOTE Always keep in mind that the number of ways in which
you can select nothing (i.e., number of ways in which you can
Hence choice (d) is the answer.
select 0 things) is 1.
8 Number of ways of selecting at most 4 tees = selecting no
tee or 1 tee or 2 tees or 3 tees or 4 tees 13 Number of ways in which you can select 1 candle of each
colour = No. of ways of selecting 1 white candle × No. of
=1 + 1 + 1 + 1 + 1 = 5
ways of selecting 1 red candle × No. of ways of selecting
Hence choice (a) is the answer. 1 green candle × No. of ways of selecting 1 yellow candle
= 1 × 1 × 1 × 1 = 1. Hence choice (d) is the answer.
Hint Number of ways of selecting 1 article from n identical articles
= 1.
1100 QUANTUM CAT
14 Number of ways of selecting at least one candle of each Case II : 1, 1, 0, 0 {WR}, {WG}, {WY}, {RG}, {RY}, {GY}
colour = 2 × 3 × 4 × 3 = 72 Therefore there are total 10 ways of selection (or
Hence choice (b) is the answer. combination) of 2 candles. In case I both the candles have
same colour and in case II each of the two candles has
Hint Number of ways of selecting r articles from n( r ≤ n)
different colour.
identical articles = 1.
No. of Case I Case II
15 Number of ways in which you can select 2 candles of each Combinations White Red Green Yellow White Red Green Yellow
colour = No. of ways of selecting 2 white candles × No. of
ways of selecting 2 red candles × No. of ways of selecting 1 2 0 0 0 1 1 0 0
2 green candles × No. of ways of selecting 2 yellow candles 2 0 2 0 0 1 0 1 0
= 1 × 1 × 1 × 1 = 1. 3 0 0 2 0 1 0 0 1
Hence choice (b) is the answer. 4 0 0 0 2 0 1 1 0
5 – – – – 0 1 0 1
Hint Number of ways of selecting r article from n( r ≤ n)
6 – – – – 0 0 1 1
identical articles = 1.

16 Number of ways of selection of at least 2 white candles = 1 19 Number of ways of selecting 1 white, 2 red, 3 green and
2 yellow candles = 1 × 1 × 1 × 1 = 1
Number of ways of selection of at least 2 red candles
Hence choice (b) is the answer.
=1 + 1 = 2
Number of ways of selection of at least 2 green candles Hint Number of ways of selecting r articles from n( r ≤ n)
= 1 + 1 + 1 = 3. Number of ways of selection of at least 2 identical articles = 1.
yellow candles = 1 + 1 = 2
20 Number of ways of selecting 2 white, 3 red, 4 green and
Therefore number of ways of selection of at least 2 candles
2 yellow candles = 1 × 1 × 1 × 1 = 1
of each colour = 1 × 2 × 3 × 2 = 12
Hence choice (a) is the answer. Hence choice (b) is the answer.
17 Number of ways of selection of at most 2 white candles Hint Number of ways of selecting n articles from n identical
=1 + 1 + 1 = 3 articles = 1.
Number of ways of selection of at most 2 red candles 21 Number of ways of choosing 4 candles from 2 candles of
=1 + 1 + 1 = 3 white colour, 3 candles of red colour, 4 candles of green
Number of ways of selection of at most 2 green candles colour and 3 candles of yellow colour is the coefficient of
=1 + 1 + 1 = 3 x 4 in the expansion (1 + x + x 2 )(1 + x + x 2 + x 3 )
Number of ways of selection of at most 2 yellow candles (1 + x + x 2 + x 3 + x 4 )(1 + x + x 2 + x 3 )
=1 + 1 + 1 = 3 By multiplication we get (1 + 4 x + 10 x 2 + 19 x 3
Therefore number of ways of selection of at most 2 candles + 29 x 4 + 37 x 5 + 40 x 6 + 37 x7 + 29 x 8 + 19 x 9 + 10 x10
of each colour = 3 × 3 × 3 × 3 = 81
+ 4 x11 + x12 ).
Hence choice (d) is the answer.
Thus we can see that the coefficient of x 4 is 29.
18 The number of ways of choosing r objects from p objects of
one kind, q objects of second kind, and so on is the Hence choice (b) is the answer.
coefficient of x r in the expansion (1 + x + x 2 + K + x p ) Alternatively

(1 + x + x 2 + K + x q ). . . Case I : 4, 0, 0, 0 → No. of selections = 1


Case II : 3, 1, 0, 0 → No. of selections = 9
Therefore number of ways of choosing 2 candles from 2
candles of white colour, 3 candles of red colour, 4 candles Case III : 2, 2, 0, 0 → No. of selections = 6
of green colour and 3 candles of yellow colour is the Case IV : 2, 1, 1, 0 → No. of selections = 12
coefficient of x 2 in the expansion Case V : 1, 1, 1, 1 → No. of selections = 1
(1 + x + x 2 )(1 + x + x 2 + x 3 ) Therefore total number of required selections
(1 + x + x 2 + x 3 + x 4 )(1 + x + x 2 + x 3 ) = 1 + 9 + 6 + 12 + 1 = 29
By multiplication we get (1 + K + 10 x 2 + K + x12 ). 22 Number of ways of choosing 6 candles from 2 candles of
2
Thus we can see that the coefficient of x is 10. white colour, 3 candles of red colour, 4 candles of green
Hence choice (d) is the answer. colour and 3 candles of yellow colour is the coefficient of
Alternatively x 6 in the expansion (1 + x + x 2 ) (1 + x + x 2 + x 3 )

Case I : 2, 0, 0, 0 {WW}, {RR}, {GG}, {YY} (1 + x + x 2 + x 3 + x 4 )(1 + x + x 2 + x 3 )


By multiplication we get
Permutations & Combinations 1101

(1 + 4 x + 10 x 2 + 19 x 3 + 29 x 4 + 37 x 5 + 40 x 6 object of each kind must be included is the coefficient of x k


in the expansion ( x + x 2 + K + x p )( x + x 2 + K + x q ) K
+ 37 x7 + 29 x 8 + 19 x 9 + 10 x10 + 4 x11 + x12 ).
Therefore required number of ways of selecting 6 candles,
Thus we can see that the coefficient of r6 is 40. such that at least one candle of each colour must be there,
Hence choice (c) is the answer. is the coefficient of x 6 in the expansion of ( x + x 2 )
Alternatively ( x + x 2 + x 3 )( x + x 2 + x 3 + x 4 )( x + x 2 + x 3 )
Case I : 4, 2, 0, 0 → No. of selections = 3 By multiplication we get, ( x 4 + 4 x 5 + 9 x 6 + . . + x12 ).
Case II : 4, 1, 1, 0 → No. of selections = 3 It shows that the coefficient of x 6 is 9. Therefore the
Case III : 3, 3, 0, 0 → No. of selections = 3 required number of selections = 9.
Case IV : 3, 2, 1, 0 → No. of selections = 18 26 Number of ways of selection of zero or more things from p
Case V : 3, 1, 1, 1 → No. of selections = 3
identical things of one kind, q identical things of second
Case VI : 2, 2, 2, 0 → No. of selections = 4 kind, r identical things of third kind, and remaining n
Case VII : 2, 2, 1, 1 → No. of selections = 6 things are different = [( p + 1)(q + 1)(r + 1) 2n]
Therefore total number of required selections
Therefore the number of ways of selection of zero or more
= 3 + 3 + 3 + 18 + 3 + 4 + 6 = 40.
pen-drives = (3 + 1)(4 + 1)(25 ) = 640
Hint Please keep in mind that you can select more than 2 candles
Hence choice (d) is the answer.
from the set of red candles, not more than 3 candles from the set
of red and yellow candles. There is only one set, of green candles, 27 The number of ways of section of zero or more pen-drives
which have 4 candles. So it must be obvious to you that you = 640
cannot select more than 4 candles from any set of candles. Number of ways of selection of zero pen-drives = 1
23 Number of ways of selecting at least 6 candles = Number of Therefore number of ways of selection of one or more
ways in which one can select either 6 candles or 7 candles pen-drives = 640 − 1 = 639.
or 8 candles or ... or 12 candles Hence choice (a) is the answer.
= 40 + 37 + 29 + 19 + 10 + 4 + 1 = 140 28 The number of ways of selecting at most 1 pen-drive
Hence choice (b) is the answer. = number of ways of selecting no pen-drive
24 Number of ways of selecting at most 6 candles = Number of + number of ways of selecting 1 pen drive.
ways in which one can select either 6 candles or 5 candles Number of ways of not selecting any pen-drive = 1.
or 4 candles or … or 1 candle or no candle Number of ways of selecting 1 pen-drive is shown below:
= 40 + 37 + 29 + 19 + 10 + 4 + 1 = 140 Number of pen-drives from
Hence choice (d) is the answer.
Box 1 Box 2 Box 3
25 In order to select the 6 candles such that there must be
1 candle of each colour, first of all we can take out Case I 1 0 0
4 candles 1 from each colour. Now 2 more candles have to Case II 0 1 0
be selected which can be done in the following ways: Case III 0 0 1
Case I: 2, 0, 0, 0 → No. of selections = 3
Case II: 1, 1, 0, 0 → No. of selections = 6 Case I : 1 × 1 × 1 = 1
Therefore total number of required selections = 3 + 6 = 9 Case II : 1 × 1 × 1 = 1
Hence choice (a) is the answer. Case III : 1 × 1 × 5 = 5
Alternatively Therefore total number of ways in which 1 pen-drive can
Case I: 3, 1, 1, 1 → No. of selections = 3 be selected = 1 + 1 + 5 = 7.
Case II: 2, 2, 1, 1 = No. of selections = 6 Thus the total number of ways in which at most 1 pen-drive
Therefore total number of required selections = 3 + 6 = 9 can be selected = 1 + 7 = 8.
Hence choice (a) is the answer. Hence choice (b) is the answer.
29 The number of ways of selecting 1 pen-drive from each box
Hint Remember that you cannot select more than 2 candles of
white colour, so in case-I, you will have only 3 ways of = No. of ways of selecting 1 pen-drive from first box × No.
selections, instead of 4 ways. of ways of selecting 1 pen-drive from second box × No. of
ways of selecting 1 pen-drive from third box = 1 × 1 × 5 = 5
Alternatively
Hence choice (a) is the answer.
The number of ways of choosing k objects, from p objects of
one kind, q objects of second kind, and so on, such that one
1102 QUANTUM CAT
30 The number of ways of selecting 3 pen-drives is shown first box = 31
below: Therefore the required number of ways
= 3×4×31
Number of pen-drives from No. of ways of
Case Total = 372
Box 1 Box 2 Box 3 selection
Hence choice (a) is the answer.
I 3 0 0 1 ×1 ×1 =1 12 34 The number of ways of selecting any 2 identical pen-drives
0 3 0 1 ×1 ×1 =1 = Number of ways of selecting 2 pen-drives either from box
0 0 3 1 × 1 × 10 = 10 1 or 2 pen-drives from box 2 = 1 + 1 = 2
II 2 1 0 1 ×1 ×1 =1 32 Hence choice (c) is the answer.
2 0 1 1 ×1 × 5= 5 35 Number of ways of selecting any two distinct pen-drives is
1 0 2 1 × 1 × 10 = 10 shown below:
1 2 0 1 ×1 ×1 =1 Number of pen-drives from No. of ways of
0 1 2 1 × 1 × 10 = 10 selection
Box 1 Box 2 Box 3
0 2 1 1 ×1 × 5= 5
Case I 0 0 2 1 × 1 × 10 = 10
III 1 1 1 1 ×1 × 5= 5 5
Case II 0 1 1 1 ×1 × 5= 5
Therefore total number of ways in which 3 pen-drives can 1 0 1 1 ×1 × 5= 5
be selected = 12 + 32 + 5 = 49. Case III 1 1 0 1 ×1 ×1 =1
Hence choice (a) is the answer. Therefore total number of required ways
31 The number of ways of selecting 3 distinct pen-drives is = 10 + 5 + 5 + 1 = 21.
shown below: Hence choice (c) is the answer.

Number of pen-drives from No. of ways of 36 We have to select 7 coupons from 4 identical coupons and
selection 6 distinct coupons. Then the coupons can be selected in the
Box 1 Box 2 Box 3 following ways.
Case I 0 0 3 1 × 1 × 10 = 10 4 identical and 3 distinct coupons in 1 × 6C 3 ways
Case II 0 1 2 1 × 1 × 10 = 10 3 identical and 4 distinct coupons in 1 × 6C 4 ways
1 0 2 1 × 1 × 10 = 10 2 identical and 5 distinct coupons in 1 × 6C 5 ways
Case III 1 1 1 1 ×1 × 5= 5 1 identical and 6 distinct coupons in 1 × 6C 6 ways
Hence the required number of ways
Therefore total number of ways in which 3 pen-drives can = 6C 3 + 6C 4 + 6C 5 + 6C 6
be selected 10 + 10 + 10 + 5 = 35 = 26 − (6C 0 + 6C1 + 6C 2 ) = 64 − 22 = 42
Hence choice (c) is the answer.
37 We have to select 3 coupons from 4 identical coupons and
32 The number of ways of selecting 2 pen-drives of 4 GB,
6 distinct coupons. Then the coupons can be selected in the
3 pen-drives of 2 GB and 4 pen-drives of different storage
following ways.
capacities other than 2 GB and 4 GB = 1×1×5 = 5
3 identical and 0 distinct coupons in 1 × 6C 0 ways
Hence choice (a) is the answer. 2 identical and 1 distinct coupons in 1 × 6C1 ways
33 Number of ways of selecting at most 2 pen-drives from the 1 identical and 2 distinct coupons in 1 × 6C 2 ways
first box = 3 0 identical and 3 distinct coupons in 1 × 6C 3 ways
Number of ways of selecting at most 3 pen-drives from the
Hence the required number of ways
first box = 4
= 6C 0 + 6C1 + 6C 2 + 6C 3
Number of ways of selecting at most 4 pen drives from the
= 26 − (6C 4 + 6C 5 + 6C 6 ) = 64 − 22 = 42
Permutations & Combinations 1103

Introductory Exercise 19.7


1 The required number of ways = 10 − 3 + 1 = 8 7 The number of ways of selecting 4 months, out of
12 months, such that no two months are consecutive
2 When there is no restriction, the number of ways of = 12 − 4 + 1C 4 = 9C 4 = 126
selection of 3 candles = 10
C 3 = 120. The number of ways of
Hence choice (c) is the answer.
selection of 3 consecutive candles = 8
8 The number of ways of selecting 4 bogies out of 10 bogies,
Therefore the required number of ways = 120 − 8 = 112 such that no two bogies are adjacent ones = 10 − 4+ 1C 4 = 35
3 The number of ways of selection of 3 consecutive candles Number of ways of selecting 4 terrorists out of 8 terrorists
= 10 − 3 + 1 = 8
who board the train = 8C 4 = 70
The number of ways of selection of 2 consecutive candles
And the number of ways of arranging 4 terrorists in the four
= 10 − 2 + 1 = 9
bogies = 4 ! = 24
The number of ways of selection of 3 candles such that the
third candle is not the adjacent to any of the two Therefore the number of ways in which 4 terrorists board
consecutive candles the train in the required manner = 35 × 70 × 24 = 58800
= 7 + 6 + 6 + 6 + 6 + 6 + 6 + 6 + 7 = 56 9 Total number of teams = 4, Total number of members = 16
Thus the number of selection of 3 candles such that at least Total number of chairs = 21, Extra number of chairs = 5
2 candles are consecutive = 8 + 56 = 64
Now for a moment, solve the problem considering the four
Hint Let a, b, c, d, e, f, g, h, i, j be 10 consecutive candles in that teams as 4 individuals.
order only. Then we have the following explanation.
So, the number of seats available for 4 teams = 4 + 5 = 9
Total Number of ways of selecting 4 seats out of 9 seats for the
2 consecutive
Third candle number of 4 teams, such that no teams sit in the adjacent chairs
candles
cases = 9 − 4 + 1C 4 = 15
ab Except c, any 7 candles 7 Number of ways of mutually arranging the 4 teams
bc Except a and d, any 6 candles 6 = 4 ! = 24
cd Except b and e, any 6 candles 6
Number of ways of arranging the individuals within a team
de Except c and f , any 6 candles 6
when they are sitting next to each other = 2! × 3! × 5! × 6 !
ef Except d and g, any 6 candles 6
= 1036800
fg Except e and h, any 6 candles 6
Number of ways of selecting 16 chairs out of 21 chairs so
gh Except f and i, any 6 candles 6
that no two members of different teams sit together and
hi Except g and j, any 6 candles 6 members of each team must sit together
ij Except h, any 7 candles 7 = 15 × 24 × 1036800 = 373248000
4 Number of selections of k consecutive things out of n things 10 Number of couples = 6, Number of seats required = 12
in a row = (n − k + 1). Therefore number of selection of 5 Number of seats available = 20, Number of extra seats = 8
consecutive seats = 10 − 5 + 1 = 6 For a moment consider each couple as an individual, then
Hence choice (c) is the answer. the number of seats available for the couples = 6 + 8 = 14
Hint (1-5), (2-6), (3-7), (4-8), (5-9), (6-10) Number of ways of selecting 6 seats out of 14 seats for the
6 couples, such that no two couples sit in the adjacent
5 Number of selections of k consecutive things out of n things chairs = 14 − 6 + 1C 6 = 84.
in a row = (n − k + 1)
Number of ways of mutually arranging the 6 couples
Therefore number of selection of 4 consecutive seats = 6 ! = 720
= 10 − 4 + 1 = 7
As each couple can be arranged in 2!ways so all the couples
Hence choice (a) is the answer. can be internally arranged in
6 Number of selections of k consecutive things out of n things 2! × 2! × 2! × 2! × 2! × 2! = 64 ways.
in a row = (n − k + 1) Number of ways of selecting 12 seats out of 20 seats so that
Therefore number of selection of 4 consecutive seats no two couples sit together and each couple must sit
=7 − 4 + 1 = 4 together = 84 × 720 × 64 = 3870720

Hence choice (a) is the answer.


1104 QUANTUM CAT
11 Number of selections of k consecutive things out of n things Therefore the number of ways of selection of 2 points
in a circle = n ; when k < n. which are non-consecutive = 21 − 6 = 15.

Therefore number of selections of 6 consecutive lights out Since the first point (in this assumption it is P1) can be
of 12 lights = 12 . selected in 10 ways, as there are total 10 points. So the total
number of selections of 3 points such that none of them is
Hence choice (d) is the answer.
adjacent to each other = 10 × 15 = 150.
12 Number of selections of k consecutive things out of n things
But, since in the above solution each combination occurs
in a circle = n; when k < n . 150
3 times, so the required number of ways = = 50.
Therefore number of selections of 2 consecutive chairs out 3
of 5 chairs = 5.
Hence choice (d) is the answer.
NOTE When you select the first point P1, then the other two
points will be (say) P3 and P5 . But when you select the first point
13 Number of selections of 4 consecutive chairs out of P3 , then the other two points will be (say) P5 and P1. Similarly,
10 chairs = 10. when you select the first point P5 , then the other two points will
Hence choice (b) is the answer. be (say) P1 and P3 . That’s why I’m saying that each combination
occurs 3 times.
14 Let V1, V2, V3, ... V10 be the 10 vertices of the 10-sided
polygon, then there will be 10 consecutive pairs of vertices 17 (i) First of all keep all the chairs of same color together.
Now the two groups can be arranged in 2! ways. Now
viz. V1V2, V2V3, V3V4, ..., V10V1.
the red chairs can be arranged in 6 ! ways and blue
Now each pair can be connected with 7 other vertices. chairs can be arranged in 4 ! ways. Therefore the
1 vertex is not counted here to avoid the repetition.
required number of arrangements = 2! × (6 ! × 4 !)
For example, V1V2 can be connected to V3, V4, V5, V6, V7 ,
(ii) At first, consider all the 6 red chairs as a single bundle.
V8, V9. Now, there are 4 blue chairs and 1 bundle of red chairs,
Similarly, V2V3 can be connected to V4, V5, V6, V7 , V8, V9, V10 . so total 5 articles are to be arranged, which can be done
… in 5!ways. But just a while back we have assumed that
… 6 red chairs as a bundle, which can be arranged in
Similarly, V10V1 can be connected to V2, V3, V4, V5, V6, V7 , V8. 6 ! ways. Therefore the total number of arrangements
So the total number of required triangles = 10 × 7 = 70. = 5! × 6 !
(iii) First of all arrange all the 6 red chairs, which can be
15 Total number of triangles obtained by connecting
done in 6! ways. Now after arranging 6 chairs there will
10 vertices of the polygon = 10C 3 = 120
be 7 places available for the 4 blue chairs to be
And the number of triangles with at least one coinciding arranged, which can be done in 7 C 4 × 4 ! = 840 ways.
side = 70
Therefore, the required number of ways = 7C 4 × 4 ! × 6 !
Therefore the number of triangles with no coinciding sides (iv) First of all arrange all the 5 red chairs in 5! ways and
= 120 − 70 = 50.
then arrange all the blue chairs in the right side of each
16 The problem is equivalent to the selection of 3 points on the red color chair in 5!. ways. Similarly all the blue chairs
circumference of a circle so that none of the 3 points are can be arranged in the left side of the each red chair.
adjacent to each other. Therefore the total number of arrangements
Let us consider the 10 points P1, P2, P3, ...,P10 in that order = 5! × 5! + 5! × 5! = 2(5! × 5!)
on the circumference. Now let us select a point P1, then we
(v) First of all arrange all the 6 identical red chairs in a row
cannot select the points P2 and P10 as they are the adjacent that can be done in 1 way. Now there will be 7 places
points to P1. for the 4 blue chairs to be arranged and that can be
Now we have 7 points P3, P4, P5, P6, P7 , P8, P9, from which done in 7 C 4 ways. Therefore, the required number of
we have to select 2 points such that they are not the ways = 1 × 7C 4 = 7C 4
adjacent to each other. This is like selecting (vi) First of all arrange all the 6 identical red chairs in a
2 non-consecutive points from the 7 points in a row. row, which can be done in 1 way. Now there will be
Number of ways of selection of 2 points out of 7 points, 7 places for the 4 blue chairs to be arranged and that
without any restriction = 7C 2 = 21 can be done in 7 C 4 × 4 ! ways. Therefore, the required
Number of ways of selection of 2 consecutive points out of number of ways = 1 × 7C 4 × 4 ! = 7C 4 × 4 !
7 points in a row = 7 − 2 + 1 = 6 Thus we see that none of the statements is wrong.
Permutations & Combinations 1105

18 On the first day of the week she can choose any of the 19 If Siddhartha practices with Deepika only once in a week, it
7 activities, but on the remaining days she can choose only can be done in 7 ways. If Siddhartha practices with her
6 activities as she cannot choose the activity that she twice a week, it can be done in 6 ways. Similarly, if
chooses the previous day. Siddhartha practices with her for 3, 4, 5, 6 and 7 days, it
So the required number of ways can be done in 5, 4, 3, 2 and 1 ways.
= 7 × 6 × 6 × 6 × 6 × 6 × 6 = 7 × 66 Therefore the required number of ways
= 7 + 6 + 5 + 4 + 3 + 2 + 1 = 28

Introductory Exercise 19.8


1 8 CDs can be selected from 12 CDs in 12C 8 ways and 4 CDs 5 6 CDs can be selected from 12 CDs in 12C 6 ways and other
can be selected from the remaining 4 CDs in 4C 4 ways. 6 CDs can be selected from the remaining 6 CDs in
6
C 6 ways.
Therefore total number of ways
12! 4! 12! Therefore total number of ways
= 12C 8 × 4C 4 = × = = 495 1 12! 6! 1
8! × 4! 4! × 0! 8! × 4! = 12C 6 × 6C 6 × = × ×
2! 6 ! × 6 ! 6 ! × 0 ! 2!
Hence choice (b) is the answer.
12! 1 12! 1
2 8 CDs can be selected from 12 CDs in 12C 8 ways and 4 CDs = × = × = 462
6 ! × 6 ! 2! (6 !)2 2!
can be selected from the remaining 4 CDs in 4C 4 ways. And
The division by 2!indicates that since the number of CDs in
since these CDs are given to 2 different students which can
each group is same so only half of the combinations will be
be exchanged mutually; that means if one student receives
8 CDs then another student will receive 4 CDs and there.
vice-versa. Hence choice (c) is the answer.
Therefore total number of ways in which 12 CDs can be 6 Number of ways in which 12 CDs can be distributed to two
given to 2 students  12! 1
French students equally =  ×  × 2! = 924
 12!   (6 !)2 2!
= (12C 8 × 4C 4 ) × 2! =   × 2! = 990
 8 ! × 4 ! The division by 2!indicates that the two groups are having
Hence choice (a) is the answer. equal number of CDs, so they are indistinguishable. And
the multiplication by 2! indicates that since the two
3 6 CDs can be selected from 12 CDs in 12C 6 ways and 4 CDs students are distinguishable so they can exchange the
can be selected from 6 CDs in 6C 4 ways and the remaining packets in 2! ways.
2 CDs can be selected from remaining 2 CDs in 2C 2 ways. Hence choice (d) is the answer.
Therefore total number of ways 7 Number of ways in which 12 CDs can be divided equally
= 12C 6 × 6C 4 × 2C 2 into three groups =
12!
×
1
=
12!
×
1
= 5775
12! 6! 2! 4 ! × 4 ! × 4 ! 3! (4 !)3
3!
= × ×
6 ! × 6 ! 4 ! × 2! 2! × 0 ! The division by 3! indicates that the three groups are
12! having equal number of CDs, so they are indistinguishable.
= = 13860
6 ! × 4 ! × 2! Hence choice (d) is the answer.
Hence choice (c) is the answer. 8. Number of ways in which 12 CDs can be distributed among
12! 1
4 The number of ways in which these CDs can be divided in three French students equally = 3
× × 3! = 34650
(4 !) 3 !
12!
the groups of 6, 4 and 2 is The division by 3! indicates that the three groups are
6 ! × 4 ! × 2!
having equal number of CDs, so they are indistinguishable.
But since these CDs can be exchanged among 3 students in And the multiplication by 3! indicates that the three
3! ways, so the number of ways in which these CDs can be students are distinguishable so they can exchange the
12!
given to three students = × 3! = 13860 × 6 packets in 3! ways. Hence choice (a) is the answer.
6 ! × 4 ! × 2!
9 Number of ways in which 12 CDs can be distributed among
= 83160 12! 1 12!
Hence choice (d) is the answer. six French students equally = × × 6! =
(2!)6 6 ! (2!)6
Hence choice (a) is the answer .
1106 QUANTUM CAT
10 Number of ways in which 12 CDs can be distributed to 14 The number of ways of dividing n identical items among
6 French students r persons (where 0 ≤ r ≤ n), each one of whom must receive
= First CD can be given to any 6 students, similarly second at least one item = n−1
Cr −1
CD, third CD….etc. can be given to any of the 6 students
= 6 × 6 × 6 × 6 × 6 × 6 × 6 × 6 × 6 × 6 × 6 × 6 = 612 Therefore the number of ways in which 12 identical
chocolates can be distributed among 4 kids such that each
Hence choice (a) is the answer.
of them must receive at least one chocolate
11 First of all distribute 2 CDs to each of the four French = 12 − 1C 4 − 1 = 11C 3 = 165
students. Then distribute the remaining 4 CDs in all the
possible ways. After distributing all the 12 CDs there will be Hence choice (d) is the answer.
five cases, as presented in the following table. Suppose A, Alternatively Since each kid has to receive at least one
B, C and D are four French students. chocolate, so first of all give 1 chocolate to each of the
4 kids, which will be done in 1 way as the chocolates are
Case A B C D identical and each one is getting equal number of
i 2 2 2 6 chocolates. Now you have to distribute just 8 chocolates
among 4 kids in such a way that anyone receives any
ii 2 2 3 5 number of chocolates. Since each kid has already received
iii 2 2 4 4 minimum 1 chocolate, so it won’t matter anymore that any
iv 2 3 3 4 particular kid is not getting any chocolate from the
remaining 8 chocolates.
v 3 3 3 3
Now, the number of ways in which 8 chocolates can be
4! distributed among 4 kids such that any kid can get any
Case (i) (12C 2 × 10C 2 × 8C 2 × 6C 6 ) ×
3! number of chocolates = 8 + 4 − 1C 4 − 1 = 165
= 83160 × 4 = 110880
Therefore the number of ways in which 12 identical
4!
Case (ii) ( C 2 × 10C 2 × 8C 3 × 5C 5 ) ×
12
chocolates can be distributed among 4 kids such that each
2! of them must receive at least one chocolate = 165 × 1 = 165
= 166320 × 12 = 1995840
15 Since each kid has to receive at least two chocolates, so first
4!
Case (iii) (12C 2 × 10C 2 × 8C 4 × 4C 4 ) × of all give 2 chocolates to each of the 4 kids, which will be
2! 2!
done in 1 way, as the chocolates are identical and each one
= 207900 × 6 = 1247400 is getting equal number of chocolates.
4!
Case (iv) (12C 2 × 10C 3 × 7C 3 × 4C 4 ) × Now you have to distribute just 4 chocolates among 4 kids
2! in such a way that anyone receives any number of
= 277200 × 12 = 3326400 chocolates. Since each kid has already received minimum
4! 2 chocolates, so it won’t matter anymore that any
Case (v) ( C 3 × 9C 3 × 6C 3 × 3C 3 ) ×
12
4! particular kid is not getting any chocolate from the
= 369600 × 1 = 369600 remaining 4 chocolates. Now, the number of ways in
which 4 chocolates can be distributed among 4 kids such
Therefore total number of the required ways = 7050120
that any kid can get any number of chocolates
12 Since none of the 4 students can have more than 3 CDs, 4 + 4 −1
= C 4 − 1 = 35
which implies that each one has to have exactly 3 CDs. This
can be easily done using the following formula. Therefore the number of ways in which 12 identical
The number of ways in which 12 CDs can be distributed chocolates can be distributed among 4 kids such that each
among 4 students equally of them must receive at least two chocolates = 35 × 1 = 35.
12! 1 12! 16. Since each of the 4 kids has to receive at least 3 chocolates,
= 4
× × 4! = = 369600
(3!) 4! (3!)4 it means all the 12 chocolates will have to be distributed
evenly (or equally). That means every one of them will get
13 The number of ways of dividing n identical items among
any 3 chocolates and this can be done in only 1 way as
r persons (where 0 ≤ r ≤ n ), each one of whom can receive
any number (0, 1, 2,…, n) of items = n + r − 1C r − 1 chocolates are identical and being distributed evenly.
17 Since each kid has to receive at least two chocolates, so first
Therefore the number of ways in which 12 identical
of all give 2 chocolates to each of the 4 kids, which will be
chocolates can be distributed among 4 kids
done in 1 way as the chocolates are identical and each one
= 12 + 4 − 1C 4 − 1 = 15C 3 = 455 is getting equal number of chocolates.
Hence choice (a) is the answer.
Permutations & Combinations 1107

After distributing 8 chocolates, now you have to distribute 21 Total number of non-negative integral solutions of the
30 + 3 − 1
remaining 4 chocolates among 4 kids. given equation = C3 −1 = C 2 = 496
32

(i) Since each kid has to receive at least 2 chocolates, so


now it’s not necessary to give any chocolate to every 22 Total number of positive integral solutions of the given
30 − 1
equation = C3 −1 = 29
C 2 = 406
kid, from the remaining 4 chocolates.
(ii) Since each kid has to receive not more than 23 When a = 2, b + c = 28, and the number of even positive
4 chocolates, so now it is obvious that no kid gets more integral solutions is 13, namely (2, 26), (4, 24), (6, 22), …,
than 2 chocolates, from the remaining 4 chocolates.
(26, 2)
This can be done in the following ways:
Similarly we can find the number of solutions for every
Case I 0, 0, 2, 2 → No. of selections = 6
value of a, as shown in the following table.
Case II 0, 1, 1, 2 → No. of selections = 12
a b+ c Number of solutions
Case III 1, 1, 1, 1 → No. of selections = 1
2 28 13
Therefore total number of required selection
4 26 12
= 6 + 12 + 1 = 19
6 24 11
Hence choice (b) is the answer.
... ... ...
Alternatively 26 4 1
Case I 2, 2, 4, 4 → No. of selections = 6
Therefore the total number of required solutions
Case II 2, 3, 3, 4 → No. of selections = 12
= 1 + 2 + 3 + K + 13 = 91
Case III 3, 3, 3, 3 = No. of selections = 1
Therefore total number of required selections 24 As you know, odd + odd + odd = odd, but since 30 is an
even number.
= 6 + 12 + 1 = 19.
So the required number of solutions = 0.
Alternatively The number of ways in which n identical
things can be divided into r groups so that no group 25 For a = 1, b + c = 29 and then it has 13 solutions such that a
contains less than m items and more than k items (where < b < c, namely (b, c)= (2, 27), (3, 26), (4, 25), …, (14, 15)
m < k) is coefficient of x n in the expansion of Similarly, we can find the number of solutions for a = 2, 3,
( x m + x m + 1 + K + x k )r 4, ...as shown in the table below.
Therefore the required number of ways of distribution of a b+c Number of solutions
chocolates = coefficient of x12 in the expansion of 1 29 13
( x 2 + x 3 + x 4 )4 2 28 11
Now, ( x 2 + x 3 + x 4 )4 = ( x 2 + x 3 + x 4 )2 × ( x 2 + x 3 + x 4 )2 3 27 10
4 26 8
⇒ ( x 2 + x 3 + x 4 )4 = ( x 4 + 2x 5 + 3x 6 + 2x7 + x 8 )
5 25 7
( x 4 + 2x 5 + 3x 6 + 2x7 + x 8 )
6 24 5
⇒ ( x 2 + x 3 + x 4 )4 = ( x 8 + K + 19 x12 + K + x16 ) 7 23 4
8 22 2
Since the coefficient of x12 is 19, so the required number
of ways = 19. Hence choice (b) is the answer. 9 21 1

18 Since the first kid can get any one lot out of 4 lots (1 or 2 or Therefore the number of solutions
4 or 5) of chocolates, then the second kid can get any one
= (1 + 4 + 7 + 10 + 13) + (2 + 5 + 8 + 11) = 61
lot of remaining 3 lots of chocolates and so on.
Therefore the number of ways of distributing Since a, b, c can be mutually arranged in 3! ways, so the
12 chocolates = 4 × 3 × 2 × 1 = 24 required number of solutions
Hence choice (b) is the answer. = 61 × 3! = 366
19 Since it’s already decided that who gets how many
26 For a = 0, b + c = 30 and then it has 14 solutions such that
chocolates, then the number of ways in which
12 chocolates can be distributed is 1. a < b < c, namely (b, c) = (1, 29), (2, 28), (3, 27), (4, 26), …,
Hence choice (d) is the answer. (14, 16)
20 Since each kid gets 3 identical chocolates so the number of Similarly, we can find the number of solutions for a = 2, 3,
ways in which 12 chocolates can be distributed = 1. 4, ... as shown in the table below.
Hence choice (d) is the answer.
1108 QUANTUM CAT
a b+ c Number of solutions NOTE The above sum can be broken into two parts for the
sake of convenience, as in (13 + 10 + 7 + 4 + 1) + (11 + 8 + 5 + 2)
0 30 14
= 61. Clearly there are 2 arithmetic progressions.
1 29 13
2 28 11 29 Let the number of red, pink and yellow roses be r, p and y,
3 27 10 respectively. Then, r + p + y = 30, such that r, p, y ≥ 0.
4 26 8
Therefore the required number of ways = Number of ways
5 25 7
of distributing 30 identical things among 3 persons such
6 24 5
that each may get 0 or more things
7 23 4
8 22 2 = 30 + 3 − 1C 3 − 1 = 32 C 2 = 496
9 21 1
30. Let the number of red, pink and yellow roses be r, p and y,
Therefore the number of solutions respectively. Then, r + p + y = 30, such that r, p, y ≥ 1.
= (1 + 14) + (2 + 13) + (4 + 11) + (5 + 10) + (7 + 8) = 75 Therefore the required number of ways = Number of ways
of distributing 30 identical things among 3 persons such
27 For a = 0, b + c = 30 and then it has 31 solutions such that
that each may get 1 or more things
a ≤ b and 0 ≤ c ≤ 30, namely (b, c) = (0, 30), (1, 29), (2, 28), 30 − 1
= C3 −1 = C 2 = 406
29
(3, 27 ), (4, 26), ..., (29, 1), (30, 0)
Similarly, we can find the number of solutions for a = 2, 3, 31 The number of non-negative integral solutions of the
n+ r −1
4, ... as shown in the table below. equation x1 + x 2 + x 3 + . . . + x r = n is Cr −1

a b+ c Number of solutions Therefore number of non-negative integral solutions of the


equation a + b + c + d = 12 is 12 + 4 − 1C 4 − 1 = 15C 3 = 425.
0 30 31
1 29 29 Hence choice (d) is the answer.
2 28 27
NOTE This problem is same as distribution of 12 identical
3 27 25
chocolates among 4 different persons.
4 26 23
... ... ... 32 The number of positive integral solutions of the equation
n− 1
... ... ... x1 + x 2 + x 3 + K + x r = n is C r − 1.
15 15 1
Therefore number of positive integral solutions of the
Therefore the number of solutions equation 12 − 1C 4 − 1 = 11C 3 = 165
= (1 + 3 + 5 + K + 29 + 31) = 256 Hence choice (b) is the answer.
28 Consider the following table. NOTE This problem is same as distribution of 12 identical
For example, when a = 1, 2b + c = 27, chocolates among 4 different persons, in such a way that
it will have 13 positive solutions. everyone gets at least 1 chocolate.
As in (b, c) = (1, 25), (1, 23), (3, 21), ..., (13, 1). 33 The number of solutions of a + b + c + d = 12 is the
Similarly, you can find the number of solutions for other coefficient of x12 in the expansion of ( x 2 + x 3 + x 4 )4
values of a.
Since, ( x 2 + x 3 + x 4 )4 = ( x 2 + x 3 + x 4 )2 × ( x 2 + x 3 + x 4 )2
a 2b + c Number of solutions
⇒ ( x 2 + x 3 + x 4 )4 = ( x 4 + 2x 5 + 3x 6 + 2x7 + x 8 )
1 27 13
( x 4 + 2x 5 + 3x 6 + 2x7 + x 8 )
2 24 11
3 21 10 ⇒ ( x 2 + x 3 + x 4 )4 = ( x 8 + K + 19 x12 + K + x16 )
4 18 8 As the coefficient of x12 is 19, so the required number of
5 15 7 solutions = 19. Hence choice (a) is the answer.
6 12 5 Alternatively Let us assume that there are 12 identical
7 9 4 balls that you are supposed to distribute among four kids
8 6 2 such that each of them receives at least 2 balls but not more
9 3 1
than 4 balls.
Therefore total number of solutions So, first of all give 2 balls to each of the four kids, now you
are left with four balls to distribute. That can be done in the
= 13 + 11 + 10 + 8 + 7 + 5 + 4 + 2 + 1 = 61
following way.
Permutations & Combinations 1109

Case I 0, 0, 2, 2 → No. of selections = 4C 2 = 6 (a − 1) + (b − 2) + (c − 3) + (d − 4)


Case II 0, 1, 1, 2 → No. of selections = 4C1 × 3C 2 = 12 = 12 − (1 + 2 + 3 + 4)
⇒ w + x + y + z = 2;
Case III 1, 1, 1, 1 → No. of selections = 1
(Where w = a − 1, x = b − 2, y = c − 3, z = d − 4)
Therefore total number of required selections
= 6 + 12 + 1 = 19 Now the number of non-negative integral solutions of
In case I, you can select any two kids out of four kids to dole w + x + y + z = 2 is
2+ 4 −1
out 2 balls to each one. C 4 − 1 = 5C 3 = 10
In case II, you can select 1 kid out of four kids to give her Therefore the number of integral solutions of
2 balls, then you can select any 2 kids out of the remaining
a + b + c + d = 12 , when a > 0, b > 1, c > 2, d > 3 is 10.
3 kids to dole out 1 ball to each of them.
Hence choice (a) is the answer.
In case III, you can give 1 ball to each one of the four kids in
1 way. NOTE This problem is same as distribution of 12 identical
Alternatively The number of integral solutions of the chocolates among 4 different persons, in such a way that one
equation a + b + c + d = 12, such that 2 ≤ a, b, c, d ≤ 4 is particular person gets more than 1 chocolate, another
same as the number of non-negative integral solutions of particular person gets more than 2 chocolates, another
particular person gets more than 3 chocolates and there is
p + q + r + s = 4, such that 0 ≤ a, b, c, d ≤ 2 for p = a − 2, another person who gets more than 4 chocolates.
q = b − 2, r = c − 2, s = d − 2.
36 Finding the number of integral solutions of the equation,
Now the number of solutions of p + q + r + s = 4, such that such that a > − 4, b > − 3, c > − 2, d > − 1 is same as finding
0 ≤ a, b, c, d ≤ 2 = (number of solutions of the number of integral solutions of the equation, such that
p + q + r + s = 4, such that 0 ≤ a, b, c, d ≤ 4) − (number of a ≥ − 3, b ≥ − 2, c ≥ − 1, d ≥ 0.
solutions of p + q + r + s = 4, such that 3 ≤ a, b, c, d ≤ 4)
The given equation a + b + c + d = 12 can be expressed as
But number of solutions of p + q + r + s = 4, such that
4 + 4 −1
following.
0 ≤ a, b, c, d ≤ 4 is C 4 − 1 = 7C 3 = 35
(a + 3) + (b + 2) + (c + 1) + (d + 0) = 12 + (3 + 2 + 1 + 0)
And number of solutions of p + q + r + s = 4, such that ⇒ w + x + y + z = 18;
3 ≤ a, b, c, d ≤ 4 is 4 [( 4 − 3) + 4 − 1C 4 − 1] = 4 [ 4C 3] = 16 (Where w = a + 3, x = b + 2, y = c + 1, z = d + 0)
Now the number of non-negative integral solutions of
Therefore, number of solutions of p + q + r + s = 4, such
w + x + y + z = 18 is
that 0 ≤ a, b, c, d ≤ 2, = 35 − 16 = 19. 18 + 4 − 1
C4 −1 = C 3 = 1330
21
So the number of integral solutions of the equation
a + b + c + d = 12, such that 2 ≤ a, b, c, d ≤ 4 is 19. Therefore the number of integral solutions of
a + b + c + d = 12 when a > − 4, b > − 3, c > − 2 ,
34 The given equation a + b + c + d = 12 can be expressed as
d > − 1 is 1330
following.
Hence choice (a) is the answer.
(a − 2) + (b − 2) + (c − 2) + (d − 2) = 12 − (2 + 2 + 2 + 2)
37 The given equation a + b + c + d = 12 can be expressed as
⇒ w + x + y + z = 4;
following a + b + 6 + 0 = 12 ⇒ a + b = 6
(Where w = a − 2, x = b − 2, y = c − 2, z = d − 2)
Now the number of non-negative integral solutions of the
Now the number of non-negative integer solutions of
w + x + y + z = 4 is 4 + 4 − 1C 4 − 1 = 7C 3 = 35 equation a + b = 6 such that a ≥ 1 and b ≤ 4.

Therefore the number of integral solutions of a b a+ b


a + b + c + d = 12, when a, b, c, d ≥ 2 is 35. 2 4 6
Hence choice (a) is the answer. 3 3 6
NOTE This problem is same as distribution of 12 identical 4 2 6
chocolates among 4 different persons, in such a way that 5 1 6
everyone gets at least 2 chocolates. 6 0 6
35 Finding the number of integral solutions of the equation,
Therefore the total number of solutions of the given
such that a > 0, b > 1 , c > 2, d > 3 is same as finding the
equation will be 5. Hence choice (c) is the correct one.
number of integral solutions of the equation, such that
a ≥ 1, b ≥ 2, c ≥ 3 d ≥ 4 38 The given equation a + b + c + d = 12 can be expressed as
Now the given equation a + b + c + d = 12can be expressed following.
as following. (a + 2) + (b + 2) + (c + 2) + (d + 2) = 12 + (2 + 2 + 2 + 2)
1110 QUANTUM CAT
⇒ w + x + y + z = 20 ; 41 The number of integral solutions of a + b + c = 8, if
(where w = a + 2, x = b + 2, y = c + 2, z = d + 2) 0 ≤ a, b, c ≤ 4 = (Number of integral solutions of
Now, finding the number of integral solutions of the a + b + c = 8, if 0 ≤ a, b, c ≤ 8) − (number of integral
equation a + b + c + d = 12, such that − 2 ≤ a, b , c, d ≤ 19 is solutions of a + b + c = 8, if any of a, b or c is such that
same as finding the number of integral solutions of the
5 ≤ a, b, c ≤ 8). Number of integral solutions of
equation w + x + y + z = 20, such that 0 ≤ w, x, y, z ≤ 21.
a + b + c = 8, if 0 ≤ a, b, c ≤ 8 = 8 + 3 + 1C 3 − 1 = 10C 2 = 45.
Since none of w, x, y or z can have any value greater than
20, so the upper limit (21) on w, x, y and z is irrelevant. Number of integral solutions of a + b + c = 8, if any of a, b
or c is such that 5 ≤ a, b, c ≤ 8)
Thus the given problem is same as finding the number of
non-negative integral solutions of w + x + y + z = 20. = 3[( 8 − 5) + 3 − 1C 3 − 1] = 3[ 5C 2] = 30
Therefore the required number of solutions Therefore number of solutions of a + b + c = 8, if
= 20 + 4 − 1C 4 − 1 = 23C 3 = 1771 0 ≤ a, b, c ≤ 4 = 45 − 30 = 15.
39 The following two problems are just the same, the only Hence choice (a) is the answer.
difference lies in the sign. Hint The above solution works on the basis of inclusion-exclusion
principle. According to that principle, the set of integral solutions
There are four numbers There are four numbers of a + b + c = 8, if 0 ≤ ( a , b, c) ≤ 4 do not include those solutions in
a, b, c, d such that a, b, c, d such that which either of a , b or c is greater than 4.
a + b + c + d = − 12 . a + b + c + d = 12 . Find However, the set of integral solutions of a + b + c = 8, if
Find the number of integral the number of integral 5 ≤ ( a , b , c) ≤ 8 includes only those solutions which have at
solutions of the equation, solutions of the equation, least one variable out of a , b and c greater than or equal to
such that a, b, c, d < 0 such that a, b , c, d > 0 5; so it’s obvious that few of the variables will be certainly
Answer : 165 Answer : 165 greater than or equal to 5 and few of them must be less
than 5. So in this case each of the variables may not be
40 The following two problems are just the same, the only greater than or equal to 5. Why does it happen? It’s because
difference lies in the sign. of the objective.
As our objective is to exclude all those solutions in which
There are four numbers There are four numbers even if one variable is greater than or equal to 5 rather than
a, b, c, d such that a, b, c, d such that to include all those solutions in which every variable is
a + b + c + d = − 12 . Find a + b + c + d = 12 . Find greater than or equal to 5. For better understanding have a
the number of integral the number of integral look at the above list of solutions.
solutions of the equation, solutions of the equation, Alternatively The number of solutions of a + b + c = 8
such that a, b, c, d ≤ 0 such that a, b , c, d ≥ 0 is the coefficient of x 8 in the expansion of
Answer : 425 Answer : 425 (1 + x + x 2 + x 3 + x 4 )3

Solutions (for Q. Nos. 41 to 44) a + b + c = 8 Since, (1 + x + x 2 + x 3 + x 4 )3


Case I 0 0 8 → 3 ways/solutions = (1 + x + x 2 + x 3 + x 4 )2 × (1 + x + x 2 + x 3 + x 4 )
Case II 0 1 7 → 6 ways/solutions ⇒ (1 + x + x 2 + x 3 + x 4 )3
Case III 0 2 6 → 6 ways/solutions
= (1 + 2x + 3x 2 + 4 x 3 + 5x 4 + 4 x 5
Case IV 0 3 5 → 6 ways/solutions
Case V 0 4 4 → 3 ways/solutions + 3x 6 + 2x7 + x 8 )(1 + x + x 2 + x 3 + x 4 )
Case VI 1 1 6 → 3 ways/solutions ⇒ (1 + x + x 2 + x 3 + x 4 )3
Case VII 1 2 5 → 6 ways/solutions = (1 + 3x + . . . + 15x 8 + . . . + 3x11 + x12 )
Case VIII 1 3 4 → 6 ways/solutions
Case IX 2 2 4 → 3 ways/solutions As the coefficient of x 8 is 15, so the required number of
Case X 2 2 33 →3 ways/solutions solutions = 15.
Alternatively
NOTE Please keep in mind that this is just a reference list to
aid to your visualization of solutions. Actually, when the a 4 4 4 4 4 3 3 3 3 2 2 2 1 1 0
number of solutions are too big to determine by preparing the b 4 3 2 1 0 4 3 2 1 4 3 2 4 3 4
list we have to resort to more logical methods. So it is important
to learn the following methods, otherwise whenever large c 0 1 2 3 4 1 2 3 4 2 3 4 3 4 4
figures (or numbers) will be involved you will be grouping in
Thus there are total 15 solutions.
the dark with no way out to get the solutions.
Permutations & Combinations 1111

42 For the number of integral solutions of a + b + c = 8, for 0 ≤ a, b, c ≤ 2 is 45 − 45 = 0.


0 ≤ a, b, c ≤ 3, first of all find the total number of integral It shows that there are no solutions to the equation
solutions of a + b + c = 8 for 0 ≤ a, b, c ≤ 8 and then a + b + c = 8 for 0 ≤ a, b, c ≤ 2
subtract the number of integral solutions of a + b + c = 8 Hence choice (d) is the answer.
if any of a, b or c is such that 4 ≤ a, b, c ≤ 8. Alternatively The number of solutions of a + b + c = 8
The number of integral solutions of a + b + c = 8 for is the coefficient of x 8 in the expansion of (1 + x + x 2 )3,
0 ≤ (a, b, c) ≤ 8 is 8 + 3 − 1C 3 − 1 = 10C 2 = 45 . which is 0, as the highest coefficient in this expansion is x 6.
And the number of integral solutions of a + b + c = 8 if 44 To find the number of integral solutions of a + b + c = 8, for
any of a, b or c is such that 4 ≤ a, b, c ≤ 8 is 0 ≤ (a, b, c) ≤ 1, first of all find the total number of solutions
3[ ( 8 − 4) + 3 − 1C 3 − 1] − 3[ ( 8 − 2 × 4) + 3 − 1C 3 − 1] of a + b + c = 8 for 0 ≤ (a, b, c) ≤ 8 and then subtract the
= 3[ 6C 2] − 3[ 2C 2] = 42 number of solutions of a + b + c = 8 if any of a, b or c is such
that 2 ≤ (a, b, c) ≤ 8. The number of solutions of
Therefore number of integral solutions of a + b + c = 8, for a + b + c = 8 for 0 ≤ (a, b, c) ≤ 8 is 45.
0 ≤ a, b, c ≤ 3 is 45 − 42 = 3.
And the number of solutions of a + b + c = 18 if any of a, b
Hence choice (c) is the answer. or c is such that 2 ≤ (a, b, c) ≤ 8 is
Alternatively The number of solutions of a + b + c = 8
3[( 8 − 3) + 3 − 1C 3 − 1] − 3[( 8 − 2 × 2) + 3 − 1C 3 − 1] +
is the coefficient of x 8 in the expansion of
(1 + x + x + x ) which is 3 as
2 3 3 [( 8 − 2 × 3) + 3 − 1C 3 − 1] = 3[ 8C 2] − 3[ 6C 2] + [ 4C 2] = 45

(1 + x + x 2 + x 3 )3 = (1 + 3x + . . . . + 3x 8 + x 9 ) Therefore number of integral solutions of a + b + c = 8,for


Alternatively Given that a + b + c = 8 and
0 ≤ (a, b, c) ≤ 1 = 45 − 45 = 0
0 ≤ a, b, c ≤ 3, then we have the following solutions. Hence choice (d) is the answer.
Alternatively The number of solutions of a + b + c = 8
a b c is the coefficient of x 8 in the expansion of (1 + x + x 2 )3,
3 3 2 which is 0 as the highest coefficient in this expansion is x 6.
3 2 3 Alternatively Since the highest value of a, b and c is 2,
so the total sum a + b + c ≤ 6. Thus we cannot attain
2 3 3
a + b + c = 8 even once.
Therefore the required number of total solutions = 3 Therefore we have no solution for the given equation.
43 To find the number of integral solutions of a + b + c = 8, 45 The number of integral solutions of a + b + c + d = 30 , if
for 0 ≤ a, b, c ≤ 2 , first of all find the total number of 0 ≤ (a, b, c, d ) ≤ 15 = (Number of integral solutions of
integral solutions of a + b + c = 8 for 0 ≤ a, b, c ≤ 8 and a + b + c + d = 30, if 0 ≤ (a, b, c, d ) ≤ 30) − (number of
then subtract the number of solutions of a + b + c = 8 if any integral solutions of a + b + c + d = 30, if any of a, b, c or d
of a, b or c is such that 3 ≤ a, b, c ≤ 8. The number of is such that 16 ≤ (a, b, c, d ) ≤ 30).
solutions of a + b + c = 8 for 0 ≤ a, b, c ≤ 8 is 45. Number of integral solutions of a + b + c + d = 30, if
The equation a + b + c = 8 for 3 ≤ a, b, c ≤ 8 suggests that 0 ≤ (a, b, c, d ) ≤ 30 = 30 + 4 − 1C 4 − 1 = 33C 3
we have to find the solutions of this equation in which at
Number of integral solutions of a + b + c + d = 30, if any of
least one variable must be greater than or equal to 3. So, we
a, b, c or d is such that 16 ≤ (a, b, c, d ) ≤ 30
see that there can be at most two variables that will be
greater than or equal to 3. But we also know that the = 4[( 30 − 16) + 4 − 1C 4 − 1] = 4[17 C 3]
solutions which have only one variable greater than or Therefore number of solutions of a + b + c + d = 30, if
equal to 3 also include those solutions which have two 0 ≤ (a, b, c, d ) ≤ 6 = 33C 3 − 4(17 C 3 ) = 2736.
variables greater than or equal to 3.
Hence choice (a) is the answer.
Therefore number of solutions of a + b + c = 8 for
Alternatively The number of integral solutions of
3 ≤ a, b, c ≤ 8 is
( 8 − 3) + 3 − 1 ( 8 − 2 × 3) + 3 − 1 a + b + c + d = 30, if 0 ≤ (a, b, c, d ) ≤ 15 = Coefficient of x 30
3[ C 3 − 1] − 3[ C 3 − 1]
in the expansion of (1 + x + x 2 + x 3 + . . . + x15 )4
= 3[ C 2] − 3[ C 2] = 45
7 4
Now (1 + x + x 2 + x 3 + . . . + x15 )4
Thus the number of solutions of a + b + c = 8 for
1112 QUANTUM CAT
30 + 4 − 1
= (1 + 2x + 3x 2 + . . . + 14 x13 + 15x14 0 ≤ (a, b, c, d ) ≤ 30 = C4 −1 = 33
C3
+ 16 x 15
+ 15x 16
+ 14 x 17
+ . . . + 2x 29
+ x )
30 2
Number of integral solutions of a + b + c + d = 30, if any of
Therefore coefficient of a, b, c or d is such that
x 30 = 2(12 + 22 + 32 + . . . + 152 ) + 162 = 2736 9 ≤ (a, b, c, d ) ≤ 30 = 4[( 30 − 9) + 4−1C 4 − 1]

46 The number of integral solutions of a + b + c + d = 30 , if − 6[( 30 − 2 × 9) + 4 − 1C 4 − 1] + 4[( 30 − 3 × 9) + 4 − 1C 4 − 1]


0 ≤ (a, b, c, d ) ≤ 10 = (Number of integral solutions of = 4[ 24C 3] − 6[15C 3] + 4 [ 6C 3] = 5446
a + b + c + d = 30, if 0 ≤ (a, b, c, d ) ≤ 30) − (Number of
Therefore number of solutions of a + b + c + d = 30, if
integral solutions of a + b + c + d = 30, if any of a, b, c or d
0 ≤ (a, b, c, d ) ≤ 8 = 33C 3 − [ 4 (24C 3 ) − 6 (15C 3 ) + 4(6C 3 )]
is such that 11 ≤ (a, b, c, d ) ≤ 30).
= 5456 − 5446 = 10
Number of integral solutions of a + b + c + d = 30, if
Hence choice (b) is the answer.
0 ≤ (a, b, c, d ) ≤ 30 = 30 + 4 − 1C 4 − 1 = 33C 3
Alternatively The number of integral solutions of
Number of integral solutions of a + b + c + d = 30, if any a + b + c + d = 30, if 0 ≤ (a, b, c, d ) ≤ 10 = Coefficient of x 30
a, b, c or d is such that 11 ≤ (a, b, c, d ) ≤ 30 in the expansion of (1 + x + x 2 + x 3 + . . . + x 8 )4 which
( 30 + 11 ) 4 − 1 ( 30 − 2 × 11 ) + 4 − 1
= 4[ C 4 − 1] − 6[ C 4 − 1] is 10.
= 4[ 22C 3] − 6[11C 3] . 48 The number of ways in which (m + n + p + q + r) different
things can be divided into five different groups containing
Therefore number of solutions of a + b + c + d = 30, if
m, n, p, q and r things respectively, if the order of the group
0 < a, b, c, d ≤ 6
(m + n + p + q + r)!
= 33C 3 − [ 4( 22C 3 ) − 6(11C 3 )] = 286. is important = × 5!
m !n ! p!q!r !
Hence choice (b) is the answer.
Therefore number of ways of distributing 15 rings among
Hint If any of a , b, c or d is such that 11≤ a , b, c, d ≤ 30 then
his 5 daughters such that one daughter gets 1 ring, another
there are two cases. 15!
daughter gets 2 rings, and so on = × 5!
(a) When one variable out of a , b, c is certainly at least 11, then 1 ! 2! 3! 4 ! 5!
one 11 is taken out from 30 and then remaining 19 is to be Hence choice (d) is the answer.
divided among a , b, c, d in 19 + 4 − 1C3 = 22C3 = 22C3 ways.
NOTE Please take stock of the situation that the articles
And this one 11 is later on arranged in 4 C1 = 4 ways. (rings) as well as receivers (girls) are different.
(b) When any two variables out of a , b, c, d are certainly at least
11, then two 11s are taken out from 30 and then remaining 8
Since the articles are different so we will use the division
(m + n + p + q + r)!
is to be divided among a , b, c, d in 8 + 4 − 1C3 = 11C3 ways. And formula for distinct things.
these two 11s are later on arranged in 4 C2 = 6 ways. m!n ! p ! q! r !
But since case (a) includes case (b) also, so we have to Further, since the receivers are distinct (as girls are
subtract case (b) from case (a) as per inclusion-exclusion distinguishable) so we can arrange the different sets of rings
principle. among different girls. That means the order is important, so we
have to multiply the possible divisions of groups by 5! In order
Alternatively The number of integral solutions of to bring out all the possibilities in which these 15 rings can be
a + b + c + d = 30, if 0 ≤ a, b, c, d ≤ 10 = Coefficient of x 30 distributed among his 5 daughters.
in the expansion of (1 + x + x 2 + x 3 + . . . + x10 )4 49 Again rings as well as girls are distinct from each other.
Now, (1 + x + x 2 + x 3 + . . . + x10 )4 However, it is certain that who gets how many rings, so we
cannot interchange the set of rings among these girls. That
= (1 + 2x + 3x + . . . + 9 x + 10 x +
2 8 9
means we can only choose the different rings but not the
11 x10 + 10 x11 + 9 x12 + K + 2x19 + x 20 )2 number of rings for each girl. Since it is already decided
Therefore coefficient of that the eldest daughter gets 1 ring, second eldest daughter
x 30 = 2(11 × 1 + 10 × 2 + 9 × 3 + 8 × 4 gets 2 rings, the middle one gets 3 rings, the second
youngest daughter gets 4 rings and the youngest daughter
+ 7 × 5) + (6 × 6) = 286 gets 5 rings. Therefore number of ways of distributing 15
47 The number of integral solutions of a + b + c + d = 30, if rings among his 5 daughters =
15!
0 ≤ (a, b, c, d ) ≤ 8 = (Number of integral solutions of 1 ! 2! 3! 4 ! 5!
a + b + c + d = 30, if 0 ≤ (a, b, c, d ) ≤ 30) − (Number of Hence choice (b) is the answer.
integral solutions of a + b + c + d = 30, if any of a, b, c or d
50 Primarily, there are two different cases in which
is such that 9 ≤ (a, b, c, d ) ≤ 30).
15 different rings can be divided. As per the statement
Number of integral solutions of a + b + c + d = 30, if there will be only one girl who is eldest and only one girl
Permutations & Combinations 1113

who is youngest. Since the problem states that there are no multiplication by 5! indicates that there are 5 distinct girls
twins but it does not say that there can’t be any triplet (i.e. among which all the rings are to be distributed.
three kids born at the same time). So the case II arises. Case III Number of ways of distributing 15 rings
Case I. 1, 2, 3, 4, 5; Case II. 1, 3, 3, 3, 5 15! 1
= × × 5!
Case I. Number of ways of distributing 15 rings 3! 3! 3! 3! 3! 5!
15!
= Here multiplication by 1!/5! indicates that there are
1 ! 2! 3! 4 ! 5! 5 groups containing the same number of rings. And the
Here we are not multiplying by 5! because it is already multiplication by 5! indicates that there are 5 distinct girls
known that which particular daughter gets the particular among which all the rings are to be distributed.
number of rings. Therefore total number of ways of distributing 15 different
Case II. Number of ways of distributing 15 rings rings
15! 1  
= × × 3! = 15! × 5! × 
1
+
1
+
1
1 ! 3! 3! 3! 5! 3! 
(2!) × 3! × (4 !) (2!) × (3!) × 4 ! (3!) × 5!
4 2 4 5

Here multiplication by 1!/3! indicates that there are


Hence choice (d) is the answer.
3 groups containing the same number of rings. And the
multiplication by 3! indicates that the 3 groups of equal 52 The number of ways of distributing 15 different rings
15! 1 15!
number of rings can be arranged among 3 distinct girls. among 5 girls equally = × × 5! =
Note that the age of girls is same, but girls are 3! 3! 3! 3! 3! 5! (3!)5
distinguishable by their names, features and all sorts of Here multiplication by 1!/5! indicates that there are
attributes that may distinguish two or more individuals. 5 groups containing the same number of rings. And the
Also, note that we are arranging it among the 3 girls only, multiplication by 5! indicates that there are 5 distinct girls
because we already know that the youngest daughter gets
among which all the rings are to be distributed.
5 rings and the oldest daughter gets 1 ring, so these two
Hence choice (c) is the answer.
girls cannot be considered for arrangement.
Therefore total number of ways of distributing 15 different
53 The number of ways of distributing 15 different rings
among 5 girls such that a girl may get nothing or all the
rings
15! 15! 15! rings
= + = ×
1 ! 2! 3! 4 ! 5! 1 ! 3! 3! 3! 5! 3! 5! = 5 × 5 × 5 × 5 × 5 × . . . . × 5 = 515
14444244443
 1 1  15! 7 15 times
 2! × 4 ! + 3! × 3! = 3! 5! × 144 Hence choice (d) is the answer.
 
Hence choice (a) is the answer. 54 The number of ways of selection of 1 ring, 2 rings, 3 rings,
4 rings and 5 rings= 1 × 1 × 1 × 1 × 1 = 1
51 Primarily, there are three different cases in which
15 different rings can be divided such that none of them Number of ways of distributing these 5 sets of rings
gets less than 2 and more than 4 rings. containing 1, 2, 3, 4, 5 rings = 5 × 4 × 3 × 2 × 1 = 120
Case I: 2, 2, 3, 4, 4; Case II: 2, 3, 3, 3, 4; Hence choice (a) is the answer.
Case III: 3, 3, 3, 3, 3
Hint Number of ways of selecting 1 ring out of 15 rings = 1,
Case I Number of ways of distributing 15 rings
selecting 2 rings out of remaining 14 rings = 1, and so on. Thus
15! 1
= × × 5! there will be 5 different sets of rings.
2! 2! 3! 4 ! 4 ! 2! × 2! First set can be given to any 5 girls, second set can be given to
1! any 4 girls, third set can be given to any three girls, fourth set
Here multiplication by indicates that there are can be given to any 2 girls and the last and fifth set can be
2! 2!
given to the remaining girl.
2 groups containing 2 rings in each group and there are
2 groups containing 4 rings in each group. And the 55 The number of ways of selection of 1 ring, 2 rings, 3 rings,
multiplication by 5! indicates that there 5 distinct girls 4 rings and 5 rings = 1 × 1 × 1 × 1 × 1 = 1
among which all the ring are to be distributed. Since it is already known that who gets how many rings, so
Case II Number of ways of distributing 15 rings the number of ways of distributing 5 different sets of rings = 1
15! 1 Hence choice (a) is the answer.
= × × 5!
2! 3! 3! 3! 4 ! 3! 56 Primarily, there are two different cases in which
Here multiplication by 1!/3! indicates that there are 15 identical rings can be divided. As per the statement
3 groups containing the same number of rings. And the there will be only one girl who is the eldest and only one
girl who is the youngest. Since the problem states that
1114 QUANTUM CAT
there are no twins but it does not say that there can’t be (number of solutions to the equation
triplets. So the case II arises. u + v + w + x + y = 5, such that 0 ≤ u, v, w, x, y ≤ 5) −
Case I 1, 2, 3, 4, 5; Case II 1, 3, 3, 3, 5 (number of solutions of the equation u + v + w + x + y = 5,
Case I Number of ways of distributing 15 rings = 1 such that at least one of the variables is 3 ≤ u, v, w, x, y, ≤ 5)

Case II Number of ways of distributing 15 rings = 1 Now the solution to the equation
u + v + w + x + y = 5, such that 0 ≤ u, v, w, x, y ≤ 5 is
Therefore total number of ways of distributing 15 rings = 2 5 + 5 −1
C 5 − 1 = 9C 4 = 126.
Hence choice (b) is the answer.
Alternatively Let us consider a, b, c, d, e be his five And the solution to the equation u + v + w + x + y = 5,
daughters, then such that at least one of the variables is 3 ≤ u, v, w, x, y ≤ 5
is 5 [( 5 − 3) + 5 − 1C 5 − 1] = 5[ 6C 4] = 75
a + b + c + d + e = 15 1 + 2 + 3 + 4 + 5 = 15
1 + 3 + 3 + 3 + 5 = 15 Therefore number of solutions to the equation
Thus there are only 2 ways of distribution as per the given u + v + w + x + y = 5, such that 0 ≤ u, v, w, x, y ≤ 2 is
= 126 − 75
conditions. Please note that the absence of twins in the
family does not rule out the possibility of triplets. In turn, the number of solutions of the equation
a + b + c + d + e = 15, such that 2 ≤ a, b, c, d, e ≤ 4 is 51.
57 The number of ways of dividing 15 identical rings among
5 girls such that none of them receives less than 2 or more It implies that the number of ways in which 15 identical
than 4 = coefficient of x15 in the expansion of rings can be distributed among 5 girls such that none of
them gets less than 2 or more than 4 is 51.
(x + x + x )
2 3 4 5

58 The number of ways of dividing 15 rings equally into 5 parts


But ( x 2 + x 3 + x 4 )5 = ( x 2 + x 3 + x 4 )4 ( x 2 + x 3 + x 4 )
=1
⇒ ( x 2 + x 3 + x 4 )5 = ( x 4 + 2x 5 + 3x 6 Now arranging the 5 sets of rings containing 5 rings each
+ 2x7 + x 8 )2( x 2 + x 3 + x 4 ) =1
⇒ ( x 2 + x 3 + x 4 )5 = (x 8 + 4 x 9 + 10 x10 + 16 x11 + 19 x12 Therefore number of ways of distributing 15 identical rings
equally among 5 girls = 1 × 1 = 1
+ 16 x13 + 10 x14 + 4 x15 + x16 )( x 2 + x 3 + x 4 )
Hence choice (b) is the answer.
⇒ ( x 2 + x 3 + x 4 )5 = ( x10 + . . . + 51 x15 + K + x 20 )
Alternatively This problem is similar to finding the
Since the coefficient of x15 in the expansion of number of solutions to the equation a + b + c + d + e = 15,
( x 2 + x 3 + x 4 )5 is 51. Therefore the required number of such that a = b = c = d = e.
ways = 51. As a + b + c + d + e = 15, therefore a = b = c = d = e = 3.
Hence choice (a) is the answer. Thus we have only 1 solution to this equation. So we can
Alternatively Primarily, there are three different cases
say that there is only 1 way of distribution of the 15 rings
in which 15 different rings can be divided such that none of equally among his daughters.
them gets less than 2 and more than 4 rings.
59 The number of ways of dividing 15 identical rings among
5!
Case I: 2, 2, 3, 4, 4; ways of distribution = = 30 5 girls such that these girls may get any number of rings
2! × 2!
= 15 + 5 − 1C 5 − 1 = 19C 4 = 3876
5!
Case II: 2, 3, 3, 3, 4; ways of distribution = = 20 Hence choice (c) is the answer.
3!
5! 60 Case I 1, 1, 3
Case III: 3, 3, 3, 3, 3; ways of distribution = =1
5! Number of ways of selection of rolls = 5C1 × 4C1 × 3C 3 = 20
Therefore the number of ways in which 15 identical rings Number of ways of arranging the 3 distinct boxes, where
can be distributed among 5 girls such that none of them 3!
2 boxes have the same number of rolls =
gets less than 2 or more than 4 is 51 (= 30 + 20 + 1). 2!
Alternatively This problem is similar to finding the Number of ways of distribution of rolls
number of solutions to the equation a + b + c + d + e = 15, 3!
= 5C1 × 4C1 × 3C 3 × = 60
such that 2 ≤ a, b, c, d, e ≤ 4. 2!
In turn, it is equivalent to finding the number of solutions Case II: 1, 2, 2
to the equation u + v + w + x + y = 5, such that Number of ways of selection of rolls = 5C1 × 4C 2 × 2C 2
0 ≤ u, v, w, x, y ≤ 2. Number of ways of arranging the 3 distinct boxes, where
3!
In turn number of solutions of the equation 2 boxes have the same number of rolls =
2!
u + v + w + x + y = 5, such that 0 ≤ u, v, w, x, y ≤ 2 =
Number of ways of distribution of rolls
Permutations & Combinations 1115

3! Case II (1, 2, 2)
= 5C1 × 4C 2 × 2C 2 ×
= 90
2! Number of ways of selection of rolls
Therefore the required number of ways = 60 + 90 = 150 5! 1
= ×
Alternatively Consider the following table. 1 ! 2! 2! 2!

Number of ways of Number of ways of distribution of rolls


Case Box 1 Box 2 Box 3 5! 1
distribution =1 × × = 15
1 ! 2! 2! 2!
i 1 1 3 5!
= 20 Therefore the required number of ways = 10 + 15 = 25
1 !1 ! 3!
ii 1 3 1 5! 63 Case I : (1, 1, 3)
= 20
1 ! 3!1 ! Number of ways of selection of rolls = 1 × 1 × 1 = 1
iii 3 1 1 5! Number of ways of distribution of rolls = 1
= 20
3!1 !1 !
Case II : (1, 2, 2)
iv 1 2 2 5!
= 30 Number of ways of selection of rolls = 1 × 1 × 1 = 1
1 ! 2! 2!
5! Number of ways of distribution of rolls = 1
v 2 1 2 = 30
2!1 ! 2! Therefore the required number of ways = 1 + 1 = 2
vi 2 2 1 5! 64 Consider the following table. Assume that the crate
= 30
2! 2!1 ! 1 contains the least number of bouquets in every case.
Therefore the required number of ways Total number
Case Crate 1 Crate 2 Crate 3
= 3 × (20 + 30) = 150 of ways

61 Case I 1, 1, 3 0, 1, 2, 3, 12, 11, 10, 9,


Number of ways of selection of rolls = 1 i 0 7
4, 5, 6 8, 7, 6
Number of ways of arranging the 3 distinct boxes, where 2
3! ii 1 1, 2, 3, 4, 5 10, 9, 8, 7, 6 5
boxes have the same number of rolls =
2! iii 2 2, 3, 4, 5 8, 7, 6, 5 4
3!
Number of ways of distribution of rolls = 1 × =3
2! iv 3 3, 4 6, 5 2
Case II 1, 2, 2 v 4 4 4 1
Number of ways of selection of rolls = 1
Therefore the required number of ways
Number of ways of arranging the 3 distinct boxes, where 2
3! = 7 + 5 + 4 + 2 + 1 = 19
boxes have the same number of rolls =
2! Alternatively Let us consider, for a moment, that the
3! crates are distinct.
Number of ways of distribution of rolls = 1 × =3
2! The number of ways of distribution of 12 identical
Therefore the required number of ways = 3 + 3 = 6 bouquets into 3 distinct crates = 12 + 3 − 1C 3 − 1 = 91
Alternatively
Case I When each crate has same number of bouquets.
Keeping 5 distinct rolls in 3 identical boxes so that none of
(4, 4, 4)
the box remains empty implies that at least 1 roll must be
Then it can be done in only 1 way.
kept in each box.
Even though the crates are distinct, the number of ways
Therefore the required number of ways
of distribution will be 1 only, as each crate receives equal
= 5 − 1C 3 − 1 = 4C 2 = 6
number of bouquets.
62 Case I (1, 1, 3) Case II When any 2 crates have same number of bouquets.
Number of ways of selection of rolls (0,0,12), (1,1,10), (2, 2, 8), (3,3,6), (5,5,2), (6,6,0)
5! 1 Number of ways = 6
= ×
1 !1 ! 3! 2! When we assume that the crates are distinct the number of
Number of ways of distribution of rolls 3!
ways of distribution of bouquets = × 6 = 18
5! 1 2!
=1 × × = 10
1 !1 ! 3! 2!
1116 QUANTUM CAT
Case III When all the crates have distinct number of to note that the number of selection in each case will be 1
bouquets. Let us assume that the number of ways of as all the bouquets are identical. The only thing we need to
distribution of 12 identical bouquets among 3 identical calculate is the number of ways of distribution among 3
crates is k. Then the number of ways of distribution of 12 distinct crates.
identical bouquets among 3 distinct crates = 3! × k = 6 × k Case I (0, 1, 11), (0, 2, 10), (0, 3, 9), (0, 4, 8), (0, 5, 7)
Therefore, we have 1 + 18 + 6k = 91 Case II (1, 2, 9), (1, 3, 8), (1, 4, 7), (1, 5, 6)
Or k = 12 Case III (2, 3, 7), (2, 4, 6)
Thus the required number of ways = 1 + 6 + 12 = 19. Case IV (3, 4, 5)
65 The number of ways in which 12 distinct bouquets can be Total number of combinations = 5 + 4 + 2 + 1 = 12
kept in 3 distinct crates = 12 + 3 − 1C 3 − 1 = 91 And since crates are distinct and no two crates has same
number of bouquets, so each combination further can be
66 The number of ways in which 12 distinct bouquets can be
arranged in 3! ways.
kept in 3 distinct crates, so that no box remains empty
Therefore the required number of ways of distributions
= 12 − 1C 3 − 1 = 55
3! × 12 = 72
67 In order to keep at least 2 bouquets in each of the crates Alternatively
you can take away 6 bouquets separately. Now you are left The number of ways of distribution of 12 identical
with just 6 bouquets to distribute among 3 distinct crates, bouquets among 3 distinct crates = 12 + 3 − 1C 3 − 1 = 91
which you can now distribute any number of bouquets
among 3 crates. This is equivalent to the distribution of Case I When each crate has the same number of bouquets.
6 identical bouquets among 3 distinct crates so that any (4, 4, 4)
number of bouquets can be kept in any crate Then it can be done in only 1 way.
= 6 + 3 − 1C 3 − 1 = 28 Case II When any 2 crates have the same number of
68 Let us consider that a, b and c are three distinct crates, so bouquets.
that a ≥ 2, b ≥ 3, c ≥ 4 and a + b + c = 12. (0,0,12), (1,1,10), (2, 2, 8), (3,3,6), (5,5,2), (6,6,0)
Consider a ≥ 2⇒ a − 2 ≥ 0 ⇒ x ≥ 0 Number of ways of selection of 12 bouquets = 6
Similarly, b ≥ 3⇒ b − 3 ≥ 0 ⇒ y ≥ 0 Since there are 3 distinct crates out of which the 2 crates
And, c ≥ 4 ⇒c − 4 ≥ 0⇒ x ≥ 0 are having the same number of bouquets, therefore the
3!
Therefore a + b + c = 12 number of arrangements of 3 crates = .
2!
⇒ x + y + z = 12 − (2 + 3 + 4)
⇒ x+ y+z=3 Therefore the number of ways of distribution of
12 identical bouquets so that exactly 2 crates have the same
Thus the number of ways of distribution of 12 identical 3!
bouquets among 3 distinct crates so that one particular number of bouquets = × 6 = 18
2!
crate has at least 2 bouquets and another one has at least
Case III When all the crates have distinct number of
3 bouquets and the third one has at least 4 bouquets is
bouquets.
same as the number of distribution of 3 identical bouquets
so that each of them can have any number of bouquets. Then, let us assume that the number of ways of distribution
Therefore the required number of ways = 3 + 3 − 1C 3 − 1 = 10 of 12 identical bouquets among 3 distinct crates = k
Therefore, we have 1 + 18 + k = 91 Or k = 72
Alternatively First of all take away 2, 3 and 4 bouquets
for crate 1, 2 and 3, assuming crate 1 is the smallest one Thus the required number of ways = 72
and crate 3 is the largest one. Now you have to distribute 70 The number of selection of identical bouquets in three
the remaining bouquets, which can be done in the groups = 1 × 1 × 1 = 1
following ways. Further, since in each crate the number of bouquets are
3! same, so the number of arrangements of crates = 1
Case I (0, 0, 3) Number of ways of distribution = =3
2!
Therefore the required number of ways of distribution
Case II (0, 1, 2) Number of ways of distribution = 3! = 6 = number of ways of selection of bouquets
Case III (1, 1, 1) Number of ways of distribution = 1 × number of ways of arrangements of crates = 1
Therefore total number of required ways of distribution of It’s like an equation a + b + c = 12, such that a = b = c = 4.
12 bouquets = 3 + 6 + 1 = 10 This obviously has 1 solution as in 4 + 4 + 4 = 12
69 Let us consider that (a, b, c) is the notation for the number
71 Number of ways of selection of 4 balls from 10 balls of
of bouquets distributed among three crates. It is imperative distinct colours = 10C 4 = 210
Permutations & Combinations 1117

72 Number of ways of selection of 4 balls from a pool of 73 Number of ways of selection of 10 cookies from a pool of
unlimited balls of 10 distinct colours = number of unlimited cookies of 4 distinct types = number of positive
integral solutions of x1 + x 2 + x 3 + x 4 = 10,
non-negative integral solutions of
x1 + x 2 + . . . + x 9 + x10 = 4, such that x1, x 2, x 3, x 4 ≥ 1
Therefore, the required number of selections
such that x1, x 2, . . . , x 9, x10 ≥ 0
= 10 − 1C 4 − 1 = 9C 3 = 84

Therefore, the required number of selections


= 4 + 10 − 1C10 − 1 = 13C 9 = 715

Introductory Exercise 19.9


1 Since the number of ways in which n different letters can be Number of ways when 4 letters are wrongly placed
placed in their n addressed envelopes so that all the letters  1 1 1 1
= 6P4 1 − + − + = 135
are in the wrong envelopes  1 ! 2! 3! 4 !
 1 1 1 1 Number of ways when 3 letters are wrongly placed
= n! 1 − + − + K + (−1)n
 1 ! 2! 3! n !  1 1 1
= 6P3 1 − + − = 40
 1 ! 2! 3!
Therefore the required number of ways
 1 1 1 1 Number of ways when 2 letters are wrongly placed
= 6! 1 − + − + K + (−1)6
 1 ! 2! 3! 6 !  1 1
= 6P2 1 − + = 15
 1 ! 2!
 1 1 1 1 1 1 
= 6 ! 1 − + + + − +  Number of ways when 1 letter is wrongly placed
 1 2 6 24 120 720
 1
 360 − 120 + 30 − 6 + 1 = 6P1 1 − =0
= 720   = 265  1 !
 720 
Number of ways when none of the letters is wrongly placed = 1
2 The number of ways in which exactly r letters can be placed Therefore the required number of ways
in wrongly addressed envelopes when n letters are placed = 135 + 40 + 15 + 0 + 1 = 191
in n addressed envelopes 4 Given that a, b, c, d, e, f can take any value from the set
 1 1 1 1
= Pr 1 −
n
+ − + K + (−1)r {1, 2, 3, 4, 5, 6}.
 1 ! 2! 3! r !
N is a non-zero integer only when a ≠ 1, b ≠ 2, c ≠ 3,
Therefore the required number of ways d ≠ 4, e ≠ 5, f ≠ 6. That is a can be anything but not 1, b can
 1 1 1 1 be anything but not 2, …, f can be anything but not 6.
= 6P4 1 − + − +
 1 ! 2! 3! 4 !
It means this problem is same as placing 6 letters in
 1 1 1  6 envelops such that no letter is placed in the right
= 60 1 − 1 + − +
 2 6 24  envelope.
12 − 4 + 1  Therefore the required number of ways
= 360 = 135
 24   1 1 1 1
= 6! 1 − + − + . . . + (− 1)6
 1 ! 2! 3! 6 !
3 At least 2 letters are correctly placed
 1 1 1 1 1 1 
= At most 4 letters are correctly placed = 6 ! 1 − + − + − + 
 1 2 6 24 120 720
= Either 4 letters are wrongly placed or 3 letters are
 360 − 120 + 30 − 6 + 1
wrongly placed or 2 letters are wrongly placed or 1 letter is = 720   = 265
wrongly placed or no letter is wrongly placed.  720 
1118 QUANTUM CAT
Introductory Exercise 19.10
Solutions (for Q. No. 1 and 2) : Consider the following Solutions (for Q. Nos. 11 to 14) :
diagram. 11 Total number of lines if 12 non-collinear points are
A B
connected = 12C 2 = 66
Number of lines if 4 non-collinear points are connected
= 4C 2 = 6
D C
1 There are 4 points,A, B, C and D. So using these 4 points Since 4 points are collinear, so there would be only 1 line
instead of 6 lines.
we have total 6 lines :AB, BC , CD, AD, AC and BD.
Thus, the maximum number of lines = (66 − 6) + 1 = 61.
Since each line needs 2 points to be connected, therefore
12 Since out of 12 points 4 points are collinear, so the
the total number of lines = 4C 2 = 6
maximum number of triangles
2 Using any 3 of the 4 points we have total 4 triangles: = 12C 3 − 4C 3 = 220 − 4 = 216
ACD, ABC , DAB and BCD.
13 Since out of 12 points 4 points are collinear, so the
Since each triangle needs 3 points to be connected, maximum number of quadrilaterals
therefore the total number of triangles = 4C 3 = 4
= 12C 4 − 4C 4 = 495 − 1 = 494
Solutions (for Q Nos. 3 to 6): In these sums you have to 14 Since 4 points are not sufficient to form a hexagon, so it
consider those figures (lines, triangles, hexagons and
does not matter that the 4 points are collinear. Therefore
decagons), whose vertices are the points already given in the
the maximum number of hexagons = 12C 6 = 924
plane.
3 The maximum number of straight lines that can be formed Solutions (for Q. Nos. 15 to18) :
using 10 non-collinear points = 10C 2 = 45
15 Maximum number of straight lines = nC 2 − mC 2 + 1
4 The maximum number of triangles that can be formed 16 Maximum number of triangles = nC 3 − mC 3
using 10 non-collinear points = 10C 3 = 120
17 Maximum number of quadrilaterals = nC 4 − mC 4
5 The maximum number of hexagons that can be formed
using 10 non-collinear points = 10C 6 = 210 18 Maximum number of hexagons = nC 6 − mC 6

6 The maximum number of diagonals in a decagon that can Solutions (for Q. Nos. 19 to 22) :
be formed using 10 non collinear points 19 Number of sides in a heptagon = 7
= 10C 2 − 10 = 45 − 10 = 35
Then the total number of lines that can be formed by
Hint Number of diagonals = total number of lines connecting 7 points (vertices) of the heptagon = 7C 2
− number of sides Therefore the total number of diagonals in the heptagon
Solutions (for Q. Nos. 7 to10) : = 7C 2 − 7 = 21 − 7 = 14

7 Since all the 15 points are on the circumference of a circle, 20 Number of sides in a polygon = n
it implies that no 3 lines are collinear. Therefore the Then the total number of lines that can be formed by
number of straight lines formed by joining the 15 points connecting n points of the polygon = nC 2
= 15C 2 = 105 Therefore the total number of diagonals in the polygon
8 Since no three points are collinear, so the number of = nC 2 − n
triangles that can be formed by joining 15 points 21 Number of diagonals in a polygon
= 15C 3 = 455 n!
= nC 2 − n = −n
(n − 2)! × 2!
9 Since no three points are collinear, so the number of
n(n − 1) n(n − 3)
quadrilaterals that can be formed by joining 15 points = −n=
= 15C 4 = 1365 2 2
n(n − 3)
10 Since no three points are collinear, so the number of Q = 54
2
octagons that can be formed by joining 15 points ⇒ n = 12
= 15C 8 = 6435
Permutations & Combinations 1119

11
22 Q 9C 3 − mC 3 = 28 ⇒ m
C 3 = 56 Therefore instead of getting C 2 points, we get only one
m (m − 1)(m − 2) point B. Hence the number of intersection points of the
⇒ = 56
6 lines is 37 C 2 − 13C 2 − 11C 2 + 2 = 535
⇒ m(m − 1)(m − 2) = 8 × 7 × 6
Solutions (for Q. Nos. 28 to 31) :
⇒ m=8
28 The number of regions into which this plane will be divided
Solutions (for Q. Nos. 23 and 24) : by 6 lines = 6C 0 + 6C1 + 6C 2 = 1 + 6 + 15 = 22
23 P Q n2 + n + 2
29 = 22 ⇒ n = 6
2
n( n + 1) n2 + n + 2
1 2 3 4 5 6 7 8 Hint n C 0 + nC1 + nC2 = + 1=
2 2
A triangle can be formed by selecting 1 point out of P and Q
and selecting 2 points out of 8 points on the other parallel 30 n2 − n + 2 is the number of disjoint regions that can be
line. formed in the plane by n simple closed curves which
or a triangle can be formed by selecting 2 points out of pair-wise meet in at most two points. It is same as you see
2 points (P and Q) and selecting 1 point out of 8 points. with the Venn-diagrams.
∴Required number of triangles 31 If n points are given on the circumference of a circle and the
= ( C1 × C 2 ) + ( C 2 × C1 )
2 8 2 8
chords determined by them are drawn. If no three chords
have a common point, then the number of triangles all of
= (2 × 28) + (1 × 8) = 64
n!
24 Number of triangles whose vertex is P = total number of whose vertices lie inside the circle = nC 6 =
6 !(n − 6)!
triangles – number of triangles which do not contain P as a
vertex.
∴Number of triangles which do not contain P as a vertex
= 1C1 × 8C 2 = 28
∴ Required number of triangles = 64 − 28 = 36

Solutions (for Q. Nos. 25 - 27) :


25 An intersection point is formed by the intersection of two
Therefore required number of triangles = 6C 6 = 1
lines. Hence number of intersection points is equal to the
number of ways of selecting 2 lines out of the given
Solutions (for Q. Nos. 32 to 38) :
20 non-parallel and non-concurrent lines.
32 Maximum number of pieces of pizza
i.e., Required number of points = 20
C 2 = 190.
= 8C 0 + 8C1 + 8C 2 = 1 + 8 + 28 = 37
26 A parallelogram is formed by choosing two straight lines.
n(n + 1) n2 + n + 2
From the set of m parallel lines and two straight lines from 33 Q nC 0 + nC1 + nC 2 = +1=
the set of n parallel lines. 2 2
n2 + n + 2
Two straight lines from the set of m parallel lines can be ∴ = 20 ⇒ n (n + 1) = 38 ⇒ n ≥ 6
chosen in mC 2 ways and two straight lines from the set of 2
n2 + n + 2
n parallel lines can be chosen in nC 2 ways. Hence the If n = 5, n(n + 1) < 38 ⇒ < 20, which is
2
number of parallelograms formed
inadmissible.
m (m − 1) n (n − 1)
= mC 2 × nC 2 = × 34 Number of friends present, initially = maximum number of
2 2
72 + 7 + 2
mn (m − 1)(n − 1) pieces obtained after making 7 cuts = = 29
= 2
4
Total number of friends including the ones who arrived
27 The number of points of intersection of 37 lines is 37 C 2. But
later = maximum number of pieces obtained after making
13 straight lines out of the given 37 straight lines pass 82 + 8 + 2
through the same point A. Therefore instead of getting 13C 2 8 cuts = = 37
2
points, we get only one point A. Similarly 11 straight lines
Therefore the maximum number of friends that arrived
out of the given 37 straight lines intersect at point B.
later at the party = 37 − 29 = 8.
1120 QUANTUM CAT
35 If n points lie on the circumference of a circle and are If no three chords have a common point, number of
connected by straight lines intersecting the circle such that triangles all of whose vertices lie inside the circle
no three lines are concurrent (that is no three lines ever n!
= nC 6 =
pass through the same point), then the maximum number 6 !(n − 6)!
of parts/regions into which these lines divide the circle is
42 The required number of triangles
n
C 0 + nC 2 + nC 4.
n! 6!
Therefore the required number of pieces C6 =
n
= =1
6 !(n − 6)! 6 !(0)!
= 8C 0 + 8C 2 + 8C 4 = 1 + 28 + 70 = 99
36 When n lines intersect a circle, the maximum number of 43 The maximum number of regions into which a
n − 3n + 2
2 3-dimensional cube/sphere/cylinder can be partitioned by
bounded regions created = = 15 ⇒ n = 7 exactly n planes is
2
n3 + 5n + 6
37 When n lines intersect a circle, the maximum number of
n
C 0 + nC1 + nC 2 + nC 3 =
6
bounded regions created
Therefore the maximum number of required pieces
n2 − 3n + 2 82 − 3 × 8 + 2
= = = 21 63 + 5 × 6 + 6
2 2 = = 42
6
Since there are only 21 Cheesers, so 4 (= 25 − 21) persons
n3 + 5n + 6
have to compromise with an Edger piece. 44 Since, ≥ 300
6
38 Let B and U denote the BOUNDED regions and
⇒ n3 + 5n ≥ 1794
UNBOUNDED regions, where bounded regions represent
the Cheesers and unbounded regions represent the Edgers. ⇒ n > 12 ⇒ n = 13
n2 − 3n + 2
Then, B = ⇒ U = 2n Solutions (for Q. Nos. 45 to 60) :
2
45 Total number of squares = 3
n2 − 3n + 2
As, B > U therefore > 2n ⇒ n2 − 7 n + 2 > 0 (i) B (ii) C
2
(iii) A + B + C + D
The above inequality is valid only when the minimum
integral value of n = 7. A B

Solutions (for Q. Nos. 39 and 40) :


C D
39 The maximum possible number of regions on a plane that
can be created by a Venn diagram with n circles
= n(n − 1) + 2 = n2 − n + 2 Total number of rectangles = 9
Therefore the required number of maximum regions (i) A (ii) B
= 62 − 6 + 2 = 32 (iii) C (iv) D
(v) A + B (vi) C + D
40 If n ellipses are drawn in the plane such that no two of them (vii) A + C (viii) B + D
are tangent, none of them lies entirely within or outside of (ix) A + B + C + D
another one and no three of them are concurrent, then the
46 Total number of squares in a square having n columns and
maximum number of parts/regions into which these
n(n + 1)(2n + 1)
ellipses divide the plane is 2n(n − 1) + 2 = 2(n2 − n + 1). n rows = 12 + 22 + 32 + . . . . . + n2 = Σn2 =
6
Therefore the required number of maximum regions Since, n = 3, therefore total number of squares = 14.
= 2 (52 − 5 + 1) = 42
Total number of rectangles in a square having n columns
2
Solutions (for Q. Nos. 41 and 42) :  n (n + 1)
and n rows = 13 + 23 + 32 + . . . + n3 = Σn3 =
41 If n points are given on the circumference of a circle and the  2 

chords determined by them are drawn. Since, n = 3, therefore total number of rectangles = 36.
47 Total number of squares in a rectangle having m columns
and n rows = m ⋅ n + (m − 1)(n − 1)
+ (m − 2)(n − 2) + . . . + 0
Since, m = 5, n = 2, therefore total number of squares
= 10 + 4 = 14
Permutations & Combinations 1121

Total number of rectangles in a rectangle having 56 The 8 vertical lines will create 7 columns and 15 horizontal
m columns and n rows lines will create 14 rows.
= (1 + 2 + 3 + . . . . + m)(1 + 2 + 3 + . . . . + n) Now, the total number of squares in a rectangle having
m columns and n rows
Therefore total number of rectangles = (3) × (15) = 45
= m ⋅ n + (m − 1)(n − 1) + (m − 2)(n − 2) + . . . + 0
48 Total number of squares Since, m = 7, n = 14, therefore total number of squares
= Σ102 = 12 + 22 + K + 102 = 385 = 98 + 78 + 60 + 44 + 30 + 18 + 8 = 336
49 Total number of rectangles 57 In any standard chessboard there are total 8 rows and
8 columns, so the total number of squares
= Σn = 1 + 2 + . . . . + 3 = 3025
3 3 3 3

= 12 + 22 + 32 + . . . . + n2
50 Total number of squares in a rectangle having m columns
n(n + 1)(2n + 1) 8 × 9 × 17
and n rows = Σn2 = = = 204
6 6
= m ⋅ n + (m − 1)(n − 1) + (m − 2)(n − 2) + . . . + 0
58 In any standard chessboard there are total 8 rows and
Since, m = 8, n = 15, therefore total number of squares 8 columns, so the total number of squares
= 120 + 98 + 78 + 60 + 44 + 30 + 18 + 8 = 13 + 23 + 33 + K + n3 = Σn3
= 456 2 2
 n (n + 1)  8 × 9
= = = 1296
51 Total number of rectangles in a rectangle having m  2   2 
columns and n rows = (1 + 2 + 3 + . . . . + m)
(1 + 2 + 3 + . . . + n) 59 Total number of squares

Therefore total number of rectangles =(36) × (120) = 4320 = (9 + 4 + 1) + (9 + 4) + (9 + 4) = 40

52 Total number of quadrilaterals in a square having Hint To get the answer easily, first you clear all the squares
from the central square. Then you will have 9 + 4 + 1 = 13
n columns and n rows squares of distinct sizes. Then you can proceed like this for the
2
 n(n + 1) central squares as well.
= 13 + 23 + 33 + . . . . + n3 = Σn3 =
 2 
60 There are three different sizes of triangles. Total number of
Since, n = 10, therefore total number of rectangles = 3025. triangles = 1 + 4 + 4 = 9
NOTE In this case quadrilaterals and rectangles are one and the 61 You can see in the following diagram that if you want to go
same thing. from A to B, you have to take 3 steps towards the right side
and 2 steps in the upward direction. Thus you have to take
53 Total number of quadrilaterals in a rectangle having m total 5 steps.
columns and n rows
B
= (1 + 2 + 3 + . . . + m)(1 + 2 + 3 + . . . . + n)
Therefore total number of quadrilaterals
= (36) × (120) = 4320 A
NOTE In this case quadrilaterals and rectangles are one and the So the total number of ways
same thing. 5!
= 5C 3 × ( 5 − 3)C 2 = 5C 3 × 2C 2 = = 10
54 Total number of quadrilaterals when a set of 8 parallel lines 3! 2!
intersects another set of 15 parallel lines 6!
62 Total number of paths from A to P = 6C 3 × 3C 3 = = 20
= C2 ×
8
C 2 = 28 × 105 = 2940
15 3! 3!
D C
55 Total number of rectangles when 8 vertical lines intersect
15 horizontal lines such that all the vertical lines are
parallel and equidistant to each other, and all the R
horizontal lines are also parallel and equidistant to each
other = 8C 2 × 15C 2 = 28 × 105 = 2940
P
NOTE The only difference between a quadrilateral and a
rectangle is that in a rectangle opposite sides are parallel and A B
adjacent sides intersect each other perpendicularly, while it
may not be the case in a normal quadrilateral. Total number of paths from P to
1122 QUANTUM CAT
4! 6!
R = 4C 2 × 2C 2 = =6 the square PQRS = 6C 3 × 3C 3 = = 20
2! 2! 3! 3!
Total number of paths from R Total number of paths from A to C, via P to R, when there is
6! NO grid in the square PQRS = (number of paths from A to
to C = 6C 3 × 3C 3 = = 20.
3! 3! P ) × (number of paths from P to R) × (number of paths from
Therefore the number of paths from A to B , via P and R R to C) = 2 × 2 × 2 = 8
= 20 × 6 × 20 = 2400 Therefore the required number of paths = Total number of
And the number of paths from A to C, via B and D = 2 paths from A to C, when there is NO grid in the square
Thus the required number of paths = 2400 + 2 = 2402 PQRS + (Total number of paths from A to C, via P to R, when
there is a grid in the square − Total number of paths from A
63 Let us consider you have to go from A to its opposite cornerC. to C, via P to R, when there is no grid in the square PQRS)
Total number of paths from P to R= 6C 3 × 3C 3 =
6!
= 20. = 20 + (368 − 8) = 380
3! 3! D C
Total number of paths from A to C, via P to R, when there is a
grid in the square PQRS = (number of paths from A to P) × S R
(number of paths from P to R) × (number of paths from R toC) N M
= 2 × 20 × 2 = 80
K L
Total number of paths from A to C, when there is NO grid in P Q
6!
the square PQRS = 6 C 3 × 3C 3 = = 20.
3! 3! A B
Total number of paths from A to C, via P to R, when there is
65 Total number of shortest paths from A to C = (number of
NO grid in the square PQRS = (number of paths from A to P)
× (number of paths from P to R) × (number of paths from R paths from A to Q ) × (number of paths from Q to C)
+ (number of paths from A to Y × (number of paths from Y
to C) to C ) + (number of paths from A to S) × (number of paths
= 2× 2× 2= 8 from S to C) + (number of paths from A to X)×(number of
Therefore the required number of paths = Total number of paths from X to C.)
paths from A to C, when there is NO grid in the square = 5 × 10 + 1 × 5 + 5 × 10 + 1 × 5 = 110
PQRS + (Total number of paths from A to C, via P to R, X
D C
when there is a grid in the square − Total number of paths S R
from A to C, via P to R, when there is no grid in the square O
PQRS)
= 20 + (80 − 8) = 92
P Q
A B
D C Y
S R Alternatively This problem is like you can go through
any point from A to C, but cannot pass through the point O.
D C
S R
P Q
O
A B
P Q
64 Using the same approach as in the previous sum, we can A B
say that the total number of the shortest paths from P to
The number of shortest paths from A to O = 5!/ 3! 2! = 10
R = 92
The number of shortest paths from O to C = 5!/ 3! 2! = 10
Total number of paths from A to C, via P to R, when there is
Total number of shortest paths from A to C via O
a grid in the square PQRS
= 10 × 10 = 100
= (number of paths from A to P)
10 !
× (number of paths from P to R) Total number of shortest paths from A to C = = 210
6 !4 !
× (number of paths from R to C)
Therefore the required number of shortest paths
= 2 × 92 × 2 = 368
= (Total number of paths from A to C) − (Total number of
Total number of paths from A to C, when there is NO grid in paths from A to C, via O) = 210 − 100 = 110
Permutations & Combinations 1123

66 Number of triangles with all three vertices taken from the maximum intersections when any two diagonals intersect
pentagon = 5C 3 = 10 each other inside a regular polygon with odd number of
Number of triangles with two vertices taken from the two vertices is nC 4. Therefore the maximum number of
adjacent vertices of the outer pentagon and one vertex intersections = 25
C 4 = 12650.
from the 3 vertices of inner pentagon = 5 (2C 2 × 3C1 ) = 15 Thus there are maximum 12650 warehouses in the SEZ.
Number of triangles with two vertices from the 69 When n is odd, the maximum diagonals intersect at any
non-adjacent vertices of outer pentagon and 1 vertex from point inside the polygon is 2. So, no 3 (or more than 3)
the inner pentagon = (2C 2 × 5C1 ) = 5 diagonals intersect at the same point inside the polygon
with 49 (odd number of) vertices. Therefore, there is no
Number of triangles with one vertex from outer pentagon
government office.
and two vertices from inner pentagon = 5C1 × 2C 2 = 5
Hence choice (a) is correct.
Therefore total number of required triangles 70. When n is even, but not divisible by 6, the maximum
= 10 + 15 + 5 + 5 = 35 number of diagonals intersect at any point inside the
67 A rail-loop can be formed by joining any 3 vertices of the regular polygon is 3. So there cannot be more than
regular polygon. It can be done in 16
C 3 = 560 ways. 3 diagonals intersecting at any common point inside the
polygon. Since no 5 diagonals intersect at any common
Therefore there are 560 rail-loops.
point, so the number of required intersection is zero. That
68 When n is odd, the maximum diagonals intersect at any is there is no airport in the given SEZ.
point inside the polygon is 2. Now, the number of Hence choice (d) is correct.

Level 01 Basic Level Exercise


1 9 × 9 × 9 × 9 = 94. 5 Let us arrange 4 persons in 4! ways then we place their
respective wives adjacent to them in 2! ways i.e., either left
2 9 − 1 (It is considered that the lock opens in the last
4
or right sides of the husbands.
attempt.)
Hence, the required number of ways
3 Case I. MW MW MW MW = 4 ! × 2! × 2! × 2! × 2!
Case II. WM WM WM WM
Let us arrange 4 men in 4! ways, then we arrange 4 women in 6 Let us first of all we arrange 4 men in 3! ways then
4
P4 ways at 4 places either left of the men or right of the men. 4 women can be arranged in 4 places in 4 P4 ways.

Hence required number of arrangements Hence the required number of ways = 3! × 4P4 = 144.
= 4 ! × 4P4 + 4 ! × 4P4 7 All the 4 men can be arranged in 4! ways.
= 2 × 576 = 1152 Similarly all the 4 women can be arranged in 4! ways but
4 First of all we arrange 4 women in 4! ways then we arrange the two different groups of men and women can be
4 men in 5 places in 5 P4 ways. mutually arranged in 2! ways.
Hence the required number of ways = (4 !)2 × 2! = 1152.
Hence, the total number of arrangements = 4 ! × 5P4 = 2880
1124 QUANTUM CAT
8 H L C N T A U I O 14 Total number of 5 digit numbers (including which begins
L N A I with zero) = 5! = 120.
There are total 13 letters out of which 7 are consonants and Number of 5 digit numbers which begin with zero = 4 ! = 24.
6 are vowels. Also there are 2L′ s, 2N ′ s, 2A′ s, and 2I′ s. Sum of all 5 digit numbers
If all the consonants are together then the number of = (0 + 1 + 2 + 3 + 4) × 4 ! × (11111)
7! = 240 × 11111 = 2666640
arrangements = . but the 7 consonants can be arranged
2!.2! Sum of all five digit numbers which begin with zero
7! = (1 + 2 + 3 + 4) × 3! × (1111)
themselves in ways.
2!.2!
= 66660
7! 7!
Hence the required number of ways = × Hence the sum of the required numbers
2! × 2! 2! × 2!
= sum of all 5 digit numbers including those numbers
= (1260)2 = 1587600
which begin with zero
9 There are 7 odd places and 6 vowels of which 2 vowels – sum of all 5 digit numbers which begin with zero
occur two times. = 2666640 − 66660 = 2599980
7
P6 15 Number of polygons = nC 2 − Number of sides (n)
Hence number of arrangements of vowels = .
2! × 2!
∴ C2 − n = n
n
Now the remaining 7 consonants of which two consonants
occur two times, can be arranged in 7 places in ⇒ C 2 = 2n
n

7! n (n − 1)
ways. ⇒ = 2n
2! × 2! 2
7
P6 7! ⇒ n=5
Hence, the required number of ways = ×
2! × 2! 2! × 2! 16 There are 10 letters in the given word PROPORTION and
= 1587600 we have 10 letters of 6 different kinds viz.,
(0, 0, 0), (P , P ), (R , R ), T , I, N.
10 S U C E F L
For a group of four letters, we have four cases :
S U C
(i) Three alike and one different.
S
There are 10 letters in the word SUCCESSFUL and S occurs (ii) Two alike and two other alike.
3 times, U occurs 2 times and C occurs 2 times. (iii) Two alike and other two different.
∴ The letters of the word SUCCESSFUL can be arranged in (iv) All four different.

=
10 !
= 151200 ways In case I, the number of selections = 1C1 × 5C1 = 5.
3! × 2! × 2! In case II, the number of selections = 3C 2 = 3.
11 Since all three S′ s are together, hence there will be 8 letters [Since we can select two pairs out of 3 pairs (0, 0), (P , P ),
of which C occurs 2 times and U occurs 2 times.
(R , R )]
8!
Thus the required number of arrangements = In case III, the number of selections = 3C1 × 5C 2 = 30.
2! × 2!
(∴We can select one of 3 pairs and then two
= 10080 from the remaining 5 letters say, P, R, T, I, N.)
12 (C, C), (U, U), S, S, S, E, F, L In case IV, the number of selections = 6C 4 = 15.
8!
These 8 letters now can be arranged in ways. (∴We can select 4 different letters
3!
from 6 letters O, P, R, T, I, N.)
13 Total number of 4 digit numbers without repetition of
4!
digits = 4 ! = 24. In case I, the number of arrangements = 5 × = 20.
3!
Each of the digits in each of the 4 places (thousands,
4!
hundreds and tens and unit) place will be 3! times In case II, the number of arrangements = 3 × = 18.
i.e.,6 times. 2! × 2!
Hence the sum of all the digits at each of the four places 4!
In case III, the number of arrangements = 30 × = 360.
= 6 (1 + 2 + 3 + 4) = 60 2!
Hence the sum of all the required 4 digit numbers In case IV, the number of arrangements = 15 × 4 ! = 360.
= 60 (1000 + 100 + 10 + 1) Hence the required number of arrangements
= 60 × 1111 = 66660 = 20 + 18 + 360 + 360 = 758
Permutations & Combinations 1125

4! 25 Number of ways of selecting the items from 6 identical


17 Vowels themselves can be permuted in ways
3! items = 6 + 1 = 7
6! Similarly we can select from other set of 6 identical items = 7
and consonants can be permuted in ways
2! × 2!
∴Total number of ways in which at least one item can be
4! 6!
Therefore the required number of ways = × selected from the two sets = (7 × 7 ) − 1 = 48
3! 2! × 2!
Hint Since we can select either 0, 1, 2, 3, 4, 5, or 6 items which
= 4 × 180 = 720 can be done in 7 ways.
18 Total number of ways of arranging 16 people = 15! ways 26 We can select atleast one item from 6 different items
Number of ways in which two brothers are together = (26 − 1)
= 14 ! × 2 Similarly we can select atleast one item from other set of
∴ Number of ways in which two brothers are never 6 different items in (26 − 1) ways.
together ∴ Required number of ways = (26 − 1)(26 − 1)
= 15! − 14 ! × 2
= (26 − 1)2 = 3969
= 14 !(15 − 2) = 14 ! × 13
19 A person out of remaining 14 persons can be selected in 27 We can select atleast one item from 6 identical items in
6 ways.
14
C1 = 14 ways.
Similarly from other set also we can select atleast one item
Now consider Lehman, Mckinsey and a third (selected in 6 ways.
person) as a single person, then we can arrange all of them Hence, the required number of ways = 6 × 6 = 36.
in 13! ways. But the two brothers mutually can be arranged
in 2! ways. Hence the required number of ways 28 Single step Double step Number of ways

= 14 × 13! × 2 12 0 12!
=1
= 14 ! × 2 12!
10 1 11 !
20 Every ball can be distributed in 4 ways. = 11
Hence the required number of ways = 4 × 4 × 4 × 4 × 4 × 4 10 !
8 2 10 !
= 46 = 4096 = 45
(8 ! × 2!)
21 Number of ways of distribution of ‘n’ identical things taken 9!
n+ r −1 6 3 = 84
r at a time = Cr −1
(6 ! × 3!)
6 + 4 −1
∴ Required number of ways = C 4 − 1 = 9C 3 = 84 4 4 8!
= 70
n+ r −1 (4 ! × 4 !)
22 Number of non-negative integral solutions = Cr −1
2 5 7!
= 21 + 3 − 1
C3 −1 = 21
(2! × 5!)
= 23
C 2 = 253 0 6 6!
=1
23 Number of positive integral solutions = n−1
Cr −1 6!
21 − 1 ∴ Total number of ways = 1 + 11 + 45 + 84 + 70 + 21 + 1
= C3 −1 = 20
C2
= 233
= 190 n+ r −1 8 + 3 −1
29 Using C r − 1, we get C 3 − 1 = 10C 2 = 45
24 Number of ways of selecting items from 6 different items
100 !
= 26. 30 100
C10 × 90
C 20 × 70C 30 × 40
C 40 =
10 ! × 20 ! × 30 ! × 40 !
Similarly from second set of 6 different objects we can
select items in 26 ways. 31 There are two sets of numbers 0, 1, 3, 5 and 0, 2, 3, 4.
Therefore number of 4 digit numbers using digits 0, 1, 3, 5
Therefore, total number of selections = 26 × 26
= 3 × 3 × 2 × 1 = 18
but we have to select at least one item therefore we
Similarly number of 4 digit numbers using digits 0, 2, 3, 4
subtract one case in which none of the items is selected
= 3 × 3 × 2 × 1 = 18
= 26 × 26 − 1 = 212 − 1 = 4095
Hence the total required numbers = 18 + 18 = 36.
1126 QUANTUM CAT
32 When  1 1 9 9 9 → 93 43 4 × 6 = 24
7 is at  → 93

1 9 1 9 9 44 Total number of 5 digit numbers = 9 × 104 = 90000
ten → 1 9 9 1 9 → 9 3
Number of 5 digit numbers without any repetition
thous- 
ands  = 9 × 9 × 8 × 7 × 6 = 27216
 1 9 9 9 1 → 93
place ∴Required number of 5 digit numbers in which atleast one
8 × 92 of the digit repeats = 90000 − 27216 = 62784
8 1 1 9 9 → 45 Total number of possible outcomes = 64.
8 × 92 The number of possible outcomes in which 4 does not
 8 1 9 1 9 → appear on any die is 54.
When 
7 is  Therefore the number of possible outcomes in which
8 × 92
not at  8 1 9 9 1 → atleast one die shows digit 4 = 64 − 54 = 671
→ 
the ten  46 Required number of signals
8 × 92
thous-  8 9 1 1 9 → = 5P1 + 5P2 + 5P3 + 5P4 + 5P5
ands 
 8 × 92 = 5 + 20 + 60 + 120 + 120
place  8 9 1 9 1 → = 325
8 × 92 47 The number of words begin with A is 4!.
8 9 9 1 1 → The number of words begin with E is 4!.
The number of words begin with M is 4!.
Hence the required number of ways = 4 × 9 + 6 × 8 × 9 3 2
The number of words begin with R is 4!.
= 92 (36 + 48) Number of words begin with VA is 3!
= 81 × 84 = 6804 Words begin with VE are VEAMR
15! VEARM VEMAR
33 Required number of ways = .
3! × 4 ! × 8 ! VEMRA VERAM
VERMA
34 Total number of ways = 7 !
∴ The rank of the word VERMA = 4 × 4 ! + 3! + 6
Let A has to speak before B.
= 96 + 6 + 6 = 108
Now since there are half of the total cases in which A speaks
before B (Similarly in half of the total cases B speaks before A) 48 Case I 0, 1, 2, 4, 5
1 Case II 1, 2, 3, 4, 5
∴ Required number of ways = × 7 ! = 2520. Since we know that a number is divisible by 3, if and only if
2
the sum of its digits is divisible by 3.
35 1 10 10 10 10 10 10 ⇒ 106 Case I, number of 5 digit numbers = 4 × 4 × 3 × 2 = 96
Case II, number of 5 digit numbers = 5 × 4 × 3 × 2 = 120
Total number of 7 digit telephone numbers which begin
with digit 2 = 106. ∴ Total required numbers = 216
49 We can choose two men out of 8 in 8C 2 ways. Since no
36 6
P4 = 360.
husband and wife are to play in the same game, two
Alternatively First son can accept any of the 6
women out of the remaining 6 can be chosen in 6C 2 ways.
proposals and second son can accept any of the 5 proposals
and so on. If M 1, M 2, W1, W2 are chosen, then a team may consist of
Hence required number of ways = 6 × 5 × 4 × 3 = 360 M 1 and W1 or M 1 and W2. Thus the number of ways of
arranging the game is
37 16
C 4 = 1820
= 8C 2 × 6C 2 × 2 = 840
38 16
P4 = 43680
50 Let us consider n = 5, then we have {a, b, c, d, e} in a row.
1 16 !
39 16
C 4 × 12C 4 × 8C 4 × 4C 4 × = . Now we can select 3 of them so that no two are adjacent to
4 ! (4 !)5
each other is only one possible way that is {a, c, e}.
40 C 2 = 105, where n is the number of players
n
Now, let us consider n = 6, then we have {a, b, c, d, e, f } in a
n (n − 1)
⇒ = 105 ⇒ n = 15 row. Now we can select 3 of them so that no two are
2 adjacent to each other is as follows.
41 9
P6 = 60480 {a, c, e}, {a, c, f }, {a, d, f }, {b, d, f }
6! That is total four ways.
42 = 15
2! × 4 !
Permutations & Combinations 1127

Now, we know that if n = 5, required number of ways = 1 Hence if a person enters at S1 he can have 7 different tickets
and if n = 6, required number of ways = 3. for S2, S3, … S7 and Mumbai.
n− 2 Similarly if a person enters at S2 he can have 6 different
This is true for C 3.
tickets and if a person enters at S3 he can have 5 different
Basically, we know that the number of ways of selecting r
tickets and so on.
things out of n things in a row so that no two of the selected
things are adjacent to each other = n + 1 − rC r . ∴Total numer of possible tickets
= 7 + 6 + 5 + 4 + 3 + 2 + 1 = 28
Hence choice (b) is correct.
Now, the five different tickets must be out of these
51 Since 8 does not occur in 1000, we have to count the 28 tickets. Therefore, the required number of ways
number of times 8 occurs when we list the integers from 1
= C 5 = 98280
28
to 999. Any number between 1 and 999 is of the form xyz,
where 0 ≤ x, y, z ≤ 9. Let us first count the numbers in 55 The number of ways of choosing the committee when Ms. B
which 8 occurs exactly once. Since 8 can occur at one place is a member (when Mr. A refuses to serve)
in 3C1 ways. There are 3C1 (9 × 9) = 3 × 92 such numbers. = 8C 3 × 6C 5 = 56 × 6 = 336
Next, 8 can occur in exactly two places in 3C 2 (9) = 3 × 9 The number of ways of choosing the committee when Ms. B
such numbers. Lastly, 8 can occur in all three digits in one is not a member (when Mr. A can serve)
number only. Hence, the number of times 8 occurs is = 8C 4 × 7C 5 = 70 × 21 = 1470
1 × (3 × 92 ) + 2 × (3 × 9) + 3 × 1 = 300 Thus the required number of ways = 336 + 1470 = 1806.
Alternatively Total number of ways of selecting
52 Case I 2 2 1 4 women and 5 men = 9C 4 × 7C 5 = 2646.
Number of possible arrangements Number of ways of selecting 4 women and 5 men in which
= C 2 × C 2 × C1 × 3 = 90
5 3 1 Ms. B and Mr. A are not present together
= 8C 3 × 6C 4 = 840
Case II 1 1 3
Hence the number of ways in which Ms. B and Mr. A do not
Number of possible arrangements serve together = 2646 − 840 = 1806
= 5C1 × 4C1 × 3C 3 × 3 = 60 56 Required number of ways = 5 × 5 × 5 × 5 × 5 = 3125
Hence, the total number of ways is 90 + 60 = 150. 57 Required number of arrangements = 5 × 5 × 5 × 5 = 625
53 In a chess board there are 8 columns and 8 rows. Since we fix cap C1 in box B1 then we arrange the remaining
4 caps in any of the 5 boxes.
Alternatively Total number of arrangements = 55

55
Number of arrangements in which B1 have cap C1 = = 54
5
8 rows

58 Required number of arrangements = 5P5


∴ 5
P5 = 5 × 4 × 3 × 2 × 1 = 120
59 1 cap can be selected in 5C1 ways.
Now this selected cap can be arranged in 5 boxes in
5
P1 ways. Again 2 caps can be selected in 5C 2 ways.
8 columns
Now, these selected caps can be arranged in 5 boxes in
Since number of columns = number of rows 5
P2 ways and so on.
Hence it is considered as a square.
Hence the required number of ways
∴Number of square in a square of
n × n = 12 + 22 + … + n2 = 5C1 × 5P1 + 5C 2 × 5P2 + 5C 3 × 5P3 + 5C 4 × 5P4
+ 5C 5 × 5P5
Hence required number of squares
= 12 + 22 + 32 + … + 82 = 25 + 200 + 600 + 600 + 120 = 1545
8 × 9 × 17  n (n + 1)(2n + 1) 60 Since all the boxes are filled with exactly one cap and all
= = 204 Q Σn2 =
6  6  the caps are identical.
Hence the required number of ways = 1.
54
L S1 S2 S3 S4 S5 S6 S7 M 5!
61 Required number of ways = = 60
2!
Since these five people enter the train during journey it
(Since 2 caps are identical)
means they must have enter the train after Lucknow.
1128 QUANTUM CAT
62 B 3 can be filled in 2! ways and rest can be filled in 4! ways. Number
Case Blue Pink Reason
Hence, required number of ways = 2 × 4 ! = 48. of ways

63 B1 and B 5 can be filled up in 2 × 2 = 4 ways and B 2, B 3, B 4 V 2 4 2 2 cases arise depending on


can be filled up in = 5 × 5 × 5 = 125 ways. whether 2 blue faces are adjacent
Hence the required number of ways = 4 × 125 = 500 or opposite
64 Since Juliet never writes more number of letters than the VI 1 5 1 Since there is a symmetric view
number of letters Romeo writes, it implies that out of the VII 0 6 1 Since all the faces are of the same
total 6 letters first letter is written by Romeo. Now the color
remaining 5 letters can be written in the following order.
Total number of ways = 1 + 1 + 2 + 2 + 2 + 1 + 1 = 10
Order of Letters in which they are
written by Romeo and Juliet
NOTE In case IV there are two possibilities as depicted in the
Case
following diagrams.
1 2 3 4 5 6 Blue Blue
I R R R R J J
II R R R J R J
III R R R J J R Blue Blue Blue
IV R R J R R J
V R R J R J R
Blue
VI R R J J R R
(i) (ii)
VII R J R R R J
VIII R J R R J R In the figure (i) all three blue faces are not adjacent to each
IX R J R J R R other. In figure (ii) all three faces are adjacent to each other.
X R J J R R R 66 The number of ways of selecting 4 days out of 7 days
= 7C 4 = 35
Out of the above 10 cases, case X is invalid as in this case
Juliet writes 2 letters by the time Romeo writes only 1 The number of ways of selecting a shift from the 4 days
letter, which is contradictory to what is stated in the = 24 = 16
problem. Therefore total valid possibilities = 10 − 1 = 9. Therefore total number of ways of attending the gym
Alternatively = 35 × 16 = 560
Since Juliet never writes more number of letters than that
of Romeo, it implies that out of the total 6 letters Romeo
67 If you do not repeat the use of digits you cannot form more
than 3! = 6 numbers. Since, you have to form 900
writes first letter. Number of ways in which the remaining 5
5! numbers, so you have to repeat the use of digits.
letters can be arranged = = 10. But if the second and
3! 2! Since there are 3 digits namely, 2, 5, 7 and they have to be
third both letter are written by Juliet then it would be placed at n-places. So each place can be filled in 3 ways.
contradictory and so unacceptable. That is the case when Thus n places can be filled in 3n ways.
the order of writing the letters is RJJRRR. Therefore the It implies that 3n ≥ 900 ⇒ 3n− 2 ≥ 100 ⇒ n − 2 ≥ 5 ⇒ n ≥ 7
actual possible ways of writing the love-letters = 10 − 1 = 9
65 So the least value of n is 7.
Hence choice (b) is the answer.
Number
Case Blue Pink Reason Alternatively Consider the following table.
of ways

I 6 0 1 Since all the faces are of the n Number of n digit numbers


same color
1 3 3
II 5 1 1 Since there is a symmetric view
2 3× 3 9
III 4 2 2 2 cases arise depending on
whether the 2 pink faces are 3 3 × 3× 3 27
adjacent or opposite 4 3× 3× 3× 3 81
IV 3 3 2 In one scenario all three blue faces 5 3× 3× 3× 3× 3 243
are adjacent and in another 6 3× 3× 3× 3× 3× 3 729
scenario one blue face is adjacent
7 3 × 3 × 3 × 3 × 3 × 3× 3 2187
to the other two blue faces but the
other two blue faces are not If n is less than 7, you cannot form 900 numbers. So n must
adjacent to each other be at least 7 to form 900 (or more) n-digit numbers.
Permutations & Combinations 1129

68 Let us discuss the various possibilities of arranging the Therefore the number of rectangles
3 lines in a plane. = mC1 × mC1 × nC1 × nC1 = m2n2
Case I No 2 lines are parallel and no 3 lines are
Hence choice (c) is the answer.
concurrent.
Then the 3 lines divide the plane into 7 regions. Alternatively
Number of possible rectangles
1 = [1 + 3 + 5 + . . . + (2m − 1)] × [1 + 3 + 5 + . . . + (2n − 1)]
6 2 = m2 n 2
7
71 Total number of shortest paths from A to C
5 4 3
10
C 6 × 6C 6 = 210
Number of shortest paths from A to R
Case II No 2 lines are parallel but the 3 lines are
concurrent. = 5C 4 × 1C1 = 5
Then the 3 lines divide the plane into 6 regions. Number of shortest paths from R to C
= 5C 2 × 3C 3 = 10

1
Number of shortest paths from A to C, via R
6 2
= 5 × 10 = 50
5 3
4 D (0, 4) C (6, 4)

Case III Exactly 2 lines are parallel and the third line is a
transversal.
Then the 3 lines divide the plane into 6 regions.
R (4, 1)

1 2 A (0, 0) B (6, 0)

3 4 Therefore the required number of shortest paths from A to


5 6 C, without going via R
= 210 − 50 = 160
72 The possible combinations are
Case IV All the 3 lines are parallel. (i) 1, 1, 1, 1, 1, 2, 3
Then the 3 lines divide the plane into 4 regions. (ii) 1, 1, 1, 1, 2, 2, 2
2 Hence total number of seven digits number
7! 7!
3
= + = 77
5! 4 ! 3!
4 Hence choice (c) is the answer.
73 The maximum possible number of regions created by
Thus it is evident that 3 lines can divide a plane into 4, 6 or
n triangles = 3n2 − 3n + 2.
7 regions but not into 5 regions.
69 There will be total 10 + 1 = 11 spots where 4 security posts Therefore the maximum possible number of regions
created by 6 triangles = 3(6)2 − 3(6) + 2 = 92
can be established.
Therefore number of different ways of selecting 4 spots out
of 11 spots = 11C 4 = 330
70 The rectangle is divided by 2m − 1 vertical lines, so there
are total 2m vertical lines.
Similarly the rectangle is divided by 2n − 1 horizontal lines,
so there are total 2n horizontal lines.
Since we are supposed to have the rectangles of odd
dimensions (e.g. , 1, 3, 5,…). So we will select two vertical
lines such that one is even numbered and another is odd Six overlapping triangles, dividing the plane
numbered and similarly two horizontal lines. into 92 distinct regions
1130 QUANTUM CAT
74 Here are few examples of the required numbers: Similarly, maximum number of regions when 6 lines are
123, 124, 125, . . . , 129 and 134, 135, 136,…, 139 and 145, drawn = 1 + 1 + 2 + 3 + 4 + 5 + 6 = 22
146, 147,…149. Therefore maximum additional number of regions
For all such numbers please consider the following table. = 22 − 7 = 15
Alternatively If n straight lines are intersecting a circle
a b c Total values
such that no two lines are parallel and no three lines are
2 3, 4, ...., 9 concurrent, the maximum number of parts/regions into
3 4, 5, ...., 9 which these lines divide the circle is
... .............. n(n + 1) n2 + n + 2
n
C 0 + nC1 + nC 2 = +1= .
1 ... .............. 28 2 2
6 7, 8, 9 Then the maximum number of regions when there are
1
7 8, 9 3 lines = (9 + 3 + 2) = 7
2
8 9
And, the maximum number of regions when there are
3 4, 5, ..., 9 1
6 lines = (36 + 6 + 2) = 22
4 5, 6, ...., 9 2
2 ... ................. 21 Therefore, maximum number of additional regions
... ................ = 22 − 7 = 15
8 9 76 If the number of countries in the four continents are a, b, c
4 5, 6, ..., 9 and d, so he has to visit at least
3 ... ................. 15 (a − 1) + (b − 1) + (c − 1) + (d − 1) number of countries and
then one more country from any continent.
... ................
8 9 Total
Asia Africa Europe America number of
4 ... ... 10 Countries
5 ... ... 6 Number of
6 ... ... 3 countries to be 21 16 11 6 54
7 ... ... 1 visited
Minimum
8 ... ... 0
number of
9 ... ... 0 countries he 20 15 10 5 50
must visit in
Therefore the total number of required three digit numbers
each continent
= 1 + 3 + 6 + 10 + 15 + 21 + 28 = 84
75 When there is no line the number of regions = 1 First let him visit 20 + 15 + 10 + 5 = 50 countries. Now, if
When 1st line is drawn, new regions added = 1 he visits 1 more country in any of these four continents,
then we can say that he visits at least 21 Asian or at least 16
When 2nd line is drawn, new regions added = 2 African or at least 11 European or at least 6 American
When 3rd line is drawn, new regions added = 3 countries. Thus, he should visit minimum 51 countries to
When 4th line is drawn, new regions added = 4 satisfy the given condition.
When 5th line is drawn, new regions added = 5 Hence choice (b) is the correct answer.
When 6th line is drawn, new regions added = 6 77 When A sits next to B, number of seating arrangements
= 5! × 2! = 240
When A sits next to B, and P sits next to T, number of
arrangements = 4 ! × 2! × 2! = 96
Therefore, the required number of seating arrangements
= 240 − 96 = 144
78 Given that 9 + 1 + a + b + c + d = 16
Thus maximum number of regions when 3 lines are drawn ⇒ a+ b+ c+ d = 6
=1 + 1 + 2+ 3=7 ∴Number of integral solutions = 9C 3 = 84
Permutations & Combinations 1131

79 There can be total 3 main cases (Assuming Sindhu wins the 3!


This can be done in = 3 ways
series): 2!

Total
Case III Since we know that the last match has to be
Number of Number of Number of definitely won by the winner of the series (Sindhu), so we
matches
matches matches ways to have to arrange the remaining 2 matches in which Sindhu
Case played to
won by won by arrange the wins and the two matches in which Jwala wins.
end the
Jwala Sindhu matches
series 4!
This can be done in = 6 ways
3! 2! 2!
I 0 3 3
Valid arrangements, since the last winner is Sindhu –
3!
{JJSSS, JSJSS, JSSJS, SJJSS, SJSJS, SSJJS}
II 1 3 4 3!
Invalid arrangements, since the Sindhu has already won
2!
the series before 4th/5th match of the series –
III 2 3 5 4!
{SSSJJ, SSJSJ, SJSSJ, JSSSJ}
2! × 2!
Thus, Sindhu wins the series in 10 (= 1 + 3 + 6) ways.
Case I Since all matches are won by only one person, so Similarly, there are 10 ways in which Jwala wins the series.
there is only 1 way Sindhu can win this series. Therefore, total number of ways in which this series could
Case II Out of the 4 matches, Jwala cannot win the last be won = 20.
match. If Jwala wins the last match, it means Sindhu has 80 Given that 109 < n < 1010. It implies that n is a 10-digit
already won the first 3 matches, so there is no need to play number.
the fourth match. Therefore, the last match has to be won
Case I n = 6 × 1 × 1 × 1 × 1 × 1 × 1 × 1 × 1 × 1
by the winner of the series.
Number of possible numbers = 10
Valid arrangements, since the last winner is Sindhu –
Case II n = 3 × 2 × 1 × 1 × 1 × 1 × 1 × 1 × 1 × 1
{JSSS, SJSS, SSJS}
Number of possible numbers = 10P2 = 90
Invalid arrangements, since Sindhu has already won the
Total required numbers = 10 + 90 = 100
series before 4th match of the series – {SSSJ}
Since, we have observed that in this type of questions, it is 81 First of all he picks 1 rose, 2 lilies, 3 orchids and
4 carnations in order to fulfill the minimum requirement
the winner of the series who always plays the last winning
criterion. Thus 10 flowers have already been kept aside for
match, so to solve this problem we should keep away the
the bouquet.
last winning match separately, and arrange the remaining
matches. Now, the remaining 5 flowers can be chosen from any of
In this case there are 4 matches (1 match is won by Jwala the four types of flowers, such that R + L + O + C = 5 and
and other 3 are won by Sindhu), so we will keep the last R , L, O , C ≥ 0.
match of Sindhu separately, and arrange the remaining Therefore, the required number of ways
matches (1 match of Jwala and 2 matches of Sindhu). = 5 + 4 − 1C 4 − 1 = 8C 3 = 56

Level 02 Higher Level Exercise


1 First and second prizes can be given in Mathematics in Alternatively Consider n = 1, 2, 3, … etc. and then
(30 × 29) ways. First and second prizes can be given in verify the result.
Physics in (30 × 29) ways.
First prize can be given in Chemistry in 30 ways. 3 Row-1 Row-2 Row-3 Row-4
First prize can be given in English in 30 ways. Case 1. Class 1 Class 2 Class 1 Class 2
Hence, the number of ways to give prizes in all the four Case 2. Class 2 Class 1 Class 2 Class 1
subjects Required seating arrangement can be done in two ways-
(30 × 29) × (30 × 29) × 30 × 30 = (30)4 × (29)2 case 1 and case 2. Total numer of arrangements
2 1 + 1 ⋅ P1 + 2⋅ P2 + 3⋅ P3 + 4 ⋅ P4 + … + n ⋅ Pn = Number of arrangements in case 1
+ Number of arrangements in case 2
= 1 + 1 + 2⋅ 2! + 3⋅ 3! + 4 ⋅ 4 ! + … + n ⋅ n ! Now, 12 students of class 1 can be seated in 12 chairs in 12 P12
= 1 + [(2! − 1 !) + (3! − 2!) + (4 ! − 3!) + … ways and 12 students of class 2 can be seated in 12 chairs in
12
+ ((n + 1)! − n !)] P12 ways. Hence, the total number of arrangements
= (12! × 12!) + (12! × 12!) (Q 12P12 = 12!)
= 1 + [(n + 1)! − 1 !] = (n + 1)!
= 2 (12!)2
1132 QUANTUM CAT
4 There are 5 even digits viz., 0, 2, 4, 6 and 8. Digit 3 cannot 9 5 ‘+’ (plus) signs can be arranged in 1 way, then we have
be at the tens place, since if 3 is at tens place, then 9 must 6 places to arrange 5 ‘–’ (minus) signs in 6C 5 ways but since
be at unit place which is impossible. Hence 3 can be only at all the minus (–) signs are identical, therefore minus signs
hundred’s place. Now there are two cases :
can be arranged in only 1 way in 5 chosen places out of 6.
1. When 3 is at hundred’s place : If 3 is at
Hence the required number of arrangements
hundred’s place, then 9 will be at ten’s place. So unit
= 1 × 6C 5 × 1 = 6
place can be filled up in only 5 ways by using even
digits. 10 In order that no two books on Maths be together we first
So, the total number of even numbers = 1 × 1 × 5 = 5. arrange 11 identical books on English which can be done in
2. When 3 is not at hundred’s place : In this case 1 way.
hundred’s place can be filled up in 8 ways (0 and 3 Now we can choose 9 places out of 12 (11 + 1) places for
cannot be used). In tens place we can use any one of Maths books, which can be done in 12C 9 ways.
the 10 digits except 3. So tens place can be filled up in 9 Again since all books on Maths are identical which can be
ways and unit place can be filled up in 5 ways by using mutually arranged in only 1 way.
0, 2, 4, 6 and 8. Hence the required number of arrangements = 1 × 12C 9 × 1
So, the total number of even numbers = 8 × 9 × 5 = 360 = 220
Hence the total number of required 3 digit even numbers
11 1680 = 2 × 3 × 5 × 7
4
= 5 + 360 = 365
Since if N = a p ⋅ bq ⋅ cr . …
5 Number of vehicles registered upto R Z 9999
we have number of factors of N = ( p + 1)(q + 1)(r + 1)…
= 18 × 26 × (9999)
Hence the total number of factors of the given number
Number of vehicles registered between SA-0001
= (4 + 1)(1 + 1)(1 + 1)(1 + 1) = 40
and SJ 9999 = 1 × 10 × 9999 n
m+1 m+ 2
Therefore number of vehicles registered before SK-0123 12 Σ k
C r = mC r + Cr + C r + … + ( n − 1)C r + nC r
k=m
= 18 × 26 × 9999 + 1 × 10 × 9999 + 122 Now assume some values of m and n, then verify the
= 9999 × 478 + 122 = 4779644 correct result from the choices given in the question.
6 Available digits are 1, 2, 3, 4 and 5. Now since we know 13 The number of students answering exactly k (1 ≤ k ≤ n − 1)
that a number is divisible by 4 if and only if the number questions wrongly is 2n − k − 2n − k − 1.
formed by last two digits is divisible by 4. So the following
cases are possible. The number of students answering all n questions wrongly
is 20. Thus, the total number of wrong answers is :
Thousands Hundreds Tens Unit
1 (2n − 1 − 2n − 2 ) + 2 (2n − 2 − 2n − 3 ) + K
x y 1 2
K + (n − 1)(21 − 20 ) + n (20 )
x y 2 4
= 2n − 1 + 2n − 2 + 2n − 3 + … + 20 = 2n − 1
x y 3 2
⇒ 2n − 1 = 2047 ⇒ 2n = 2048 ⇒ n = 11
x y 5 2
14 Required number of triangles = Total number of triangles
In each case thousands and hundreds places can be filled – Number of triangles having one side common
up in 3 P2 ways.
with the octagon
Hence the required number of ways = 3P2 × 4 = 24. – Number of triangles having two sides common
7 Total number of seats = 2 (3 + 4) = 14 with the octagon
Total number of persons = 3 + 9 = 12 = 8C 3 − (8C1 × 4C1 ) − 8 = 16
∴Total number of required arrangements = 14P12. 15 The number of ways of selecting any five numbers from the
8 Three girls can be seated together in a back row on given set = 25
C 5.
adjacent seats in the following ways : Now, the five consecutive numbers can be chosen in the
1, 2, 3 or 2, 3, 4 of first van and 1, 2, 3 or 2, 3 4 of second van. following ways
In each of the above four cases all the three girls can be (1, 2, 3, 4, 5);(2, 3, 4, 5, 6);(3, 4, 5, 6, 7 );…(21, 22, 23, 24, 25)
inter change among themselved in 3! ways. Thus the required number of ways = C 5 − 21 = 53109
25

So the total number of ways = 4 × 3!


16 Q a ≥ − 1, b ≥ − 1, c ≥ − 1, d ≥ − 1.
Now, 9 boys can be seated in remaining 11 seats, which can
be done in 11 P9 ways. Let u ≥ 0, v ≥ 0, w ≥ 0, x ≥ 0.
∴ a + b + c + d = 12
Hence, the required number of ways = 11P9 × 4 × 3!.
Permutations & Combinations 1133

⇒ (u − 1) + (v − 1) + (w − 1) + ( x − 1) = 12 (ii)
6!
× 4C 2 = 1080
⇒ u + v + w + x = 16 2! 2!
∴ Required number of solutions = 16 + 4 − 1C 4 − 1 = 19C 3 Therefore the required number = 480 + 1080 = 1560.

17 Assume some value of k then find ‘n’ and hence nC 2. 24 9 10 10 10 10 10 5 = 4500000


Now, verify the correct option. Since if we fix first digit then unit digit will determine the
18 42 = 2 × 3 × 7 odd/even nature of the numbers so formed.
Here each of a, b and c can take 3 values. But unit place can assume half even and half odd. Hence
Hence the required number of solutions = 3 × 3 × 3 = 27. unit place can assume just 5 digits which makes the sum of
19 Let (r, b, g) denotes the number of red, blue and green balls previous six digits even.
respectively. 25 Required number of ways = 6P4.
Therefore the possibilities are (0, 10, 0), (2, 7, 1), (4, 4, 2)
and (6, 1, 3). 26 Suppose x one rupee coins, y fifty paise coins and z twenty
paise coins are selected.
Hence, the required number of ways = 4.
Then x+ y+z=6
20 Let x1 be the number of stations before the first halting
∴Total number of ways
station, x 2 between first and second, x 3 between second
and third, x 4 between third and fourth and x 5 on the right = Number of solutions of the above equation
6 + 3 −1
of the fourth station, then = C 3 − 1 = 8C 2 = 28
x1 ≥ 0, x 2, x 3, x 4 ≥ 1 and x5 ≥ 0
27 Required number of times
such that x1 + x 2 + x 3 + x 4 + x 5 = 6. = Number of selections of 3 children taking at a time
Now, the total number of ways is the number of solutions of = 8C 3 = 56
the above equation.
28 Each child will go as often as he or she can be accompanied
Let y 2 ≥ 0, y 3 ≥ 0, y 4 ≥ 0.
by two others. Therefore the required number
∴ x1 + x 2 + x 3 + x 4 + x 5 = 6 = 7C 2 = 21
⇒ x1 + ( y 2 + 1) + ( y 3 + 1) + ( y 4 + 1) + x 5 = 6
29 Total number of points = 12
⇒ x1 + y 2 + y 3 + y 4 + x 5 = 3 Total number of triangles = 12C 3
3 + 5 −1
∴ The number of solutions = C 5 − 1 = C 4 = 35
7
But there are 3 cases which must be excluded for the
21 Let x i denote the marks assigned to the i question. Then th required number of triangles.
Case 1. The number of triangles formed by 3 points on
x1 + x 2 + x 3 + … + x7 + x 8 = 30
AB = 3C 3 = 1
where i = 1, 2, 3, … , 8 and xi ≥ 2
Case 2. The number of triangles formed by 4 points on
and y1 + y 2 + … + y7 + y 8 = 14 BC = 4C 3 = 4
where y i = x i − 2, i = 1, 2, 3, …
Case 3. The number of triangles formed by 5 points on
∴ The total number of solutions of this equation is CA = 5C 3 = 10
14 + 8 − 1
C8 −1 = 21
C7 Hence, the required number of triangles
22 ∴ The exponent of 3 in 33! is 15. = 220 − (10 + 4 + 1) = 205
30 Each question can be attempted in 3 ways i.e., either it can
3 33 be omitted or one of the two parts can be attempted.
3 11 ←  Hence required number of ways = 310 − 1 = 59048.
 15 31 Required number of ways = 16C 9.
3 3 ← 
1 ←  32 Since each bulb has two choices, either switched on or off,
therefore required number = 210 − 1 = 1023.
Hint For more information about exponents refer the first
chapter of the book. 33 Since each digit of a 10 digit number can be written as
either 2 or 3, therefore required number of 10 digit number
23 There are two distinct possibilities
is 210.
(i) When any of the four digits may repeat thrice.
34 Consider some value of n then verify the result.
(ii) When any two of the four digits may repeat twice.
35 Any divisor of 23 ⋅ 34 ⋅ 52 is of the form 2a ⋅ 3b ⋅ 5c where
6! 4
(i) × C1 = 480 0 ≤ a ≤ 3, 0 ≤ b ≤ 4 and 0 ≤ c ≤ 2. Thus, the sum of the
3!
1134 QUANTUM CAT
divisors of 23 ⋅ 34 ⋅ 52 is remaining 7 persons can be placed in 7! ways.
(1 + 2 + … + 23 )(1 + 3 + … + 34 )(1 + 5 + 52 ) Hence, the number of ways in which they can speak is
(24 − 1)(35 − 1)(53 − 1) 15 × 242 × 124 10 !
= =
10
C3 × 7 ! =
(2 − 1)(3 − 1)(5 − 1) 1 × 2× 4 3!
= 56265 Alternatively All the 10 persons can speak in 10! ways
10 + 4 − 1 in which A, B and C can speak in 6 different ways.
36 Required number of ways = C4 −1 = C 3 = 286
13
But we are required to find a single case out of 6 cases in
37 Since we know that out of n things arranged in a row then which A speaks before B and B speaks before C.
we can select r things out of n such that no two of them are 10 ! 10 !
Hence the required number of ways = =
adjacent to each other in n + 1 − r C r ways. 3! 6
Hence required number of ways = 10 + 1 − 3C 3 = 8C 3 = 56 42 Total one digit number = 2
Alternatively To each selection of 3 objects we Total two digit numbers = 22
associated a binary sequence of the form 1001000100 Total three digit numbers = 23
where 1 at ith place means the i th object is selected and 0 at Total four digit numbers = 24
ith place means the i th object is not selected. … … … … …
There exists one to one correspondence between the set of Total eight digit numbers = 28
selections of 3 objects and set of binary sequences There are total 9 digit numbers = 29
containing 7 zeros and 3 ones.
but out of these 29 numbers half of the numbers begin with
Actually we are concerned with the binary sequences in digit 1 and rest half numbers begin with digit 2.
which 2 ones are consecutive we first arrange 7 zeros.
The greatest possible number less than 2⋅ 108 is
This can be done in just one way. Now, 3 ones can be 122222222, which in turn is the greatest possible number
arranged at any of the 3 places marked with a cross in the of 9 digit begin with 1. Thus the maximum possible number
following arrangement. 29
of 9 digit numbers = = 28
×0×0×0×0×0×0×0× 2
We can arrange 3 ones at 8 places in 8C 3 ways = 56 Hence, the required total numbers
38 Let x 5 be such that x1 + x 2 + x 3 + x 4 + x 5 = n = 2 + 22 + 23 + 24 + … + 28 + 28
Now we require the non-negative integral solutions of 2 (28 − 1)
= + 28 = 29 − 2 + 28 = 28 (2 + 1) − 2 = 766
x1 + x 2 + x 3 + x 4 + x 5 = n 2−1
n+ 5 −1 n+ 4
The number of required solutions = C5 −1 = C4 43 Required number of ways = 12 + 1 − 4C 4 = 9C 4 = 126.
39 When we arrange things one at a time, the number of n
44 Since 2k < n ⇒ k <
possible permutations is n. 2
When we arrange them two at a time, the number of Hence, number of zeros is less than the number of ones.
possible permutations is n × n = n2 and so on. Now, first of all we arrange (n − k ) ones as follows
× 1 × 1 × 1 × 1 × 1 ×… × 1 ×
Hence, the total number of required permutations is
The above arrangement leaves (n − k + 1) places marked
n (nr − 1) with ×.
n + n2 + n3 + … + nr =
n −1 Now we can arrange k zeros in the (n − k + 1) places in
n− k + 1
40 In a chess board there are 8 columns and 8 rows. C k ways.
Hence the number of rectangles = 13 + 23 + 33 + … + 83 45 Part A can be attempted in 3 ways
2
 8 × 9 (i) The student do not solve the question.
=  = 1296
 2  (ii) The student attempts the first part of the question.
(iii) The student attempts the second (i.e., alternative) part
Alternatively A chessboard is a network of 9 vertical
of the question.
and 9 horizontal lines.
∴ Total number of choices in the first part = 35
Since we know that number of quadrilaterals formed by
m vertical and n horizontal lines is mC 2 × nC 2. Hence, the required number of ways in which a student
must attempt at least one question = 35 − 1 = 242
Hence the required numer of rectangles = C 2 × C 2 = 1296
9 9
Similarly we can show that there are 24 − 1
41 Order of speaking is equivalent to the order of placing the
i.e. 15 choices for part B.
things.
10
Hence the required number of ways = 242 × 15 = 3630.
Thus A, B and C can be placed in C 3 ways and the
Permutations & Combinations 1135

46 Required number of number plates of licences Remaining 4 places can be filled up in 4! ways but since
= (26)2 × (999) = 675324 zero is being repeated thrice hence the actual number of
4!
ways = =4
47 Required number of plates = 26
P2 × 10P3 = 468000 3!
48 We can arrange r persons on m chairs on a particular side in ∴The required number of ways = 1 × 4 = 4.
m
Pr ways and s persons on m chairs on the other side in 55 Digit 5 can be placed in any one of the 4 places in 4 ways.
m
P s ways. Now, we can arrange (2m − r − s ) persons on the Now the remaining 3 places can be filled up with remaining
remaining (2m − r − s ) chairs in ( 2 m − r − s)
P( 2m − r − s) ways. 8 digits in 83 ways.
Hence, the required number of ways = 4 × 83 = 2048.
Thus, the required number of ways of arranging the
persons = (m Pr )(m P s )(2m − r − s P2m − r − s ) 56 Required number of ways = 4 × 8 × 7 × 6 = 1344
49 The diagram of roads can be as follows : 57 Digit 2 can be arranged in two places out of 8 places in
8
P2
 ways.
2!


m-roads

Now, the remaining 6 places can be filled by the rest


 3 digits in 36 ways.
 8
 Hence, the required number of ways =
P2
× 36
 2!
14444444244444443 = 8C 2 × 36 = 20412
n-roads
58 Required number of ways = 38 = 6561.
Suppose the distance between two successive parallel
roads be one unit then a person can travel (m − 1) steps in 59 Since chairs are numbered, hence these are
north-south direction and he can travel (n − 1) steps in distinguishable.
east-west direction. Therefore 3 boys can be arranged in 3! ways on alternate
Thus he has to travel {(m − 1) + (n − 1)} = (m + n − 2) steps chairs and 3 girls can be arranged in remaining 3 chairs in
to reach from one corner to diagonally opposite corner. 3! ways.
Thus he can arrange his steps in Also 3 girls can be arranged before the boys are arranged.
( m + n − 2)
C( m − 1) × ( n − 1)C( n − 1) ways Hence the required number of ways = 2 × (3!)2 = 72
m + n− 2 64
i.e. C m − 1 ways. 60 First rook can be selected in 64 ways C1 .
50 The required number of ways is equal to the number of Second rook cannot be selected from the same row and
ways in which they can select the same number of toys same column from which the first rook has been selected
from the collection of their combined number of toys – the hence there are only 64 − (8 + 7 ) = 49 places for the second
number of ways in which each can select her original toys. rook.
Hence, required number of ways = (19C11 × 8C 8 ) − 1 Hence, the second rook can be selected in 49
C1 ways.
= 19
C11 − 1 ∴ Required number of ways = C1 ×
64 49
C1 = 3136.
51 Go through options. 61 Number of white shoes = 9 pairs
52 The number of ways of choosing 26 cards out of 52 cards is
52 ⇒ 9 left and 9 right shoes
C 26. There are two ways in which each card can be dealt
Number of black shoes = 6 pairs
because a card can be either from a first pack or from the
second pack. ⇒ 6 left and 6 right
Hence, the number of ways in which the card can be dealt is In such questions we always consider the worst situation.
52
C 26 × 226 Therefore we assume that initially we get 6 + 6 = 12
53 There are 4 suits and each suit contains 13 cards so we can black shoes and 9 white shoes either left or right.
select any one card from the first suit in 13 ways. Similarly Thus we have taken out 12 + 9 = 21 shoes still we don’t
we can select one card different from the first card out of have required pair of white shoes.
second suit in 12 ways and so on. Now we drawout just one white shoe which will fulfill our
Hence, the required number of ways = 13 × 12 × 11 × 10 requirement of getting one pair of white shoes.
= 13!/ 9 ! [One pair of white shoes means 1 left shoe and 1 right
54 First digit (i.e. ten thousands place) can be filled up in shoe.]
2! ways by the digit 3, but since 3 is being repeated twice, Thus we have to draw out 12 + 9 + 1 = 22 shoes to get
2! correct pair of white shoes.
hence the actual number of ways = =1
2!
1136 QUANTUM CAT
62 The worst possible situations are : Hence, total number of ways = 5C1 × 7C 3 × 4 !.

White Black Case 2. There are 2 boxes (each box contains 2 balls) is
selected in 5C 2 ways.
9 left 6 left
9 left 6 right Now, any two ball can be selected in 7 C 2 ways and for the
9 right 6 left other selected box, 2 balls can be selected from the
remaining 5 balls in 5C 2 ways and the remaining balls can
9 right 6 right
be distributed in 3! ways.
Thus before getting the required correct pair of shoes we Hence, the total number of ways = 5C 2 × (7 C 2 × 5C 2 ) × 3!
have to draw out 9 + 6 = 15 shoes.
∴The required number of ways
Now when we will draw out one more shoe which will give
the required combination of correct shoes either white or = ( 5C1 × 7C 3 × 4 !) + ( 5C 2 + 7C 2 × 5C 2 × 3!) = 16800
black. 69 Required number of ways = 7 + 5 − 1C 5 − 1
Hence we have to draw out minimum 15 + 1 = 16 shoes. n+ r −1
(using C r − 1) = 11C 4 = 330
63 We cannot put only one incorrect ball since if we make a
wrong choice it will simultaneously make another choice 70 Possible ways of distribution are as follows :
wrong. For example if we put red colour ball in either of (i) (7) (ii) (6 + 1)
green, blue or black box then we have to fill the red colour
(iii) (5 + 2) (iv) (4 + 3)
box with different colour ball, hence one colour
combination can never be wrong. (v) (5 + 1 + 1) (vi) (4 + 2 + 1)
Since atleast two combinations will be different. (vii) (3 + 2 + 2) (viii) (3 + 3 + 1)
64 Select any two balls out of 4 then interchange the (ix) (4 + 1 + 1 + 1) (x) (3 + 2 + 1 + 1)
combination of colours. (xi) (2 + 2 + 2 + 1) (xii) (3 + 1 + 1 + 1 + 1)
Hence the required number of ways = 4C 2 = 6. (xiii) (2 + 2 + 1 + 1 + 1)
Alternatively Hence total number of ways of distribution = 13.
R B G W R B G W 71 There is only one way, which is as follows :
 , 
B R G W G B R W 1, 2, 1, 2, 1.
R B G W R B G W 72 One ball can be selected for the box 2 in 7 C1 ways and two
 , 
W B G R R G B W balls can be selected out of remaining 6 balls for the box 4
R B G W R B G W in 6C 2 ways and the remaining 4 balls can be distributed in
 , 
R W G B R B W G 3 boxes in 34 ways.
65 For one digit number 1 to 9 → 1 × 9 = 9 Hence, the required number of ways
For two digit numbers 10 to 99 → 2 × 90 = 180 = 7C1 × 6C 2 × 34 = 8505
For threee digit numbers 100 to 300 → 3 × 201 = 603
73 Ball 2 can be distributed between 2 boxes (viz., box 2 and
Hence, total number of required digits = 9 + 180 + 603= 792
box 4) in 2 ways.
66 Four odd digits (1, 1, 3, 3) can be arranged in 4 odd places
4! Now, the remaining 6 balls can be distributed in any of the
in = 6 ways. 5 boxes in 56 ways.
2! 2!
Hence, the required number of ways = 2 × 56 ways
Now the remaining 3 even digits (2, 2, 4) can be arranged in = 31250 ways
3!
3 even place in = 3 ways. 74 Required number of distribution
2!
Hence the required number of ways = 6 × 3 = 18. = Total number of distribution
– Number of distribution in which ball 2
67 Each of the 7 balls can be distributed into 5 ways.
and ball 4 are together.
Hence the required number of ways = 57 .
Here total number of distribution (as calculated in question
68 There can be two cases number 68) = 16800.
(1) 1, 1, 1, 1, 3 Now, consider ball 2 and ball 4 are stuck together and this
(2) 1, 1, 1, 2, 2 arrangement is assumed to be a single ball.
Case 1. A box (in which 3 balls can be put) is selected in Thus we have 6 balls to be distributed into 5 boxes. Which
5
C1 ways. can be done as 1, 1, 1, 1, 2 in 5C1 × 6C 2 × 4 ! ways = 1800.
Now, 3 balls can be selected in 7 C 3 ways and remaining 5
C1 → Number of ways of selecting one box in which two
4 balls can be arranged in 4! ways. balls are kept together.
Permutations & Combinations 1137

6
C 2 → Number of ways of selecting 2 balls out of 6 balls. (2, 1, 1) → can be done in 2 ways
4 !→ Number of ways of distribution of remaining 4 balls either distinct pot can be filled up with
in remaining 4 boxes. 1 ball or 2 ball.
Hence, the required number of ways Hence, the required number of ways = 2 + 3 + 2 + 2 = 9.
= 16800 − 1800 = 15000
75 Required number of ways = 34. 82 The possible arrangements are as follows :
(4, 0, 0) → can be done in 3 ways
76 Following 4 distributions are possible : let the 4 coins be A, A, B and C then all
(4, 0, 0), (3, 1, 0), (2, 2, 0), (2, 1, 1) these 4 coins as a single packet of coins can
4 + 3 −1 be arranged 3 different pots in 3 ways.
77 Required number of ways = C3 −1
(3, 1, 0) → can be done in 18 ways
n+ r −1
(using C r − 1) = C 2 = 15
6
out of A, A, B and C we can select one coin
in 3 ways i.e., either A or B or C.
78 Since pots are identical then there will be 4 cases (4, 0, 0),
Now we can arrange this selected coin in any 3 pots in
(3, 1, 0), (2, 2, 0) and (2, 1, 1) but since all coins are 3 ways and the remaining 3 coins as a single packet of coins
different hence selection of coins matters. can be arranged in remaining 2 pots in 2 ways.
Therefore for the first case number of selections = 4C 4 = 1 Hence, the required number of ways = 3 × 3 × 2 = 18.
For the second case number of selections = 4C 3 × 1C1 = 4 (2, 2, 0) → can be done in 12 ways
There are two possible cases :
4
C 2 × 2C 2
For the third case number of selections = =3 (i) ( A, A ), (B, C ) (ii) ( A, B ), ( A, C )
2!
In each of two cases we assume that there are two packets
For the fourth case number of selections
of coins which can be arranged in 3 × 2 = 6 ways.
4
C 2 × 2C1 × 1C1
= =6 Since there are two cases, hence the total number of
2! required ways = 6 × 2 = 12.
Hence the total number of distributions (2, 1, 1) → can be done in 21 ways.
= 1 + 4 + 3 + 6 = 14. There are 4 possible cases :
79 Since no box is empty and all pots are identical so the (i) ( A, A ), (B ), (C ) (ii) ( A, B ), ( A ), (C )
possible case is (1, 1, 2). (iii) ( A, C ), ( A ), (B ) (iv) (B, C ), ( A ), ( A )
But since all the coins are different, then the 2 balls can be For the first 3 cases in each cases all the 3 packets of coins
2
C × 1C1 can be arranged in 3! ways.
selected in 4C 2 ways and rest can be put in 1 ways.
2! Hence, the number of arrangements = 3 × 3! = 18.
Hence, the required number of distribution Now in the fourth (iv) case two coins are identical so the
C 2 × 2C1 × 1C1
4
third packet of coins can be arranged in 3 pots in 3 ways.
= =6
2! Hence, the total number of arrangements = 18 + 3 = 21.
80 Since no pot is empty and all coins are identical, the Thus the required number of ways
possible case is (1, 1, 2). = 3 + 18 + 12 + 21 = 54 ways.
But since all three pots are different hence a pot (which 83 Since he switches off the bulb immediately after switching
contains 2 coins together) can be selected in 3C1 ways. it on, so he basically switches on 3 times (6 = 3 times on +
Hence the required number of distribution = C1 × 1 = 3. 3 3 times off). So, for every time he wants to switch a bulb on
he can choose any of the two bulbs, so for 3 different
81 The possible arrangements are as follows : occasions he has 8 options = (2×2×2).
(4, 0, 0) → can be done in 2 ways Therefore the required number of ways = 8
i.e., 4 balls can be put either one of the two 84 In order to come back to the same point he has to take same
identical pots or can be put in different number of steps in North and South directions or same
pot. number of steps in East and West directions. Then we have
the following five cases:
(3, 1, 0) → can be done in 3 ways
either distinct pot can be filled up with Case North South East West Number of ways
3 balls, or 1 ball or remained empty. I 4 4 0 0 8!
= 70
(2, 2, 0) → can be done in 2 ways 4 !4 !
II 3 3 1 1 8!
either distinct pot can be filled by 2 balls or = 1120
3! 3!
remained empty.
1138 QUANTUM CAT
Case North South East West Number of ways 3 4 42
III 2 2 2 2 8!
= 2520 ... ... ... 19
2! 2! 2! 2!
... ... ...
IV 1 1 3 3 8!
= 1120 3 22 24
3! 3!
V 0 0 4 4 8! ...
= 70
4 !4 ! ...
14 15 20
Therefore the total required number of ways
... ... ... 3
= 2(70) + 2(1120) + 2520 = 4900
14 17 18
85 It is like dividing 100 identical things among 3 persons,
such that each one must get at least 1 thing. 15 16 18 1
Therefore we have 100 − 1C 3 − 1 = 4851
Therefore total number of ways of building the toilets in 3
Since there are 3 parties and 100 seats, so it’s obvious that selected villages
all the 3 parties cannot get the same number of seats. = 1 + (3 + 4) + (6 + 7 ) + (9 + 10) + (12 + 13)
However, there can be ways in which any 2 parties can get
+ (15 + 16) + (18 + 19) + (21 + 22) + 24
the same number of seats.
= (1 + 7 + 13 + 19 + 25 + 31 + 37 + 43) + 24
These 2 parties (or allies) can be selected in 3C 2 = 3 ways. (1 + 43)8
The 2 parties can get the same number of seats in the = + 24 = 200
2
following ways.
Hence, choice (d) is the answer.
(1, 1, 98), (2, 2, 96), (3, 3, 94), ..., (48, 48, 4), (49, 49,2)
Alternatively
That means there are total 49 ways. Total number of ways of building 49 toilets in 3 villages.
Thus the total number of ways in which any 2 parties can 51 × 50
get the same number of seats = 3 × 49 = 147 = 49 + 3 − 1C 3 − 1 = 51C 2 = = 1275
2
Number of ways in which all the three parties get the same Now look out for all the possibilities when any two villages
number of seats = 0. receive equal number of toilets.
Therefore the required number of ways
Village 1 Village 2 Village 3
= 4851 − 147 = 4704.
86 If a > b > c, the number of solutions of a + b + c = 49, is 0 0 49
 n + 6 / 12 .
2
1 1 47
Therefore, the required number of solutions 2 2 45
 492 + 6   2407  ... ... ...
= =  = 200. 5833 = 200
 12   12  ... ... ...
22 22 5
Alternatively Consider the following explanation.
23 23 3
Number of toilets
Total Case 24 24 1
Village C Village B Village A
There are total 25 possibilities in which village 1 and
0 1 48 village 2 will have equal number of toilets. Similarly there
... ... ... 24 are 25 possibilities in which village 2 and village 3 will have
... ... ... equal number of toilets. And so there will be 25 possibilities
0 24 25 when village 1 and village 3 will have equal number of toilets.
1 2 46 Thus, it shows that there are total 75 possibilities in which
any two villages will have equal number of toilets.
... ... ... 22
Therefore the net possibilities in which all the three villages
... ... ...
will have distinct number of toilets = 1275 − 75 = 1200
1 23 25
But there is only 1 case out of 3! cases in which village A
2 3 44 will have more toilets than that of village B and village B
... ... ... 21 will have more toilets than that of village C
... ... ... 1200
Therefore the required number of ways = = 200
2 23 24 3!
Permutations & Combinations 1139

Hint Please note that there is no any possibility in which all the The number of ways of giving the appointments at these
three villages will have equal number of toilets, since 49 is not 2 salons = 2! × 15 × 3 × 4 = 360
divisible by 3.
Case IV When 3 persons are given the appointment at one
87 Let’s say the numbers of persons of this family seeking the salon and the 1 person takes the appointment at the other
appointments at the 6 salons are a, b, c, d, e and f . Then we salon.
have a + b + c + d + e + f = 4. The number of ways of selection of 2 distinct salons
The number of non-negative integral solutions of the above = 6C 2 = 15
equation is 4 + 6 − 1C 6 − 1 = 126 The number of ways of arranging these people between
2 salons = 2!
Since each person of the family is distinct (or
distinguishable), so each of them can be mutually arranged The number of ways of selecting 3 persons for the one salon
in 4! ways. = 4C 3 = 4
Therefore, the required number of ways in which they can The number of ways of arranging 3 persons at one salon
be given the appointments at these salons = 3! = 6
= 4 ! × 126 = 3024. The number of ways of giving the appointments at these
2 salons = 2! × 15 × 4 × 6 = 720
Alternatively
Case V When all the 4 persons are given the appointment
Case I When each person is given the appointment at
at the same salon.
distinct salons.
The number of ways of selection of 1 salon = 6C1 = 6
The number of ways of selection of 4 distinct salons
6
C 4 = 15 The number of ways arranging the 4 persons at one salon
= 4!
The number of ways of arranging these people among
The number of ways of giving the appointments at a salon
4 salons = 4 !
= 4 ! × 6 = 144
The number of ways of getting the appointments at these Therefore the required number of ways
4 salons = 4 ! × 15 = 360 = 360 + 1440 + 360 + 720 + 144 = 3024
Case II When 2 persons are given the appointment at one 88 There are two distinct cases:
salon and the other 2 persons are given the appointment at
2 distinct salons. Case I When there are 11 digits –
The number of ways of selection of 3 distinct salons x1 + x 2 + . . . . . + x11 = 3; for x1 ≥ 1
= 6C 3 = 20 and x 2, x 3, . . . . . . , x11 ≥ 0
The number of ways of arranging these people among ⇒ x1 + x 2 + . . . + x11 = 2 ; for x1, x 2, x 3, . . . . x11 ≥ 0
2 + 11 − 1
3 salons = 3! Number of solutions = C11 − 1 = 12C10 = 66
The number of ways of selection of 2 persons who are given Case II When there are 12 digits –
the appointment at the same salon = 4 C 2 = 6
x1 + x 2 + . . . + x12 = 3; for x1 ≥ 1 and x 2, x 3, . . . . x12 ≥ 0
The number of ways of arranging the 2 persons who are ⇒ x1 + x 2 + . . . + x12 = 2; for x1, x 2, x 3, K x12 ≥ 0
given the appointment at the same salon = 2! 2 + 12 − 1
Number of solutions = C12 − 1 = 13C11 = 78
The number of ways of giving the appointments at the
3 salons = 3! × 20 × 6 × 2 = 1440 Therefore, the total required number of values of
N = 66 + 78 = 144
Case III When 2 persons are given the appointment at one
Alternatively As per the given information, N can be an
salon and the other 2 persons are given the appointment at
11 digit or a 12-digit number.
the other salon.
The number of ways of selection of 2 distinct salons (A) When N is an 11 digit number
= 6C 2 = 15 Case I Out of eleven digits three digits are 1 and
The number of ways of arranging these people between eight digits are 0.
2 salons = 2! In this case a number will start with only one number that
The number of ways of selection of 2 people for each salon is 1. Then you have to arrange only 10 digits of which
1 2 digits will be same (i.e.) and other 8 digits will be same
= 4C 2 × 2C 2 × =3
2! (i.e. 0).
10 !
The number of ways of arranging the 2 persons in each Therefore the total required numbers = 1 × = 45
salon = 2! × 2! = 4 8 ! 2!
1140 QUANTUM CAT
Case II Out of eleven digits one digit is 1, another digit is 2 friends = 496 − m × f
and remaining nine digits are 0. In this case a number will Now we know that one-fifth of the girls deny becoming
start with two digits that is either 1 or 2. Then you have to Facebook friend with one-third of the boys, so we must
arrange only 10 digits of which one digits is either 1 or 2,
have 3m + 5 f = 32
while rest of nine digits will be same. (i.e., 0).
10 ! Therefore the possible values of m and f are (4, 4) and
Therefore the total required numbers = 2x = 20
9! (9, 1).
Case III Out of the eleven digits one digit is 3 and the Now the possible number of pairs who are the Facebook
remaining ten digits are 0. friends = 496 − 4 × 4 = 480 and 496 − 9 × 1 = 487
In this case a number will start with the digit 3 and the Since 487 is given in the choices, so it could be the required
other 10 digits will be same (i.e., 0). possible value.
10 !
Therefore the total required numbers = 1 × =1 Thus the number of regions that these volunteers could
10 ! have visited = 487.
Thus the total 11-digit numbers = 45 + 20 + 1 = 66. 90 Any number between 1 and 1000 must be less than 4 digits.
(B) When N is a 12 digit number Therefore it must be of the form x1 + x 2 + x 3 = 10, where
Case I Out of twelve digits three digits are 1 and nine 0 ≤ x i ≤ 9 for every i = 1, 2, 3.
Therefore, the required number
digits are 0.
In this case a number will start with only one number that = Coefficient of x10 in (1 + x + x 2 + K + x 9 )3
3
is 1. Then you have to arrange only 11 digits of which  1 − x10 
2 digits will be same (i.e., 1) and other 9 digits will be same = Coefficient of x10 in  
 1− x 
(i.e., 0).
11 ! = Coefficient of x10 in [(1 − x10 )3 (1 − x )−3]
Therefore the total required numbers = 1 × = 55
9 ! × 2! = Coefficient of x10 in [(1 − 3C1 x10 . . . )(1 − x )−3]
Case II Out of the twelve digits one digit is 1, another digit = Coefficient of x10 in (1 − x )−3 − 3C1.
is 2 and remaining ten digits are 0.
[Coefficient of x(10 − 10) in (1 − x )−3]
In this case a number will start with two digits that is either
1 or 2. Then you have to arrange only 11 digits of which = 3 + 10 − 1C 3 − 1 − 3. (3 + 0 − 1C 3 − 1 )
one digit is either 1 or 2, while rest of the 10 digits will be = 12C 2 − 3. (2C 2 ) = 66 − 3 = 63
same (i.e., 0).
11 ! Alternatively The number of non-negative integral
Therefore the total required numbers = 2 × = 22 solutions of x1 + x 2 + x 3 = 10 will be 10 + 3 − 1C 3 − 1 = 66
10 !
Case III Out of the twelve digits one digit is 3 and the But out of these 66 solutions there are the following
remaining eleven digits are 0. 3 solutions which need to be excluded.
In this case a number will start with the digit 3 and the 10 + 0 + 0 = 10, 0 + 10 + 0 = 10,
other 11 digits will be same (i.e., 0). 0 + 0 + 10 = 10
11 ! Therefore the required number of solutions = 66 − 3 = 63.
Therefore the total required numbers = 1 × =1
11 ! Alternatively Number of 1 digit number is zero.
Thus the total 11-digit numbers = 55 + 22 + 1 = 78 Number of 2-digit numbers
Therefore, the total required number of values of = Number of solutions of a + b = 10; (1 ≤ a ≤ 9, 0 ≤ b ≤ 9)
N = 66 + 78 = 144 = Number of solutions of a + b = 9; (0 ≤ a ≤ 9, 0 ≤ b ≤ 9)
89 If out of 32 volunteers everybody befriended on Facebook, = 10C1 = 10
the maximum number of friendships = 32
C 2 = 496 But, this involves a solution in which a = 10. Which is
It means at most 496 pairs can be formed, which, in turn, unacceptable, so there are only 9 numbers of 3 digit
numbers.
implies that at most 496 regions can be visited.
= Number of solutions of a + b + c = 10.
However, since certain number of guys and girls are not the (1 ≤ a ≤ 9, 0 ≤ b ≤ 9, 0 ≤ c ≤ 9 )
Facebook friends, so they are not allowed to visit the
regions together. It implies that the required answer would = Number of solutions of a + b + c = 9;
be less than 496. (0 ≤ a ≤ 9, 0 ≤ b ≤ 9, 0 ≤ c ≤ 9), 11
C 2 = 55
Let us consider that m guys and f girls don’t befriend each But, this involves a solution in which a = 10, which is
other on the Facebook. unacceptable, so there are only 54 numbers.
Therefore the total number of pairs who are Facebook Thus, we have total 63 numbers (= 0 + 9 + 54).
Permutations & Combinations 1141

91 Primarily, there are six different cases in which 15 different 5!


Case V Number of ways of distributing 15 rings =
rings can be divided such that there are twins in the family 2! × 2!
and none of the daughters gets less than one and more than 5!
Case VI Number of ways of distributing 15 rings =
five rings. 2! × 2!
Case I: 1, 1, 3, 5, 5; Case II: 1, 1, 4, 4, 5;
Therefore total number of ways of distributing 15 different
Case III: 1, 2, 2, 5, 5 Case IV: 1, 3, 3, 4, 4;
 5!  6!
Case V: 2, 2, 3, 3, 5; Case VI: 2, 2, 3, 4, 4 rings = 6 ×   = = 180
 (2!)2  (2!)2
Case I Number of ways of distributing 15 rings
Hence choice (a) is the answer.
15! 1
= × × 2! × 2! 93 462 = 2 × 3 × 7 × 11
1 ! 1 ! 3! 5! 5! 2! × 2!
Case II Number of ways of distributing 15 rings Since, there are exactly 4 prime factors, so each of the
prime number can be assigned to each x i in 4 ways.
15! 1
= × × 2! × 2! Therefore the total number of ways of assigning the prime
1 ! 1 ! 4 ! 4 ! 5! 2! × 2! factors to all the x i is 44 = 256
Case III Number of ways of distributing 15 rings
94 462 = 2 × 3 × 7 × 11
15! 1
= × × 2! × 2! Since, there are exactly 4 prime factors, so each of the four
1 ! 2! 2! 5! 5! 2! × 2! prime numbers can be assigned to x i in 4 ways. Therefore the
Case IV Number of ways of distributing 15 rings total number of assigning the prime factor to all the x is is 44.
15! 1
= × × 2! × 2! The required solution will be positive only when either 0 or
1 ! 3! 3! 4 ! 4 ! 2! × 2!
2 or 4 prime factors are negative. This can be achieved in
Case V Number of ways of distributing 15 rings 4
C 0 + 4C 2 + 4C 4 = 1 + 6 + 1 = 8 ways.
15! 1 Thus the required number of solutions = 8 × 44 = 2048.
= × × 2! × 2!
2! 2! 3! 3! 5! 2! × 2!
95 Let L( A, C )denotes the total number shortest paths from A
Case VI Number of ways of distributing 15 rings
to C, without any restriction; L ( A, P , C ) denotes the total
15! 1
= × × 2! × 2! number of shortest paths from A to C, via P ; L( A, Q , C )
2! 2! 3! 4 ! 4 ! 2! × 2! denotes the total number of shortest paths from A to C, via
Therefore total number of ways of distributing 15 different Q ; L( A, P , Q , C ) denotes the total number of shortest paths
rings from A to C, via P and Q Now,
 1 1 1 L( A, C ) = 16C 6 × 6C 6 = 8008; L( A, P , C ) = L( AP ) × L(PC )
= 15! ×  + +
 3 ! × ( 5 !)2
(4 !)2
× 5 ! (2 !)2
× (5!)2 = (7 C 4 × 3C 3 ) × (9C 6 × 3C 3 ) = 35 × 84 = 2940
1 1 1  L( A, Q , C ) = L( AQ ) × L(QC ) = (12C 8 × 4C 4 ) × (4C 2 × 2C 2 )
+ + + 2
(3!) × (4 !)
2 2
(2!) × (3!) × 5! (2!) × 3! × (4 !) 
2 2 2
= 495 × 6 = 2970
Hence choice (d) is the answer. L( A, P , Q , C ) = L( AP ) × L(PQ ) × L(QC )
92 Primarily, there are six different cases in which 15 different = (7 C 4 × 3C 3 ) × (5C 4 × 1C1 ) × (4C 2 × 2C 2 ) = 1050
rings can be divided such that there are twins in the family
By the inclusion-exclusion principle, the total number of
and none of the daughters gets less than one and more than
the shortest paths from A to C, such that Runnbeer doesn’t
five rings.
go through any of the points P or Q
Case I : 1, 1, 3, 5, 5; Case II : 1, 1, 4, 4, 5;
Case III : 1, 2, 2, 5, 5 Case IV : 1, 3, 3, 4, 4; = L( A, C ) − L( A, P , C ) − L( A, Q , C ) + L( A, P , Q , C )
Case V : 2, 2, 3, 3, 5; Case VI : 2, 2, 3, 4, 4 = 8008 − 2940 − 2970 + 1050 = 3148
5!
Case I Number of ways of distributing 15 rings = 96 Total number of delegates in all 3 countries is 60, so the
2! × 2! number of handshakes among the delegates of any two
5!
Case II Number of ways of distributing 15 rings = countries (only once) is 60C 2.
2! × 2!
Total number of delegates in all 4 states of each country is
5!
Case III Number of ways of distributing 15 rings = 20, so the number of total number of handshakes among
2! × 2! the delegates of any two states of each country (only once)
5! is 20C 2.
Case IV Number of ways of distributing 15 rings =
2! × 2!
1142 QUANTUM CAT
Total number of delegates in all 5 cities of each state is 5, so Case Polygon Value
the number of total number of handshakes among the
delegates of any two cities of each sate (only once) is 5C 2. V Heptagon 10
C7 = 120
Out of 60 delegates any two delegates shake hands 2 times. VI Octagon 10
C 8 = 45
Out of 20 delegates (in each country) any two delegates
shake hands 5 times, but since some of their handshakes VII Nonagon 10
C 9 = 10
have been already counted, so we have to consider only
VIII Decagon 10
C10 = 1
3 times, instead of 5 times.
Out of 5 delegates (in each state) any two delegates shake
Therefore the total required number of polygons such that
hands 9 times, but since some of their handshakes have
been already counted, so we have to consider only 4 times, at least one of the vertices is green and rest of the vertices are
instead of 9 times. red = 100 + 195 + 246 + 209 + 120 + 45 + 10 + 1 = 926
Thus the total number of handshakes 99 10 ! = 2a 3b5c7 d
= [ 2 (60C 2 )] + 3[ 3(20C 2 )] + 12[ 4 (5C 2 )] Since, there are four distinct prime factors so there will be
= 3540 + 1710 + 480 24 = 16 subsets including the empty one, as in
= 5730 {}, {2a }, {3b}, {5c }, {7 d }, {2a 3b}, {2a 5c }, {2a7 d },
97 Number of zones = 4 {3b5c }, {3b7 d }, {5c7 d }, {2a 3b5c }, {2a 3b7 d }, {2a 5c7 d }
Number of blocks in each zone = 5
{3b 5c 7 d }, {2a 3b5c7 d }.
Total number of blocks = 4 × 5 = 20
Total number of connections between any two blocks In this kind of problems empty set should be replaced with
across all the zones = 20C 2 = 190 a set containing the element 1.
Now we have the following combination for m × n, as
Number of connections between any two blocks in a zone
shown in the below.
= 5C 2 = 10
m × n
Total number of trucks carrying goods between any two
blocks across all the zones = 3 1 × a b c d
2 3 57
Number of trucks carrying goods between any two blocks 2a × 3b5c7 d
in each zone = 2 3b × 2a 5c7 d
5c × 2a 3b7 d
Since the number of dedicated trucks between two cities of
7d × 2a 3b5c
each block is less than the number of dedicated trucks
between two cities across all the blocks of four zones, so we 2a 3b × 5c 7 d
have to subtract the excess number of trucks. 2a 5c × 3b7 d
Total number of trucks between any two blocks across all 2a7 d × 3b5c
the zones = 3 × 190 = 570 3b5c × 2a7 d
Extra number of trucks in each zone = 1×10 = 10 3b7 d × 2a 5c
Extra number of trucks across all the 4 zones = 4 × 10 = 40 5c 7 d × 2a 3b
Therefore the actual number of trucks engaged in the 2a 3b5c × 7d
market 2a 3b7 d × 5c
= 3(20C 2 ) − 4 [1 × (5C 2 )] 2a 5c7 d × 3b
= 570 − 40 3b5c7 d × 2a
= 530 2a 3b5c7 d × 1
98 Consider the following table. In all such 16 cases there are half of the cases in which
Case Polygon Value
m < n.
m
Therefore, for 0 < < 1, the total number of required
I Triangle 10
C 3 − 6C 3 = 100 n
rational numbers
II Quadrilateral 10
C 4 − 6C 4 = 195 1
= × 24 = 8
2
III Pentagon 10
C 5 − 6C 6 = 246

IV Hexagon 10
C 6 − 6C 6 = 209
Permutations & Combinations 1143

100 Look at the following diagram and figure out all the unique ⇒ N( l > b) + N( l = b) + N( l < b) = 1225
triangles. Now, we know that when l = b, we have
{l + b} = {1 + 1}, {2 + 2} . . . , {25 + 25}
7
∴ N( l = b) = 25
6 6 But, numerically, N( l > b) = N( l < b)
1225 − 25
5 ∴ N( l > b) = = 600
5 5 2
4 4
4 3 3 Total number of rectangles = N( l > b) + N( l = b)
3 4
2
1
2 = 600 + 25 = 625
Alternatively 2(l + b) = 100 ⇒ l + b = 50. We will
consider only combinations in which l ≥ b, but we cannot
consider l < b, as practically it violates the basic definition
Since there are four slant lines emerging from each vertex
of rectangle.
at the base of the triangle.
Therefore the total number of triangles can be calculated as Number of
l+ b Solutions
shown below. solutions
1 × 1 + 2 × 2 + 3 × 3 + 4 × 4 + 3 × 5 + 2 × 6 + 1 × 7 = 64 2 1 {1, 1}
Alternatively Total number of triangles = the number 3 1 {2, 1}
of triangles with left bottom point as a vertex + the number 4 2 {2, 2} {3, 1}
of triangles with right bottom point as a vertex − the
5 2 {3, 2}, {4, 1}
number of triangles with both bottom points as the vertices
n × n × (n + 1) n × n × (n + 1) 6 3 {3, 3}, {4, 2}, {5, 1}
= + − (n × n) = n × n × n
2 2 7 3 {4, 3}, {5, 2}, {6, 1}
Therefore the required number of triangles ... ... ...
= 4 × 4 × 4 = 64 50 25 {25, 25}, {26, 24}, (27, 23), . . . , {49, 1}
101 In the 1st transaction, PayTM or PayU can receive money
Therefore total number of rectangles
from PayPal in 1 way.
In the 2nd transaction, PayPal can receive money either = 2(1 + 2 + 3 + . . . + 24) + 25 = 625.
from PayTM or from PayU in 2 ways. But PayTM and PayU 103 a + b + c = 12; {0 ≤ a, b, c ≤ 6} − {a, b, c = 5}
can receive money in 1 way from each other, as PayPal had
Number of solutions of a + b + c = 12 ; {0 ≤ a, b, c} is
no money after 1st transaction.
C 2 = 91
14
In the 3rd transaction, PayPal can receive money either
If we allocate 7 to one of a, b, c we can make sure that the
from PayTM or PayU in 2 ways. But, each of PayTM and
solution has a, b, c > 6.
PayU can receive money in 3 ways.
Therefore, number of solutions of a + b + c = 12
Remember that we have to count the number of ways
cumulatively. Now, follow the same principle and continue {0 ≤ a, b, c ≤ 6} is equivalent to the number of solutions of
counting until the 7th transaction. a + b + c = 5; {a, b, c ≥ 0} is 7 C 2 = 21.

Transaction 1st 2nd 3rd 4th 5th 6th 7th


Since any of a, b, c can take value greater than 6, so it can be
chosen in 3C1 = 3 ways.
PayPal − 2 2 6 10 22 42
Now, we have following 3 sets in which one or more teams
PayTM 1 1 3 5 11 21 43
score 5 runs in a ball –
PayU 1 1 3 5 11 21 43
{5, 6, 1} → 6
Thus we see that there are total 42 different ways in which {5, 5, 2} → 3
PayPal wallet can have same amount after the 7th {5, 4, 3} → 6
transaction that it had before the 1st transaction.
So, the number of solutions involving 5 is 15.
102 2(l + b) ≤ 100 (l, b ≥ 0) Therefore, required number of solutions
⇒ l + b ≤ 50 (l, b ≥ 0) = 91 − 3(21) − 15 = 13
⇒ l + b ≤ 48 (l, b ≥ 1) Alternatively There are certain triplets that can be
⇒ l + b + c = 48 (l, b ≥ 1) arranged to find the valid possibilities.
48 + 3 − 1
Number of solutions (N ) = C3 −1 = 50
C 2 = 1225 {6, 6, 0} → 3
1144 QUANTUM CAT
{6, 4, 2} → 6 Case II When exactly two pairs exist, the number of
{6, 3, 3} → 3 arrangements = 6 !
{4, 4, 4} → 1 Since we can select any 2 pairs out of 4 pairs in 6 ways, so
the total number of arrangements = 6 × (6 !)
Therefore, the possible number of ways
Case III When exactly three pairs exist, the number of
3 + 6 + 3 + 1 = 13 arrangements = 5!
104 105 = N(Shirts) × N (Ties) × N (Pants) Since we can select any 3 pairs out of 4 pairs in 4 ways, so
⇒ 105 = 3 × 5 × 7 the total number of arrangements = 3 × (5!)
Since there are exactly three prime factors in 105, each of Case IV When exactly two pairs exist, the number of
these 3 prime numbers can be allocated to three articles arrangements = 4 !
(shirts, ties and pants) in 3 ways. Therefore the required Since we can select any 4 pairs out of 4 pairs in 1 way only,
number of arrangements = 33 = 27. so the total number of arrangements = 1 × (4 !)
Alternatively There are certain triplets that can be By the Inclusion-Exclusion Principle, we have
arranged to find the valid possibilities. | A12 ∪ A 34 ∪ A 56 ∪ A78| = | A 12| + | A 34| + | A 56| + | A78|
{1 × 1 × 105} → 3 − | A12 ∩ A 34| − | A12 ∩ A 56| − | A12 ∩ A78| − | A 34 ∩ A 56|
{1 × 3 × 35} → 6 − | A 34 ∩ A78| − | A 56 ∩ A78| + | A12 ∩ A 34 ∩ A 56|
{1 × 5 × 25} → 6 + | A12 ∩ A 34 ∩ A78| + | A12 ∩ A 56 ∩ A78|
{1 × 7 × 15} → 6 + | A 34 ∩ A 56 ∩ A78| − | A12 ∩ A 34 ∩ A 56 ∩ A78|
{3 × 5 × 7 } → 6 Therefore,| A12 ∪ A 34 ∪ A 56 ∪ A78| = 4(7 !) + 4(5!) − 4 !
Therefore, the possible number of arrangements The total number of arrangements of numbers 1, 2,
= 3 + 6 + 6 + 6 + 6 = 27. ............. 8 is 8!.
But, we need to find out the value of 8!
105 There are four pairs (12), (34), (56) and (78).
− | A12 ∪ A34 ∪ A56 ∪ A78 |.
Case I When only one pair exists, the number of
Thus the required number of arrangements
arrangements = 7 ! .
= 8 ! − [ 4(7 !) − 6(6 !) + 4(5!) − 4 !] = 24024
Since we can select any 1 pair out of 4 pairs in 4 ways, so
the total number of arrangements = 4 × (7 !)

Level 03 Final Round


1 Number of intersections = 2 × (6C 2 ) = 6P2 = 30. 8 Let n = 2m + 1, for the three numbers are in AP we have the
2 6
C 2 = 15. following patterns :

3 3 lines intersect each other in 3C 2 = 3 points. Common


Numbers
Number of
differences ways
3 circles intersect each other in P2 = 6 points.
3

Every line cuts 3 circles into 6 points. Therefore 3 lines cuts 1 (1, 2, 3)(2, 3, 4) … (n − 2, n − 1, n) (n − 2)
3 circles into 18 points. Therefore, the maximum number 2 (1, 3, 5), (2, 4, 6)… (n − 4, n − 2, n) (n − 4)
of points = 3 + 6 + 18 = 27 3 (1, 4, 7 ), (2, 5, 8)… (n − 6, n − 3, n) (n − 6)
8!
4 = 70 — …… …… —
4 !⋅ 4 ! — …… …… —
5 Required number of triangles = m + n C 3 − mC 3 − nC 3.
m (1, m + 1, 2m + 1) 1
6 Number of even places = 4
Number of even digits = 5 (2, 2, 8, 8, 8) ∴ Favourable number of ways
Number of odd places = 5 = (n − 2) + (n − 4) + (n − 6) + … + 3 + 1 (total m terms)
Number of odd digits = 4 (3, 3, 5, 5) m (n − 1) . (n − 1) (n − 1)2
4! = (n − 2 + 1) = =
Odd digits can be arranged in ways = 6 ways. 2 2 2 4
2! × 2!
Alternatively Consider some proper value of n and
5!
Even digits can be arranged in = 10 ways. verify the result.
2! × 3! 8 8×9
Hence the required number of ways = 6 × 10 = 60 ways. 9 Required number of parts = 1 + Σ r = 1 + = 37
r =1 2
7 Required number of triangles Alternatively We can use the following formula too.
m + n+ k
= C 3 − mC 3 − nC 3 − kC 3
Permutations & Combinations 1145

n × (n + 1) n2 + n + 2 17 The possible ways are given below.


n
C 0 + nC1 + nC 2 = +1=
2 2
I II III IV V VI VII VIII IX
64 + 8 + 2
Therefore the required number of parts = = 37
2 Number of fours 25 22 19 16 13 10 7 4 1
240
10 = k ∈I Number of sixes 0 2 4 6 8 10 12 14 16
4n + 2

120 23 × 3 × 5 Therefore the required number of ways = 9.


∴ k= =
2n + 1 2n + 1 18 Total number of permutations = 8 ! = 40320
Since probable divisors are 1, 3, 5, 7, 9, 11, 13, …, (2n + 1) Number of permutations when word LURY occurs
but we have only 4 possible divisors 1, 3, 5, 15. = (8 − 4 + 1)! = 5! = 120
11 Total number of rectangles Number of permutations when word MINA occurs = 5!
= (1 + 2 + 3 + … + 12) × (1 + 2 + … + 8) = 120

12 × 13 8 × 9 Number of permutation when words LURY and MINA both


= × = 2808 occurs = 3! = 6
2 2
Required number of permutations
Total number of squares
= 40320 − (120 + 120) + 6 = 40086
= (12 × 8) + (11 × 7 ) + (10 × 6) + … + (5 × 1) = 348
19 5 students can be selected out of 10 students in 10C 5 ways
∴ Required number of rectangles = 2808 − 348 = 2460
remaining 5 students can be selected in 5C 5 ways. These
8
Hint Number of squares = Σ (13 − r) ( 9 − r) students (in each row) can be arranged mutually in
r =1
5! × 5! ways.
12 Required number of triangles = (2 n C3 − nC3 − nC3 ) + (n × n ) Besides it the two set of papers can be arranged in 2! ways
= n2 (n − 1) + n2 = n3 between two rows.
Hence, the required number of arrangements
Alternatively Required number of triangles = 10C 5 × (5!)2 × 2 = 7257600
= ( C 2 × C1 ) + ( C1 × C 2 ) + (n × n) = n
n n n n 3
20 Number of handshakes among guests = nC 2
13 There are 12 ways as follows : Number of handshakes between the hosts and guests = 2n
(9, 0, 0), (8, 1, 0), (7, 2, 0), (6, 3, 0), (5, 4, 0), (7, 1, 1), Therefore total number of handshakes = nC 2 + 2n = 65
(6, 2, 1), (5, 3, 1), (5, 2, 2), (4, 4, 1) (4, 3, 2), (3, 3, 3).
Now for your convenience go through the options and
14 First of all we give 4 coins to each of the six persons so that verify the correct one.
each one must have at least 4 coins.
21 First of all deduce 3 × 10 = 30 marks to assign atleast
Now we are left with = 30 − (6 × 4) = 6 coins.
3 marks to each of the 10 students.
These remaining 6 coins can be distributed among
Now remaining 20 marks can be assigned to 10 students in
6 persons in such a way that any one can receive any 20 + 10 − 1
C10 − 1 ways = 29C 9 ways
number of coins which can be done in
6 + 6 −1
C 6 − 1 = 11C 5 ways 1  1 1  1 2 
22 = + = +
 3  3 100   3 100 
= 462 ways
1 65  1 66 
15 Required number of circles = 10C 3 − 7C 3 = 85. =… + = + =0
 3 100   3 100 
16 0 cannot be placed as the left most digit so we have only 1 67  1 68  1 69 
and + = + = +
 3 100   3 100   3 100 
9 digits to be placed.
Also we will not consider 1 digit numbers.
1 98  1 99 
Now we can form a two digit number in 9C 2 ways. =… + = + =1
 3 100   3 100 
Now we can form a three digit number in 9C 3 ways.
1  1 1  1 99 
Now we can form a four digit number in 9C 4 ways. Hence, E= + + +…+ +
 3  3 100   3 100 
……………………
= 33 × 1 = 33
……………………
We can form 9 digit number in 9C 9 way. 23 Total numbers = 106 (1, 2, 3, 4, … , 106 )
1, 4, 9, 16, 25, … 106 
Hence the required numbers = 9C 2 + 9C 3 + 9C 4 + … + 9C 9 n2 ⇒   → 10 = 1000
3

1, 2, 3, 4, 5, … (10 ) 
3
= 502
1146 QUANTUM CAT
1, 8, 27, 64, 125, … , 106  26 The total number of balls in the box = 2 + 3 + 4 = 9.
n3 ⇒   → 10 = 100
2

1, 2 , 3 , 4 , 5 , … , (10 2
)  Total number of selections of 3 balls out of 9 balls = 9C 3
1 , 16 , 81 , 256 , … , 923521  Number of selections in which no any green ball is selected
n4 ⇒   → 31 = 6C 3
1 , 2 , 3 , 4 , … , 31 
Hence the required number of selections = 9C 3 − 6C 3 = 64.
Hint 10 = 10
4 6 6/ 4
= 10
3 /2
= 1000 ⇒ 31
27 There are four possible cases.
1, 64, 729, 4096, … , 106 
n ∩n
2 3
⇒ n  6
 → 10
1, 2, 3, 4, … , 10  H W H W
1 , 2 , 3 
12 12 12 M 0 3 M 2 1
n3 ∩ n4 ⇒ n12  →3
1, 2, 3  F 3 0
F 1 2
(i)
Hint 10 = 10
12 6 6 / 12
= 10 1/ 2
= 10 ⇒ 3 (iii)
1, 16, 81, 256, … , (31) 4  H W
n2 ∩ n4 ⇒ n4   → 31
H W
1, 2, 3, 4, … , 31  M 1 2
M 3 0
112 , 212 , 312  F 2 1
n ∩n ∩n
2 3 4
⇒ n  12
→3 F 0 3
1, 2, 3  (ii)
(iv)
Hence the number of numbers which are either perfect
square or perfect cube or perfect fourth powers or all of these H → husband’s relatives W → wife’s relatives
= n2 + n3 + n4 − (n2 ∩ n3 + n3 ∩ n4 + n2 ∩ n4 ) M → male F → female
+ n2 ∩ n3 ∩ n4 Hence the required number of ways
= 1131 − 44 + 3 = 1090 = (4C 3 × 4C 3 ) + (3C1 × 4C 2 )(4C 2 × 3C1 )
Hence, the required number of ways + (3C 2 × 4C1 )(4C1 × 3C 2 ) + (3C 3 × 3C 3 )
= Total numbers – Numbers which are perfect squares
= (4 × 4) + (3 × 6 × 6 × 3) + (3 × 4 × 4 × 3) + (1 × 1)
or perfect cubes or perfect fourth powers
= 485
= 106 − 1090 = 998910
28 Let the number of men participating in the tournament be
24 Total number of required seats = 1 + m + 2n n. Since every participant played two games with every
The Grandchildren can occupy the n seats on either side of other participant.
the table in (2nP2n) ways. Remaining seats are (1 + m). Therefore the total number of games played among men is
2 × nC 2 = n (n − 1).
Since grandfather can not occupy adjacent seats of the
grandchildren hence the grandfather can access only And the number of games played with each woman = 2n.
m + 1 − 2 = m − 1 seats. Hence he can occupy a seat in but since there are two women, hence the total number of
( m − 1)
P1ways. Now the remaining seats can be occupied in games men played with 2 women = 2 × 2n = 4n
m
Pm ways by the ‘m’ sons and daughters. ∴ {n (n − 1)} − 4n = 66
Hence the required number of ways ⇒ n2 − 5n − 66 = 0
= 2nP2n × mPm × m − 1P1 = (2n !)(m !)(m − 1)
⇒ n = 11 (Q n < 0, is not possible)
25 The 4 possible cases are as follows : ∴ Number of participants = 11 men + 2 women = 13.
C1 C2 C3 29 Number of games played by them is 2 (13C 2 ) = 156.
C1 → First column 2 3 1 30 There are two possible cases in which 12 sweets can be
distributed among 10 girls.
C 2 → Second column 2 2 2
(i) Any 9 girls get one sweets each and the remaining 1 girl
C 3 → Third column 1 4 1 gets 3 sweets.
1 3 2 (ii) Any 8 girls get one sweets each and the remaining
2 girls get 2 pieces of sweets each.
Hence, the required number of ways
Case I. 3 pieces of sweets can be given to a girl in the
= 2C 2 × 4C 3 × 2C1 + 2C 2 × 4C 2 × 2C 2 following four way :
+ 2C1 × 4C 4 × 2C1 + 2C1 + 2C1 × 4C 3 × 2C 2
Burfi 3 2 1 0
=1 × 4 × 2+ 1 × 6 ×1 + 2×1 × 2+ 2× 4 ×1
Rasgulla 0 1 2 3
= 8 + 6 + 4 + 8 = 26
Permutations & Combinations 1147

After giving 3 pieces of sweets to a single girl. We can 32 Let the form of the required numbers be a1, a2 … a9, where
distribute the remaining 9 sweets to the 9 girls in the 0 ≤ a1 ≤ 1 and 0 ≤ ai ≤ 2 for i = 2, 3, … , 9 and where all
following ways : a1, a2, … , a9 cannot be equal to zero.
9
C 3 × 6C 6 + 9C 4 × 5C 5 + 9C 5 + 4C 4 + 9C 6 × 3C 3 Now, we can choose a1 in two ways (0 or 1) and ai for
= 2 (9C 3 + 9C 4 ) i = 2, 3, … 8 in 3 ways (0, 1, 2).

One particular girl can be chosen in 10C1 ways. After selecting a1, a2, a3 … , a8 we find the sum
s = a1 + a2 + K a8 which is of the form 3m − 2, 3m − 1 or
Therefore 3 sweets can be given to a single girl in 3m. Now we can select a9 in just one way.
10
C1 × 2 × (9C 3 + 9C 4 ) = 4200 ways.
Actually a9 can be selected out of 2, 1 or 0 depending on
Case II. We can give two sweets to two girls (say A and B) whether s = 3m − 2, 3m − 1 or 3m. Therefore, we can
in the following ways : choose the numbers in 2 × 37 × 1 = 4374 ways.
A Burfi 2 1 0 2 1 0 2 1 0 But this includes the case in which each of ai = 0. Thus, the
Rasgulla 0 1 2 0 1 2 0 1 2 required number of numbers = 4374 − 1 = 4373
33 The digits which can be recognised as digits on the screen
B Burfi 2 2 2 1 1 1 0 0 0
of a calculator when they are read inverted i.e., upside
Rasgulla 0 0 0 1 1 1 2 2 2 down are 0, 1, 2, 5, 6, 8 and 9.
Now, remaining 8 pieces of sweets can be distributed Since a number cannot begin with zero hence left most
among eight girls in the following ways digit and right most digit can never be 0 as when an ‘n’ digit
number read upside down it will become a number of less
(8C 2 × 6C 6 ) + (8C 3 × 5C 5 ) + (8C 4 × 4C 4 ) + (8C 3 × 5C 5 )
than n digit. Hence,
+ (8C 4 × 4C 4 ) + (8C 5 × 3C 3 )
Number of digits Total number of numbers
+ (8C 4 × 4C 4 ) + (8C 5 × 5C 5 ) + (8C 6 × 2C 2 )
1 7
= 2 (8C 2 ) + 4 (8C 3 ) + 3 (8C 4 )
2 6 × 6 = 62
Further, two girls can be selected in 10C 2 ways.
3 6 × 7 × 6 = 62 × 7
Therefore two girls can get two sweets each in
(10C 2 )[ 2 (8C 2 ) + 4 (8C 3 ) + 3 (8C 4 )] = 22050 ways 4 6 × 7 × 7 × 6 = 62 × 7 2
Hence, the required number of ways = 4200 + 22050 5 6 × 7 × 7 × 7 × 6 = 62 × 7 3
= 26250 6 6 × 7 × 7 × 7 × 7 × 6 = 62 × 7 4
31 Number of common children of Mr. John and Ms. Bashu
= 10 − ( x + x + 1) = 9 − 2x Thus, the number of required numbers
Let N = The number of fights between children = 7 + 62 + 62 ⋅ 7 + … + 62 ⋅ 7 4
of different parents (7 5 − 1)
= 7 + 62 = 7 + 6 (7 5 − 1)
= (Total number of fights that can take place among (7 − 1)
all the children)
= 6 ⋅ 7 5 + 1 = 100843
– (The number of fights among the children
of the same parents) 34 Since rings are distinct, hence they can be named as
x+1 9 − 2x
= 10C 2 − [ x C 2 + C2 + C 2] …(i) R1, R 2, R 3, R 4 and R 5.
 x ( x − 1) ( x + 1)( x ) (9 − 2x )(8 − 2x ) The ring R1 can be placed on any of the four fingers in
= 45 − + +
 2 2 2  4 ways.
1 2 The ring R 2 can be placed on any of the four fingers in
= 45 − [ x − x + x 2 + x + 72 − 34 x + 4 x 2]
2 5 ways since the finger in which R1 is placed now has
397  17 
2 2 choices, one above the R1 and one below the ring R1.
= − 3x − 
12  6
17 Similarly R 3, R 4 and R 5 can be arrange in 6, 7 and 8 ways
For N to be maximum, x must be . As x cannot be in respectively.
6
 17  Hence, the required number of ways
fraction, we take x = 3  ≈ .
 6 = 4 × 5 × 6 × 7 × 8 = 6720
Thus, maximum value of N = 33which is attained at x = 3. 35 We can select first object out of n objects in nC1 ways.
Alternatively After making the equation (i) go through Now, number of ways of choosing two objects such that
options. they are always together (n − 4) ways.
1148 QUANTUM CAT
Since we assume two objects as a single object. Further we Hence, there are total 7 ways.
can select three objects viz., the one object which has been Now, go through options.
already selected and two objects of one either side of the
Let us consider option (b).
first object. Therefore the number of ways of choosing two
objects such that they are not together = ( n − 3)C 2 − (n − 4) Putting n = 1, we get
1
1 (1 + 1)(5 × 12 + 10 × 1 + 6) = 7
= (n − 4)(n − 5) 6
2
Hence the choice (b) is correct.
Since these two objects can be arranged in 2! ways, the
number of ways of choosing three objects (in order of the 38 Do this problem similarly as discussed in the previous
first, second and third) is problem.
1
n × (n − 4)(n − 5) × 2 = n (n − 4)(n − 5) NOTE The general solution of this type of problems
2 involves higher mathematics. But we have given in question
But, since the order in which the objects are taken is number 37 a very simple, lucid and a novel solution for the lay
immaterial, the number of ways of choosing the objects is students.
1
n (n − 4)(n − 5). 39 Let the number of male students is x, the number of female
6
students in that class will be 2x.
36 Number of Number of Number of The number of friendships among male students
similar letters different letters selections x( x − 1)
= xC 2 =
5 0
1
C1 = 1 2
The number of friendships among female students
4 1
4
C1 × 2C1 = 8
2x(2x − 1)
= 2 xC 2 =
3 2
3
C1 × 4C 2 = 18 2
3 of one type and The number of friendships between male and female
0
3
C1 × 3C1 = 9
2 of another type students = x × 2x = 2x 2
2 of one type and 4
C 2 × 3C1 = 18 Let’s assume that the number of friend requests sent by
1
2 of another type female students to male students is y. Therefore, the
2 3
4
C1 × 4C 3 = 16 number of friend requests sent by male students to female
students = 2x 2 − y.
0 5
5
C5 = 1
But you must know the following fact about intra group
Hence, the total number of selections friend requests as in among boys only or among girls only.
= 1 + 8 + 18 + 9 + 18 + 16 + 1 = 71 The number of friendships = number of friend requests sent
= number of friend requests accepted
37 Ist 2nd 3rd 4th
paper paper paper paper
So, the number of friend requests sent/accepted by male
x( x − 1)
students among themselves =
Max. marks n n n 2n 2
Similarly, the number of friend requests sent/accepted by
Let us consider n = 1, then a candidate required 3 marks 2x(2x − 1)
out of 5 marks, which can be done in the following ways : female students among themselves =
2
Ist 2nd 4th Further it is given that, The total number of friend requests
3rd paper Total
paper paper paper sent by male students = Total number of friend requests
sent by female students
0 0 1 2 3
x ( x − 1) 2x(2x − 1)
0 1 0 2 3 ∴ + (2x 2 − y ) = + y
1 0 0 2 3 2 2
1 1 1 0 3 ⇒ 4 y = x( x + 1)
Among the given choices, only choice (c) is valid.
1 1 0 1 3
1 0 1 1 3
0 1 1 1 3
Permutations & Combinations 1149

40 Let A, B, C, D, E, F, G, H, I, J, K be the 11 masseurs, then divide them in two groups say one group has 5 masseurs (A, B, C, D and E),
while another group has 6 masseurs (F, G, H, I, J and K). In the first week, employ first group of 5 masseurs.
Number of masseurs familiar Number of weeks
Case with queen required to Reason
First week Second week identify
I 5 0 1 All 5 are familiar in the first week itself
II 4 1 1 4 out of 5 are familiar in the first week itself
III 3 2 1 3 out of 5 are familiar in the first week itself
IV 2 3 >2 3 out of 6 are familiar so 3 must be strangers, thus, if all
3 strangers and any 2 familiar masseurs give the massage out of
5 days on the 2nd, 3rd or 4th week, king still can’t identify them.
So king needs 5th week to identify the masseurs familiar with the
queen.
V 1 4 2 4 out of 6 are familiar and 2 are strangers, so even if all 2 strangers
give the massage out of 5 days, 3 familiar masseurs will still be
giving the massage in the second week, so king can identify them.
VI 0 5 2 5 out of 6 are familiar and 1 is stranger, so even if 1 stranger gives
the massage out of 5 days, 4 familiar masseurs will still be giving
the massage in the second week, so king can identify them.

Suppose in the first week A, B, C, D and E give her massage 41 Let A be the set of permutations where 3 and 6 appear
and the king could not identify them. It implies that there consecutively, B be the set of permutations, where 6 and 5
are maximum 2 familiar masseurs among A, B, C, D and E. appear consecutively, and let C be the set of permutations,
In turn, it implies that there are minimum 3 familiar where 5 and 4 do. Then| A | = | B | = |C | = 9 !.
masseurs among the F, G, H, I, J, K. If there are 5 familiar
masseurs in the group F, G, H, I, J and K, so whoever the For example, permutations in A correspond to
5 masseurs, among these 6 masseurs, are giving the permutations of the alphabet {0, 1, 2, 36, 4, 5, 7, 8, 9},
where 36 is considered as a single symbol.
massage king would certainly identify them.
Similarly, if there are 4 familiar masseurs in the group Similarly,| A ∩ B | = | B ∩ C | = |C ∩ A | = 8 !
F, G, H, I, J and K, so whoever the 5 masseurs, among these For example, permutations in A ∩ B correspond to
6 masseurs, are giving the massage king would certainly permutations of the alphabet with symbol 365 replacing 3,
identify them. 5 and 6.
But when there are only 3 familiar masseurs in the group F, Similarly,| A ∩ B ∩ C | = 7 !
G, H, I, J and K, then we will have the following possibilities
∴ | A ∪ B ∪ C | =| A | + | B | + | C | −| A ∩ B | −| B ∩ C |
assuming that I, J and K are the 3 familiar masseurs.
− |C ∩ A | − | A ∩ B ∩ C |
{F , G, H , I, J} , {F , G, H , I, K }, {F , G, H , J, K }, {F , G, I, J, K },
⇒ | A ∪ B ∪ C | = 3(9 !) − 3(8 !) − 7 ! = 191(7 !) = 962640
{F , H , I, J, K }, {G, H , I, J, K }
42 Number of solutions of x1 + x 2 + x 3 + x 4 = 30 for
Out of the 6 possibilities there are 3 cases in which king
3 ≤ x1, x 2, x 3, x 4 ≤ 10 is same as the number of solutions
cannot identify the familiar masseurs.
of y1 + y 2 + y 3 + y 4 = 18 for 0 ≤ y1, y 2, y 3, y 4 ≤ 7
It means in the worst case scenario king has to wait for
But the number of solutions of y1 + y 2 + y 3 + y 4 = 18 for
5 weeks to identify the 5 familiar masseurs as illustrated
0 ≤ y1, y 2, y 3, y 4 ≤ 7 = (number of solutions of
below.
y1 + y 2 + y 3 + y 4 = 18 for 0 ≤ y1, y 2, y 3, y 4 ≤ 18) −
Week 1: A, B, C, D, E can’t identify
(number of solutions of y1 + y 2 + y 3 + y 4 = 18 for 8
Week 2: F, G, H, I, J can’t identify ≤ y1, y 2, y 3, y 4 ≤ 18)
Week 3: F, G, H, I, K can’t identify
Now, number of solutions of y1 + y 2 + y 3 + y 4 = 18 for
Week 4: F, G, H, J, K can’t identify 0 ≤ y1, y 2, y 3, y 4 ≤ 18 is 18 + 4 − 1C 4 − 1 = 21C 3 = 1330.
Week 5: F, G, I, J, K can identify
Week 6: F, H, I, J, K can identify Now the number of solutions of y1 + y 2 + y 3 + y 4 = 18 for
Week 7: G, H, I, J, K can identify 8 ≤ y1, y 2, y 3, y 4 ≤ 18 can be determined as follows.
So, in the worst case scenario, king would require at least Since, y i ≥ 8 so out of four values of y either one value of y
5 weeks to be 100% sure if he wants to identify the or two values of y can be equal to or greater than 8 as
masseurs familiar with the queen. y1 + y 2 + y 3 + y 4 = 18.
1150 QUANTUM CAT
Then number of solutions of y1 + y 2 + y 3 + y 4 = 18 for the Chinese restaurants = 20
C 2 − 20
8 ≤ y1, y 2, y 3, y 4 ≤ 18 is
Similarly, number of transactions among Thai restaurants
| A1 ∪ A2 ∪ A3 ∪ A4 | = ∑| Ai | − ∑| Ai ∩ A j| = 15C 2 − 15
1 ≤ i≤ 4 1 ≤ i< j ≤ 4

(18 − 8 ) + 4 − 1 (18 − 2 × 8 ) + 4 − 1 And, the number of transactions among Italian restaurants


= 4[ C 4 − 1] − 6 [ C 4 − 1]
= 10C 2 − 10
= 4(13C 3 ) − 6(5C 3 ) = 4 × 286 − 6 × 10 = 1084 Total number of transactions
Therefore the number of solutions of = (20C 2 − 20) + (15C 2 − 15) + (10C 2 − 10)
y1 + y 2 + y 3 + y 4 = 18 for 0 ≤ y1, y 2, y 3, y 4 ≤ 7 is But since these restaurants can make up to 2 transactions a
1330 − 1084 = 246 day, total number of transactions
So the number of solutions of x1 + x 2 + x 3 + x 4 = 30 for = 2 × {(20C 2 − 20) + (15C 2 − 15) + (10C 2 − 10)}
3 ≤ x1, x 2, x 3, x 4 ≤ 10 is 246. Therefore, total number of transactions
Hint | Ai | can be selected in 4 ways. For example, | A1| indicates = 3 × {45C 2 − (20C 2 + 15C 2+ 10C 2 )}
that z1+ y2 + y3 + y 4 = 18 − 8; where z1 = y1 − 8.
+ 2 × {(20C 2 − 20) + (15C 2 − 15) + (10C 2 − 10)}
Similarly, | Ai ∩ A j | can be selected in 6 ways. For example Now, as you know that when two people make a
| A1 ∩ A2 | indicates that z1 + z2 + y3 + y 4 = 18 − ( 8 + 8) ; where transaction so either of them can give or take change, so
z1 = y1 − 8 and z2 = y2 − 8.
the total number of times the change can be given
43 If x ≤ 3, the given expression will be zero or negative, = 2 × [ 3 × {45C 2 − (20C 2 + 15C 2 + 10C 2 )}
which is impossible. So choices (a) and (c) are wrong ones.
+ 2 × {(20C 2 − 20) + (15C 2 − 15) + (10C 2 − 10)}]
Again if you consider k as a large number, the given
relation won’t satisfy. = 2 × [ 3×(45C 2 ) −{(20C 2 )+ (15C 2 )+ (10C 2 )}− 2(20 + 15 + 10)]
Hence, choice (d) is the answer. = 5080
n− 1
Alternatively C r = (k 2 − 3)(nC r + 1 )
(n − 1)! (n)! NOTE I have used the term “transactions” which indicates, for
⇒ = (k 2 − 3) my purpose, the flow of something in one direction, that’s why
(r)! ⋅ (n − 1 − r)! (r + 1)! ⋅ (n − r − 1)! I have multiplied by 2 in the last part to accommodate the
(n − 1)! (n) ⋅ [(n − 1)] two-way transactions.
⇒ = (k 2 − 3)
(r)!(n − 1 − r)! (r + 1) ⋅ [(r)!] ⋅ (n − r − 1)!
45 Since x + 1 is a factor of ax 2 + bx + c, therefore x + 1 = 0 or
r+1 x = − 1.
⇒ (k 2 − 3) =
n
Then we have a − b + c = 0 ⇒ a + c = b. But, since a ≠ b ≠ c,
r+1
The minimum value of will be close to zero but so let us consider a < c, then we have the following
n possibilities for a, b and c.
positive when n is extremely high.
Number of
And the maximum value of r + 1/ n will be 1 when a c b
solutions
n = r + 1 ,since r + 1 cannot exceed n.
1 2, 3, ......1998 3, 4, ..., 1999 1997
Therefore, 0 < (k 2 − 3) ≤ 1 ⇒ 3 < k 2 ≤ 4 ⇒ 3 < k ≤ 2.
2 3, 4, ..., 1997 5, 6, ..., 1999 1995
Hence choice (d) is the answer. 3 4, 5, ..., 1996 7, 8, ..., 1999 1993
44 Total number of restaurants = 20 + 15 + 10 = 45 …
Number of transactions among 45 restaurants = 45
C 2. …
Number of transactions within each type of restaurants 997 998, ..., 1002 1995, ..., 1999 5
= 20C 2 + 15C 2 + 10C 2 998 999, 1000, 1997, 1998, 3
Number of transactions among different categories of 1001 1999
restaurants = 45C 2 − (20C 2 + 15C 2 + 10C 2 ) 999 1000 1999 1
Since these restaurants can make up to 3 transactions a Total number of solutions
day, total number of transactions
= 1 + 3 + 5 + . . . . + 1993 + 1995 + 1997 = 9992
= 3 × {45C 2 − (20C 2 + 15C 2 + 10C 2 )}
Number of transactions among Chinese restaurants = 20
C2 Thus for a < c, we have total 9992 number of solutions.
But since no two neighboring restaurants make any Similarly for a > c, we will have total 9992 number of
transaction, so the actual number of transactions among solutions.
Permutations & Combinations 1151

Therefore the required number of solutions (or There are many possibilities for the numbers in position 4
polynomials) = 9992 + 9992 = 2(999)2 = 1996002 and 5.
Alternatively a + c ≤ 1999 {a, c ≥ 1}
In position 4 we can have 8, 7, 6, 5, 4, 3, 2, 1
In position 5 what we have depends on what we have in
⇒ a + c ≤ 1997 {a, c ≥ 0}
previous position. If we have 8 in the previous position, we
⇒ a + c + d = 1997 {a, c, d ≥ 0} have 8 possibilities 7, 6, 5, 4, 3, 2, 1, 0. If the number is in
Number of solutions of the above equation is the 4th position is 7, we have 7 possibilities - 6, 5, 4, 3, 2, 1,
1997 + 3 − 1
C 3 − 1 = 1999C 2 = 999 × 1999 0 and so on and finally if there is 1 in position 4, we have
only 1 possibility 0.
When a = c, we have 2a + d = 1997 So basically, the number of possibilities is
{a, c, d ≥ 0} 8 + 7 + 6 + 5 + 4 + 3 + 2 + 1 = 36
Then the number of solutions of 2a + d = 1997 is 999, as Hint We can use the formula for the sum of first n natural
a = 0, 1, 2, . . . 998. n( n + 1)
numbers, .
Therefore the number of required solutions when a ≠ c, is 2
8×9
999 × 1999 − 999 Therefore, 1 + 2 + 3 + ... + 8 = = 36
2
= 999 (1998) = 9992 × 2 = 1996002
48 Total number of triangles = 10C 3 = 120
46 Let the three sides of isosceles triangle be a, a and b, then
there will be a triangle if 2a > b. Number of triangles, which share exactly one side of the
Case I If 1 ≤ a ≤ 1007 decagon: first we can choose any 10 sides. Now, we cannot
choose the two adjacent vertices of the chosen side, so we
a b
Number of can choose the third vertex out of the remaining 6(= 10 − 4)
possibilities vertices. Therefore, the number of triangles that share only
1 1 1 one side of the decagon = 10 × 6 = 60.
2 1, 2, 3 3 Number of triangles, which share exactly two sides of the
3 1, 2, 3, 4, 5 5 decagon: first we can choose any two adjacent sides in 10
ways, out of the 10 sides of the decagon. The third side will
... .... .... be naturally formed by joining the two non-common
1007 1, 2, 3, ..., 2013 2013 vertices of the two chosen sides. Therefore, the number of
triangles that share exactly two sides of the decagon = 10
Thus the number of triangles Thus the required number of triangles
= 1 + 3 + 5 + . . . + 2013 = (1007 )2 = 120 − (60 − 10) = 50
Case II If 1008 ≤ a ≤ 2014 Alternatively What we want is that any three vertices
which are not adjacent to each other. So let us assume that
Number of A, B and C are the three vertices of the triangle taken
a b
possibilities
clockwise, then have to select three vertices A, B, C such
1008 1, 2, 3, ...., 2014 2014 that at least one vertex of decagon lies between A and B
1009 1, 2, 3, ...., 2014 2014 and so between B and C. There are the various possibilities
to choose from the remaining 7 (= 10 − 3) vertices between
1010 1, 2, 3, ...., 2014 2014
A, B and C.
... ... ...
Case (i) (5, 1, 1)-Number of triangles = 10
2014 1, 2, 3, ...., 2014 2014

Thus the number of triangles


= 2014 + 2014 + 2014 + . . . + 2014
= 1007 × 2014 = 2(1007 )2
Therefore total number of isosceles triangles Case (ii) (4, 2, 1) - Number of triangles = 20
= (1007 )2 + 2(1007 )2 = 3(1007 )2= 2028098

47 The numbers in 2nd and 3rd position can only be 8 and 9


because they have to be strictly increasing.
So the first 3 numbers have to be 789.
1152 QUANTUM CAT
Case (iii) (3, 3, 1) - Number of triangles = 10 every such group must be able to open all the locks.
It shows that there are 7 C 3 = 35 groups of 3 people each,
and for each group there is a lock that the pertinent group
cannot open it. But, any group of 4 people can open every
lock. It implies that there must be at least 35 locks. If there
are fewer than 35 locks, then all these locks can be opened
Case (iv) (3, 2, 2) - Number of triangles = 10 without even majority being present, as there must be
some group of 3 people having the keys of all the locks.
Thus the minimum number of locks = 35.
Now, associate each group of 3 people (G i ) with each lock
(L i ) and give the keys of the corresponding lock to all other
people not associated with a particular lock. Therefore,
Thus the total number of require triangles
each person (in a group of 3) receives 6C 3 = 20 keys.
= 10 + 20 + 10 + 10 = 50
Thus the minimum number of keys = 20.
49 We can get a majority if we have 4 or more people together
50 When a block is at the centre and connected with all other
to unlock the box. If there are 3 or fewer people trying to
24 blocks we have minimum number of required roads
open the box, they should not be able to open it.
= 24
So, in order to have the optimal solution to this problem,
The maximum number of roads can be laid out when any
we will focus on the closest values. That is 3 and 4.
blocks are directly connected with each other. It is
If we form groups of 3 people each, no such group should 25
C 2 = 300.
be able to open all the locks. But, when we form groups of 4
people each, Therefore, we have 24 ≤ R ≤ 300.

You might also like